Paediatric

Download as pdf or txt
Download as pdf or txt
You are on page 1of 423

Question 1 of 257

A 2-month old baby is admitted to the Paediatric Ward with persistent vomiting and failure to gain weight.
Bloods taken on admission show the following:
Na+
136 mmol/l
K+
3.1 mmol/l
Cl81 mmol/l
HCO3-30 mmol/l
An ultrasound of the stomach and duodenum is performed:

Image used on license from Radiopaedia

What is the most likely diagnosis?


Duodenal atresia

Pyloric stenosis

Malrotation

Gastro-oesophageal reflux disease

Coeliac disease

Next question

The ultrasound demonstrates a thickened and elongated pylorus. The bloods also show a hypochloraemic, hypokalaemic
alkalosis in keeping with the diagnosis.

Pyloric stenosis
Pyloric stenosis typically presents in the second to fourth weeks of life with vomiting, although rarely may present later at up to
four months. It is caused by hypertrophy of the circular muscles of the pylorus
Epidemiology

incidence of 4 per 1,000 live births


4 times more common in males
10-15% of infants have a positive family history
first-borns are more commonly affected

Features

'projectile' vomiting, typically 30 minutes after a feed


constipation and dehydration may also be present
a palpable mass may be present in the upper abdomen
hypochloraemic, hypokalaemic alkalosis due to persistent vomiting

Diagnosis is most commonly made by ultrasound


Management is with Ramstedt pyloromyotomy

Question 2 of 257
A mother brings her 14-month-old son into surgery. Since yesterday he seems to be straining whilst passing stools. She describes
him screaming, appearing to be in pain and pulling his knees up towards his chest. These episodes are now occurring every 1520 minutes. This morning she noted a small amount of blood in his nappy. He is taking around 50% of his normal feeds and
vomited once this morning. On examination he appears irritable and lethargic but is well hydrated and apyrexial. Abdominal
examination is unremarkable. What is the most likely diagnosis?
Constipation

Intussusception

Gastroenteritis

Meckel's diverticulum

Volvulus

Next question

Intussusception
Intussusception describes the invagination of one portion of bowel into the lumen of the adjacent bowel, most commonly around
the ileo-caecal region.
Intussusception is usually affects infants between 6-18 months old. Boys are affected twice as often as girls
Features

paroxysmal abdominal colic pain

during paroxysm the infant will characteristically draw their knees up and turn pale
vomiting
blood stained stool - 'red-currant jelly'
sausage-shaped mass in the abdomen

Investigation

ultrasound is now the investigation of choice and may show a target -like mass

Management

the majority of children can be treated with reduction by air insufflation under radiological control, which is now widely
used first-line compared to the traditional barium enema
if this fails, or the child has signs of peritonitis, surgery is performed

Question 3 of 257
A 2-month-old boy is brought to the afternoon surgery by his mother. Since the morning he has been taking reduced feeds and
has been 'not his usual self'. On examination the baby appears well but has a temperature of 38.7C. What is the most
appropriate management?
Advise regarding antipyretics, to see if not settling

IM benzylpenicillin

Advise regarding antipyretics, booked appointment for next day

Admit to hospital

Empirical amoxicillin for 7 days

Next question

Any child less than 3 months old with a temperature > 38C is regarded as a 'red' feature in the new NICE guidelines, warranting
urgent referral to a paediatrician. Although many experienced GPs may choose not to strictly follow such advice it is important to
be aware of recent guidelines for the exam

Feverish illness in children


The 2007 NICE Feverish illness in children guidelines introduced a 'traffic light' system for risk stratification of children under the
age of 5 years presenting with a fever. These guidelines were later modified in a 2013 update.
It should be noted that these guidelines only apply 'until a clinical diagnosis of the underlying condition has been made'. A link to
the guidelines is provided but some key points are listed below.
Assessment
The following should be recorded in all febrile children:

temperature
heart rate
respiratory rate
capillary refill time

Signs of dehydration (reduced skin turgor, cool extremities etc) should also be looked for

Measuring temperature should be done with an electronic thermometer in the axilla if the child is < 4 weeks or with an
electronic/chemical dot thermometer in the axilla or an infra-red tympanic thermometer.
Risk stratification
Please see the link for the complete table, below is a modified version
Green - low risk
Normal colour
Responds normally to
social cues
Content/smiles
Stays awake or awakens
quickly
Strong normal cry/not
crying

Colour

Activity

Amber - intermediate risk


Pallor reported by parent/carer

Red - high risk


Pale/mottled/ashen/blue

Not responding normally to social cues


No smile
Wakes only with prolonged stimulation
Decreased activity

No response to social cues


Appears ill to a healthcare
professional
Does not wake or if roused does not
stay awake
Weak, high-pitched or continuous cry

Nasal flaring
Tachypnoea: respiratory rate

Respiratory

>50 breaths/minute, age 6-12 Grunting


months;
Tachypnoea: respiratory rate >60
>40 breaths/minute, age >12 breaths/minute
Moderate or severe chest indrawing
months

Oxygen saturation <=95% in air


Crackles in the chest
Tachycardia:

Circulation and
hydration

Normal skin and eyes


Moist mucous
membranes

>160 beats/minute, age <12


months
>150 beats/minute, age 12-24
months
>140 beats/minute, age 2-5
years

Reduced skin turgor

Capillary refill time >=3 seconds


Dry mucous membranes
Poor feeding in infants
Reduced urine output

Other

No amber or red signs

Age 3-6 months, temperature >=39C


Fever for >=5 days
Rigors
Swelling of a limb or joint
Non-weight bearing limb/not using an
extremity

Age <3 months, temperature >=38C


Non-blanching rash
Bulging fontanelle
Neck stiffness
Status epilepticus
Focal neurological signs
Focal seizures

Management
If green:

Child can be managed at home with appropriate care advice, including when to seek further help

If amber:

provide parents with a safety net or refer to a paediatric specialist for further assessment
a safety net includes verbal or written information on warning symptoms and how further healthcare can be accessed, a
follow-up appointment, liaison with other healthcare professionals, e.g. out-of-hours providers, for further follow-up

If red:

refer child urgently to a paediatric specialist

Other key points include

oral antibiotics should not be prescribed to children with fever without apparent source
if a pneumonia is suspected but the child is not going to be referred to hospital then a chest x-ray does not need to be
routinely performed

Recommendations
1.1 Thermometers and the detection of fever
1.2 Clinical assessment of children with fever
1.3 Management by remote assessment
1.4 Management by the non-paediatric practitioner
1.5 Management by the paediatric specialist
1.6 Antipyretic interventions
1.7 Advice for home care
1 Recommendations
The following guidance is based on the best available evidence. The full guideline gives
details of the methods and the evidence used to develop the guidance.
The wording used in the recommendations in this guideline denotes the certainty with
which the recommendation is made (the strength of the recommendation). See About
this guideline for details.
This guideline is intended for use by healthcare professionals for the assessment and
initial management in young children with feverish illness. The guideline should be
followed until a clinical diagnosis of the underlying condition has been made. Once a
diagnosis has been made, the child should be treated according to national or local
guidance for that condition.
Parents or carers of a child with fever may approach a range of different healthcare
professionals as their first point of contact, for example, a GP, a pharmacist or an
emergency care practitioner. The training and experience of the healthcare
professionals involved in the child's care will vary and each should interpret the
guidance according to the scope of their own practice.
For the purposes of this guideline, fever was defined as 'an elevation of body
temperature above the normal daily variation'.
This guideline should be read in conjunction with:
Bacterial meningitis and meningococcal septicaemia (NICE clinical guideline 102).
Urinary tract infection in children (NICE clinical guideline 54).
Diarrhoea and vomiting in children under 5 (NICE clinical guideline 84).
1.1 Thermometers and the detection of fever
1.1.1 Oral and rectal temperature measurements
1.1.1.1 Do not routinely use the oral and rectal routes to measure the body temperature
of children aged 05 years. [2007]
1.1.2 Measurement of body temperature at other sites
1.1.2.1 In infants under the age of 4 weeks, measure body temperature with an
electronic thermometer in the axilla. [2007]

1.1.2.2 In children aged 4 weeks to 5 years, measure body temperature by one of the
following methods:
electronic thermometer in the axilla
chemical dot thermometer in the axilla
infra-red tympanic thermometer. [2007]
1.1.2.3 Healthcare professionals who routinely use disposable chemical dot
thermometers should consider using an alternative type of thermometer when multiple
temperature measurements are required. [2007]
1.1.2.4 Forehead chemical thermometers are unreliable and should not be used by
healthcare professionals. [2007]
1.1.3 Subjective detection of fever by parents and carers
1.1.3.1 Reported parental perception of a fever should be considered valid and taken
seriously by healthcare professionals. [2007]
1.2 Clinical assessment of children with fever
1.2.1 Life-threatening features of illness in children
1.2.1.1 First, healthcare professionals should identify any immediately life-threatening
features, including compromise of the airway, breathing or circulation, and decreased
level of consciousness. [2007]
1.2.2 Assessment of risk of serious illness
1.2.2.1 Assess children with feverish illness for the presence or absence of symptoms
and signs that can be used to predict the risk of serious illness using the traffic light
system (see table 1). [2013]
1.2.2.2 When assessing children with learning disabilities, take the individual child's
learning disability into account when interpreting the traffic light table. [new 2013]
1.2.2.3 Recognise that children with any of the following symptoms or signs are in a
high-risk group for serious illness:
pale/mottled/ashen/blue skin, lips or tongue
no response to social cues[3]
appearing ill to a healthcare professional
does not wake or if roused does not stay awake
weak, high-pitched or continuous cry
grunting
respiratory rate greater than 60 breaths per minute
moderate or severe chest indrawing
reduced skin turgor
bulging fontanelle. [new 2013]
1.2.2.4 Recognise that children with any of the following symptoms or signs are in at
least an intermediate-risk group for serious illness:
pallor of skin, lips or tongue reported by parent or carer
not responding normally to social cues[3]
no smile
wakes only with prolonged stimulation

decreased activity
nasal flaring
dry mucous membranes
poor feeding in infants
reduced urine output
rigors. [new 2013]
1.2.2.5 Recognise that children who have all of the following features, and none of the
high- or intermediate-risk features, are in a low-risk group for serious illness:
normal colour of skin, lips and tongue
responds normally to social cues[3]
content/smiles
stays awake or awakens quickly
strong normal cry or not crying
normal skin and eyes
moist mucous membranes. [new 2013]
1.2.2.6 Measure and record temperature, heart rate, respiratory rate and capillary refill
time as part of the routine assessment of a child with fever. [2007]
1.2.2.7 Recognise that a capillary refill time of 3 seconds or longer is an intermediaterisk group marker for serious illness ('amber' sign). [2013]
1.2.2.8 Measure the blood pressure of children with fever if the heart rate or capillary
refill time is abnormal and the facilities to measure blood pressure are available. [2007]
1.2.2.9 In children older than 6 months do not use height of body temperature alone to
identify those with serious illness. [2013]
1.2.2.10 Recognise that children younger than 3 months with a temperature of 38C or
higher are in a high-risk group for serious illness. [2013]
1.2.2.11 Recognise that children aged 36 months with a temperature of 39C or higher
are in at least an intermediate-risk group for serious illness. [new 2013]
1.2.2.12 Do not use duration of fever to predict the likelihood of serious illness.
However, children with a fever lasting more than 5 days should be assessed for
Kawasaki disease (see recommendation 1.2.3.10). [new 2013]
1.2.2.13 Recognise that children with tachycardia are in at least an intermediate-risk
group for serious illness. Use the Advanced Paediatric Life Support (APLS)[4] criteria
below to define tachycardia: [new 2013]

Age

Heart rate (bpm)

<12 months

>160

1224 months >150


25 years

>140

1.2.2.14 Assess children with fever for signs of dehydration. Look for:

prolonged capillary refill time


abnormal skin turgor
abnormal respiratory pattern
weak pulse
cool extremities. [2007]
1.2.3 Symptoms and signs of specific illnesses
1.2.3.1 Look for a source of fever and check for the presence of symptoms and signs
that are associated with specific diseases (see table 2). [2007]
1.2.3.2 Consider meningococcal disease in any child with fever and a non-blanching
rash, particularly if any of the following features are present[5]:
an ill-looking child
lesions larger than 2 mm in diameter (purpura)
a capillary refill time of 3 seconds or longer
neck stiffness. [2007]
1.2.3.3 Consider bacterial meningitis in a child with fever and any of the following
features[5]:
neck stiffness
bulging fontanelle
decreased level of consciousness
convulsive status epilepticus. [2007, amended 2013]
1.2.3.4 Be aware that classic signs of meningitis (neck stiffness, bulging fontanelle, highpitched cry) are often absent in infants with bacterial meningitis[5]. [2007]
1.2.3.5 Consider herpes simplex encephalitis in children with fever and any of the
following features:
focal neurological signs
focal seizures
decreased level of consciousness. [2007]
1.2.3.6 Consider pneumonia in children with fever and any of the following signs:
tachypnoea (respiratory rate greater than 60 breaths per minute, age 05 months;
greater than 50 breaths per minute, age 612 months; greater than 40 breaths per
minute, age older than 12 months)
crackles in the chest
nasal flaring
chest indrawing
cyanosis
oxygen saturation of 95% or less when breathing air. [2007]
1.2.3.7 Consider urinary tract infection in any child younger than 3 months with fever[6].
[2007]
1.2.3.8 Consider urinary tract infection in a child aged 3 months or older with fever and
1 or more of the following[6]:
vomiting
poor feeding

lethargy
irritability
abdominal pain or tenderness
urinary frequency or dysuria. [new 2013]
1.2.3.9 Consider septic arthritis/osteomyelitis in children with fever and any of the
following signs:
swelling of a limb or joint
not using an extremity
non-weight bearing. [2007]
1.2.3.10 Consider Kawasaki disease in children with fever that has lasted longer than
5 days and who have 4 of the following 5 features:
bilateral conjunctival injection
change in mucous membranes in the upper respiratory tract (for example, injected
pharynx, dry cracked lips or strawberry tongue)
change in the extremities (for example, oedema, erythema or desquamation)
polymorphous rash
cervical lymphadenopathy
Be aware that, in rare cases, incomplete/atypical Kawasaki disease may be diagnosed
with fewer features. [2007]
1.2.4 Imported infections
1.2.4.1 When assessing a child with feverish illness, enquire about recent travel abroad
and consider the possibility of imported infections according to the region visited.
[2007]
Table 1 Traffic light system for identifying risk of serious illness
[new 2013]
Children with fever and any of the symptoms or signs in the red column should be
recognised as being at high risk. Similarly, children with fever and any of the symptoms
or signs in the amber column and none in the red column should be recognised as being
at intermediate risk. Children with symptoms and signs in the green column and none in
the amber or red columns are at low risk. The management of children with fever
should be directed by the level of risk.
This traffic light table should be used in conjunction with the recommendations in this
guideline on investigations and initial management in children with fever.
A colour version of this table is available.

Colour
(of skin, lips or
tongue)

Green low risk

Amber intermediate risk Red high risk

Normal colour

Pallor reported by
parent/carer

Pale/mottled/ashen/blue

Activity

Responds normally to Not responding normally


social cues
to social cues
Content/smiles

No smile

Appears ill to a healthcare


professional

Stays awake or
awakens quickly

Wakes only with


prolonged stimulation

Does not wake or if roused


does not stay awake

Strong normal cry/not Decreased activity


crying
Respiratory

No response to social cues

Weak, high-pitched or
continuous cry

Nasal flaring

Grunting

Tachypnoea: respiratory
rate

Tachypnoea: respiratory rate


>60 breaths/minute

>50 breaths/minute, age


612 months;

Moderate or severe chest


indrawing

>40 breaths/minute, age


>12 months
Oxygen saturation 95% in
air
Crackles in the chest
Circulation
Normal skin and eyes Tachycardia:
and hydration
Moist mucous
>160 beats/minute, age
membranes
<12 months
>150 beats/minute,
age 1224 months
>140 beats/minute, age 2
5 years
Capillary refill time
3 seconds
Dry mucous membranes
Poor feeding in infants
Reduced urine output

Reduced skin turgor

Other

None of the amber or Age 36 months,


red symptoms or
temperature 39C
signs
Fever for 5 days
Rigors

Age <3 months, temperature


38C
Non-blanching rash
Bulging fontanelle

Swelling of a limb or joint Neck stiffness


Non-weight bearing
limb/not using an
extremity

Status epilepticus
Focal neurological signs
Focal seizures

Table 2 Summary table for symptoms and signs suggestive of specific diseases
[2013]

Diagnosis to be
considered

Symptoms and signs in conjunction with fever

Meningococcal
disease

Non-blanching rash, particularly with 1 or more of the following:


an ill-looking child
lesions larger than 2 mm in diameter (purpura)
capillary refill time of 3 seconds
neck stiffness

Bacterial meningitis Neck stiffness


Bulging fontanelle
Decreased level of consciousness
Convulsive status epilepticus
Herpes simplex
encephalitis

Focal neurological signs


Focal seizures
Decreased level of consciousness

Pneumonia

Tachypnoea (respiratory rate >60 breaths/minute, age 05 months; >50


breaths/minute, age 612 months; >40 breaths/minute, age >12 months)

Crackles in the chest


Nasal flaring
Chest indrawing
Cyanosis
Oxygen saturation 95%
Urinary tract
infection

Vomiting
Poor feeding
Lethargy
Irritability
Abdominal pain or tenderness
Urinary frequency or dysuria

Septic arthritis

Swelling of a limb or joint


Not using an extremity
Non-weight bearing

Kawasaki disease

Fever for more than 5 days and at least 4 of the following:


bilateral conjunctival injection
change in mucous membranes
change in the extremities
polymorphous rash
cervical lymphadenopathy

1.3 Management by remote assessment


Remote assessment refers to situations in which a child is assessed by a healthcare
professional who is unable to examine the child because the child is geographically
remote from the assessor (for example, telephone calls to NHS Direct[7]). Therefore,
assessment is largely an interpretation of symptoms rather than physical signs. The
guidance in this section may also apply to healthcare professionals whose scope of
practice does not include the physical examination of a young child (for example,
community pharmacists).

1.3.1 Management according to risk of serious illness


1.3.1.1 Healthcare professionals performing a remote assessment of a child with fever
should seek to identify symptoms and signs of serious illness and specific diseases as
described in section 1.2 and summarised in tables 1 and 2. [2007]
1.3.1.2 Children whose symptoms or combination of symptoms suggest an immediately
life-threatening illness (see recommendation 1.2.1.1) should be referred immediately
for emergency medical care by the most appropriate means of transport (usually 999
ambulance). [2007]
1.3.1.3 Children with any 'red' features but who are not considered to have an
immediately life-threatening illness should be urgently assessed by a healthcare
professional in a face-to-face setting within 2 hours. [2007]
1.3.1.4 Children with 'amber' but no 'red' features should be assessed by a healthcare
professional in a face-to-face setting. The urgency of this assessment should be
determined by the clinical judgement of the healthcare professional carrying out the
remote assessment. [2007]
1.3.1.5 Children with 'green' features and none of the 'amber' or 'red' features can be
cared for at home with appropriate advice for parents and carers, including advice on
when to seek further attention from the healthcare services (see section 1.7). [2007,
amended 2013]
1.4 Management by the non-paediatric practitioner
In this guideline, a non-paediatric practitioner is defined as a healthcare professional
who has not had specific training or who does not have expertise in the assessment and
treatment of children and their illnesses. This term includes healthcare professionals
working in primary care, but it may also apply to many healthcare professionals in
general emergency departments.
1.4.1 Clinical assessment
1.4.1.1 Management by a non-paediatric practitioner should start with a clinical
assessment as described in section 1.2. Healthcare practitioners should attempt to
identify symptoms and signs of serious illness and specific diseases as summarised in
tables 1 and 2. [2007]
1.4.2 Management according to risk of serious illness
1.4.2.1 Children whose symptoms or combination of symptoms and signs suggest an
immediately life-threatening illness (see recommendation 1.2.1.1) should be referred
immediately for emergency medical care by the most appropriate means of transport
(usually 999 ambulance). [2007]
1.4.2.2 Children with any 'red' features but who are not considered to have an
immediately life-threatening illness should be referred urgently to the care of a
paediatric specialist. [2007]
1.4.2.3 If any 'amber' features are present and no diagnosis has been reached, provide
parents or carers with a 'safety net' or refer to specialist paediatric care for further
assessment. The safety net should be 1 or more of the following:

providing the parent or carer with verbal and/or written information on warning
symptoms and how further healthcare can be accessed (see section 1.7.2)
arranging further follow-up at a specified time and place
liaising with other healthcare professionals, including out-of-hours providers, to ensure
direct access for the child if further assessment is required. [2007]
1.4.2.4 Children with 'green' features and none of the 'amber' or 'red' features can be
cared for at home with appropriate advice for parents and carers, including advice on
when to seek further attention from the healthcare services (see section 1.7). [2007,
amended 2013]
1.4.3 Tests by the non-paediatric practitioner
1.4.3.1 Children with symptoms and signs suggesting pneumonia who are not admitted
to hospital should not routinely have a chest X-ray. [2007]
1.4.3.2 Test urine in children with fever as recommended in Urinary tract infection in
children (NICE clinical guideline 54). [2007]
1.4.4 Use of antibiotics by the non-paediatric practitioner
1.4.4.1 Do not prescribe oral antibiotics to children with fever without apparent source.
[2007]
1.4.4.2 Give parenteral antibiotics to children with suspected meningococcal disease at
the earliest opportunity (either benzylpenicillin or a third-generation cephalosporin)[5].
[2007]
1.5 Management by the paediatric specialist
In this guideline, the term paediatric specialist refers to a healthcare professional who
has had specific training or has recognised expertise in the assessment and treatment of
children and their illnesses. Examples include paediatricians, or healthcare professionals
working in children's emergency departments.
1.5.1 Children younger than 5 years
1.5.1.1 Management by the paediatric specialist should start with a clinical assessment
as described in section 1.2. The healthcare professional should attempt to identify
symptoms and signs of serious illness and specific diseases as summarised in tables 1
and 2. [2007]
1.5.2 Children younger than 3 months
1.5.2.1 Infants younger than 3 months with fever should be observed and have the
following vital signs measured and recorded:
temperature
heart rate
respiratory rate. [2007]
1.5.2.2 Perform the following investigations in infants younger than 3 months with
fever:
full blood count
blood culture
C-reactive protein
urine testing for urinary tract infection[6]

chest X-ray only if respiratory signs are present


stool culture, if diarrhoea is present. [2013]
1.5.2.3 Perform lumbar puncture in the following children with fever (unless
contraindicated):
infants younger than 1 month
all infants aged 13 months who appear unwell
infants aged 13 months with a white blood cell count (WBC) less than 5 109/litre or
greater than 15 109/litre. [2007, amended 2013]
1.5.2.4 When indicated, perform a lumbar puncture without delay and, whenever
possible, before the administration of antibiotics. [2007]
1.5.2.5 Give parenteral antibiotics to:
infants younger than 1 month with fever
all infants aged 13 months with fever who appear unwell
infants aged 13 months with WBC less than 5 109/litre or greater than 15 109/litre.
[2007, amended 2013]
1.5.2.6 When parenteral antibiotics are indicated for infants younger than 3 months of
age, a third-generation cephalosporin (for example cefotaxime or ceftriaxone) should be
given plus an antibiotic active against listeria (for example, ampicillin or amoxicillin).
[2007]
1.5.3 Children aged 3 months or older
1.5.3.1 Perform the following investigations in children with fever without apparent
source who present to paediatric specialists with 1 or more 'red' features:
full blood count
blood culture
C-reactive protein
urine testing for urinary tract infection[6]. [2013]
1.5.3.2 The following investigations should also be considered in children with 'red'
features, as guided by the clinical assessment:
lumbar puncture in children of all ages (if not contraindicated)
chest X-ray irrespective of body temperature and WBC
serum electrolytes and blood gas. [2007]
1.5.3.3 Children with fever without apparent source presenting to paediatric specialists
who have 1 or more 'amber' features, should have the following investigations
performed unless deemed unnecessary by an experienced paediatrician.
urine should be collected and tested for urinary tract infection[6]
blood tests: full blood count, C-reactive protein and blood cultures
lumbar puncture should be considered for children younger than 1 year
chest X-ray in a child with a fever greater than 39C and WBC greater than 20
109/litre. [2007]
1.5.3.4 Children who have been referred to a paediatric specialist with fever without
apparent source and who have no features of serious illness (that is, the 'green' group),

should have urine tested for urinary tract infection[6] and be assessed for symptoms
and signs of pneumonia (see table 2). [2007]
1.5.3.5 Do not routinely perform blood tests and chest X-rays in children with fever who
have no features of serious illness (that is, the 'green' group). [2007]
1.5.4 Viral co-infection
1.5.4.1 Febrile children with proven respiratory syncytial virus or influenza infection
should be assessed for features of serious illness. Consideration should be given to urine
testing for urinary tract infection[6]. [2007]
1.5.5 Observation in hospital
1.5.5.1 In children aged 3 months or older with fever without apparent source, a period
of observation in hospital (with or without investigations) should be considered as part
of the assessment to help differentiate non-serious from serious illness. [2007]
1.5.5.2 When a child has been given antipyretics, do not rely on a decrease or lack of
decrease in temperature at 12 hours to differentiate between serious and non-serious
illness. Nevertheless, in order to detect possible clinical deterioration, all children in
hospital with 'amber' or 'red' features should still be reassessed after 12 hours. [new
2013]
1.5.6 Immediate treatment by the paediatric specialist (for children of all ages)
1.5.6.1 Children with fever and shock presenting to specialist paediatric care or an
emergency department should be:
given an immediate intravenous fluid bolus of 20 ml/kg; the initial fluid should normally
be 0.9% sodium chloride
actively monitored and given further fluid boluses as necessary. [2007]
1.5.6.2 Give immediate parenteral antibiotics to children with fever presenting to
specialist paediatric care or an emergency department if they are:
shocked
unrousable
showing signs of meningococcal disease. [2007]
1.5.6.3 Immediate parenteral antibiotics should be considered for children with fever
and reduced levels of consciousness. In these cases symptoms and signs of meningitis
and herpes simplex encephalitis should be sought (see table 2 and Bacterial meningitis
and meningococcal septicaemia [NICE clinical guideline 102]). [2007]
1.5.6.4 When parenteral antibiotics are indicated, a third-generation cephalosporin (for
example, cefotaxime or ceftriaxone) should be given, until culture results are available.
For children younger than 3 months, an antibiotic active against listeria (for example,
ampicillin or amoxicillin) should also be given. [2007]
1.5.6.5 Give intravenous aciclovir to children with fever and symptoms and signs
suggestive of herpes simplex encephalitis (see recommendation 1.2.3.5). [2007]
1.5.6.6 Oxygen should be given to children with fever who have signs of shock or oxygen
saturation (SpO2) of less than 92% when breathing air. Treatment with oxygen should
also be considered for children with an SpO2 of greater than 92%, as clinically indicated.
[2007]

1.5.7 Causes and incidence of serious bacterial infection


1.5.7.1 In a child presenting to hospital with a fever and suspected serious bacterial
infection, requiring immediate treatment, antibiotics should be directed against
Neisseria meningitidis, Streptococcus pneumoniae, Escherichia coli, Staphylococcus
aureus and Haemophilus influenzae type b. A third-generation cephalosporin (for
example, cefotaxime or ceftriaxone) is appropriate, until culture results are available.
For infants younger than 3 months, an antibiotic active against listeria (for example,
ampicillin or amoxicillin) should be added. [2007]
1.5.7.2 Refer to local treatment guidelines when rates of bacterial antibiotic resistance
are significant. [2007]
1.5.8 Admission to and discharge from hospital
1.5.8.1 In addition to the child's clinical condition, consider the following factors when
deciding whether to admit a child with fever to hospital:
social and family circumstances
other illnesses that affect the child or other family members
parental anxiety and instinct (based on their knowledge of their child)
contacts with other people who have serious infectious diseases
recent travel abroad to tropical/subtropical areas, or areas with a high risk of endemic
infectious disease
when the parent or carer's concern for their child's current illness has caused them to
seek healthcare advice repeatedly
where the family has experienced a previous serious illness or death due to feverish
illness which has increased their anxiety levels
when a feverish illness has no obvious cause, but the child remains ill longer than
expected for a self-limiting illness. [2007]
1.5.8.2 If it is decided that a child does not need to be admitted to hospital, but no
diagnosis has been reached, provide a safety net for parents and carers if any 'red' or
'amber' features are present. The safety net should be 1 or more of the following:
providing the parent or carer with verbal and/or written information on warning
symptoms and how further healthcare can be accessed (see section 1.7.2)
arranging further follow-up at a specified time and place
liaising with other healthcare professionals, including out-of-hours providers, to ensure
direct access for the child if further assessment is required. [2007]
1.5.8.3 Children with 'green' features and none of the 'amber' or 'red' features can be
cared for at home with appropriate advice for parents and carers, including advice on
when to seek further attention from the healthcare services (see section 1.7). [2007,
amended 2013]
1.5.9 Referral to paediatric intensive care
1.5.9.1 Children with fever who are shocked, unrousable or showing signs of
meningococcal disease should be urgently reviewed by an experienced paediatrician
and consideration given to referral to paediatric intensive care. [2007]

1.5.9.2 Give parenteral antibiotics to children with suspected meningococcal disease at


the earliest opportunity (either benzylpenicillin or a third-generation cephalosporin).
[2007]
1.5.9.3 Children admitted to hospital with meningococcal disease should be under
paediatric care, supervised by a consultant and have their need for inotropes assessed.
[2007]
1.6 Antipyretic interventions
1.6.1 Effects of body temperature reduction
1.6.1.1 Antipyretic agents do not prevent febrile convulsions and should not be used
specifically for this purpose. [2007]
1.6.2 Physical interventions to reduce body temperature
1.6.2.1 Tepid sponging is not recommended for the treatment of fever. [2007]
1.6.2.2 Children with fever should not be underdressed or over-wrapped. [2007]
1.6.3 Drug interventions to reduce body temperature
1.6.3.1 Consider using either paracetamol or ibuprofen in children with fever who
appear distressed. [new 2013]
1.6.3.2 Do not use antipyretic agents with the sole aim of reducing body temperature in
children with fever. [new 2013]
1.6.3.3 When using paracetamol or ibuprofen in children with fever:
continue only as long as the child appears distressed
consider changing to the other agent if the child's distress is not alleviated
do not give both agents simultaneously
only consider alternating these agents if the distress persists or recurs before the next
dose is due. [new 2013]
1.7 Advice for home care
1.7.1 Care at home
1.7.1.1 Advise parents or carers to manage their child's temperature as described in
section 1.6. [2007]
1.7.1.2 Advise parents or carers looking after a feverish child at home:
to offer the child regular fluids (where a baby or child is breastfed the most appropriate
fluid is breast milk)
how to detect signs of dehydration by looking for the following features:
sunken fontanelle
dry mouth
sunken eyes
absence of tears
poor overall appearance
to encourage their child to drink more fluids and consider seeking further advice if they
detect signs of dehydration
how to identify a non-blanching rash
to check their child during the night

to keep their child away from nursery or school while the child's fever persists but to
notify the school or nursery of the illness. [2007]
1.7.2 When to seek further help
1.7.2.1 Following contact with a healthcare professional, parents and carers who are
looking after their feverish child at home should seek further advice if:
the child has a fit
the child develops a non-blanching rash
the parent or carer feels that the child is less well than when they previously sought
advice
the parent or carer is more worried than when they previously sought advice
the fever lasts longer than 5 days
the parent or carer is distressed, or concerned that they are unable to look after their
child. [2007]

Question 4 of 257
A 3-year-old girl is brought in by her mother. Her mother reports that she has been eating less and refusing food for the past few
weeks. Despite this her mother has noticed that her abdomen is distended and she has developed a 'beer belly'. For the past y ear
she has opened her bowels around once every other day, passing a stool of 'normal' consistency. There are no urinary symptoms.
On examination she is on the 50th centile for height and weight. Her abdomen is soft but slightly distended and a non -tender
ballotable mass can be felt on the left side. Her mother has tried lactulose but there has no significant improvement. What is the
most appropriate next step in management?
Switch to polyethylene glycol 3350 + electrolytes (Movicol Paediatric Plain) and review in two weeks

Speak to a local paediatrician

Reassure normal findings and advise Health Visitor review to improve oral intake

Prescribe a Microlax enema

Continue lactulose and add ispaghula husk sachets

Next question

The history of constipation is not particularly convincing. A child passing a stool of normal consistency every other day is within the
boundaries of normal. The key point to this question is recognising the abnormal examination finding - a ballotable mass
associated with abdominal distension. Whilst an adult with such a 'red flag' symptom/sign would be fast-tracked it is more
appropriate to speak to a paediatrician to determine the best referral pathway, which would probably be clinic review the sam e
week.

Wilms' tumour
Wilms' nephroblastoma is one of the most common childhood malignancies. It typically presents in children under 5 years of age,
with a median age of 3 years old.
Features

abdominal mass (most common presenting feature)


painless haematuria
flank pain

other features: anorexia, fever


unilateral in 95% of cases
metastases are found in 20% of patients (most commonly lung)

Associations

Beckwith-Wiedemann syndrome
as part of WAGR syndrome with Aniridia, Genitourinary malformations, mental Retardation
hemihypertrophy
around one-third of cases are associated with a mutation in the WT1 gene on chromosome 11

Management

nephrectomy
chemotherapy
radiotherapy if advanced disease
prognosis: good, 80% cure rate

Image used on license from PathoPic

Histological features include epithelial tubules, areas of necrosis, immature glomerular structures, stroma with spindle cells and small cell
blastomatous tissues resembling the metanephric blastema

Question 5 of 257
A 6-month-old baby who was born in Bangladesh is brought to surgery. Around one week ago he started with coryzal symptoms.
His mother reports he has not been feeding well for the past two days and has started to vomit today. Her main concern is a
cough which occurs in bouts and is so severe he often turns red. No inspiratory or expiratory noises are noted. Clinical
examination reveals an apyrexial child with a clear chest. What is the most likely diagnosis?
Bronchiolitis

Mycoplasma pneumonia

Pertussis

Afebrile pneumonia syndrome

Tuberculosis

Next question

The inspiratory 'whoop' is uncommon in patients this young.

Whooping cough (pertussis)


Overview

caused by the Gram negative bacterium Bordetella pertussis


incubation period = 10-14 days
infants are routinely immunised at 2, 3, 4 months and 3-5 years. Newborn infants are particularly vulnerable, which is why
the vaccincation campaign for pregnant women was introduced
neither infection nor immunisation results in lifelong protection - hence adolescents and adults may develop whooping
cough despite having had their routine immunisations
around 1,000 cases are reported each year in the UK

Features, 2-3 days of coryza precede onset of:

coughing bouts: usually worse at night and after feeding, may be ended by vomiting & associated central cyanosis
inspiratory whoop: not always present (caused by forced inspiration against a closed glottis)
persistent coughing may cause subconjunctival haemorrhages or even anoxia leading to syncope & seizures
symptoms may last 10-14 weeks* and tend to be more severe in infants
marked lymphocytosis

Diagnosis

per nasal swab culture for Bordetella pertussis - may take several days or weeks to come back
PCR and serology are now increasingly used as their availability becomes more widespread

Management

oral erythromycin to eradicate the organism and reduce spread


has not been shown to alter the course of the illness

Complications

subconjunctival haemorrhage
pneumonia
bronchiectasis
seizures

Vaccination of pregnant women


In 2012 there was an outbreak of whooping cough (pertussis) which resulted in the death of 14 newborn children. As a temporary
measure a vaccination programme was introduced in 2012 for pregnant women. This has successfully reduced the number of
cases of whooping cough (the vaccine is thought to be more than 90% effective in preventing newborns developing whooping
cough). It was however decided in 2014 to extend the whooping cough vaccination programme for pregnant women. This decision
was taken as there was a 'great deal of uncertainty' about the timing of future outbreaks.

Women who are between 28-38 weeks pregnant will be offered the vaccine.
*weeks, not days

Question 6 of 257
At what age would the average child acquire the ability to walk unsupported?
6-7 months

8-9 months

10-11 months

13-15 months

2 years

Next question

Developmental milestones: gross motor


The table below summarises the major gross motor developmental milestones
Age

Milestone
Little or no head lag on being pulled to sit
3 months
Lying on abdomen, good head control
Held sitting, lumbar curve
Lying on abdomen, arms extended
Lying on back, lifts and grasps feet
6 months
Pulls self to sitting
Held sitting, back straight
Rolls front to back
7-8 months Sits without support (Refer at 12 months)
Pulls to standing
9 months
Crawls
Cruises
12 months
Walks with one hand held
13-15 monthsWalks unsupported (Refer at 18 months)
18 months Squats to pick up a toy
Runs
2 years
Walks upstairs and downstairs holding on to rail
Rides a tricycle using pedals
3 years
Walks up stairs without holding on to rail
4 years
Hops on one leg
Notes

the majority of children crawl on all fours before walking but some children 'bottom -shuffle'. This is a normal variant and
runs in families

Question 7 of 257
A male child from a travelling community is diagnosed with measles. Which one of the following complications is he at risk from in
the immediate aftermath of the initial infection?
Arthritis

Pancreatitis

Infertility

Subacute sclerosing panencephalitis

Pneumonia

Next question

Subacute sclerosing panencephalitis is seen but develops 5-10 years following the illness. Pancreatitis and infertility may follow
mumps infection

Measles
Overview

RNA paramyxovirus
spread by droplets
infective from prodrome until 4 days after rash starts
incubation period = 10-14 days

Features

prodrome: irritable, conjunctivitis, fever


Koplik spots (before rash): white spots ('grain of salt') on buccal mucosa
rash: starts behind ears then to whole body, discrete maculopapular rash becoming blotchy & confluent

Image used on license from DermNet NZ

Koplik spots

Complications

encephalitis: typically occurs 1-2 weeks following the onset of the illness)
subacute sclerosing panencephalitis: very rare, may present 5-10 years following the illness
febrile convulsions

giant cell pneumonia


keratoconjunctivitis, corneal ulceration
diarrhoea
increased incidence of appendicitis
myocarditis

Image used on license from DermNet NZ

The rash typically starts behind the ears and then spreads to the whole body

Management of contacts

if a child not immunized against measles comes into contact with measles then MMR should be offered (vaccine-induced
measles antibody develops more rapidly than that following natural infection)
this should be given within 72 hours

Question 8 of 257
Which one of the following is the most common cause of nephrotic syndrome in children?
Minimal change disease

IgA nephropathy

Focal segmental glomerulosclerosis

Chronic pyelonephritis

Infantile microcystic disease

Next question

Minimal change glomerulonephritis nearly always presents as nephrotic syndrome, accounting for 80% of cases in children and
25% in adults. The majority of cases are idiopathic and respond well to steroids

Nephrotic syndrome in children


Nephrotic syndrome is classically defined as a triad of

proteinuria (> 1 g/m^2 per 24 hours)


hypoalbuminaemia (< 25 g/l)
oedema

In children the peak incidence is between 2 and 5 years of age. Around 80% of cases in children are due to a condition called
minimal change glomerulonephritis. The condition generally carries a good prognosis with around 90% of cases responding to
high-dose oral steroids.
Other features include hyperlipidaemia, a hypercoagulable state (due to loss of antithrombin III) and a predisposition to infection
(due to loss of immunoglobulins)

Question 9 of 257
A 5-year-old boy from a travelling community presents to the Emergency Department with breathing difficulties. On examination
he has a temperature of 38.2C, stridor and a toxic looking appearance. A diagnosis of acute epiglottitis is suspected. Which one
of the following organisms is most likely to be responsible?
Epstein Barr Virus

Streptococcus pneumoniae

Neisseria meningitidis

Haemophilus influenzae

Staphylococcus aureus

Next question

Patients from travelling communities may not always receive a full course of immunisation

Acute epiglottitis
Acute epiglottitis is rare but serious infection caused by Haemophilus influenzae type B. Prompt recognition and treatment is
essential as airway obstruction may develop. Epiglottitis was generally considered a disease of childhood but in the UK it is now
more common in adults due to the immunisation programme. The incidence of epiglottitis has decreased since the introduction o f
the Hib vaccine
Features

rapid onset
high temperature, generally unwell
stridor

drooling of saliva

Birth

Question 10 of 257
At what age would the average child start to smile?

2 weeks

6 weeks

3 months

4 months

Next question

Developmental milestones: social behaviour and play


The table below summarises the major social behaviour and play milestones

Age
Milestone
6 weeks Smiles (Refer at 10 weeks)
Laughs
3 months
Enjoys friendly handling
6 monthsNot shy
Shy
9 months
Takes everything to mouth

Feeding
Age
Milestone
May put hand on bottle when being fed
6 months
Drinks from cup + uses spoon, develops over 3 month period 12 -15 months
Competent with spoon, doesn't spill with cup
2 years
Uses spoon and fork
3 years
Uses knife and fork
5 years

Dressing
Age
Milestone
Helps getting dressed/undressed
12-15 months
Takes off shoes, hat but unable to replace
18 months
Puts on hat and shoes
2 years
Can dress and undress independently except for laces and buttons 4 years

Play
Age
Milestone
Plays 'peek-a-boo'
9 months
Waves 'bye-bye'
12 months
Plays 'pat-a-cake'
Plays contentedly alone
18 months
Plays near others, not with them 2 years
Plays with other children
4 years

Question 11 of 257
A 7-year-old boy is brought in to the GP surgery with an exacerbation of asthma. On examination he has a bilateral expiratory
wheeze but there are no signs of respiratory distress. His respiratory rate is 36 / min and PEF around 60% of normal. What is the
most appropriate action with regards to steroid therapy?
Oral prednisolone for 3 days

Admit for intravenous steroids

Give a stat dose of oral dexamethasone

Double his usual beclometasone dose

Do not give steroids

Next question

Asthma in children: management of acute attacks


Children with severe or life threatening asthma should be transferred immediately to hospital.
Children between 2 and 5 years of age
Moderate attack

Severe attack

Life-threatening attack
SpO2 <92%
SpO2 < 92%
Silent chest
Too breathless to talk or feed
SpO2 > 92%
Poor respiratory effort
Heart rate > 140/min
No clinical features of severe asthma
Agitation
Respiratory rate > 40/min
Altered consciousness
Use of accessory neck muscles
Cyanosis
Children greater than 5 years of age
Attempt to measure PEF in all children aged > 5 years.
Moderate attack

Severe attack
Life-threatening attack
SpO2 < 92%
SpO2 < 92%
SpO2 > 92%
PEF 33-50% best or predicted
PEF < 33% best or predicted
PEF > 50% best or predicted Can't complete sentences in one breath or too breathless to talk or feed Silent chest
No clinical features of
Heart rate > 125/min
Poor respiratory effort
severe asthma
Respiratory rate > 30/min
Altered consciousness
Use of accessory neck muscles
Cyanosis
For children with mild to moderate acute asthma:
Bronchodilator therapy

give a beta-2 agonist via a spacer (for a child < 3 years use a close-fitting mask)
give 1 puff every 15-30 seconds up to a maximum of 10 puffs; repeat dose after 10-20 minutes if necessary
if symptoms are not controlled repeat beta-2 agonist and refer to hospital

Steroid therapy

should be given to all children with an asthma exacerbation


treatment should be given for 3-5 days

Usual prednisolone dose


Age
Dose as per BTSDose as per cBNF
2 - 5 years20 mg od
1-2 mg/kg od (max 40mg)
> 5 years 30 - 40 mg od
1-2 mg/kg od (max 40mg)

Question 12 of 257

A 9-year-old boy is brought to surgery with recurrent headaches. What is the most common cause of headaches in children?
Migraine

Depression

Refractive errors

Tension-type headache

Cluster headache

Next question
Migraine is the most common cause of headache in children

Headache in children
Some of the following is based on an excellent review article on the Great Ormond Street Hospital website. Please see the link for
more details.
Epidemiology

up to 50 per cent of 7-year-olds and up to 80 per cent of 15-year-old have experienced at least one headache
equally as common in boys/girls until puberty then strong (3:1) female preponderance

Migraine
Migraine without aura is the most common cause of primary headache in children. The International Headache Society (IHS) have
produced criteria for paediatric migraine without aura:
A>= 5 attacks fulfilling features B to D
B Headache attack lasting 4-72 hours
Headache has at least two of the following four features:

bilateral or unilateral (frontal/temporal) location


pulsating quality
moderate to severe intensity
aggravated by routine physical activity

At least one of the following accompanies headache:

nausea and/or vomiting


photophobia and phonophobia (may be inferred from behaviour)

Acute management

ibuprofen is thought to be more effective than paracetamol for paediatric migraine


the use of triptans in children should only be initiated by a specialist
sumatriptan nasal spay (licensed) is the only triptan that has proven efficacy but it is poorly tolerated by young people
who don't like the taste in the back of the throat
orodispersible zolmitriptan (unlicensed) is widely used in children aged 8-years and older

side-effects of triptans include tingling, heat and heaviness/pressure sensations

Prophylaxis

the evidence base is limited and no clear consensus guidelines exist


the GOSH website states: 'in practice, pizotifen and propranolol should be used as first line preventatives in children.
Second line preventatives are valproate, topiramate and amitryptiline'

Tension-type headache (TTH)


Tension-type headache is the second most common cause of headache in children. The IHS diagnostic criteria for TTH in children
is reproduced below:
AAt least 10 previous headache episodes fulfilling features B to D
B Headache lasting from 30 minutes to 7 days
At least two of the following pain characteristics:

pressing/tightening (non/pulsating) quality


mild or moderate intensity (may inhibit but does not prohibit activity)
bilateral location
no aggravation by routine physical activity

Both of the following:

no nausea or vomiting
photophobia and phonophobia, or one, but not the other is present

Question 13 of 257
The parents of a 14-month-old girl present to their GP. They have noticed that in some photos there is no 'red eye' on the left hand
side. When you examine the girl you notice an esotropic strabismus and a loss of the red-reflex in the left eye. There is a family
history of a grandparent having an enucleation as a child. What is the most likely diagnosis?
Congenital hypertrophy of the retinal pigment epithelium

Uveal malignant melanoma

Neuroblastoma

Retinoblastoma

Congenital cataract

Next question

A congenital cataract may cause a loss of the red-reflex but is likely to have been detected at birth or during the routine babychecks. It would also not explain the family history of enucleation.

Retinoblastoma
Retinoblastoma is a the most common ocular malignancy found in children. The average age of diagnosis is 18 months.
Pathophysiology

caused by a loss of function of the retinoblastoma tumour suppressor gene on chromosome 13

around 10% of cases are hereditary

Possible features

absence of red-reflex, repalced by a white pupil (leukocoria) - the most common presenting symptom
strabismus
visual problems

Management

enucleation is not the only option


depending on how advanced the tumour is other options include external beam radiation therapy, chemotherapy and
photocoagulation

Prognosis

excellent, with > 90% surviving into adulthood

Question 14 of 257
Which one of the following is an example of a purely secondary accident prevention strategy?
Speed limits

Teaching road safety

Window safety catches

Stair guards

Cycling helmets

Next question

Speed limits are an example of both a primary and secondary accident prevention strategy, whereas cycling helmets are purely a
secondary strategy.

Accidents in children
Around 15-20% of children attend Emergency Departments in the course of a year due to an accident. Accidents account for a
third of all childhood deaths and are the single most common cause of death in children aged between 1 - 15 years of age.
Key points

road traffic accidents are the most common cause of fatal accidents
boys and children from lower social classes are more likely to have an accident

Accident prevention
Accident prevention can be divided up into primary (preventing the accident from happening), secondary (prevent injury from t he
accident) and tertiary (limit the impact of the injury) prevention strategies
The table below gives examples of accident prevention strategies

Primary prevention Secondary prevention Tertiary prevention


Stair guards
Wearing seat belts
Speed limits*
Cycling helmets
Teaching parents first aid
Teaching road safety Smoke alarms
Window safety catches Laminated safety glass
*some strategies such as reducing driving speed may have a role in both primary and secondary accident prevention

Question 15 of 257
A mother is concerned about the risk of her son developing influenza. Her son is fit and otherwise well. Following NHS
immunisation guidance, at what age should the child first be offered the influenza vaccine?
3 months

4 months

12-13 months

2-3 years

65 years

Next question

Influenza vaccination
Seasonal influenza still accounts for a significant morbidity and mortality in the UK each winter, with the influenza season typically
starting in the middle of November. This may vary year from year so it is recommended that vaccination occurs between
September and early November. There are three types of influenza virus; A, B and C. Types A and B account for the majority of
clinical disease.
Prior to 2013 flu vaccination was only offered to the elderly and at risk groups.
Remember that the type of vaccine given routinely to children and the one given to the elderly and at risk groups is different (live
vs. inactivated) - this explains the different contraindications
Children
A new NHS influenza vaccination programme for children was announced in 2013. There are three key things to remember about
the children's vaccine:

it is given intranasally
the first dose is given at 2-3 years, then annually after that
it is a live vaccine (cf. injectable vaccine below)

Some other points

children who were traditionally offered the flu vaccine (e.g. asthmatics) will now be given intranasal vaccine unless this is
inappropriate, for example if they are immunosuppressed. In this situation the inactivated, injectable vaccine should be
given
only children aged 2-9 years who have not received an influenza vaccine before need 2 doses
it is more effective than the injectable vaccine

Contraindications

immunocompromised
aged < 2 years
current febrile illness or blocked nose/rhinorrhoea
current wheeze (e.g. ongoing viral-induced wheeze/asthma) or history of severe asthma (BTS step 4)
egg allergy
pregnancy/breastfeeding
if the child is taking aspirin (e.g. for Kawasaki disease) due to a risk of Reye's syndrome

Side-effects

blocked-nose/rhinorrhoea
headache
anorexia

Adults and at-risk groups


Current vaccines are trivalent and consist of two subtypes of influenza A and one subtype of influenza B.
The Department of Health recommends annual influenza vaccination for people older than 65 years and those older than 6
months if they have:

chronic respiratory disease (including asthmatics who use inhaled steroids)


chronic heart disease (heart failure, ischaemic heart disease, including hypertension if associated with cardiac
complications)
chronic kidney disease
chronic liver disease: cirrhosis, biliary atresia, chronic hepatitis
chronic neurological disease: (e.g. Stroke/TIAs)
diabetes mellitus (including diet controlled)
immunosuppression due to disease or treatment (e.g. HIV)
asplenia or splenic dysfunction
pregnant women

Other at risk individuals include:

health and social care staff directly involved in patient care (e.g. NHS staff)
those living in long-stay residential care homes
carers of the elderly or disabled person whose welfare may be at risk if the carer becomes ill (at the GP's discretion)

The influenza vaccine

it is an inactivated vaccine, so cannot cause influenza. A minority of patients however develop fever and malaise which
may last 1-2 days
should be stored between +2 and +8C and shielded from light
contraindications include hypersensitivity to egg protein.
in adults the vaccination is around 75% effective, although this figure decreases in the elderly
it takes around 10-14 days after immunisation before antibody levels are at protective levels

Question 16 of 257
Which one of the following statements regarding croup is true?
Symptoms are typically worse during the day

Most common in children under the age of 6 months

Throat examination is important prior to making the diagnosis

Most commonly caused by parainfluenza viruses

More common in spring

Next question

Throat examination should be avoided as it may precipitate airway obstruction.

Croup
Croup is a form of upper respiratory tract infection seen in infants and toddlers. It is characterised by stridor which is caused by a
combination of laryngeal oedema and secretions. Parainfluenza viruses account for the majority of cases.
Epidemiology

peak incidence at 6 months - 3 years


more common in autumn

Features

stridor
barking cough (worse at night)
fever
coryzal symptoms

Clinical Knowledge Summaries (CKS) suggest using the following criteria to grade the severity*:
Mild

Moderate

Severe
Frequent barking cough
Frequent barking cough
Occasional barking cough
Prominent inspiratory (and occasionally, expiratory)
Easily audible stridor at rest
No audible stridor at rest
stridor at rest
Suprasternal and sternal wall retraction
No or mild suprasternal and/or
Marked sternal wall retractions
at rest
intercostal recession
Significant distress and agitation, or lethargy or
No or little distress or agitation
The child is happy and is prepared to
restlessness (a sign of hypoxaemia)
The child can be placated and is
eat, drink, and play
Tachycardia occurs with more severe obstructive
interested in its surroundings
symptoms and hypoxaemia
CKS suggest admitting any child with moderate or severe croup. Other features which should prompt admission include:

< 6 months of age


known upper airway abnormalities (e.g. Laryngomalacia, Down's syndrome)
uncertainty about diagnosis (important differentials include acute epiglottitis, bacterial tracheitis, peritonsillar abscess and
foreign body inhalation)

Management

CKS recommend giving a single dose of oral dexamethasone (0.15mg/kg) to all children regardless of severity
prednisolone is an alternative if dexamethasone is not available

Emergency treatment

high-flow oxygen
nebulised adrenaline

*these in turn are based partly on the Alberta Medical Association (2008) Guideline for the diagnosis and management of croup.

Croup - Summary

Croup (laryngotracheitis) is a common childhood disease that is usually caused by a


virus. It is characterized by the sudden onset of a seal-like barking cough usually
accompanied by stridor (predominantly inspiratory), hoarse voice, and respiratory
distress due to upper airways obstruction. Symptoms are usually worse at night. There
may be a fever up to 40C.
There is often a preceding 14-day history of a non-specific cough, rhinorrhoea, and
fever.
Croup most commonly affects children 6 months to 3 years of age, with a peak
incidence during the second year of life.
Mild:
o Occasional barking cough and no audible stridor at rest.
o No or mild suprasternal and/or intercostal recession.
o The child is happy and is prepared to eat, drink, and play.
Moderate:
o Frequent barking cough and easily audible stridor at rest.
o Suprasternal and sternal wall retraction at rest.
o No or little distress or agitation.
o The child can be placated and is interested in its surroundings.
Severe:
o Frequent barking cough with prominent inspiratory (and occasionally, expiratory)
stridor at rest.
o Marked sternal wall retractions.
o Significant distress and agitation, or lethargy or restlessness (a sign of hypoxaemia).
o Tachycardia occurs with more severe obstructive symptoms and hypoxaemia.
Impending respiratory failure may develop regardless of the severity of the symptoms:
o Change in mental state, such as lethargy and listlessness or decreased level of
consciousness.
o Dusky appearance.
o Tachycardia.
In children with impending respiratory failure, breathing may be laboured, a barking
cough may not be prominent, stridor at rest may be hard to hear, and sternal wall
retractions may not be marked. A child who appears to be deteriorating but whose
stridor appears to be improving has worsening airways obstruction and is at high risk of
complete airway occlusion.
A child should be immediately admitted when presenting with:
o Moderate or severe croup, or impending respiratory failure.
o A serious disorder such as epiglottitis, bacterial tracheitis, peritonsillar abscess,
retropharyngeal abscess, foreign body or angioneurotic oedema.
Mild croup can usually be managed at home. However, reasons for admission include: a
history of severe obstruction, age less than 6 months, immunocompromised, an

inadequate fluid intake, an uncertain diagnosis, a poor response to initial treatment,


parental anxiety, no transport, or living far from the hospital.
All children with mild, moderate, or severe croup should receive a single dose of oral
dexamethasone (0.15 mg per kg body weight).
Oral prednisolone (12 mg per kg body weight) is an alternative if dexamethasone is
not available. A second dose should be considered if residual symptoms of stridor are
still present the following day.
Croup is self limiting and symptoms usually resolve within 48 hours. The use of
paracetamol or ibuprofen to control fever and pain should be advised, and
arrangements made to review the child within a few hours. Parents should be advised
to seek urgent medical advice if there is any deterioration.

Question 17 of 257
A newborn baby is transferred to the neonatal intensive care unit shortly after birth due to respiratory distress. An x-ray
taken on arrival is shown below:

Image used on license from Radiopaedia

What is the diagnosis?


Bronchopulmonary dysplasia

Respiratory distress syndrome

Left-sided neonatal bronchiectasis

Congenital diaphragmatic hernia

Left pneumothorax

Next question

Bowel loops can be seen in the left side of the thoracic cavity.

Congenital diaphramatic hernia


Congenital diaphramatic hernia (CDH) occurs in around 1 in 2,000 newborns. It is characterised by the herniation of
abdominal viscera into the chest cavity due to incomplete formation of the diaphragm. This can result in pulmonary
hypoplasia and hypertension which causes respiratory distress shortly after birth.
The most common type of CDH is a left-sided posterolateral Bochdalek hernia which accounts for around 85% of cases.
Only around 50% of newborns with CDH survive despite modern medical intervention.

Question 18 of 257
Which one of the following is responsible for causing scarlet fever?
Group A haemolytic streptococci

Staphylococcus aureus

Human herpesvirus type 6

Parvovirus B19

Coxsackie A16

Next question

Scarlet fever
Scarlet fever is a reaction to erythrogenic toxins produced by Group A haemolytic streptococci (usually Streptococcus pyogenes).
It is more common in children aged 2 - 6 years with the peak incidence being at 4 years.
Scarlet fever has an incubation period of 2-4 days and typically presents with:

fever
malaise
tonsillitis
'strawberry' tongue
rash - fine punctate erythema ('pinhead') which generally appears first on the torso and spares the face although children
often have a flushed appearance with perioral pallor. The rash often has a rough 'sandpaper' texture. Desquamination
occurs later in the course of the illness, particularly around the fingers and toes

Image used on license from DermNet NZ

Diagnosis

a throat swab is normally taken but antibiotic treatment should be commenced immediately, rather than waiting for the
results

Management

oral penicillin V
patients who have a penicillin allergy should be given azithromycin
children can return to school 24 hours after commencing antibiotics
scarlet fever is a notifiable disease

Complications

otitis media: the most common complication


rheumatic fever: typically occurs 20 days after infection
acute glomerulonephritis: typically occurs 10 days after infection

Image sourced from Wikipedia

Question 19 of 257
Which of the following conditions is inherited in an autosomal recessive fashion?
Homocystinuria

Huntington's disease

Hereditary non-polyposis colorectal carcinoma

Hereditary spherocytosis

Hypokalaemic periodic paralysis

Next question
Autosomal recessive conditions are 'metabolic' - exceptions: inherited ataxias
Autosomal dominant conditions are 'structural' - exceptions: Gilbert's, hyperlipidaemia type II

Huntington's disease, hereditary non-polyposis colorectal carcinoma, hereditary spherocytosis and hypokalaemic periodic
paralysis are all inherited in an autosomal dominant fashion

Autosomal recessive conditions


Autosomal recessive conditions are often thought to be 'metabolic' as opposed to autosomal dominant conditions being
'structural', notable exceptions:

some 'metabolic' conditions such as Hunter's and G6PD are X-linked recessive whilst others such as hyperlipidemia type
II and hypokalemic periodic paralysis are autosomal dominant
some 'structural' conditions such as ataxia telangiectasia and Friedreich's ataxia are autosomal recessive

The following conditions are autosomal recessive:

Albinism
Ataxia telangiectasia
Congenital adrenal hyperplasia
Cystic fibrosis
Cystinuria
Familial Mediterranean Fever
Fanconi anaemia
Friedreich's ataxia
Gilbert's syndrome*
Glycogen storage disease
Haemochromatosis
Homocystinuria
Lipid storage disease: Tay-Sach's, Gaucher, Niemann-Pick
Mucopolysaccharidoses: Hurler's
PKU
Sickle cell anaemia
Thalassaemias
Wilson's disease

*this is still a matter of debate and many textbooks will list Gilbert's as autosomal dominant

Question 20 of 257

A mother brings her 6-year-old son into surgery. He is not doing well at school and she worries he may be 'hyperactive'. Which one of the
following features is not consistent with a diagnosis of Attention Deficit Hyperactivity Disorder (ADHD)?
Poor concentration

Impulsiveness

Uncontrolled activity

Repetitive behaviour

Extreme restlessness

Next question

Repetitive behaviour is not part of the diagnostic criteria for ADHD and may suggest a disorder on the autistic spectrum.

Attention Deficit Hyperactivity Disorder


Attention Deficit Hyperactivity Disorder (ADHD) is characterised by

extreme restlessness
poor concentration
uncontrolled activity
impulsiveness

ADHD is diagnosed in about 5% of American children, In the UK, where the term hyperkinetic syndrome is preferred, only 0.1% of children
are diagnosed with the condition. The male:female ratio is 5:1
Management

specialist assessment is required in all cases


unless a food diary has shown a link between diet and behaviour there is no basis for recommending the avoidance of artificial
colourings or the use of fatty acid supplements
methylphenidate (Ritalin) - side-effects include abdominal pain, nausea, dyspepsia. Growth is not usually affected but it is
advised to monitor growth during treatment every 6 months. The BNF also advises monitoring for psychiatric disorders and
checking blood pressure/pulse every 6 months
atomoxetine

1 Guidance

1.1 Prerequisites of treatment and care for all people with ADHD
1.2 Identification, pre-diagnostic intervention in the community and referral to secondary
services
1.3 Diagnosis of ADHD
1.4 Post-diagnostic advice
1.5 Treatment for children and young people
1.6 Transition to adult services
1.7 Treatment of adults with ADHD
1.8 How to use drugs for the treatment of ADHD

1 Guidance
March 2013: In treatment for children and young people footnote 6 has been
updated. Recommendations 1.5.1.7 and 1.5.1.9 have been removed and are
replaced by recommendations 1.5.2 and 1.5.4 in 'Antisocial behaviour and
conduct disorders in children and young people' (NICE clinical guideline 158).
Recommendation 1.5.1.10 has been removed.
The following guidance is based on the best available evidence. The full
guideline gives details of the methods and the evidence used to develop the
guidance.
1.1 Prerequisites of treatment and care for all people with ADHD

People with ADHD require integrated care that addresses a wide range of
personal, social, educational and occupational needs. Care should be provided
by adequately trained healthcare and education professionals.
1.1.1 Organisation and planning of services

1.1.1.1 People with ADHD would benefit from improved organisation of care and
better integration of paediatric, child and adolescent mental health services
(CAMHS) and adult mental health services

1.1.1.2 Mental health trusts, and children's trusts that provide mental health/child
development services, should form multidisciplinary specialist ADHD teams
and/or clinics for children and young people and separate teams and/or clinics for
adults. These teams and clinics should have expertise in the diagnosis and
management of ADHD, and should:

provide diagnostic, treatment and consultation services for people with


ADHD who have complex needs, or where general psychiatric services are
in doubt about the diagnosis and/or management of ADHD

put in place systems of communication and protocols for information sharing


among paediatric, child and adolescent, forensic, and adult mental health
services for people with ADHD, including arrangements for transition
between child and adult services

produce local protocols for shared care arrangements with primary care
providers, and ensure that clear lines of communication between primary
and secondary care are maintained

ensure age-appropriate psychological services are available for children,


young people and adults with ADHD, and for parents or carers.
The size and time commitment of these teams should depend on local
circumstances (for example, the size of the trust, the population covered and
the estimated referral rate for people with ADHD).

1.1.1.3 Every locality should develop a multi-agency group, with representatives


from multidisciplinary specialist ADHD teams, paediatrics, mental health and
learning disability trusts, forensic services, child and adolescent mental health
services (CAMHS), the Children and Young People's Directorate (CYPD)
(including services for education and social services), parent support groups and
others with a significant local involvement in ADHD services. The group should:

oversee the implementation of this guideline

start and coordinate local training initiatives, including the provision of


training and information for teachers about the characteristics of ADHD and
its basic behavioural management

oversee the development and coordination of parent-training/education


programmes

consider compiling a comprehensive directory of information and services for


ADHD including advice on how to contact relevant services and assist in the
development of specialist teams.

1.1.2 Information, consent, the law and support for people with ADHD and their carers

Many people with ADHD, and their parents or carers, experience stigma and
other difficulties because of the symptoms and impairment associated with
ADHD and current practice within healthcare and education. The following
recommendations have been developed based on the experiences of people with
ADHD and their families.

1.1.2.1 Healthcare professionals should develop a trusting relationship with


people with ADHD and their families or carers by:

respecting the person and their family's knowledge and experience of ADHD

being sensitive to stigma in relation to mental illness.

1.1.2.2 Healthcare professionals should provide people with ADHD and their
families or carers with relevant, age-appropriate information (including written
information) about ADHD at every stage of their care. The information should
cover diagnosis and assessment, support and self-help, psychological treatment,
and the use and possible side effects of drug treatment.
1.1.2.3 When assessing a child or young person with ADHD, and throughout
their care, healthcare professionals should:

allow the child or young person to give their own account of how they feel,
and record this in the notes

involve the child or young person and the family or carer in treatment
decisions

take into account expectations of treatment, so that informed consent can be


obtained from the child's parent or carer or the young person before
treatment is started.

1.1.2.4 Healthcare professionals working with children and young people with
ADHD should be:

familiar with local and national guidelines on confidentiality and the rights of
the child

able to assess the young person's understanding of issues related to ADHD


and its treatment (including Gillick competence)

familiar with parental consent and responsibilities, child protection issues,


the Mental Health Act (2007) and the Children Act (1989).

1.1.2.5 Healthcare professionals should work with children and young people
with ADHD and their parents or carers to anticipate major life changes (such as
puberty, starting or changing schools, the birth of a sibling) and make appropriate
arrangements for adequate personal and social support during times of
increased need. The need for psychological treatment at these times should be
considered.
1.1.2.6 Adults with ADHD should be given written information about local and
national support groups and voluntary organisations.
1.1.2.7 Healthcare professionals should ask families or carers about the impact
of ADHD on themselves and other family members, and discuss any concerns
they may have. Healthcare professionals should:

offer family members or carers an assessment of their personal, social and


mental health needs

encourage participation in self-help and support groups where appropriate

offer general advice to parents and carers about positive parent and carer
child contact, clear and appropriate rules about behaviour, and the
importance of structure in the child or young person's day

explain that parent-training/education programmes do not necessarily imply


bad parenting, and that their aim is to optimise parenting skills to meet the
above-average parenting needs of children and young people with ADHD.

1.1.3 Training

Healthcare and education professionals require training to better address the


needs of people with ADHD.
1.1.3.1 Trusts should ensure that specialist ADHD teams for children, young
people and adults jointly develop age-appropriate training programmes for the
diagnosis and management of ADHD for mental health, paediatric, social care,
education, forensic and primary care providers and other professionals who have
contact with people with ADHD.
1.1.3.2 Child and adult psychiatrists, paediatricians, and other child and adult
mental health professionals (including those working in forensic services) should
undertake training so that they are able to diagnose ADHD and provide treatment
and management in accordance with this guideline.
1.1.3.3 The Department for Children, Schools and Families should consider
providing more education to trainee teachers about ADHD by working with the
Training and Development Agency for Schools (TDA) and relevant health service
organisations to produce training programmes and guidance for supporting
children with ADHD.
Care pathway for the treatment and care of people with ADHD
The recommendations in sections 1.21.7 form a care pathway that sets out how
children, young people and adults should receive help, treatment and care from
different services, from the community (including primary care and education),
through to secondary and tertiary services. Most of the recommendations in
sections 1.21.5 describe the approach for children but some of these also apply
to adults. The pathway also covers transition between child and adult services
(section 1.6) and specific treatment for adults (section 1.7), including those who
were first diagnosed with ADHD in adulthood.
Specific recommendations on the use of drugs, monitoring side effects,
improving adherence and discontinuing drug treatment are in section 1.8.
1.2 Identification, pre-diagnostic intervention in the community and referral to
secondary services

Children and young people with behavioural problems suggestive of ADHD can
be referred by their school or primary care practitioner for parenttraining/education programmes without a formal diagnosis of ADHD. The
diagnosis of ADHD in children, young people and adults should take place in
secondary care.

1.2.1 Identification and referral in children and young people with ADHD

1.2.1.1 Universal screening for ADHD should not be undertaken in nursery,


primary and secondary schools.
1.2.1.2 When a child or young person with disordered conduct and suspected
ADHD is referred to a school's special educational needs coordinator (SENCO),
the SENCO, in addition to helping the child with their behaviour, should inform
the parents about local parent-training/education programmes.
1.2.1.3 Referral from the community to secondary care may involve health,
education and social care professionals (for example, GPs, paediatricians,
educational psychologists, SENCOs, social workers) and care pathways can vary
locally. The person making the referral to secondary care should inform the child
or young person's GP.
1.2.1.4 When a child or young person presents in primary care with behavioural
and/or attention problems suggestive of ADHD, primary care practitioners should
determine the severity of the problems, how these affect the child or young
person and the parents or carers and the extent to which they pervade different
domains and settings.
1.2.1.5 If the child or young person's behavioural and/or attention problems
suggestive of ADHD are having an adverse impact on their development or
family life, healthcare professionals should consider:

a period of watchful waiting of up to 10 weeks

offering parents or carers a referral to a parent-training/education


programme (this should not wait for a formal diagnosis of ADHD).
If the behavioural and/or attention problems persist with at least moderate
impairment, the child or young person should be referred to secondary care
(that is, a child psychiatrist, paediatrician, or specialist ADHD CAMHS) for
assessment.

1.2.1.6 If the child or young person's behavioural and/or attention problems are
associated with severe impairment, referral should be made directly to secondary
care (that is, a child psychiatrist, paediatrician, or specialist ADHD CAMHS) for
assessment.
1.2.1.7 Group-based parent-training/education programmes are recommended in
the management of children with conduct disorders [4].
1.2.1.8 Primary care practitioners should not make the initial diagnosis or start
drug treatment in children or young people with suspected ADHD.
1.2.1.9 A child or young person who is currently treated in primary care with
methylphenidate, atomoxetine, dexamfetamine, or any other psychotropic drug
for a presumptive diagnosis of ADHD, but has not yet been assessed by a
specialist in ADHD in secondary care, should be referred for assessment to a
child psychiatrist, paediatrician, or specialist ADHD CAMHS as a matter of
clinical priority.

1.2.2 Identification and referral in adults with ADHD

1.2.2.1 Adults presenting with symptoms of ADHD in primary care or general


adult psychiatric services, who do not have a childhood diagnosis of ADHD,
should be referred for assessment by a mental health specialist trained in the
diagnosis and treatment of ADHD, where there is evidence of typical
manifestations of ADHD (hyperactivity/impulsivity and/or inattention) that:

began during childhood and have persisted throughout life

are not explained by other psychiatric diagnoses (although there may be


other coexisting psychiatric conditions)

have resulted in or are associated with moderate or severe psychological,


social and/or educational or occupational impairment.

1.2.2.2 Adults who have previously been treated for ADHD as children or young
people and present with symptoms suggestive of continuing ADHD should be
referred to general adult psychiatric services for assessment. The symptoms
should be associated with at least moderate or severe psychological and/or
social or educational or occupational impairment.
1.3 Diagnosis of ADHD

ADHD is a valid clinical disorder that can be distinguished from coexisting


conditions (although it is most commonly comorbid) and the normal spectrum.
ADHD differs from the normal spectrum because there are high levels of
hyperactivity/impulsivity and/or inattention that result in significant psychological,
social and/or educational or occupational impairment that occurs across multiple
domains and settings and persists over time.
1.3.1.1 A diagnosis of ADHD should only be made by a specialist psychiatrist,
paediatrician or other appropriately qualified healthcare professional with training
and expertise in the diagnosis of ADHD, on the basis of:

a full clinical and psychosocial assessment of the person; this should include
discussion about behaviour and symptoms in the different domains and
settings of the person's everyday life, and

a full developmental and psychiatric history, and

observer reports and assessment of the person's mental state.

1.3.1.2 A diagnosis of ADHD should not be made solely on the basis of rating
scale or observational data. However rating scales such as the Conners' rating
scales and the Strengths and Difficulties questionnaire are valuable adjuncts,
and observations (for example, at school) are useful when there is doubt about
symptoms.
1.3.1.3 For a diagnosis of ADHD, symptoms of hyperactivity/impulsivity and/or
inattention should:

meet the diagnostic criteria in DSM-IV or ICD-10 (hyperkinetic


disorder),[5]and

be associated with at least moderate psychological, social and/or


educational or occupational impairment based on interview and/or direct
observation in multiple settings, and

be pervasive, occurring in two or more important settings including social,


familial, educational and/or occupational settings.
As part of the diagnostic process, include an assessment of the person's
needs, coexisting conditions, social, familial and educational or occupational
circumstances and physical health. For children and young people, there
should also be an assessment of their parents' or carers' mental health.

1.3.1.4 ADHD should be considered in all age groups, with symptom criteria
adjusted for age-appropriate changes in behaviour.
1.3.1.5 In determining the clinical significance of impairment resulting from the
symptoms of ADHD in children and young people, their views should be taken
into account wherever possible.
1.4 Post-diagnostic advice

After diagnosis people with ADHD and their parents or carers may benefit from
advice about diet, behaviour and general care.
1.4.1 General advice

1.4.1.1 Following a diagnosis of ADHD, healthcare professionals should consider


providing all parents or carers of all children and young people with ADHD selfinstruction manuals, and other materials such as videos, based on positive
parenting and behavioural techniques.
1.4.2 Dietary advice

1.4.2.1 Healthcare professionals should stress the value of a balanced diet, good
nutrition and regular exercise for children, young people and adults with ADHD.
1.4.2.2 The elimination of artificial colouring and additives from the diet is not
recommended as a generally applicable treatment for children and young people
with ADHD.
1.4.2.3 Clinical assessment of ADHD in children and young people should
include asking about foods or drinks that appear to influence their hyperactive
behaviour. If there is a clear link, healthcare professionals should advise parents
or carers to keep a diary of food and drinks taken and ADHD behaviour. If the
diary supports a relationship between specific foods and drinks and behaviour,
then referral to a dietitian should be offered. Further management (for example,
specific dietary elimination) should be jointly undertaken by the dietitian, mental
health specialist or paediatrician, and the parent or carer and child or young
person.
1.4.2.4 Dietary fatty acid supplementation is not recommended for the treatment
of ADHD in children and young people.

1.5 Treatment for children and young people


1.5.1 Treatment for pre-school children

Parent-training/education programmes are the first-line treatment for parents or


carers of pre-school children. These programmes are the same as those
recommended for the parents or carers of other children with conduct disorder. If
more help is needed the child can be referred to a tertiary service.
1.5.1.1 Drug treatment is not recommended for pre-school children with ADHD.
1.5.1.2 Following a diagnosis of ADHD in a child of pre-school age, healthcare
professionals should, with the parents' or carers' consent, contact the child's
nursery or pre-school teacher to explain:

the diagnosis and severity of symptoms and impairment

the care plan

any special educational needs.

1.5.1.3 Healthcare professionals should offer parents or carers of pre-school


children with ADHD a referral to a parent-training/education programme as the
first-line treatment if the parents or carers have not already attended such a
programme or the programme has had a limited effect.
1.5.1.4 Group-based parent-training/education programmes, developed for the
treatment and management of children with conduct disorders [6], should be fully
accessible to parents or carers of children with ADHD whether or not the child
also has a formal diagnosis of conduct disorder.
1.5.1.5 Individual-based parent-training/education programmes[6] are
recommended in the management of children with ADHD when:

a group programme is not possible because of low participant numbers

there are particular difficulties for families in attending group sessions (for
example, because of disability, needs related to diversity such as language
differences, parental ill-health, problems with transport, or where other
factors suggest poor prospects for therapeutic engagement)

a family's needs are too complex to be met by group-based parenttraining/education programmes.

1.5.1.6 When individual-based parent-training/education programmes for preschool children with ADHD are undertaken, the skills training stages should
involve both the parents or carers and the child.
1.5.1.7 This recommendation has been replaced by recommendations 1.5.2 and
1.5.4 in 'Antisocial behaviour and conduct disorders in children and young
people' (NICE clinical guideline 158).
1.5.1.8 Consideration should be given to involving both of the parents or all
carers of children or young people with ADHD in parent-training/education
programmes wherever this is feasible.

1.5.1.9 This recommendation has been replaced by recommendations 1.5.2 and


1.5.4 in 'Antisocial behaviour and conduct disorders in children and young
people' (NICE clinical guideline 158).
1.5.1.10 This recommendation has been deleted.
1.5.1.11 If overall treatment, including parent-training/education programmes,
has been effective in managing ADHD symptoms and any associated impairment
in pre-school children, before considering discharge from secondary care
healthcare professionals should:

review the child, with their parents or carers and siblings, for any residual
coexisting conditions and develop a treatment plan for these if needed

monitor for the recurrence of ADHD symptoms and any associated


impairment that may occur after the child starts school.

1.5.1.12 If overall treatment, including parent-training/education programmes,


has not been effective in managing ADHD symptoms and any associated
impairment in pre-school children, healthcare professionals should consider
referral to tertiary services for further care.
1.5.2 Treatment for school-age children and young people with ADHD and moderate impairment

Group-based parent-training/education programmes are usually the first-line


treatment for parents and carers of children and young people of school age with
ADHD and moderate impairment. This may also include group psychological
treatment (cognitive behavioural therapy [CBT] and/or social skills training) for
the younger child. For older age groups, individual psychological treatment may
be more acceptable if group behavioural or psychological approaches have not
been effective, or have been refused. See section 1.5.1 for recommendations on
conducting parent-training/education programmes that also apply to school-age
children with ADHD. Drug treatment may be tried next for those children and
young people with ADHD and moderate levels of impairment.
1.5.2.1 Drug treatment is not indicated as the first-line treatment for all schoolage children and young people with ADHD. It should be reserved for those with
severe symptoms and impairment or for those with moderate levels of
impairment who have refused non-drug interventions, or whose symptoms have
not responded sufficiently to parent-training/education programmes or group
psychological treatment.
1.5.2.2 Following a diagnosis of ADHD in a school-age child or young person
healthcare professionals should, with the parents' or carers' consent, contact the
child or young person's teacher to explain:

the diagnosis and severity of symptoms and impairment

the care plan

any special educational needs.

1.5.2.3 Teachers who have received training about ADHD and its management
should provide behavioural interventions in the classroom to help children and
young people with ADHD.
1.5.2.4 If the child or young person with ADHD has moderate levels of
impairment, the parents or carers should be offered referral to a group parenttraining/education programme, either on its own or together with a group
treatment programme (CBT and/or social skills training) for the child or young
person.
1.5.2.5 When using group treatment (CBT and/or social skills training) for the
child or young person in conjunction with a parent-training/education programme,
particular emphasis should be given to targeting a range of areas, including
social skills with peers, problem solving, self-control, listening skills and dealing
with and expressing feelings. Active learning strategies should be used, and
rewards given for achieving key elements of learning.
1.5.2.6 For older adolescents with ADHD and moderate impairment, individual
psychological interventions (such as CBT or social skills training) may be
considered as they may be more effective and acceptable than group parenttraining/education programmes or group CBT and/or social skills training.
1.5.2.7 For children and young people (including older age groups) with ADHD
and a learning disability, a parent-training/education programme should be
offered on either a group or individual basis, whichever is preferred following
discussion with the parents or carers and the child or young person.
1.5.2.8 When parents or carers of children or young people with ADHD
undertake parent-training/education programmes, the professional delivering the
sessions should consider contacting the school and providing the child or young
person's teacher with written information on the areas of behavioural
management covered in these sessions. This should only be done with parental
consent.
1.5.2.9 Following successful treatment with a parent-training/education
programme and before considering discharge from secondary care, the child or
young person should be reviewed, with their parents or carers and siblings, for
any residual problems such as anxiety, aggression or learning difficulties.
Treatment plans should be developed for any coexisting conditions.
1.5.2.10 Following treatment with a parent-training/education programme,
children and young people with ADHD and persisting significant impairment
should be offered drug treatment.
1.5.3 Treatment for school-age children and young people with severe ADHD (hyperkinetic disorder) and severe
impairment

The first-line treatment for school-age children and young people with severe
ADHD (hyperkinetic disorder) and severe impairment is drug treatment. If the
child or young person wishes to refuse medication and/or the parents or carers
reject it, a psychological intervention may be tried but drug treatment has more
benefits and is superior to other treatments for this group.

1.5.3.1 In school-age children and young people with severe ADHD, drug
treatment should be offered as the first-line treatment. Parents should also be
offered a group-based parent-training/education programme.
1.5.3.2 Drug treatment should only be initiated by an appropriately qualified
healthcare professional with expertise in ADHD and should be based on a
comprehensive assessment and diagnosis. Continued prescribing and
monitoring of drug therapy may be performed by general practitioners, under
shared care arrangements[7].
1.5.3.3 If drug treatment is not accepted by the child or young person with severe
ADHD, or their parents or carers, healthcare professionals should advise parents
or carers and the child or young person about the benefits and superiority of drug
treatment in this group. If drug treatment is still not accepted, a group parenttraining/education programme should be offered.
1.5.3.4 If a group parent-training/education programme is effective in children
and young people with severe ADHD who have refused drug treatment,
healthcare professionals should assess the child or young person for possible
coexisting conditions and develop a longer-term care plan.
1.5.3.5 If a group parent-training/education programme is not effective for a child
or young person with severe ADHD, and if drug treatment has not been
accepted, discuss the possibility of drug treatment again or other psychological
treatment (group CBT and/or social skills training), highlighting the clear benefits
and superiority of drug treatment in children or young people with severe ADHD.
1.5.3.6 Following a diagnosis of severe ADHD in a school-age child or young
person healthcare professionals should, with the parents' or carers' consent,
contact the child or young person's teacher to explain:

the diagnosis and severity of symptoms and impairment

the care plan

any special educational needs.

1.5.3.7 Teachers who have received training about ADHD and its management
should provide behavioural interventions in the classroom to help children and
young people with ADHD.
1.5.4 Pre-drug treatment assessment

It is important that before starting drug treatment baseline measures of a range of


parameters, including height and weight, are taken.
1.5.4.1 Before starting drug treatment, children and young people with ADHD
should have a full pre-treatment assessment, which should include:

full mental health and social assessment

full history and physical examination, including:


o

assessment of history of exercise syncope, undue breathlessness and


other cardiovascular symptoms

heart rate and blood pressure (plotted on a centile chart)

height and weight (plotted on a growth chart)

family history of cardiac disease and examination of the cardiovascular


system

an electrocardiogram (ECG) if there is past medical or family history of


serious cardiac disease, a history of sudden death in young family members
or abnormal findings on cardiac examination

risk assessment for substance misuse and drug diversion (where the drug is
passed on to others for non-prescription use).

1.5.4.2 Drug treatment for children and young people with ADHD should always
form part of a comprehensive treatment plan that includes psychological,
behavioural and educational advice and interventions.
1.5.5 Choice of drug for children and young people with ADHD

Depending on a range of factors such as the presence of coexisting conditions,


side effects and patient preference, the child or young person may be offered
methylphenidate, atomoxetine or dexamfetamine.
1.5.5.1 Where drug treatment is considered appropriate, methylphenidate,
atomoxetine and dexamfetamine are recommended, within their licensed
indications, as options for the management of ADHD in children and
adolescents[7].
1.5.5.2 The decision regarding which product to use should be based on the
following:

the presence of comorbid conditions (for example, tic disorders, Tourette's


syndrome, epilepsy)

the different adverse effects of the drugs

specific issues regarding compliance identified for the individual child or


adolescent, for example problems created by the need to administer a midday treatment dose at school

the potential for drug diversion (where the medication is forwarded on to


others for non-prescription uses) and/or misuse

the preferences of the child/adolescent and/or his or her parent or


guardian[7].

1.5.5.3 When a decision has been made to treat children or young people with
ADHD with drugs, healthcare professionals should consider:

methylphenidate for ADHD without significant comorbidity

methylphenidate for ADHD with comorbid conduct disorder

methylphenidate or atomoxetine when tics, Tourette's syndrome, anxiety


disorder, stimulant misuse or risk of stimulant diversion are present

atomoxetine if methylphenidate has been tried and has been ineffective at


the maximum tolerated dose, or the child or young person is intolerant to low
or moderate doses of methylphenidate.

1.5.5.4 When prescribing methylphenidate for the treatment of children or young


people, modified-release preparations should be considered for the following
reasons:

convenience

improving adherence

reducing stigma (because the child or young person does not need to take
medication at school)

reducing problems schools have in storing and administering controlled


drugs

their pharmacokinetic profiles.


Alternatively, immediate-release preparations may be considered if more
flexible dosing regimens are required, or during initial titration to determine
correct dosing levels.

1.5.5.5 When starting drug treatment children and young people should be
monitored for side effects. In particular, those treated with atomoxetine should be
closely observed for agitation, irritability, suicidal thinking and self-harming
behaviour, and unusual changes in behaviour, particularly during the initial
months of treatment, or after a change in dose. Parents and/or carers should be
warned about the potential for suicidal thinking and self-harming behaviour with
atomoxetine and asked to report these to their healthcare professionals. Parents
or carers should also be warned about the potential for liver damage in rare
cases with atomoxetine (usually presenting as abdominal pain, unexplained
nausea, malaise, darkening of the urine or jaundice).
1.5.5.6 If there is a choice of more than one appropriate drug, the product with
the lowest cost (taking into account the cost per dose and number of daily doses)
should be prescribed[7].
1.5.5.7 Antipsychotics are not recommended for the treatment of ADHD in
children and young people.
1.5.6 Poor response to treatment

If there has been a poor response to parent-training/education programmes,


psychological treatment and drug treatment with methylphenidate and
atomoxetine, a comprehensive review is required. The following are further
options for treatment: higher doses of methylphenidate or atomoxetine; switching
to dexamfetamine; further or alternative psychological treatments; or referral to
regional specialists for alternative drug treatment.
1.5.6.1 If there has been a poor response following parent-training/education
programmes and/or psychological treatment and treatment with methylphenidate

and atomoxetine in a child or young person with ADHD, there should be a further
review of:

the diagnosis

any coexisting conditions

response to drug treatment, occurrence of side effects and treatment


adherence

uptake and use of psychological interventions for the child or young person
and their parents or carers

effects of stigma on treatment acceptability

concerns related to school and/or family

motivation of the child or young person and the parents or carers

the child or young person's diet.

1.5.6.2 Following review of poor response to treatment, a dose higher than that
licensed for methylphenidate or atomoxetine should be considered following
consultation with a tertiary or regional centre. This may exceed 'British national
formulary' (BNF) recommendations: methylphenidate can be increased to
0.7 mg/kg per dose up to three times a day or a total daily dose of 2.1 mg/kg/day
(up to a total maximum dose of 90 mg/day for immediate release; or an
equivalent dose of modified-release methylphenidate)[8]; atomoxetine may be
increased to 1.8 mg/kg/day (up to a total maximum dose of 120 mg/day). The
prescriber should closely monitor the child or young person for side effects.
1.5.6.3 Dexamfetamine should be considered in children and young people
whose ADHD is unresponsive to a maximum tolerated dose of methylphenidate
or atomoxetine.
1.5.6.4 In children and young people whose ADHD is unresponsive to
methylphenidate, atomoxetine and dexamfetamine, further treatment should only
follow after referral to tertiary services. Further treatment may include the use of
medication unlicensed for the treatment of ADHD (such as bupropion, clonidine,
modafinil and imipramine)[9] or combination treatments (including psychological
treatments for the parent or carer and the child or young person). The use of
medication unlicensed for ADHD should only be considered in the context of
tertiary services.
1.5.6.5 A cardiovascular examination and ECG should be carried out before
starting treatment with clonidine in children or young people with ADHD.
1.6 Transition to adult services

Young people with ADHD receiving treatment and care from CAMHS or
paediatric services should normally be transferred to adult services if they
continue to have significant symptoms of ADHD or other coexisting conditions
that require treatment. Transition should be planned in advance by both referring

and receiving services. If needs are severe and/or complex, use of the care
programme approach should be considered.
1.6.1.1 A young person with ADHD receiving treatment and care from CAMHS or
paediatric services should be reassessed at school-leaving age to establish the
need for continuing treatment into adulthood. If treatment is necessary,
arrangements should be made for a smooth transition to adult services with
details of the anticipated treatment and services that the young person will
require. Precise timing of arrangements may vary locally but should usually be
completed by the time the young person is 18 years.
1.6.1.2 During the transition to adult services, a formal meeting involving CAMHS
and/or paediatrics and adult psychiatric services should be considered, and full
information provided to the young person about adult services. For young people
aged 16 years and older, the care programme approach (CPA) should be used
as an aid to transfer between services. The young person, and when appropriate
the parent or carer, should be involved in the planning.
1.6.1.3 After transition to adult services, adult healthcare professionals should
carry out a comprehensive assessment of the person with ADHD that includes
personal, educational, occupational and social functioning, and assessment of
any coexisting conditions, especially drug misuse, personality disorders,
emotional problems and learning difficulties.
1.7 Treatment of adults with ADHD

Drug treatment is the first-line treatment for adults with ADHD with either
moderate or severe levels of impairment. Methylphenidate is the first-line drug.
Psychological interventions without medication may be effective for some adults
with moderate impairment, but there are insufficient data to support this
recommendation. If methylphenidate is ineffective or unacceptable, atomoxetine
or dexamfetamine can be tried. If there is residual impairment despite some
benefit from drug treatment, or there is no response to drug treatment, CBT may
be considered. There is the potential for drug misuse and diversion in adults with
ADHD, especially in some settings, such as prison, although there is no strong
evidence that this is a significant problem.
1.7.1.1 For adults with ADHD, drug treatment[10] should be the first-line treatment
unless the person would prefer a psychological approach.
1.7.1.2 Drug treatment for adults with ADHD should be started only under the
guidance of a psychiatrist, nurse prescriber specialising in ADHD, or other clinical
prescriber with training in the diagnosis and management of ADHD.
1.7.1.3 Before starting drug treatment for adults with ADHD a full assessment
should be completed, which should include:

full mental health and social assessment

full history and physical examination, including:


o

assessment of history of exercise syncope, undue breathlessness and


other cardiovascular symptoms

heart rate and blood pressure (plotted on a centile chart)

weight

family history of cardiac disease and examination of the cardiovascular


system

an ECG if there is past medical or family history of serious cardiac disease,


a history of sudden death in young family members or abnormal findings on
cardiac examination

risk assessment for substance misuse and drug diversion.

1.7.1.4 Drug treatment for adults with ADHD should always form part of a
comprehensive treatment programme that addresses psychological, behavioural
and educational or occupational needs.
1.7.1.5 Following a decision to start drug treatment in adults with ADHD,
methylphenidate should normally be tried first.
1.7.1.6 Atomoxetine or dexamfetamine should be considered in adults
unresponsive or intolerant to an adequate trial of methylphenidate (this should
usually be about 6 weeks)[10]. Caution should be exercised when prescribing
dexamfetamine to those likely to be at risk of stimulant misuse or diversion.
1.7.1.7 When starting drug treatment, adults should be monitored for side effects.
In particular, people treated with atomoxetine should be observed for agitation,
irritability, suicidal thinking and self-harming behaviour, and unusual changes in
behaviour, particularly during the initial months of treatment, or after a change in
dose. They should also be warned of potential liver damage in rare cases
(usually presenting as abdominal pain, unexplained nausea, malaise, darkening
of the urine or jaundice). Younger adults aged 30 years or younger should also
be warned of the potential of atomoxetine to increase agitation, anxiety, suicidal
thinking and self-harming behaviour in some people, especially during the first
few weeks of treatment.
1.7.1.8 For adults with ADHD stabilised on medication but with persisting
functional impairment associated with the disorder, or where there has been no
response to drug treatment, a course of either group or individual CBT to address
the person's functional impairment should be considered. Group therapy is
recommended as the first-line psychological treatment because it is the most
cost effective.
1.7.1.9 For adults with ADHD, CBT may be considered when:

the person has made an informed choice not to have drug treatment

drug treatment has proved to be only partially effective or ineffective or the


person is intolerant to it

people have difficulty accepting the diagnosis of ADHD and accepting and
adhering to drug treatment

symptoms are remitting and psychological treatment is considered sufficient


to target residual (mild to moderate) functional impairment.

1.7.1.10 Where there may be concern about the potential for drug misuse and
diversion (for example, in prison services), atomoxetine may be considered as
the first-line drug treatment for ADHD in adults[10].
1.7.1.11 Drug treatment for adults with ADHD who also misuse substances
should only be prescribed by an appropriately qualified healthcare professional
with expertise in managing both ADHD and substance misuse. For adults with
ADHD and drug or alcohol addiction disorders there should be close liaison
between the professional treating the person's ADHD and an addiction specialist.
1.7.1.12 Antipsychotics are not recommended for the treatment of ADHD in
adults.
1.8 How to use drugs for the treatment of ADHD

Good knowledge of the drugs used in the treatment of ADHD and their different
preparations is essential (refer to the BNF and summaries of product
characteristics). It is important to start with low doses and titrate upwards,
monitoring effects and side effects carefully. Higher doses may need to be
prescribed to some adults. The recommendations on improving adherence in
children and young people may also be of use in adults.
1.8.1 General principles

1.8.1.1 Prescribers should be familiar with the pharmacokinetic profiles of all the
modified-release and immediate-release preparations available for ADHD to
ensure that treatment is tailored effectively to the individual needs of the child,
young person or adult.
1.8.1.2 Prescribers should be familiar with the requirements of controlled drug
legislation governing the prescription and supply of stimulants.
1.8.1.3 During the titration phase, doses should be gradually increased until there
is no further clinical improvement in ADHD (that is, symptom reduction,
behaviour change, improvements in education and/or relationships) and side
effects are tolerable.
1.8.1.4 Following titration and dose stabilisation, prescribing and monitoring
should be carried out under locally agreed shared care arrangements with
primary care.
1.8.1.5 Side effects resulting from drug treatment for ADHD should be routinely
monitored and documented in the person's notes.
1.8.1.6 If side effects become troublesome in people receiving drug treatment for
ADHD, a reduction in dose should be considered.
1.8.1.7 Healthcare professionals should be aware that dose titration should be
slower if tics or seizures are present in people with ADHD.
1.8.2 Initiation and titration of methylphenidate, atomoxetine and dexamfetamine in children and young people

1.8.2.1 During the titration phase, symptoms and side effects should be recorded
at each dose change on standard scales (for example, Conners' 10-item scale)
by parents and teachers, and progress reviewed regularly (for example, by
weekly telephone contact and at each dose change) with a specialist clinician.
1.8.2.2 If using methylphenidate in children and young people with ADHD aged
6 years and older:

initial treatment should begin with low doses of immediate-release or


modified-release preparations consistent with starting doses in the BNF

the dose should be titrated against symptoms and side effects over 4
6 weeks until dose optimisation is achieved

modified-release preparations should be given as a single dose in the


morning

immediate-release preparations should be given in two or three divided


doses.

1.8.2.3 If using atomoxetine in children and young people with ADHD aged
6 years and older:

for those weighing up to 70 kg, the initial total daily dose should be
approximately 0.5 mg/kg; the dose should be increased after 7 days to
approximately 1.2 mg/kg/day

for those weighing more than 70 kg, the initial total daily dose should be
40 mg; the dose should be increased after 7 days up to a maintenance dose
of 80 mg/day

a single daily dose can be given; two divided doses may be prescribed to
minimise side effects.

1.8.2.4 If using dexamfetamine in children and young people with ADHD:

initial treatment should begin with low doses consistent with starting doses in
the BNF

the dose should be titrated against symptoms and side effects over 4
6 weeks

treatment should be given in divided doses increasing to a maximum of


20 mg/day

for children aged 618 years, doses up to 40 mg/day may occasionally be


required.

1.8.3 Initiation and titration of methylphenidate, atomoxetine and dexamfetamine in adults

1.8.3.1 In order to optimise drug treatment, the initial dose should be titrated
against symptoms and side effects over 46 weeks.
1.8.3.2 During the titration phase, symptoms and side effects should be recorded
at each dose change by the prescriber after discussion with the person with
ADHD and, wherever possible, a carer (for example, a spouse, parent or close

friend). Progress should be reviewed (for example, by weekly telephone contact


and at each dose change) with a specialist clinician.
1.8.3.3 If using methylphenidate in adults with ADHD:

initial treatment should begin with low doses (5 mg three times daily for
immediate-release preparations; the equivalent dose for modified-release
preparations)

the dose should be titrated against symptoms and side effects over 4
6 weeks

the dose should be increased according to response up to a maximum of


100 mg/day

modified-release preparations should usually be given once daily and no


more than twice daily

modified-release preparations may be preferred to increase adherence and


in circumstances where there are concerns about substance misuse or
diversion

immediate-release preparations should be given up to four times daily.

1.8.3.4 If using atomoxetine in adults with ADHD:

for people with ADHD weighing up to 70 kg, the initial total daily dose should
be approximately 0.5 mg/kg; the dose should be increased after 7 days to
approximately 1.2 mg/kg/day

for people with ADHD weighing more than 70 kg, the initial total daily dose
should be 40 mg; the dose should be increased after 7 days up to a
maintenance dose of 100 mg/day

the usual maintenance dose is either 80 or 100 mg, which may be taken in
divided doses

a trial of 6 weeks on a maintenance dose should be allowed to evaluate the


full effectiveness of atomoxetine.

1.8.3.5 If using dexamfetamine in adults with ADHD:

initial treatment should begin with low doses (5 mg twice daily)

the dose should be titrated against symptoms and side effects over 4
6 weeks

treatment should be given in divided doses

the dose should be increased according to response up to a maximum of


60 mg per day

the dose should usually be given between two and four times daily.

1.8.4 Monitoring side effects and the potential for misuse in children, young people and adults

1.8.4.1 Healthcare professionals should consider using standard symptom and


side effect rating scales throughout the course of treatment as an adjunct to
clinical assessment for people with ADHD.
1.8.4.2 In people taking methylphenidate, atomoxetine, or dexamfetamine:

height should be measured every 6 months in children and young people

weight should be measured 3 and 6 months after drug treatment has started
and every 6 months thereafter in children, young people and adults

height and weight in children and young people should be plotted on a


growth chart and reviewed by the healthcare professional responsible for
treatment.

1.8.4.3 If there is evidence of weight loss associated with drug treatment in adults
with ADHD, healthcare professionals should consider monitoring body mass
index and changing the drug if weight loss persists.
1.8.4.4 Strategies to reduce weight loss in people with ADHD, or manage
decreased weight gain in children, include:

taking medication either with or after food, rather than before meals

taking additional meals or snacks early in the morning or late in the evening
when the stimulant effects of the drug have worn off

obtaining dietary advice

consuming high-calorie foods of good nutritional value.

1.8.4.5 If growth is significantly affected by drug treatment (that is, the child or
young person has not met the height expected for their age), the option of a
planned break in treatment over school holidays should be considered to allow
'catch-up' growth to occur.
1.8.4.6 In people with ADHD, heart rate and blood pressure should be monitored
and recorded on a centile chart before and after each dose change and routinely
every 3 months.
1.8.4.7 For people taking methylphenidate, dexamfetamine and atomoxetine,
routine blood tests and ECGs are not recommended unless there is a clinical
indication.
1.8.4.8 Liver damage is a rare and idiosyncratic adverse effect of atomoxetine
and routine liver function tests are not recommended.
1.8.4.9 For children and young people taking methylphenidate and
dexamfetamine, healthcare professionals and parents or carers should monitor
changes in the potential for drug misuse and diversion, which may come with
changes in circumstances and age. In these situations, modified-release
methylphenidate or atomoxetine may be preferred.
1.8.4.10 In young people and adults, sexual dysfunction (that is, erectile and
ejaculatory dysfunction) and dysmenorrhoea should be monitored as potential
side effects of atomoxetine.

1.8.4.11 For people taking methylphenidate, dexamfetamine or atomoxetine who


have sustained resting tachycardia, arrhythmia or systolic blood pressure greater
than the 95th percentile (or a clinically significant increase) measured on two
occasions should have their dose reduced and be referred to a paediatrician or
adult physician.
1.8.4.12 If psychotic symptoms (for example, delusions and hallucinations)
emerge in children, young people and adults after starting methylphenidate or
dexamfetamine, the drug should be withdrawn and a full psychiatric assessment
carried out. Atomoxetine should be considered as an alternative.
1.8.4.13 If seizures are exacerbated in a child or young person with epilepsy, or
de novo seizures emerge following the introduction of methylphenidate or
atomoxetine, the drug should be discontinued immediately. Dexamfetamine may
be considered as an alternative in consultation with a regional tertiary specialist
treatment centre.
1.8.4.14 If tics emerge in people taking methylphenidate or dexamfetamine,
healthcare professionals should consider whether:

the tics are stimulant-related (tics naturally wax and wane)

tic-related impairment outweighs the benefits of ADHD treatment.


If tics are stimulant-related, reduce the dose of methylphenidate or
dexamfetamine, consider changing to atomoxetine, or stop drug treatment.

1.8.4.15 Anxiety symptoms, including panic, may be precipitated by stimulants,


particularly in adults with a history of coexisting anxiety. Where this is an issue,
lower doses of the stimulant and/or combined treatment with an antidepressant
used to treat anxiety can be used; switching to atomoxetine may be effective.
1.8.5 Improving adherence to drug treatment

For children and young people with ADHD, the strategies outlined in the
recommendations below should be considered to improve treatment adherence.
Similar strategies, adapted for age, may be considered for adults.
1.8.5.1 Communication between the prescriber and the child or young person
should be improved by educating parents or carers and ensuring there are
regular three-way conversations between prescriber, parent or carer and the
child or young person. For adults with ADHD, and with their permission, a
spouse, partner, parent, close friend or carer wherever possible should be part of
these conversations. Clear instructions about how to take the drug should be
offered in picture or written format, which may include information on dose,
duration, side effects, dosage schedule, the need for supervision and how this
should be done.
1.8.5.2 Healthcare professionals should consider suggesting peer-support
groups for the child or young person with ADHD and their parents or carers if
adherence to drug treatment is difficult or uncertain.

1.8.5.3 Simple drug regimens (for example, once-daily modified-release doses)


are recommended for people with ADHD.
1.8.5.4 Healthcare professionals should encourage children and young people
with ADHD to be responsible for their own health, including taking their
medication as required, and support parents and carers in this endeavour.
1.8.5.5 Healthcare professionals should advise parents or carers to provide the
child or young person with visual reminders to take medication regularly (for
example, alarms, clocks, pill boxes, or notes on calendars or fridges).
1.8.5.6 Healthcare professionals should advise children and young people and
their parents or carers that taking medication should be incorporated into daily
routines (for example, before meals or after brushing teeth).
1.8.5.7 Where necessary, healthcare professionals should help parents or carers
develop a positive attitude and approach in the management of medication,
which might include praise and positive reinforcement for the child or young
person with ADHD.
1.8.6 Duration, discontinuation and continuity of treatment in children and young people

It is advisable to review each year whether the child or young person needs to
continue drug treatment and to ensure that the long-term pattern of use is
tailored to the person's needs, preferences and circumstances.
1.8.6.1 Following an adequate treatment response, drug treatment for ADHD
should be continued for as long as it remains clinically effective. This should be
reviewed at least annually. The review should include a comprehensive
assessment of clinical need, benefits and side effects, taking into account the
views of the child or young person, as well as those of parents, carers and
teachers, and how these views may differ. The effect of missed doses, planned
dose reductions and brief periods of no treatment should be taken into account
and the preferred pattern of use should also be reviewed. Coexisting conditions
should be reviewed, and the child or young person treated or referred if
necessary. The need for psychological and social support for the child or young
person and for the parents or other carers should be assessed.
1.8.6.2 Drug holidays are not routinely recommended; however, consideration
should be given to the parent or carer and child or young person with ADHD
working with their healthcare professional to find the best pattern of use, which
may include periods without drug treatment.
1.8.7 Duration, discontinuation and continuity of treatment in adults

1.8.7.1 Following an adequate response, drug treatment for ADHD should be


continued for as long as it is clinically effective. This should be reviewed
annually. The review should include a comprehensive assessment of clinical
need, benefits and side effects, taking into account the views of the person and
those of a spouse, partner, parent, close friends or carers wherever possible, and
how these accounts may differ. The effect of missed doses, planned dose
reductions and brief periods of no treatment should be taken into account and the
preferred pattern of use should also be reviewed. Coexisting conditions should

be reviewed, and the person treated or referred if necessary. The need for
psychological, social and occupational support for the person and their carers
should be assessed.
1.8.7.2 An individual treatment approach is important for adults, and healthcare
professionals should regularly review (at least annually) the need to adapt
patterns of use, including the effect of drug treatment on coexisting conditions
and mood changes.

Question 21 of 257

A 3-year-old boy is brought to surgery by his father. He is currently being managed with paracetamol for a middle ear infection but his father
is concerned his heart is racing. What is the normal heart rate of a 3-year-old?
90 - 140 bpm

50 - 70 bpm

60 - 100 bpm

110 - 160 bpm

70 - 120 bpm

Next question

Paediatric vital signs


The table below indicate age-appropriate vital signs
Age Heart rate Respiratory rate
<1

110 - 160 30 - 40

1 - 2 100 - 150 25 - 35
2 - 5 90 - 140

25 - 30

5 - 12 80 - 120

20 - 25

> 12 60 - 100

15 - 20

Question 22 of 257

A 2-year-old boy is seen in the Emergency Department with watery diarrhoea for the past two day. What is the most likely causative agent?

Norovirus

Rotavirus

E. coli

Norwalk virus

Adenovirus

Next question

Diarrhoea and vomiting in children


Diarrhoea and vomiting is very common in younger children. The most common cause of gastroenteritis in children in the UK is rotavirus.
Much of the following is based around the 2009 NICE guidelines (please see the link for more details).
Clinical features
NICE suggest that typically:

diarrhoea usually lasts for 5-7 days and stops within 2 weeks
vomiting usually lasts for 1-2 days and stops within 3 days

When assessing hydration status NICE advocate using normal, dehydrated or shocked categories rather than the traditional normal, mild,
moderate or severe categories.
Clinical dehydration

Clinical shock

Appears to be unwell or deteriorating


Decreased level of consciousness
Decreased urine output
Skin colour unchanged
Cold extremities
Warm extremities
Altered responsiveness (for example, irritable, lethargic)Pale or mottled skin
Sunken eyes
Dry mucous membranes
Tachycardia
Tachypnoea
Normal peripheral pulses
Normal capillary refill time
Reduced skin turgor
Normal blood pressure

Tachycardia
Tachypnoea
Weak peripheral pulses
Prolonged capillary refill time
Hypotension

The following children are at an increased risk of dehydration:

children younger than 1 year, especially those younger than 6 months


infants who were of low birth weight
children who have passed six or more diarrhoeal stools in the past 24 hours
children who have vomited three times or more in the past 24 hours
children who have not been offered or have not been able to tolerate supplementary fluids before presentation
infants who have stopped breastfeeding during the illness
children with signs of malnutrition

Features suggestive of hypernatraemic dehydration:

jittery movements
increased muscle tone
hyperreflexia
convulsions
drowsiness or coma

Diagnosis
NICE suggest doing a stool culture in the following situations:

you suspect septicaemia or


there is blood and/or mucus in the stool or
the child is immunocompromised

You should consider doing a stool culture if:

the child has recently been abroad or


the diarrhoea has not improved by day 7 or
you are uncertain about the diagnosis of gastroenteritis

Management
If clinical shock is suspected children should be admitted for intravenous rehydration.
For children with no evidence of dehydration

continue breastfeeding and other milk feeds


encourage fluid intake
discourage fruit juices and carbonated drinks

If dehydration is suspected:

give 50 ml/kg low osmolarity oral rehydration solution (ORS) solution over 4 hours, plus ORS solution for maintenance, often
and in small amounts
continue breastfeeding
consider supplementing with usual fluids (including milk feeds or water, but not fruit juices or carbonated drinks)

Question 23 of 257

You are asked to give a talk to local district nurses about childhood diseases. In the UK, what is the most common cause of death in
children greater than one year old?
Cancer

Non-accidental injury

Congenital disorders

Infection

Accidents

Next question

Accidents in children
Around 15-20% of children attend Emergency Departments in the course of a year due to an accident. Accidents account for a third of all
childhood deaths and are the single most common cause of death in children aged between 1 - 15 years of age.
Key points

road traffic accidents are the most common cause of fatal accidents
boys and children from lower social classes are more likely to have an accident

Accident prevention
Accident prevention can be divided up into primary (preventing the accident from happening), secondary (prevent injury from the accident)
and tertiary (limit the impact of the injury) prevention strategies
The table below gives examples of accident prevention strategies
Primary prevention

Secondary preventionTertiary prevention

Stair guards
Wearing seat belts
Speed limits*
Cycling helmets
Teaching parents first aid
Teaching road safety Smoke alarms
Window safety catchesLaminated safety glass

*some strategies such as reducing driving speed may have a role in both primary and secondary accident prevention

Question 24 of 257

A 16-month-old girl is reviewed by her GP. She has a 3 day history of fever and coryzal symptoms. Overnight she has developed a harsh
cough. On examination she has a temperature of 38C and inspiratory stridor is noticed although there are no signs of intercostal
recession. What is the most likely diagnosis?
Bronchiolitis

Croup

Bacterial tracheitis

Pertussis

Acute epiglottitis

Next question

Croup
Croup is a form of upper respiratory tract infection seen in infants and toddlers. It is characterised by stridor which is caused by a
combination of laryngeal oedema and secretions. Parainfluenza viruses account for the majority of cases.
Epidemiology

peak incidence at 6 months - 3 years


more common in autumn

Features

stridor
barking cough (worse at night)
fever
coryzal symptoms

Clinical Knowledge Summaries (CKS) suggest using the following criteria to grade the severity*:
Mild

Moderate

Severe

Occasional barking cough


No audible stridor at rest
No or mild suprasternal and/or
intercostal recession
The child is happy and is prepared to
eat, drink, and play

Frequent barking cough


Frequent barking cough
Prominent inspiratory (and occasionally, expiratory)
Easily audible stridor at rest
stridor at rest
Suprasternal and sternal wall retraction at
Marked sternal wall retractions
rest
Significant distress and agitation, or lethargy or
No or little distress or agitation
restlessness (a sign of hypoxaemia)
The child can be placated and is interested
Tachycardia occurs with more severe obstructive
in its surroundings
symptoms and hypoxaemia

CKS suggest admitting any child with moderate or severe croup. Other features which should prompt admission include:

< 6 months of age


known upper airway abnormalities (e.g. Laryngomalacia, Down's syndrome)
uncertainty about diagnosis (important differentials include acute epiglottitis, bacterial tracheitis, peritonsillar abscess and
foreign body inhalation)

Management

CKS recommend giving a single dose of oral dexamethasone (0.15mg/kg) to all children regardless of severity
prednisolone is an alternative if dexamethasone is not available

Emergency treatment

high-flow oxygen
nebulised adrenaline

*these in turn are based partly on the Alberta Medical Association (2008) Guideline for the diagnosis and management of croup.
Question 25 of 257

Which one of the following statements concerning infantile colic is false?


It is characterised by excessive crying

Symptoms are often worse in the evening

Is most common at around 6 months of age

The cause is unknown

Occurs in up to 20% of infants

Next question

Infantile colic typically occurs in infants less than 3 months old

Infantile colic
Infantile colic describes a relatively common and benign set of symptoms seen in young infants. It typically occurs in infants less than 3
months old and is characterised by bouts of excessive crying and pulling-up of the legs, often worse in the evening
Infantile colic occurs in up to 20% of infants. The cause of infantile colic is unknown

Colic - infantile - Summary

Infantile colic is defined for clinical purposes as repeated episodes of excessive and
inconsolable crying in an infant that otherwise appears to be healthy and thriving.
The underlying cause is unknown.
Estimates of prevalence range from 520% of infants, depending on the definition used.
Infantile colic can cause significant distress and suffering to the parents.
Typically:
o Colic starts in the first weeks of life and resolves by around 4 months of age.
o Crying most often occurs in the late afternoon or evening.
o The baby draws its knees up to its abdomen, or arches its back when crying.
The history and examination should include the:
o General health of the baby.
o Antenatal and perinatal history.
o Onset and length of crying.
o Nature of the stools.
o Feeding assessment.
o Mother's diet if breastfeeding.
o Family history of allergy.
o Parent's response to the baby's crying.
o Factors which lessen or worsen the crying.

The most useful intervention is support for parents and reassurance that infantile colic
will resolve.
Holding the baby through the crying episode may be helpful. Other strategies include:
o Gentle motion (e.g. pushing the pram, rocking the crib).
o 'White noise' (e.g. vacuum cleaner, hairdryer, running water).
o Bathing in a warm bath.
Parents should be encouraged to look after their own well-being by:
o Resting when possible.
o Asking family and friends for support. CRY-SIS is a support group for families
(www.cry-sis.org.uk).
o Meeting other parents with babies of the same age.
Health visitors will also provide advice and support.
Medical treatments should only be tried if parents feel unable to cope despite advice
and reassurance. The options for medical treatments are:
o A 1-week trial of simeticone drops (breastfed or bottle-fed).
o A 1-week trial of diet modification to exclude cow's milk protein breastfed babies:
dairy-free diet for the mother; bottle-fed babies: hypoallergenic formula.
o A 1-week trial of lactase drops (breastfed or bottle-fed).
Treatment should only be continued if there is a response (i.e. the duration of crying
shortens). If no response, another treatment can be considered.
Breastfeeding mothers should take a calcium supplement if they are on a dairy-free diet
long term.
If the baby responds to lactase or a hypoallergenic diet, the parents should be
reassured that this does not necessarily mean that they are lactose intolerant or
allergic to cow's milk.
If there is a response to treatment: after the age of 3 months (and by 6 months of age at
the latest), treatment can be weaned off over a period of about 1 week.
Advice from a paediatrician is required if:
o The parents are not coping.
o There is diagnostic doubt.
o It is not possible to wean the baby off treatment by the age of 6 months.

Question 26 of 257

A mother brings her 4-year-old daughter into surgery as she thinks she may have 'nits'. Which one of the following statements regarding
head lice is incorrect?
Malathion is an option for first-line treatment

Fine-toothed combing of wet or dry hair is the standard way of diagnosing head lice

Children should be excluded from school until treatment has been started

Caused by Pediculus capitis

Wet combing alone is a suitable first-line treatment

Next question

School exclusion is not advised for children with head lice

Head lice
Head lice (also known as pediculosis capitis or 'nits') is a common condition in children caused by the parasitic insect Pediculus capitis,
which lives on and among the hair of the scalp of humans
Diagnosis

fine-toothed combing of wet or dry hair

Management

treatment is only if living lice are found


a choice of treatments should be offered - malathion, wet combing, dimeticone, isopropyl myristate and cyclomethicone

School exclusion is not advised for children with head lice

Question 1 of 231

The chance of a 35-year-old mother giving birth to a child with Down's syndrome is approximately:
1 in 125

1 in 350

1 in 550

1 in 1,000

1 in 2,000

Next question

Down's syndrome risk - 1/1,000 at 30 years then divide by 3 for every 5 years

Down's syndrome: epidemiology and genetics


Risk of Down's syndrome with increasing maternal age

risk at 30 years = 1/1000


35 years = 1/350
40 years = 1/100
45 years = 1/30

One way of remembering this is by starting at 1/1,000 at 30 years and then dividing the denominator by 3 (i.e. 3 times more common) for
every extra 5 years of age
Cytogenetics
Mode

% of cases Risk of recurrence

Non-disjunction

94%

Robertsonian translocation
5%
(usually onto 14)
Mosaicism

1 in 100 if under mother < 35 years


10-15% if mother is translocation carrier
2.5% if father is translocation carrier

1%

The chance of a further child with Down's syndrome is approximately 1 in 100 if the mother is less than 35 years old. If the trisomy 21 is a
result of a translocation the risk is much higher

Question 2 of 231

Each one of the following statements regarding autism is correct, except:


There is a global impairment of language and communication

The majority of children have normal or increased intelligence

Children may perform ritualistic behaviour

75% of children are male

Usually develops before 3 years of age

Next question

Autism
Epidemiology

75% of children are male


usually develops before 3 years of age

All 3 of the following features must be present for a diagnosis to be made

global impairment of language and communication


impairment of social relationships
ritualistic and compulsive phenomena

Other features

most children have a decreased IQ - the 'idiot savant' is rare

Associated conditions

Fragile X
Rett's syndrome

Question 3 of 231

A 14-month-old boy is brought to surgery. Mum says he has been off his food for the past few days and is a bit 'niggly'. Clinical examination
reveals the following:

Image used on license from DermNet NZ

His temperature is 37.8degC. What is the most likely diagnosis?


Fifth disease

Erythema multiforme

Kawasaki disease

Hand, foot and mouth disease

Measles

Next question

Hand, foot and mouth disease


Hand, foot and mouth disease is a self-limiting condition affecting children. It is caused by the intestinal viruses of the Picornaviridae family
(most commonly coxsackie A16 and enterovirus 71). It is very contagious and typically occurs in outbreaks at nursery
Clinical features

mild systemic upset: sore throat, fever


oral ulcers
followed later by vesicles on the palms and soles of the feet

Image used on license from DermNet NZ

Image used on license from DermNet NZ

Management

general advice about hydration and analgesia


reassurance no link to disease in cattle
children do not need to be excluded from school*

*The HPA recommends that children who are unwell should be kept off school until they feel better. They also advise that you contact them
if you suspect that there may be a large outbreak.

Question 4 of 231

A 6-year-old boy is brought for review. You can see from his records that he has been treated for constipation in the past but is otherwise fit
and well. His mother reports that he is currently passing only one hard stool every 4-5 days. The stool is described as being like 'rabbit
droppings'. There is no history of overflow soiling or diarrhoea. Examination of the abdomen is unremarkable. What is the most appropriate
first-line intervention?
Advice on diet/fluid intake + lactulose

Advice on diet/fluid intake

Advice on diet/fluid intake + Movicol Paediatric Plain

Advice on diet/fluid intake + refer to local Constipation Nurse for support

Advice on diet/fluid intake + senna

Next question

Constipation in children: Movicol is first-line

Please see the new NICE guidelines for more information.

Constipation in children
The frequency at which children open their bowels varies widely, but generally decreases with age from a mean of 3 times per day for
infants under 6 months old to once a day after 3 years of age.
NICE produced guidelines in 2010 on the diagnosis and management of constipation in children. A diagnosis of constipation is suggested
by 2 or more of the following:
Child < 1 year

Child > 1 year

Fewer than 3 complete stools per week


Fewer than 3 complete stools per week (type 3 or 4 on Bristol Stool
(type 3 or 4)
Form Scale) (this does not apply to exclusively breastfed babies after 6 Overflow soiling (commonly very loose,
weeks
very smelly, stool passed without
of age)
sensation)
Hard large stool
'Rabbit droppings' (type 1)
'Rabbit droppings' (type 1)
Large, infrequent stools that can block the
toilet

Stool pattern

Distress on passing stool


Symptoms associated
Bleeding associated with hard stool
with defecation
Straining

Previous episode(s) of constipation


Previous or current anal fissure

History

Poor appetite that improves with passage


of large stool
Waxing and waning of abdominal pain with
passage of stool
Evidence of retentive posturing: typical
straight legged, tiptoed, back arching
posture
Straining
Anal pain
Previous episode(s) of constipation
Previous or current anal fissure
Painful bowel movements and bleeding
associated with hard stools

The vast majority of children have no identifiable cause - idiopathic constipation. Other causes of constipation in children include:

dehydration
low-fibre diet
medications: e.g. Opiates
anal fissure
over-enthusiastic potty training
hypothyroidism
Hirschsprung's disease
hypercalcaemia
learning disabilities

After making a diagnosis of constipation NICE then suggesting excluding secondary causes. If no red or amber flags are present then a
diagnosis of idiopathic constipation can be made:

Indicates idiopathic constipation

Timing

'Red flag' suggesting underlying


disorder

Starts after a few weeks of life


Reported from birth or first few
Obvious precipitating factors coinciding with the start of symptoms: fissure, change
weeks of life
of diet, timing of potty/toilet training or acute events such as infections, moving
house, starting nursery/school, fears and phobias, major change in family, taking

Indicates idiopathic constipation

'Red flag' suggesting underlying


disorder

medicines
Passage of
meconium

< 48 hours

Stool pattern
Growth

> 48 hours

'Ribbon' stools
Generally well, weight and height within normal limits, fit and active

Faltering growth is an amber flag

Neuro/locomotor No neurological problems in legs, normal locomotor development

Previously unknown or
undiagnosed weakness in legs,
locomotor delay

Abdomen

Distension

Diet

Changes in infant formula, weaning, insufficient fluid intake or poor diet


Amber flag: Disclosure or evidence
that raises concerns over possibility
of child maltreatment

Other

Prior to starting treatment the child needs to be assessed for faecal impaction. Factors which suggest faecal impaction include:

symptoms of severe constipation


overflow soiling
faecal mass palpable in abdomen (digital rectal examination should only be carried out by a specialist)

NICE guidelines on management


If faecal impaction is present

polyethylene glycol 3350 + electrolytes (Movicol Paediatric Plain) using an escalating dose regimen as the first-line treatment
add a stimulant laxative if Movicol Paediatric Plain does not lead to disimpaction after 2 weeks
substitute a stimulant laxative singly or in combination with an osmotic laxative such as lactulose if Movicol Paediatric Plain is
not tolerated
inform families that disimpaction treatment can initially increase symptoms of soiling and abdominal pain

Maintenance therapy

very similar to the above regime, with obvious adjustments to the starting dose, i.e.
first-line: Movicol Paediatric Plain
add a stimulant laxative if no response
substitute a stimulant laxative if Movicol Paediatric Plain is not tolerated. Add another laxative such as lactulose or docusate if
stools are hard
continue medication at maintenance dose for several weeks after regular bowel habit is established, then reduce dose gradually

General points

do not use dietary interventions alone as first-line treatment although ensure child is having adequate fluid and fibre intake
consider regular toileting and non-punitive behavioural interventions
for all children consider asking the Health Visitor or Paediatric Continence Advisor to help support the parents.

The NICE guidelines do not specifically discuss the management of very young child. The following recommendations are largely based on

the old Clinical Knowledge Summaries recommendations.


Infants not yet weaned (usually < 6 months)

bottle-fed infants: give extra water in between feeds. Try gentle abdominal massage and bicycling the infant's legs
breast-fed infants: constipation is unusual and organic causes should be considered

Infants who have or are being weaned

offer extra water, diluted fruit juice and fruits


if not effective consider adding lactulose

Constipation in children - Summary

Constipation is a decrease in the frequency of bowel movements characterized by the


passing of hardened stools which may be large and associated with straining and pain.
Soiling of the clothes may result from overflow from the overloaded bowel.
Normal stool frequency ranges from an average of four per day in the first week of life
to two per day at 1 year of age. Passing between three stools per day and three per
week is usually attained by 4 years of age.
Constipation is termed idiopathic (functional) if it cannot be explained by any
anatomical or physiological abnormality.
The exact cause of constipation is poorly understood. Contributing factors include pain,
fever, dehydration, dietary and fluid intake, toilet training problems, the effects of
medicines, psychological issues, and a family history of constipation.
Two or more of the following indicate that the child is constipated:
o Fewer than three complete stools per week (unless exclusively breastfed when
stools may be infrequent).
o Hard, large stool.
o 'Rabbit droppings' stool.
o Overflow soiling (commonly very loose, very smelly stools, which are passed without
sensation or awareness; may also be thick and sticky, or dry and flaky).
No specific investigations are required in primary care.
If constipation is diagnosed, red or amber flags and anatomical and physiological causes
should be excluded.
Faecal impaction should be identified because the child will require disimpaction prior
to prescribing maintenance treatment. Impaction should be suspected if there is:
o A history of severe symptoms of constipation.
o Overflow soiling.
o A faecal mass palpable on abdominal examination (or rectally, although digital rectal
examination is not routinely recommended in children).
If red flag features are present, referral for urgent specialist assessment should be
arranged.
Children should be treated in primary care only if a positive diagnosis of idiopathic
constipation has been made.
o Offer reassurance that underlying causes of constipation have been excluded.
o Advise that the condition is treatable, but laxatives may need to be taken for several
months in order to resolve the problem.
o Written advice should be offered.

An adequate fluid intake and a balanced diet with sufficient fibre (unless exclusively
breastfed) should be recommended.
o Faecal impaction should be treated using the recommended disimpaction regimen.
o If impaction is not present or has been treated, the child should be promptly treated
with a laxative (even if the history of constipation is very short).
o Polyethylene glycol 3350 plus electrolytes (Movicol Paediatric Plain) is preferred
as first-line management. If this approach does not work, a stimulant laxative,
lactulose, or docusate should be considered.
Advice should be offered on behavioural interventions such as scheduled toileting, use
of a bowel diary, and reward systems.
Follow up should be arranged regularly to assess adherence and response to treatment
o

Guidance

1.1 History-taking and physical examination


1.2 Digital rectal examination
1.3 Clinical investigations
1.4 Clinical management
1.5 Diet and lifestyle
1.6 Psychological interventions
1.7 Antegrade colonic enema procedure
1.8 Information and support

1 Guidance
The following guidance is based on the best available evidence. The full
guideline gives details of the methods and the evidence used to develop the
guidance.
1.1 History-taking and physical examination

1.1.1 Establish during history-taking whether the child or young person has
constipation. Two or more findings from table 1 indicate constipation.
Table 1 Key components of history-taking to diagnose constipation
Key
components
Stool patterns

Potential findings in a child


younger than 1 year

Potential findings in a
child/young person older than
1 year

Fewer than three complete


stools per week (type 3 or
4, see Bristol Stool Form
Scale appendix D) (this
does not apply to

exclusively breastfed
babies after 6 weeks of
age)

Hard large stool

'Rabbit droppings' (type 1,

Fewer than three complete


stools per week (type 3 or 4,
see Bristol Stool Form Scale
appendix D)
Overflow soiling (commonly
very loose [no form], very
smelly [smells more
unpleasant than normal
stools], stool passed without
sensation. Can also be thick

see Bristol Stool Form


Scale appendix D)

Symptoms

associated with
defecation

History

and sticky or dry and flaky.)

'Rabbit droppings' (type 1,


see Bristol Stool Form Scale
appendix D)

Large, infrequent stools that


can block the toilet

Distress on stooling

Bleeding associated with


hard stool

Poor appetite that improves


with passage of large stool

Waxing and waning of


abdominal pain with
passage of stool

Evidence of retentive
posturing: typical straight
legged, tiptoed, back
arching posture

Straining

Anal pain

Straining

Previous episode(s) of
constipation

Previous episode(s) of
constipation

Previous or current anal


fissure

Previous or current anal


fissure

Painful bowel movements


and bleeding associated
with hard stools

1.1.2 If the child or young person has constipation take a history using table 2 to
establish a positive diagnosis of idiopathic constipation by excluding underlying
causes. If a child or young person has any 'red flag' symptoms, do not treat them
for constipation. Instead, refer them urgently to a healthcare professional with
experience in the specific aspect of child health that is causing concern.
Table 2 Key components of history-taking to diagnose idiopathic
constipation
Key
components

Timing of onset
of constipation

Findings and diagnostic clues that 'Red flag' findings and


indicate idiopathic constipation
diagnostic clues that
indicate an underlying
disorder or condition:
not idiopathic
constipation

In a child younger than 1 year:


o

Starts after a few weeks of

Reported from birth


or first few weeks

and potential
precipitating
factors

Passage of
meconium

life

Obvious precipitating factors


coinciding with the start of
symptoms: fissure, change of
diet, infections

In a child/young person older


than 1 year:

Starts after a few weeks of


life

Obvious precipitating factors


coinciding with the start of
symptoms: fissure, change of
diet, timing of potty/toilet
training or acute events such
as infections, moving house,
starting nursery/school, fears
and phobias, major change in
family, taking medicines

Normal (within 48 hours after birth


[in term baby])

Failure to pass
meconium/delay
(more than 48
hours after birth [in
term baby])

'Ribbon stools'
(more likely in a
child younger than
1 year)

No 'red flag', but


see 'amber flag'
below.

Previously
unknown or
undiagnosed
weakness in legs,
locomotor delay

Stool patterns

Growth and

general wellbeing

Symptoms in
legs/locomotor
development

of life

In a child younger than 1 year:


o

Generally well, weight and


height within normal limits

In a child/young person older


than 1 year:

Generally well, weight and


height within normal limits, fit
and active

No neurological problems in legs


(such as falling over in a
child/young person older than 1
year), normal locomotor
development

Abdomen

Diet and fluid


intake

Abdominal
distension with
vomiting

In a child younger than 1 year:


o

Changes in infant formula,


weaning, insufficient fluid
intake

In a child/young person older


than 1 year:

History of poor diet and/or


insufficient fluid intake

'Amber flag': possible idiopathic constipation


Growth and general wellbeing: Faltering growth (see recommendation 1.1.4)
Personal/familial/social factors: Disclosure or evidence that raises concerns
over possibility of child maltreatment (see recommendation 1.1.5)
1.1.3 Do a physical examination. Use table 3 to establish a positive diagnosis of
idiopathic constipation by excluding underlying causes. If a child or young person
has any 'red flag' symptoms do not treat them for constipation. Instead, refer
them urgently to a healthcare professional with experience in the specific aspect
of child health that is causing concern.
Table 3 Key components of physical examination to diagnose idiopathic
constipation
Key components

Findings and
'Red flag' findings and
diagnostic clues that diagnostic clues that indicate an
indicate idiopathic underlying disorder or
constipation
condition: not idiopathic
constipation

Inspection of perianal
area: appearance,
position, patency, etc

Normal
appearance of
anus and
surrounding area

Abdominal
examination

Soft abdomen.

Flat or distension
that can be
explained
because of age or
excess weight

Spine/lumbosacral

Normal

Abnormal
appearance/position/patency
of anus: fistulae, bruising,
multiple fissures, tight or
patulous anus, anteriorly
placed anus, absent anal wink
Gross abdominal distension

Abnormal: asymmetry or

region/gluteal
examination

appearance of the
skin and
anatomical
structures of
lumbosacral/glute
al regions

Lower limb
neuromuscular
examination including
tone and strength

Lower limb
neuromuscular
examination: reflexes
(perform only if 'red
flags' in history or
physical examination
suggest new onset
neurological
impairment)

flattening of the gluteal


muscles, evidence of sacral
agenesis, discoloured skin,
naevi or sinus, hairy patch,
lipoma, central pit (dimple that
you can't see the bottom of),
scoliosis

Normal gait.
Normal tone and
strength in lower
limbs

Deformity in lower limbs such


as talipes

Abnormal neuromuscular
signs unexplained by any
existing condition, such as
cerebral palsy

Reflexes present
and of normal
amplitude

Abnormal reflexes

1.1.4 If the history-taking and/or physical examination show evidence of faltering


growth treat for constipation and test for coeliac disease [2] and hypothyroidism.
1.1.5 If either the history-taking or the physical examination show evidence of
possible maltreatment treat for constipation and refer to NICE guidance on 'When
to suspect child maltreatment', NICE clinical guideline 89 (2009) [3].
1.1.6 If the physical examination shows evidence of perianal streptococcal
infection, treat for constipation and also treat the infection.
1.1.7 Inform the child or young person and his or her parents or carers of a
positive diagnosis of idiopathic constipation and also that underlying causes have
been excluded by the history and/or physical examination. Reassure them that
there is a suitable treatment for idiopathic constipation but that it may take
several months for the condition to be resolved.
1.2 Digital rectal examination

1.2.1 A digital rectal examination should be undertaken only by healthcare


professionals competent to interpret features of anatomical abnormalities or
Hirschsprung's disease.
1.2.2 If a child younger than 1 year has a diagnosis of idiopathic constipation that
does not respond to optimum treatment within 4 weeks, refer them urgently to a
healthcare professional competent to perform a digital rectal examination and
interpret features of anatomical abnormalities or Hirschsprung's disease.

1.2.3 Do not perform a digital rectal examination in children or young people


older than 1 year with a 'red flag' (see tables 2 and 3) in the history-taking and/or
physical examination that might indicate an underlying disorder. Instead, refer
them urgently to a healthcare professional competent to perform a digital rectal
examination and interpret features of anatomical abnormalities or Hirschsprung's
disease.
1.2.4 For a digital rectal examination ensure:

privacy

informed consent is given by the child or young person, or the parent or legal
guardian if the child is not able to give it, and is documented

a chaperone is present

the child or young person's individual preferences about degree of body


exposure and gender of the examiner are taken into account

all findings are documented.

1.3 Clinical investigations

Endoscopy
1.3.1 Do not use gastrointestinal endoscopy to investigate idiopathic
constipation.
Coeliac disease and hypothyroidism
1.3.2 Test for coeliac disease[2] and hypothyroidism in the ongoing management
of intractable constipation in children and young people if requested by specialist
services.
Manometry
1.3.3 Do not use anorectal manometry to exclude Hirschsprung's disease in
children and young people with chronic constipation.
Radiography
1.3.4 Do not use a plain abdominal radiograph to make a diagnosis of idiopathic
constipation.
1.3.5 Consider using a plain abdominal radiograph only if requested by specialist
services in the ongoing management of intractable idiopathic constipation.
Rectal biopsy
1.3.6 Do not perform rectal biopsy unless any of the following clinical features of
Hirschsprung's disease are or have been present:

delayed passage of meconium (more than 48 hours after birth in term


babies)

constipation since first few weeks of life

chronic abdominal distension plus vomiting

family history of Hirschsprung's disease

faltering growth in addition to any of the previous features.

Transit studies
1.3.7 Do not use transit studies to make a diagnosis of idiopathic constipation.
1.3.8 Consider using transit studies in the ongoing management of intractable
idiopathic constipation only if requested by specialist services.
Ultrasound
1.3.9 Do not use abdominal ultrasound to make a diagnosis of idiopathic
constipation.
1.3.10 Consider using abdominal ultrasound in the ongoing management of
intractable idiopathic constipation only if requested by specialist services.
1.4 Clinical management

Disimpaction
1.4.1 Assess all children and young people with idiopathic constipation for faecal
impaction, including children and young people who were originally referred to
the relevant services because of 'red flags' but in whom there were no significant
findings following further investigations (see tables 2 and 3). Use a combination
of history-taking and physical examination to diagnose faecal impaction look for
overflow soiling and/or faecal mass palpable abdominally and/or rectally if
indicated.
1.4.2 Start maintenance therapy if the child or young person is not faecally
impacted.
1.4.3 Offer the following oral medication regimen for disimpaction if indicated:

Polyethylene glycol 3350 + electrolytes, using an escalating dose regimen


(see table 4), as the first-line treatment[4].

Polyethylene glycol 3350 + electrolytes may be mixed with a cold drink.

Add a stimulant laxative (see table 4) if polyethylene glycol 3350 +


electrolytes does not lead to disimpaction after 2 weeks.

Substitute a stimulant laxative singly or in combination with an osmotic


laxative such as lactulose (see table 4) if polyethylene glycol 3350 +
electrolytes is not tolerated.

Inform families that disimpaction treatment can initially increase symptoms of


soiling and abdominal pain.

Table 4 Laxatives: recommended doses


Laxatives

Recommended doses

Macrogols
Polyethylene glycol Paediatric formula: Oral powder: macrogol 3350

3350 + electrolytes (polyethylene glycol 3350)a 6.563 g; sodium bicarbonate 89.3


mg; sodium chloride 175.4 mg; potassium chloride 25.1
mg/sachet (unflavoured)
Disimpaction

Child under 1 year: 1 sachet daily (non-BNFC


recommended dose)

Child 15 years: 2 sachets on 1st day, then 4 sachets


daily for 2 days, then 6 sachets daily for 2 days, then 8
sachets daily (non-BNFC recommended dose)

Child 512 years: 4 sachets on 1st day, then increased


in steps of 2 sachets daily to maximum of 12 sachets
daily (non-BNFC recommended dose)

Ongoing maintenance (chronic constipation, prevention of


faecal impaction)

Child under 1 year: 1 sachet daily (non-BNFC


recommended dose)

Child 16 years: 1 sachet daily; adjust dose to produce


regular soft stools (maximum 4 sachets daily) (for
children under 2, non-BNFC recommended dose)

Child 612 years: 2 sachets daily; adjust dose to


produce regular soft stools (maximum 4 sachets daily)

Adult formula: Oral powder: macrogol 3350 (polyethylene


glycol 3350) 13.125 g; sodium bicarbonate 178.5 mg;
sodium chloride 350.7 mg; potassium chloride
46.6 mg/sachet (unflavoured)
Disimpaction

Child/young person 1218 years: 4 sachets on 1st day,


then increased in steps of 2 sachets daily to maximum
of 8 sachets daily (non-BNFC recommended dose)

Ongoing maintenance (chronic constipation, prevention of


faecal impaction)

Child/young person 1218 years: 13 sachets daily in


divided doses adjusted according to response;
maintenance, 12 sachets daily

Child 1 month to 1 year: 2.5 ml twice daily, adjusted


according to response

Child 15 years: 2.510 ml twice daily, adjusted


according to response (non-BNFC recommended dose)

Osmotic laxatives
Lactulose

Child/young person 518 years: 520 ml twice daily,


adjusted according to response (non-BNFC
recommended dose)

Stimulant laxatives
Sodium picosulfateb Non-BNFC recommended doses
Elixir (5 mg/5 ml)

Child 1 month to 4 years: 2.510 mg once a day

Child/young person 418 years: 2.520 mg once a day

Non-BNFC recommended dose


Perlesc (1 tablet = 2.5mg)

Bisacodyl

Child/young person 418 years: 2.520mg once a day

Non-BNFC recommended doses


By mouth

Child/young person 418 years: 520 mg once daily

By rectum (suppository)

Sennad

Child/young person 218 years: 510 mg once daily

Senna syrup (7.5 mg/5 ml)

Child 1 month to 4 years: 2.510 ml once daily

Child/young person 418 years: 2.520 ml once daily

Senna (non-proprietary) (1 tablet = 7.5 mg)

Docusate sodiume

Child 24 years: 2 tablets once daily

Child 46 years: 4 tablets once daily

Child/young person 618 years: 14 tablets once daily

Child 6 months2 years: 12.5 mg three times daily (use


paediatric oral solution)

Child 212 years: 12.525 mg three times daily (use


paediatric oral solution)

Child/young person 1218 years: up to 500 mg daily in


divided doses

All drugs listed above are given by mouth unless stated otherwise.
Unless stated otherwise, doses are those recommended by the British National
Formulary for Children (BNFC) 2009. Informed consent should be obtained and
documented whenever medications/doses are prescribed that are different from
those recommended by the BNFC.

At the time of publication (May 2010) Movicol Paediatric Plain is the only
macrogol licensed for children under 12 years that includes electrolytes. It does
not have UK marketing authorisation for use in faecal impaction in children under
5 years, or for chronic constipation in children under 2 years. Informed consent
should be obtained and documented. Movicol Paediatric Plain is the only
macrogol licensed for children under 12 years that is also unflavoured.
b

Elixir, licensed for use in children (age range not specified by manufacturer).
Perles not licensed for use in children under 4 years. Informed consent should be
obtained and documented.
c

Perles produced by Dulcolax should not be confused with Dulcolax tablets


which contain bisacodyl as the active ingredient
d

Syrup not licensed for use in children under 2 years. Informed consent should
be obtained and documented.
e

Adult oral solution and capsules not licensed for use in children under 12 years.
Informed consent should be obtained and documented.
1.4.4 Do not use rectal medications for disimpaction unless all oral medications
have failed and only if the child or young person and their family consent.
1.4.5 Administer sodium citrate enemas only if all oral medications for
disimpaction have failed.
1.4.6 Do not administer phosphate enemas for disimpaction unless under
specialist supervision in hospital/health centre/clinic, and only if all oral
medications and sodium citrate enemas have failed.
1.4.7 Do not perform manual evacuation of the bowel under anaesthesia unless
optimum treatment with oral and rectal medications has failed.
1.4.8 Review children and young people undergoing disimpaction within 1 week.
Maintenance therapy
1.4.9 Start maintenance therapy as soon as the child or young person's bowel is
disimpacted.
1.4.10 Reassess children frequently during maintenance treatment to ensure
they do not become reimpacted and assess issues in maintaining treatment such
as taking medicine and toileting. Tailor the frequency of assessment to the
individual needs of the child and their families (this could range from daily contact
to contact every few weeks). Where possible, reassessment should be provided
by the same person/team.
1.4.11 Offer the following regimen for ongoing treatment or maintenance therapy:

Polyethylene glycol 3350 + electrolytes as the first-line treatment[4].

Adjust the dose of polyethylene glycol 3350 + electrolytes according to


symptoms and response. As a guide for children and young people who
have had disimpaction the starting maintenance dose might be half the
disimpaction dose (see table 4).

Add a stimulant laxative (see table 4) if polyethylene glycol 3350 +


electrolytes does not work.

Substitute a stimulant laxative if polyethylene glycol 3350 + electrolytes is


not tolerated by the child or young person. Add another laxative such as
lactulose or docusate (see table 4) if stools are hard.

Continue medication at maintenance dose for several weeks after regular


bowel habit is established this may take several months. Children who are
toilet training should remain on laxatives until toilet training is well
established. Do not stop medication abruptly: gradually reduce the dose over
a period of months in response to stool consistency and frequency. Some
children may require laxative therapy for several years. A minority may
require ongoing laxative therapy.

1.5 Diet and lifestyle

1.5.1 Do not use dietary interventions alone as first-line treatment for idiopathic
constipation.
1.5.2 Treat constipation with laxatives and a combination of:

Negotiated and non-punitive behavioural interventions suited to the child or


young person's stage of development. These could include scheduled
toileting and support to establish a regular bowel habit, maintenance and
discussion of a bowel diary, information on constipation, and use of
encouragement and rewards systems.

Dietary modifications to ensure a balanced diet and sufficient fluids are


consumed.

1.5.3 Advise parents and children and young people (if appropriate) that a
balanced diet should include:

Adequate fluid intake (see table 5).

Adequate fibre. Recommend including foods with a high fibre content (such
as fruit, vegetables, high-fibre bread, baked beans and wholegrain breakfast
cereals) (not applicable to exclusively breastfed infants). Do not recommend
unprocessed bran, which can cause bloating and flatulence and reduce the
absorption of micronutrients.

Table 5 American dietary recommendations


(Institute of Medicine, 2005). Dietary reference intakes for water, potassium,
sodium chloride and sulfate. Washington DC: The National Academies Press
Total water intake per day, including
water contained in food
Infants 06
months

Water obtained from


drinks per day

700 ml
assumed to be from breast milk

712 months 800 ml

600 ml

from milk and complementary foods and


beverages
13 years

1300 ml

900 ml

48 years

1700 ml

1200 ml

Boys 913
years

2400 ml

1800 ml

Girls 913
years

2100 ml

1600 ml

Boys 1418
years

3300 ml

2600 ml

Girls 1418
years

2300 ml

1800 ml

The above recommendations are for adequate intakes and should not be
interpreted as a specific requirement. Higher intakes of total water will be
required for those who are physically active or who are exposed to hot
environments. It should be noted that obese children may also require higher
total intakes of water.
1.5.4 Provide children and young people with idiopathic constipation and their
families with written information about diet and fluid intake.
1.5.5 In children with idiopathic constipation, start a cows' milk exclusion diet only
on the advice of the relevant specialist services.
1.5.6 Advise daily physical activity that is tailored to the child or young person's
stage of development and individual ability as part of ongoing maintenance in
children and young people with idiopathic constipation.
1.6 Psychological interventions

1.6.1 Do not use biofeedback for ongoing treatment in children and young people
with idiopathic constipation.
1.6.2 Do not routinely refer children and young people with idiopathic
constipation to a psychologist or child and adolescent mental health services
unless the child or young person has been identified as likely to benefit from
receiving a psychological intervention.
1.7 Antegrade colonic enema procedure

1.7.1 Refer children and young people with idiopathic constipation who still have
unresolved symptoms on optimum management to a paediatric surgical centre to
assess their suitability for an antegrade colonic enema (ACE) procedure.
1.7.2 Ensure that all children and young people who are referred for an ACE
procedure have access to support, information and follow-up from paediatric

healthcare professionals with experience in managing children and young people


who have had an ACE procedure.
1.8 Information and support

1.8.1 Provide tailored follow-up to children and young people and their parents or
carers according to the child or young person's response to treatment, measured
by frequency, amount and consistency of stools. Use the Bristol Stool Form
Scale to assess this (see appendix D). This could include:

telephoning or face-to-face talks

giving detailed evidence-based information about their condition and its


management, using, for example, NICE's Information for the public for this
guideline

giving verbal information supported by (but not replaced by) written or


website information in several formats about how the bowels work,
symptoms that might indicate a serious underlying problem, how to take their
medication, what to expect when taking laxatives, how to poo, origins of
constipation, criteria to recognise risk situations for relapse (such as
worsening of any symptoms, soiling etc.) and the importance of continuing
treatment until advised otherwise by the healthcare professional.

1.8.2 Offer children and young people with idiopathic constipation and their
families a point of contact with specialist healthcare professionals, including
school nurses, who can give ongoing support.
1.8.3 Healthcare professionals should liaise with school nurses to provide
information and support, and to help school nurses raise awareness of the issues
surrounding constipation with children and young people and school staff.
1.8.4 Refer children and young people with idiopathic constipation who do not
respond to initial treatment within 3 months to a practitioner with expertise in the
problem.

Question 5 of 231

What is the carrier rate of cystic fibrosis in the UK?


1 in 5

1 in 10

1 in 25

1 in 100

1 in 500

Next question

Cystic fibrosis
Cystic fibrosis (CF) is an autosomal recessive disorder causing increased viscosity of secretions (e.g. lungs and pancreas). It is due to a
defect in the cystic fibrosis transmembrane conductance regulator gene (CFTR), which codes a cAMP-regulated chloride channel
In the UK 80% of CF cases are due to delta F508 on the long arm of chromosome 7. Cystic fibrosis affects 1 per 2500 births, and the
carrier rate is c. 1 in 25
Organisms which may colonise CF patients

Staphylococcus aureus
Pseudomonas aeruginosa
Burkholderia cepacia*
Aspergillus

*previously known as Pseudomonas cepacia

Question 6 of 231

A 10-month-old boy is brought to surgery as his mother has noticed some noisy breathing. For the past 2-3 days he has had a runny nose
and has felt hot. There is no past medical history of note and he is currently feeding satisfactorily. On examination temperature is 37.6C,
respiratory rate is 36 / min and there is no intercostal recession noted. Chest auscultation reveals a mild expiratory wheeze bilaterally with
the occasional fine crackle. What is the most appropriate management?
Amoxicillin + chest x-ray + review

Paracetamol + review

Bronchodilator via spacer device + course of prednisolone + review

Admit

Bronchodilator via spacer device + review

Next question

SIGN guidelines do not support the use of bronchodilators in children with bronchiolitis.

Bronchiolitis
Bronchiolitis is a condition characterised by acute bronchiolar inflammation. Respiratory syncytial virus (RSV) is the pathogen in 75-80% of
cases. SIGN released guidelines on bronchiolitis in 2006. Please see the link for more details.
Epidemiology

most common cause of a serious lower respiratory tract infection in < 1yr olds (90% are 1-9 months, with a peak incidence of 36 months). Maternal IgG provides protection to newborns against RSV
higher incidence in winter

Basics

respiratory syncytial virus (RSV) is the pathogen in 75-80% of cases


other causes: mycoplasma, adenoviruses
may be secondary bacterial infection
more serious if bronchopulmonary dysplasia (e.g. Premature), congenital heart disease or cystic fibrosis

Features

coryzal symptoms (including mild fever) precede:


dry cough
increasing breathlessness
wheezing, fine inspiratory crackles (not always present)
feeding difficulties associated with increasing dyspnoea are often the reason for hospital admission

SIGN suggested the following criteria for referral to hospital

poor feeding (< 50% normal)


lethargy
apnoea
respiratory rate > 70/min
nasal flaring or grunting
severe chest wall recession
cyanosis
oxygen saturation < 94%
uncertainty regarding diagnosis

Investigation

immunofluorescence of nasopharyngeal secretions may show RSV

Management is largely supportive

humidified oxygen is given via a head box

Question 7 of 231

An emergency call is put out as a 9-year-old boy has collapsed in the waiting room. The receptionists have already dialled 999. He does not
respond to stimulation and no signs of respiration can be found after 10 seconds. There is no obvious foreign body in the mouth. What is
the most appropriate next step?
Start chest compressions/ventilations at a ratio of 15:2

Give 2 rescue breaths

Give 5 rescue breaths

Start chest compressions/ventilations at a ratio of 30:2

Check for a carotid pulse

Next question

Current paediatric basic life support guidelines do not yet reflect the change in emphasis away from rescue breaths in adults. This partly
reflects the fact that the majority of paediatric arrests are secondary to airway or breathing problems.

Paediatric basic life support


The 2010 Resuscitation Council guidelines made the following changes to paediatric basic life support

compression:ventilation ratio: lay rescuers should use a ratio of 30:2. If there are two or more rescuers with a duty to respond
then a ratio of 15:2 should be used
age definitions: an infant is a child under 1 year, a child is between 1 year and puberty

Key points of algorithm (please see link attached for more details)

unresponsive?
shout for help
open airway
look, listen, feel for breathing
give 5 rescue breaths
check for signs of circulation
15 chest compressions:2 rescue breaths (see above)

Question 8 of 231

A mother comes to surgery with her 6-year-old son. During the MMR scare she decided not to have her son immunised. However, due to a
recent measles outbreak she asks if he can still receive the MMR vaccine. What is the most appropriate action?
Arrange for measles immunoglobulin to be given

Cannot vaccinate at this age as live vaccine

Give separate measles vaccine

Give MMR with repeat dose in 3 months

Give MMR with repeat dose in 5 years

Next question

The Green Book recommends allowing 3 months between doses to maximise the response rate. A period of 1 month is considered
adequate if the child is greater than 10 years of age. In an urgent situation (e.g. an outbreak at the child's school) then a shorter period of 1
month can be used in younger children.

MMR vaccine
Children in the UK receive two doses of the Measles, Mumps and Rubella (MMR) vaccine before entry to primary school. This currently
occurs at 12-15 months and 3-4 years as part of the routine immunisation schedule
Contraindications to MMR

severe immunosuppression
allergy to neomycin
children who have received another live vaccine by injection within 4 weeks
pregnancy should be avoided for at least 1 month following vaccination
immunoglobulin therapy within the past 3 months (there may be no immune response to the measles vaccine if antibodies are
present)

Adverse effects

malaise, fever and rash may occur after the first dose of MMR. This typically occurs after 5-10 days and lasts around 2-3 days

Question 9 of 231

A 3-month-old boy is brought to surgery as his mother is concerned he is 'floppy'. Examination confirms a greater degree of hypotonia than
would be expected at this age. Which one of the following is least likely to be responsible?
Cystic fibrosis

Acutely unwell child

Prader-Willi syndrome

Down's syndrome

Cerebral palsy

Next question

Cystic fibrosis is not a cause of hypotonia in infancy

Hypotonia
Hypotonia, or floppiness, may be central in origin or related to nerve and muscle problems. An acutely ill child (e.g. septicaemic) may be
hypotonic on examination. Hypotonia associated with encephalopathy in the newborn period is most likely caused by hypoxic ischaemic
encephalopathy
Central causes

Down's syndrome
Prader-Willi syndrome
hypothyroidism
cerebral palsy (hypotonia may precede the development of spasticity)

Neurological and muscular problems

spinal muscular atrophy


spina bifida
Guillain-Barre syndrome
myasthenia gravis
muscular dystrophy
myotonic dystrophy

Question 10 of 231

Which one of the following statements regarding scabies is false?


All members of the household should be treated

Typically affects the fingers, interdigital webs and flexor aspects of the wrist in adults

Scabies causes a delayed type IV hypersensitivity reaction

Patients who complain of pruritus 4 weeks following treatment should be retreated

Malathion is suitable for the eradication of scabies

Next question

It is normal for pruritus to persist for up to 4-6 weeks post eradication

Scabies
Scabies is caused by the mite Sarcoptes scabiei and is spread by prolonged skin contact. It typically affects children and young adults.
The scabies mite burrows into the skin, laying its eggs in the stratum corneum. The intense pruritus associated with scabies is due to a
delayed type IV hypersensitivity reaction to mites/eggs which occurs about 30 days after the initial infection.
Features

widespread pruritus
linear burrows on the side of fingers, interdigital webs and flexor aspects of the wrist
in infants the face and scalp may also be affected
secondary features are seen due to scratching: excoriation, infection

Management

permethrin 5% is first-line
malathion 0.5% is second-line
give appropriate guidance on use (see below)
pruritus persists for up to 4-6 weeks post eradication

Patient guidance on treatment (from Clinical Knowledge Summaries)

avoid close physical contact with others until treatment is complete


all household and close physical contacts should be treated at the same time, even if asymptomatic

launder, iron or tumble dry clothing, bedding, towels, etc., on the first day of treatment to kill off mites.

The BNF advises to apply the insecticide to all areas, including the face and scalp, contrary to the manufacturer's recommendation.
Patients should be given the following instructions:

apply the insecticide cream or liquid to cool, dry skin


pay close attention to areas between fingers and toes, under nails, armpit area, creases of the skin such as at the wrist and
elbow
allow to dry and leave on the skin for 8-12 hours for permethrin, or for 24 hours for malathion, before washing off
reapply if insecticide is removed during the treatment period, e.g. If wash hands, change nappy, etc
repeat treatment 7 days later

Crusted (Norwegian) scabies


Crusted scabies is seen in patients with suppressed immunity, especially HIV.

Image used on license from DermNet NZ

The crusted skin will be teeming with hundreds of thousands of organisms.


Ivermectin is the treatment of choice and isolation is essential

Question 11 of 231

Which one of the following is a risk factor for developmental dysplasia of the hip?
Vertex presentation

Oligohydramnios

Male sex

Maternal diabetes mellitus

Polyhydramnios

Next question

Developmental dysplasia of the hip


Developmental dysplasia of the hip (DDH) is gradually replacing the old term 'congenital dislocation of the hip' (CDH). It affects around 13% of newborns.
Risk factors

female sex: 6 times greater risk


breech presentation
positive family history
firstborn children
oligohydramnios
birth weight > 5 kg
congenital calcaneovalgus foot deformity

DDH is slightly more common in the left hip. Around 20% of cases are bilateral.
Clinical examination is made using the Barlow and Ortolani tests:

Barlow test: attempts to dislocate an articulated femoral head


Ortolani test: attempts to relocate a dislocated femoral head

Ultrasound is used to confirm the diagnosis if clinically suspected


Management

most unstable hips will spontaneously stabilise by 3-6 weeks of age


Pavlik harness (flexion-abduction orthosis) in children younger than 4-5 months
older children may require surgery

Question 12 of 231

Which one of the following vaccines is not given routinely in the first 6 months of life?
MMR

Inactivated polio vaccine

Pneumococcal Conjugate Vaccine

Tetanus vaccine

Meningitis C vaccine

Next question

Immunisation schedule
The current UK immunisation schedule is as follows. Please note that this table includes the changes announced in 2010 which merged the
12 and 13 month visits into one.
Age

Recommended immunisations

At birth

BCG / hepatitis B vaccine if risk factors (see below)

2 months

DTaP/IPV/Hib + PCV
Oral rotavirus vaccine

3 months

DTaP/IPV/Hib + Men C
Oral rotavirus vaccine

4 months

DTaP/IPV/Hib + PCV

12-13 months Hib/Men C + MMR + PCV


2-3 years

Flu vaccine (annual)

3-4 years

MMR + DTaP/IPV

12-13 years HPV vaccination for girls


13-18 years DT/IPV + MenC

At birth the BCG vaccine should be given if the baby is deemed at risk of tuberculosis (e.g. Tuberculosis in the family in the past 6 months).
Hepatitis B vaccine should be given at birth if the mother is HBsAg +ve.
Key

DTaP = Diphtheria, Tetanus, acellular Pertussis vaccine


IPV = Inactivated Polio Vaccine
Hib = Haemophilus influenzae B vaccine
PCV = Pneumococcal Conjugate Vaccine
Men C = Meningococcal C vaccine
MMR = Measles, Mumps, Rubella vaccine
DT = Diphtheria, Tetanus vaccine
HPV = Human Papilloma Vaccine

Question 13 of 231

A 10-month-old boy is brought to surgery. Around 4 days ago he developed a fever after being irritable the previous day. The fever settled
after around 3 days but following this he developed a rash, which prompted his mother to bring him to surgery. He is taking around 75% of
his normal feeds, is producing wet nappies and has had two episodes of loose stools. On examination he is alert, temperature is 37.0C,
chest is clear, ears/throat unremarkable. There are a number of blanching, rose pink macules present on his trunk. What is the most likely
diagnosis?
Rubella

Chickenpox

Roseola infantum

Pityriasis rosea

Measles

Next question

Roseola infantum - fever followed later by rash

Roseola infantum
Roseola infantum (also known as exanthem subitum, occasionally sixth disease) is a common disease of infancy caused by the human
herpes virus 6 (HHV6). It has an incubation period of 5-15 days and typically affects children aged 6 months to 2 years.
Features

high fever: lasting a few days, followed by a


maculopapular rash
febrile convulsions occur in around 10-15%
diarrhoea and cough are also commonly seen

Other possible consequences of HHV6 infection

aseptic meningitis
hepatitis

Question 14 of 231

A 3-year-old child is brought to surgery as her mother has noticed that she is 'cross-eyed'. The corneal light reflection test confirms this.
What is the most appropriate management?
Advise that referral to secondary care should be delayed until 5 years of age, when surgery may be
contemplated
Refer to ophthalmology

Refer to paediatric physiotherapy for eye movement exercises

Reassure mother that the majority of squints improve with age

Advise the mother to restrict the amount of television she watches

Next question

Refer children with a squint immediately to ophthalmology

This child needs to be reviewed by an ophthalmologist and orthoptist

Squint
Squint (strabismus) is characterised by misalignment of the visual axes. Squints may be divided into concomitant (common) and paralytic
(rare)
Concomitant

Paralytic

Due to imbalance in extraocular muscles


Due to paralysis of extraocular muscles
Convergent is more common than divergent

Detection of a squint may be made by the corneal light reflection test - holding a light source 30cm from the child's face to see if the light
reflects symmetrically on the pupils
The cover test is used to identify the nature of the squint

ask the child to focus on a object


cover one eye
observe movement of uncovered eye
cover other eye and repeat test

Management

eye patches may help prevent amblyopia


referral to secondary care is appropriate

Question 15 of 231

A 2-month-old girl is brought to surgery with poor feeding and vomiting. Mother reports that her urine has a strong smell. A urinary tract
infection is suspected. What is the most appropriate management?
Urine culture + empirical oral antibiotic therapy + ultrasound as soon as possible

Urine culture + empirical oral antibiotic therapy + outpatient referral to paediatrics

Urine culture + oral antibiotics based upon results + ultrasound within 6 weeks

Urine culture + empirical oral antibiotic therapy + ultrasound within 6 weeks

Refer immediately to hospital

Next question

Urinary tract infection in children: features, diagnosis and management


Urinary tract infections (UTI) are more common in boys until 3 months of age (due to more congenital abnormalities) after which the
incidence is substantially higher in girls. At least 8% of girls and 2% of boys will have a UTI in childhood

Presentation in childhood depends on age:

infants: poor feeding, vomiting, irritability


younger children: abdominal pain, fever, dysuria
older children: dysuria, frequency, haematuria
features which may suggest an upper UTI include: temperature > 38C, loin pain/tenderness

NICE guidelines for checking urine sample in a child

if there are any symptoms or signs suggestive or a UTI


with unexplained fever of 38C or higher (test urine after 24 hours at the latest)
with an alternative site of infection but who remain unwell (consider urine test after 24 hours at the latest)

Urine collection method

clean catch is preferable


if not possible then urine collection pads should be used
cotton wool balls, gauze and sanitary towels are not suitable
invasive methods such as suprapubic aspiration should only be used if non-invasive methods are not possible

Management

infants less than 3 months old should be referred immediately to a paediatrician


children aged more than 3 months old with an upper UTI should be considered for admission to hospital. If not admitted oral
antibiotics such as cephalosporin or co-amoxiclav should be given for 7-10 days
children aged more than 3 months old with a lower UTI should be treated with oral antibiotics for 3 days according to local
guidelines, usually trimethoprim, nitrofurantoin, cephalosporin or amoxicillin. Parents should be asked to bring the children back
if they remain unwell after 24-48 hours
antibiotic prophylaxis is not given after the first UTI but should be considered with recurrent UTIs

Urinary tract infection - children - Summary

Urinary tract infection (UTI) is illness caused by micro-organisms in the urinary tract.
Most UTIs are caused by a single species of bacteria from the gastrointestinal tract.
Common organisms causing UTI in children include Escherichia coli (about 75% or more
of cases), Klebsiella species, and Staphylococcus saprophyticus.
Around 1 in 10 girls and 1 in 30 boys will have had a UTI by the time they turn 16 years
of age.
Most UTIs are not associated with any risk factor.
The prognosis after childhood UTI is generally excellent and a course of antibiotics
cures UTI in most infants and children. However, UTI may recur and rarely, long-term
complications (such as renal scarring and hypertension) may occur.
In a child less than 3 months old, UTI should be suspected if there is a combination of:
o Fever (without obvious cause), vomiting, irritability, lethargy.
o Poor feeding, failure to thrive.
o Failure to respond adequately to appropriate treatment of another presumed cause
of the illness.
o Abdominal pain, jaundice, haematuria, offensive urine.

An urgent admission should be made in all infants less than 3 months of age with
suspected UTI. Urine testing is not necessary in primary care.
In a child over 3 months of age, UTI should be suspected if there is a combination of:
o Frequency, dysuria.
o Fever without an obvious cause (if the temperature is greater than 38C urine
should be tested within 24 hours).
o Fever with a presumed cause (other than UTI) but poor response to treatment (if
there is doubt about the cause of fever urine should be tested within 24 hours).
o Abdominal pain, loin tenderness.
o Voluntary withholding or incontinence of urine or faeces.
o Vomiting, poor feeding.
o Malaise, lethargy, irritability.
o Haematuria, offensive urine, cloudy urine.
o Failure to thrive.
Management of suspected UTI in a child over 3 months of age involves:
o Assessing the risk of serious illness and arranging a referral (or admission) with the
appropriate urgency. This should include taking a full history; measurement of blood
pressure; abdominal palpation; inspection of external genitalia, anus and urethral
meatus; assessment of lower back and limbs, and examination of urine.
o Considering the need for antibiotics. A urine specimen should be sent for culture
and sensitivities prior to starting antibiotics.
o Arranging a review (e.g. after 48 hours) to ensure response to treatment and to
reassess the antibiotic choice, if applicable.
o Offering appropriate advice to parents or carers (such as to return for assessment if
the child is still unwell after 2448 hours of treatment or if they suspect a repeat
UTI).
Infants/children aged over 3 months with suspected UTI are at:
o High risk of serious illness if they are systemically unwell, dehydrated, or vomiting
and cannot tolerate oral fluids and medication or have a history or clinical features
suggesting urinary tract obstruction.
o Low risk of serious illness if temperature is less than 38C with no history of fever
and there is no loin pain/tenderness.
o Intermediate risk of serious illness if they do not satisfy the criteria for being at high
or low risk.

Question 16 of 231

A 4-week-old child is brought to clinic with a red rash on her scalp associated with yellow flakes. What is the most likely diagnosis?
Irritant contact dermatitis

Seborrhoeic dermatitis

Psoriasis

Atopic eczema

Candidiasis

Next question

Seborrhoeic dermatitis in children


Seborrhoeic dermatitis is a relatively common skin disorder seen in children. It typically affects the scalp ('Cradle cap'), nappy area, face
and limb flexures.
Cradle cap is an early sign which may develop in the first few weeks of life. It is characterised by an erythematous rash with coarse yellow
scales.
Management depends on severity

mild-moderate: baby shampoo and baby oils


severe: mild topical steroids e.g. 1% hydrocortisone

Seborrhoeic dermatitis in children tends to resolve spontaneously by around 8 months of age

Question 17 of 231

What is the most common identified trigger of anaphylaxis in children?


Hair dye

Wasp venom

Medication

Food

Blood products

Next question

Anaphylaxis
Anaphylaxis may be defined as a severe, life-threatening, generalised or systemic
hypersensitivity reaction.
Anaphylaxis is one of the few times when you would not have time to look up the dose of a medication. The Resuscitation Council
guidelines on anaphylaxis have recently been updated. Adrenaline is by far the most important drug in anaphylaxis and should be given as
soon as possible. The recommended doses for adrenaline, hydrocortisone and chlorphenamine are as follows:
Adrenaline

Hydrocortisone Chlorphenamine

Adrenaline

Hydrocortisone Chlorphenamine

< 6 months

150 mcg (0.15ml 1 in 1,000)25 mg

250 mcg/kg

6 months - 6 years

150 mcg (0.15ml 1 in 1,000)50 mg

2.5 mg

6-12 years

300 mcg (0.3ml 1 in 1,000) 100 mg

5 mg

Adult and child > 12 years 500 mcg (0.5ml 1 in 1,000) 200 mg

10 mg

Adrenaline can be repeated every 5 minutes if necessary. The best site for IM injection is the anterolateral aspect of the middle third of the
thigh.
Common identified causes of anaphylaxis

food (e.g. Nuts) - the most common cause in children


drugs
venom (e.g. Wasp sting)

Question 18 of 231

A 2-month-old girl is brought in by her mother. She was breastfed for the first two weeks of life before being switched to formula. For the
past six weeks she has experienced a number of problems including regurgitation, vomiting, diarrhoea and eczema. Despite these
problems she has kept to the 50th centile for weight. Clinical examination is unremarkable other than some dry skin on her torso. What is
the most likely diagnosis?
Coeliac disease

Lactose intolerance

Cow's milk protein intolerance

Gastro-oesophageal reflux disease

Cystic fibrosis

Next question

The emergence of symptoms following the introduction of formula is very suggestive of cow's milk protein intolerance.

Cow's milk protein intolerance/allergy


Cow's milk protein intolerance/allergy (CMPI/CMPA) occurs in around 3-6% of all children and typically presents in the first 3 months of life
in formula fed infants, although rarely it is seen in exclusively breastfed infants.
Both immediate (IgE mediated) and delayed (non-IgE mediated) reactions are seen. The term CMPA is usually used for immediate
reactions and CMPI for mild-moderate delayed reactions.
Features

regurgitation and vomiting


diarrhoea
urticaria, atopic eczema
'colic' symptoms: irritability, crying

wheeze, chronic cough


rarely angioedema and anaphylaxis may occur

Diagnosis is often clinical (e.g. improvement with cow's milk protein elimination). Investigations include:

skin prick/patch testing


total IgE and specific IgE (RAST) for cow's milk protein

Management
If the symptoms are severe (e.g. failure to thrive) refer to a paediatrician.
Management if formula-fed

extensive hydrolysed formula (eHF) milk is the first-line replacement formula for infants with mild-moderate symptoms
amino acid-based formula (AAF) in infants with severe CMPA or if no response to eHF
around 10% of infants are also intolerant to soya milk

Management if breast-fed

continue breastfeeding
eliminate cow's milk protein from maternal diet
use eHF milk when breastfeeding stops, until 12 months of age and at least for 6 months

CMPI usually resolves by 1-2 years of age. A challenge is often performed in the hospital setting as anaphylaxis can occur.

Question 19 of 231

A 9-year-old boy who is having an asthma attack is brought to surgery. Which one of the following findings would be categorise the asthma
attack as life-threatening, rather than just severe, according to the British Thoracic Society guidelines?
Heart rate 140 bpm

Peak flow 30% of best

Use of accessory neck muscles

Sats 93%

Respiratory rate 36/min

Next question

A PEF of 30% of best would indicate a life-threatening attack.

Asthma in children: assessment of acute attacks

The 2014 BTS/SIGN guidelines suggest the following criteria are used to assess the severity of asthma in general practice:
Children between 2 and 5 years of age
Moderate attack

Severe attack

Life-threatening attack

SpO2 <92%
SpO2 < 92%
Silent chest
Too breathless to talk or feed
SpO2 > 92%
Poor respiratory effort
Heart rate > 140/min
No clinical features of severe asthma
Agitation
Respiratory rate > 40/min
Altered consciousness
Use of accessory neck muscles
Cyanosis

Children greater than 5 years of age


Attempt to measure PEF in all children aged > 5 years.
Moderate attack

Severe attack

Life-threatening attack

SpO2 < 92%


SpO2 < 92%
SpO2 > 92%
PEF 33-50% best or predicted
PEF < 33% best or predicted
PEF > 50% best or predicted Can't complete sentences in one breath or too breathless to talk or feed Silent chest
No clinical features of
Heart rate > 125/min
Poor respiratory effort
severe asthma
Respiratory rate > 30/min
Altered consciousness
Use of accessory neck muscles
Cyanosis

Question 20 of 231

A 6-year-old boy is brought to surgery by his mother. For the past 2 months he has been complaining of pain in his shins and ankles at
night-time. His symptoms are bilateral he is otherwise well. There is no family history of note. Clinical examination is unremarkable. What is
the most likely diagnosis?
Legg-Calve-Perthes disease

Osteoid osteoma

Growing pains

Talipes equinovarus

Osteochondritis dissecans

Next question

Growing pains
A common presentation in General Practice is a child complaining of pain in the legs with no obvious cause. Such presentations, in the
absence of any worrying features, are often attributed to 'growing pains'. This is a misnomer as the pains are often not related to growth the current term used in rheumatology is 'benign idiopathic nocturnal limb pains of childhood'
Growing pains are equally common in boys and girls and occur in the age range of 3-12 years.

Features of growing pains

never present at the start of the day after the child has woken
no limp
no limitation of physical activity
systemically well
normal physical examination
motor milestones normal
symptoms are often intermittent and worse after a day of vigorous activity

Question 21 of 231

A 7-year-old child is brought to surgery by her father after he collected her from school. She was unwell in the morning with a fever and
cough but has become gradually worse during the day. His father is panicked because he has noticed a red rash on her legs which doesn't
blanch following the 'tumbler test'. The girl is now also complaining of a headache and cool hands and feet. On examination she has a
petechial rash on her lower legs and is pyrexial with a temperature of 38.4C. Her father reports she is allergic to penicillin. On reviewing
the records you note she developed a rash and vomiting after been given amoxicillin for otitis media 4 years ago. What is the most
appropriate next step?
Phone 999 + administer intramuscular benzylpenicillin

Reassure petechiae are likely secondary to cough but admit to paediatrics for further tests.=

Phone the paediatric registrar for advice

Phone 999

Phone 999 + administer oral clarithromycin 250mg stat

Next question

This girl has meningococcal septicaemia until proven otherwise. The question looks at the urgency of your response and also how you
would deal with a history of 'allergy' to penicillin. NICE state the following:

Withhold benzylpenicillin only in children and young people who have a clear history of anaphylaxis after a
previous dose; a history of a rash following penicillin is not a contraindication.
A cough would not explain petechiae in the legs.

Meningococcal septicaemia
Meningococcal septicaemia is a frightening condition for patients, parents and doctors. It is associated with a high morbidity and mortality
unless treated early - meningococcal disease is the leading infectious cause of death in early childhood. A high index of suspicion is
therefore needed. Much of the following is based on the 2010 NICE guidelines (please see link).
Presentation of meningococcal disease:

15% - meningitis
25% - septicaemia
60% - a combination of meningitis and septicaemia

NICE divide the features of meningococcal septicaemia into:

common non-specific symptoms/signs e.g. fever, vomiting, lethargy


less common non-specific symptoms/signs e.g. Chills, shivering
more specific symptoms/signs e.g. Non-blanching rash, altered mental state, capillary refill time more than 2 seconds, unusual
skin colour, shock, hypotension, leg pain, cold hands/feet

Management if suspected meningococcal septicaemia

give intramuscular or intravenous benzylpenicillin unless there is a history of anaphylaxis (do not give if this will delay hospital
transfer)
NICE recommend phoning 999

Guidance

1.1 Bacterial meningitis and meningococcal septicaemia in children and young people
symptoms, signs and initial assessment
1.2 Pre-hospital management of suspected bacterial meningitis and meningococcal
septicaemia
1.3 Diagnosis in secondary care
1.4 Management in secondary care
1.5 Long-term management

1 Guidance
The following guidance is based on the best available evidence. The full
guideline gives details of the methods and the evidence used to develop the
guidance.
This guideline assumes that fever in children younger than 5 years will be
managed according to Feverish illness in children (NICE clinical guideline 47)
until bacterial meningitis or meningococcal septicaemia is suspected.
1.1 Bacterial meningitis and meningococcal septicaemia in children and young
people symptoms, signs and initial assessment

1.1.1 Consider bacterial meningitis and meningococcal septicaemia in children


and young people who present with the symptoms and signs in table 1 (below).

Be aware that:
o

some children and young people will present with mostly non-specific
symptoms or signs, and the conditions may be difficult to distinguish
from other less important (viral) infections presenting in this way

children and young people with the more specific symptoms and signs
are more likely to have bacterial meningitis or meningococcal
septicaemia, and the symptoms and signs may become more severe
and more specific over time.

Recognise shock (see table 1 below) and manage urgently in secondary


care.

Table 1 Symptoms and signs of bacterial meningitis and meningococcal


septicaemia
Symptom/sign Bacterial
Meningococcal Meningococca Notes
meningitis
disease
l septicaemia
(meningococca (meningococca
l meningitis
l meningitis
and meningitis and/or
caused by
meningococcal
other bacteria) septicaemia)
Common non-specific symptoms/signs

Vomiting/nausea

Irritable/unsettle
d

Ill appearance

Refusing
food/drink

Headache

Muscle
ache/joint pain

Respiratory
symptoms/signs
or breathing
difficulty

Fever

Lethargy

Less common non-specific symptoms/signs


Chills/shivering

Diarrhoea,
abdominal
pain/distension

NK

Sore
throat/coryza or
other ear, nose

NK

Not always
present,
especially in
neonates

and throat
symptoms/signs
More specific symptoms/signs
Non-blanching
rash

Stiff neck

NK

Altered mental
state

Capillary refill
time more than
2 seconds

NK

Unusual skin
colour

NK

Shock

Hypotension

NK

Leg pain

NK

Cold hands/feet NK

Back rigidity

NK

Bulging
fontanelle

NK

Photophobia

Kernig's sign

Brudzinski's sign

Be aware that
a rash may
be less visible
in darker skin
tones check
soles of feet,
palms of
hands and
conjunctivae

Includes
confusion,
delirium and
drowsiness,
and impaired
consciousnes
s

Only relevant
in children
aged under
2 years

Unconsciousnes
s

Toxic/moribund
state

Focal
neurological
deficit including
cranial nerve
involvement and
abnormal pupils

Paresis

Seizures
Signs of shock

Capillary refill time more than 2 seconds

Unusual skin colour

Tachycardia and/or hypotension

Respiratory symptoms or breathing difficulty

Leg pain

Cold hands/feet

Toxic/moribund state

Altered mental state/decreased conscious level

Poor urine output

symptom/sign present
X symptom/sign not present
NK not known if a symptom/sign is present (not reported in the evidence)
1.1.2 Be alert to the possibility of bacterial meningitis or meningococcal
septicaemia when assessing children or young people with acute febrile illness.
1.1.3 Healthcare professionals should be aware that classical signs of meningitis
(neck stiffness, bulging fontanelle, high-pitched cry) are often absent in infants
with bacterial meningitis[3].
1.1.4 Be aware that children and young people with bacterial meningitis
commonly present with non-specific symptoms and signs, including fever,
vomiting, irritability, and upper respiratory tract symptoms. Some children with
bacterial meningitis present with seizures[4].
1.1.5 Consider other non-specific features of the child's or young person's
presentation, such as:

the level of parental or carer concern (particularly compared with previous


illness in the child or young person or their family),

how quickly the illness is progressing, and

clinical judgement of the overall severity of the illness.

1.1.6 In children and young people with suspected bacterial meningitis or


meningococcal septicaemia, undertake and record physiological observations of
heart rate, respiratory rate, oxygen saturations, blood pressure, temperature,
perfusion (capillary refill) and neurological assessment (for example the Alert,
Voice, Pain, Unresponsive [AVPU] scale) at least hourly.
1.1.7 Healthcare professionals should be trained in the recognition and
management of meningococcal disease.
1.1.8 Notify a proper officer of the local authority urgently on suspicion of
meningitis or meningococcal septicaemia. This is a legal requirement under the
Health Protection (Notification) Regulations 2010 [5][6].
1.1.9 Be aware of 'Guidance for Public Health Management of Meningococcal
Disease in the UK' (Health Protection Agency Meningococcus Forum, 2006) [7].
1.2 Pre-hospital management of suspected bacterial meningitis and meningococcal
septicaemia

1.2.1 Primary care healthcare professionals should transfer children and young
people with suspected bacterial meningitis or suspected meningococcal
septicaemia to secondary care as an emergency by telephoning 999.
Suspected bacterial meningitis without non-blanching rash
1.2.2 Transfer children and young people with suspected bacterial meningitis
without non-blanching rash directly to secondary care without giving parenteral
antibiotics.
1.2.3 If urgent transfer to hospital is not possible (for example, in remote
locations or adverse weather conditions), administer antibiotics to children and
young people with suspected bacterial meningitis.
Suspected meningococcal disease (meningitis with non-blanching rash or
meningococcal septicaemia)
1.2.4 Give parenteral antibiotics (intramuscular or intravenous benzylpenicillin) at
the earliest opportunity, either in primary or secondary care, but do not delay
urgent transfer to hospital to give the parenteral antibiotics.
1.2.5 Withhold benzylpenicillin only in children and young people who have a
clear history of anaphylaxis after a previous dose; a history of a rash following
penicillin is not a contraindication.
1.3 Diagnosis in secondary care

1.3.1 Perform a very careful examination for signs of meningitis or septicaemia in


children and young people presenting with petechial rashes (see table 1 above).

Investigation and management in children and young people with petechial


rash
1.3.2 Give intravenous ceftriaxone immediately to children and young people with
a petechial rash if any of the following occur at any point during the assessment
(these children are at high risk of having meningococcal disease):

petechiae start to spread

the rash becomes purpuric

there are signs of bacterial meningitis (see table 1 above)

there are signs of meningococcal septicaemia (see table 1 above)

the child or young person appears ill to a healthcare professional.

1.3.3 If a child or young person has an unexplained petechial rash and fever (or
history of fever) carry out the following investigations:

full blood count

C-reactive protein (CRP)

coagulation screen

blood culture

whole-blood polymerase chain reaction (PCR) for N meningitidis

blood glucose

blood gas.

1.3.4 In a child or young person with an unexplained petechial rash and fever (or
history of fever) but none of the high-risk clinical manifestations (see table 1
above):

Treat with intravenous ceftriaxone immediately if the CRP and/or white blood
cell count (especially neutrophil count) is raised, as this indicates an
increased risk of having meningococcal disease.

Be aware that while a normal CRP and normal white blood cell count mean
meningococcal disease is less likely, they do not rule it out. The CRP may
be normal and the white blood cell count normal or low even in severe
meningococcal disease.

Assess clinical progress by monitoring vital signs (respiratory rate, heart


rate, blood pressure, conscious level [Glasgow Coma Scale and/or APVU],
temperature), capillary refill time, and oxygen saturations. Carry out
observations at least hourly over the next 46 hours.

If doubt remains, treat with antibiotics and admit to hospital.

1.3.5 If the child or young person is assessed as being at low risk of


meningococcal disease and is discharged after initial observation, advise parents
or carers to return to hospital if the child or young person appears ill to them.

1.3.6 Be aware that in children and young people who present with a nonspreading petechial rash without fever (or history of fever) who do not appear ill
to a healthcare professional, meningococcal disease is unlikely, especially if the
rash has been present for more than 24 hours. In such cases consider:

other possible diagnoses

performing a full blood count and coagulation screen.

Investigation and management in children and young people with


suspected bacterial meningitis
1.3.7 In children and young people with suspected bacterial meningitis, perform a
CRP and white blood cell count:

If the CRP and/or white blood cell count is raised and there is a
non-specifically abnormal cerebrospinal fluid (CSF) (for example consistent
with viral meningitis), treat as bacterial meningitis.

Be aware that a normal CRP and white blood cell count does not rule out
bacterial meningitis.

Regardless of the CRP and white blood cell count, if no CSF is available for
examination or if the CSF findings are uninterpretable, manage as if the
diagnosis of meningitis is confirmed.

Polymerase chain reaction (PCR) tests for bacterial meningitis and


meningococcal disease
1.3.8 Perform whole blood real-time PCR testing (EDTA[8] sample) for
N meningitidis to confirm a diagnosis of meningococcal disease.
1.3.9 The PCR blood sample should be taken as soon as possible because early
samples are more likely to be positive.
1.3.10 Use PCR testing of blood samples from other hospital laboratories if
available, to avoid repeating the test.
1.3.11 Be aware that a negative blood PCR test result for N meningitidis does not
rule out meningococcal disease.
1.3.12 Submit CSF to the laboratory to hold for PCR testing for N meningitidis
and S pneumoniae, but only perform the PCR testing if the CSF culture is
negative.
1.3.13 Be aware that CSF samples taken up to 96 hours after admission to
hospital may give useful results.
Skin samples and throat swabs for meningococcal disease
1.3.14 Do not use any of the following techniques when investigating for possible
meningococcal disease: skin scrapings, skin biopsies, petechial or purpuric
lesion aspirates (obtained with a needle and syringe), or throat swabs.
Performing lumbar puncture and interpreting CSF parameters for
suspected bacterial meningitis

1.3.15 Perform a lumbar puncture as a primary investigation unless this is


contraindicated.
1.3.16 Do not allow lumbar puncture to delay the administration of parenteral
antibiotics.
1.3.17 CSF examination should include white blood cell count and examination,
total protein and glucose concentrations, Gram stain and microbiological culture.
A corresponding laboratory-determined blood glucose concentration should be
measured.
1.3.18 In children and young people with suspected meningitis or suspected
meningococcal disease, perform a lumbar puncture unless any of the following
contraindications are present:

signs suggesting raised intracranial pressure


o

reduced or fluctuating level of consciousness (Glasgow Coma Scale


score less than 9 or a drop of 3 or more)

relative bradycardia and hypertension

focal neurological signs

abnormal posture or posturing

unequal, dilated or poorly responsive pupils

papilloedema

abnormal 'doll's eye' movements

shock (see table 1 above)

extensive or spreading purpura

after convulsions until stabilised

coagulation abnormalities
o

coagulation results (if obtained) outside the normal range

platelet count below 100 x 109/litre

receiving anticoagulant therapy

local superficial infection at the lumbar puncture site

respiratory insufficiency (lumbar puncture is considered to have a high risk of


precipitating respiratory failure in the presence of respiratory insufficiency).

1.3.19 In children and young people with suspected bacterial meningitis, if


contraindications to lumbar puncture exist at presentation consider delaying
lumbar puncture until there are no longer contraindications. Delayed lumbar
puncture is especially worthwhile if there is diagnostic uncertainty or
unsatisfactory clinical progress.

1.3.20 CSF white blood cell counts, total protein and glucose concentrations
should be made available within 4 hours to support the decision regarding
adjunctive steroid therapy.
1.3.21 Start antibiotic treatment for bacterial meningitis if the CSF white blood
cell count is abnormal:

in neonates at least 20 cells/microlitre (be aware that even if fewer


than 20 cells/microlitre, bacterial meningitis should still be considered if other
symptoms and signs are present see table 1 above)

in older children and young people more than 5 cells/microlitre or more than
1 neutrophil/microlitre, regardless of other CSF variables.

1.3.22 In children and young people with suspected bacterial meningitis, consider
alternative diagnoses if the child or young person is significantly ill and has CSF
variables within the accepted normal ranges.
1.3.23 Consider herpes simplex encephalitis as an alternative diagnosis.
1.3.24 If CSF white cell count is increased and there is a history suggesting a risk
of tuberculous meningitis, evaluate for the diagnosis of tuberculous meningitis in
line with Tuberculosis: clinical diagnosis and management of tuberculosis, and
measures for its prevention and control (NICE clinical guideline 33).
1.3.25 Perform a repeat lumbar puncture in neonates with:

persistent or re-emergent fever

deterioration in clinical condition

new clinical findings (especially neurological findings) or

persistently abnormal inflammatory markers.

1.3.26 Do not perform a repeat lumbar puncture in neonates:

who are receiving the antibiotic treatment appropriate to the causative


organism and are making a good clinical recovery

before stopping antibiotic therapy if they are clinically well.

Cranial computed tomography in suspected bacterial meningitis


1.3.27 Use clinical assessment and not cranial computed tomography (CT), to
decide whether it is safe to perform a lumbar puncture. CT is unreliable for
identifying raised intracranial pressure.
1.3.28 If a CT scan has been performed, do not perform a lumbar puncture if the
CT scan shows radiological evidence of raised intracranial pressure.
1.3.29 In children and young people with a reduced or fluctuating level of
consciousness (Glasgow Coma Scale score less than 9 or a drop of 3 or more)
or with focal neurological signs, perform a CT scan to detect alternative
intracranial pathology.
1.3.30 Do not delay treatment to undertake a CT scan.

1.3.31 Clinically stabilise children and young people before CT scanning.


1.3.32 If performing a CT scan consult an anaesthetist, paediatrician or
intensivist.
1.4 Management in secondary care

Antibiotics for suspected bacterial meningitis or meningococcal disease


1.4.1 Treat children and young people aged 3 months or older with suspected
bacterial meningitis without delay using intravenous ceftriaxone.
1.4.2 Treat children younger than 3 months with suspected bacterial meningitis
without delay using intravenous cefotaxime plus either amoxicillin or ampicillin.
1.4.3 Treat suspected meningococcal disease without delay using intravenous
ceftriaxone.
1.4.4 Treat children and young people with suspected bacterial meningitis who
have recently travelled outside the UK or have had prolonged or multiple
exposure to antibiotics (within the past 3 months) with vancomycin in addition to
the above antibiotics.
1.4.5 Where ceftriaxone is used, do not administer it at the same time as
calcium-containing infusions. Instead, use cefotaxime[9].
1.4.6 In children younger than 3 months, ceftriaxone may be used as an
alternative to cefotaxime (with or without ampicillin or amoxicillin), but be aware
that ceftriaxone should not be used in premature babies or in babies with
jaundice, hypoalbuminaemia or acidosis as it may exacerbate
hyperbilirubinaemia.
1.4.7 If tuberculous meningitis is part of the differential diagnosis use antibiotic
treatment appropriate for tuberculous meningitis in line with Tuberculosis (NICE
clinical guideline 33) (replaced by Tuberculosis [NICE clinical guideline 117]).
1.4.8 If herpes simplex meningoencephalitis is part of the differential diagnosis
give appropriate antiviral treatment.
Treatment for specific infections in confirmed bacterial meningitis
Children and young people aged 3 months or older
1.4.9 Treat H influenzae type b meningitis with intravenous ceftriaxone for 10
days in total unless directed otherwise by the results of antibiotic sensitivities.
1.4.10 Treat S pneumoniae meningitis with intravenous ceftriaxone for 14 days in
total unless directed otherwise by the results of antibiotic sensitivities.
Children younger than 3 months
1.4.11 Treat Group B streptococcal meningitis with intravenous cefotaxime for at
least 14 days. If the clinical course is complicated [10] consider extending the
duration of treatment and consulting an expert in paediatric infectious diseases.

1.4.12 Treat bacterial meningitis due to L monocytogenes with intravenous


amoxicillin or ampicillin for 21 days in total, plus gentamicin for at least the first 7
days.
1.4.13 Treat bacterial meningitis due to Gram-negative bacilli with intravenous
cefotaxime for at least 21 days unless directed otherwise by the results of
antibiotic sensitivities. If the clinical course is complicated [10] consider extending
the duration of treatment and consulting an expert in paediatric infectious
diseases.
Treatment of unconfirmed bacterial meningitis
1.4.14 In children and young people aged 3 months or older with unconfirmed,
uncomplicated but clinically suspected bacterial meningitis, treat with intravenous
ceftriaxone for at least 10 days depending on symptoms and signs and course of
the illness.
1.4.15 In children younger than 3 months with unconfirmed but clinically
suspected bacterial meningitis, treat with cefotaxime plus either ampicillin or
amoxicillin for at least 14 days. If the clinical course is complicated [10], consider
extending the duration of treatment and consulting an expert in paediatric
infectious diseases.
Meningococcal disease
1.4.16 In children and young people with confirmed meningococcal disease, treat
with intravenous ceftriaxone for 7 days in total unless directed otherwise by the
results of antibiotic sensitivities.
1.4.17 In children and young people with unconfirmed but clinically suspected
meningococcal disease, treat with intravenous ceftriaxone for 7 days in total.
Other aspects of management in bacterial meningitis and meningococcal
septicaemia
Metabolic disturbances
1.4.18 In children and young people with suspected or confirmed meningococcal
septicaemia, anticipate, monitor and correct the following metabolic disturbances
using local or national protocols:

hypoglycaemia

acidosis

hypokalaemia

hypocalcaemia

hypomagnesaemia

anaemia

coagulopathy.

Seizures

1.4.19 Use local or national protocols for management of seizures in children and
young people with suspected bacterial meningitis or meningococcal septicaemia.
Raised intracranial pressure
1.4.20 Use local or national protocols to treat raised intracranial pressure.
Fluid management in suspected or confirmed bacterial meningitis
1.4.21 Assess for all of the following:

signs of shock (see table 1 above)

raised intracranial pressure

signs of dehydration.

Refer to Diarrhoea and vomiting in children (NICE clinical guideline 84) for
assessment of shock and dehydration.
1.4.22 If present, correct dehydration using enteral fluids or feeds, or intravenous
isotonic fluids (for example, sodium chloride 0.9% with glucose 5% or sodium
chloride 0.9% with dextrose 5%).
1.4.23 Do not restrict fluids unless there is evidence of:

raised intracranial pressure, or

increased antidiuretic hormone secretion [11].

1.4.24 Give full-volume maintenance fluids to avoid hypoglycaemia and maintain


electrolyte balance.
1.4.25 Use enteral feeds as maintenance fluid if tolerated.
1.4.26 If intravenous maintenance fluid is required, use isotonic fluids (for
example, sodium chloride 0.9% with glucose 5% or sodium chloride 0.9% with
dextrose 5%). In neonates, use glucose 10% and added sodium chloride for
maintenance.
1.4.27 Monitor fluid administration and urine output to ensure adequate hydration
and avoid overhydration.
1.4.28 Monitor electrolytes and blood glucose regularly (at least daily while the
child or young person is receiving intravenous fluids).
1.4.29 If there are signs of raised intracranial pressure or evidence of shock,
initiate emergency management for these conditions and discuss ongoing fluid
management with a paediatric intensivist.
Intravenous fluid resuscitation in meningococcal septicaemia
1.4.30 In children and young people with suspected or confirmed meningococcal
septicaemia:

If there are signs of shock, give an immediate fluid bolus of 20 ml/kg sodium
chloride 0.9% over 510 minutes. Give the fluid intravenously or via an

intraosseous route and reassess the child or young person immediately


afterwards.

If the signs of shock persist, immediately give a second bolus of 20 ml/kg of


intravenous or intraosseous sodium chloride 0.9% or human albumin 4.5%
solution over 510 minutes.

If the signs of shock still persist after the first 40 ml/kg:

immediately give a third bolus of 20 ml/kg of intravenous or


intraosseous sodium chloride 0.9% or human albumin 4.5% solution
over 510 minutes

call for anaesthetic assistance for urgent tracheal intubation and


mechanical ventilation

start treatment with vasoactive drugs

be aware that some children and young people may require large
volumes of fluid over a short period of time to restore their circulating
volume

consider giving further fluid boluses at 20 ml/kg of intravenous or


intraosseous sodium chloride 0.9% or human albumin 4.5% solution
over 510 minutes based on clinical signs and appropriate laboratory
investigations including urea and electrolytes

discuss further management with a paediatric intensivist.

Vasoactive therapy for shock in meningococcal septicaemia


1.4.31 If shock persists despite fluid resuscitation (more than 40 ml/kg) and
treatment with either intravenous adrenaline or intravenous noradrenaline, or
both, consider potential reasons (such as persistent acidosis, incorrect dilution,
extravasation) and discuss further management options with a paediatric
intensivist.
1.4.32 Use local or national protocols for the administration of vasoactive agents
in children and young people with suspected or confirmed bacterial meningitis or
meningococcal septicaemia.
Respiratory support in children and young people with suspected or confirmed
bacterial meningitis or meningococcal septicaemia
1.4.33 In self-ventilating children and young people with signs of respiratory
distress, administer 15-litre face mask oxygen via a reservoir rebreathing mask.
1.4.34 If there is a threatened loss of airway patency, implement airway-opening
manoeuvres, and start bagvalve mask ventilation in preparation for tracheal
intubation.
1.4.35 A healthcare professional with expertise in paediatric airway management
should undertake tracheal intubation.
1.4.36 Be aware that children and young people with suspected or confirmed
bacterial meningitis or meningococcal septicaemia are very ill and at grave risk of

sudden deterioration during intubation. Anticipate aspiration, pulmonary oedema


or worsening shock during intubation. Ensure that they are nil by mouth from
admission to hospital and that the following are available before intubation:

facilities to administer fluid boluses

appropriate vasoactive drugs

access to a healthcare professional experienced in the management of


critically ill children.

1.4.37 Undertake tracheal intubation and mechanical ventilation for the following
indications:

threatened (for example, loss of gag reflex), or actual loss of airway patency

the need for any form of assisted ventilation, for example


bagmask ventilation

clinical observation of increasing work of breathing

hypoventilation or apnoea

features of respiratory failure, including:


o

irregular respiration (for example, CheyneStokes breathing)

hypoxia (PaO2 less than 13 kPa or 97.5 mmHg) or decreased oxygen


saturations in air

hypercapnia (PaCO2 greater than 6 kPa or 45 mmHg)

continuing shock following infusion of a total of 40 ml/kg of resuscitation fluid

signs of raised intracranial pressure

impaired mental status:


o

reduced or fluctuating level of consciousness (Glasgow Coma Scale


score less than 9 or a drop of 3 or more)

moribund state

control of intractable seizures

need for stabilisation and management to allow brain imaging or transfer to


the paediatric intensive care unit or another hospital.

1.4.38 Use local or national protocols for intubation.


Corticosteroids
Bacterial meningitis
1.4.39 Do not use corticosteroids in children younger than 3 months with
suspected or confirmed bacterial meningitis.
1.4.40 Give dexamethasone (0.15 mg/kg to a maximum dose of 10 mg, four
times daily for 4 days)[12] for suspected or confirmed bacterial meningitis as soon
as possible if lumbar puncture reveals any of the following:

frankly purulent CSF

CSF white blood cell count greater than 1000/microlitre

raised CSF white blood cell count with protein concentration greater than
1 g/litre

bacteria on Gram stain.

1.4.41 If tuberculous meningitis is in the differential diagnosis, refer to


Tuberculosis (NICE clinical guideline 33) (replaced by Tuberculosis [NICE clinical
guideline 117]) before administering steroids, because steroids may be harmful if
given without antituberculous therapy.
1.4.42 If dexamethasone was not given before or with the first dose of antibiotics,
but was indicated, try to administer the first dose within 4 hours of starting
antibiotics, but do not start dexamethasone more than 12 hours after starting
antibiotics.
1.4.43 After the first dose of dexamethasone discuss the decision to continue
dexamethasone with a senior paediatrician.
Meningococcal septicaemia
1.4.44 Do not treat with high-dose corticosteroids (defined as dexamethasone
0.6 mg/kg/day or an equivalent dose of other corticosteroids).
1.4.45 In children and young people with shock that is unresponsive to
vasoactive agents, steroid replacement therapy using low-dose corticosteroids
(hydrocortisone 25 mg/m2 four times daily)[12] should be used only when directed
by a paediatric intensivist.
Adjunctive therapies
1.4.46 Do not use activated protein C or recombinant bacterial permeabilityincreasing protein in children and young people with meningococcal septicaemia.
Monitoring for deterioration for meningococcal disease
1.4.47 Monitor children and young people closely after admission to hospital for
signs of deterioration (monitor respiration, pulse, blood pressure, oxygen
saturation and Glasgow Coma Scale score).
1.4.48 Be aware that children and young people with meningococcal disease can
deteriorate rapidly, regardless of the results of any initial assessment of severity.
Retrieval and transfer to tertiary care
1.4.49 Children and young people who need resuscitation should be discussed
with a paediatric intensivist as soon as possible.
1.4.50 Transfer of children and young people to tertiary care should be
undertaken by an experienced paediatric intensive care retrieval team comprising
medical and nursing staff.

1.5 Long-term management

Long-term effects of bacterial meningitis and meningococcal septicaemia


1.5.1 Before discharging children and young people from hospital:

consider their requirements for follow-up, taking into account potential


sensory, neurological, psychosocial, orthopaedic, cutaneous and renal
morbidities, and

discuss potential long-term effects of their condition and likely patterns of


recovery with the child or young person and their parents or carers, and
provide them with opportunities to discuss issues and ask questions.

1.5.2 Offer children and young people and their parents or carers:

information about and access to further care immediately after discharge,


and

contact details of patient support organisations including meningitis charities


that can offer support, befriending, in-depth information, advocacy,
counselling, and written information to signpost families to further help, and

advice on accessing future care.

1.5.3 Offer a formal audiological assessment as soon as possible, preferably


before discharge, within 4 weeks of being fit to test.
1.5.4 Offer children and young people with a severe or profound deafness an
urgent assessment for cochlear implants as soon as they are fit to undergo
testing (further guidance on the use of cochlear implants for severe to profound
deafness can be found in Cochlear implants for children and adults with severe
to profound deafness [NICE technology appraisal guidance 166]).
1.5.5 Children and young people should be reviewed by a paediatrician with the
results of their hearing test 46 weeks after discharge from hospital to discuss
morbidities associated with their condition and offered referral to the appropriate
services. The following morbidities should be specifically considered:

hearing loss (with the child or young person having undergone an urgent
assessment for cochlear implants as soon as they are fit)

orthopaedic complications (damage to bones and joints)

skin complications (including scarring from necrosis)

psychosocial problems

neurological and developmental problems

renal failure.

1.5.6 Inform the child's or young person's GP, health visitor and school nurse (for
school-age children and young people) about their bacterial meningitis or
meningococcal septicaemia.
1.5.7 Healthcare professionals with responsibility for monitoring the child's or
young person's health should be alert to possible late-onset sensory,

neurological, orthopaedic and psychosocial effects of bacterial meningitis and


meningococcal septicaemia.
Immune testing
1.5.8 Test children and young people for complement deficiency if they have had
either:

more than one episode of meningococcal disease, or

one episode of meningococcal disease caused by serogroups other than B


(for example A, C, Y, W135, X, 29E), or

meningococcal disease caused by any serogroup and a history of other


recurrent or serious bacterial infections.

1.5.9 Children and young people with recurrent episodes of meningococcal


disease should be assessed by a specialist in infectious disease or immunology.
1.5.10 Do not test children and young people for complement deficiency who
have had either:

a single episode of meningococcal disease caused by serogroup B


meningococcus, or

unconfirmed meningococcal disease.

1.5.11 Discuss appropriate testing for complement deficiency with local


immunology laboratory staff.
1.5.12 If a child or young person who has had meningococcal disease has a
family history of meningococcal disease or complement deficiency, test the child
or young person for complement deficiency.
1.5.13 If a child or young person who has had meningococcal disease is found to
have complement deficiency, test their parents and siblings for complement
deficiency.
1.5.14 Refer children and young people with complement deficiency to a
healthcare professional with expertise in the management of the condition.
1.5.15 Do not test children and young people for immunoglobulin deficiency if
they have had meningococcal disease, unless they have a history suggestive of
an immunodeficiency (that is, a history of serious, persistent, unusual, or
recurrent infections).
Question 22 of 231

A 4-year-old girl is reviewed by her GP due to poor control of asthma. She currently takes a regular steroid inhaler and a salbutamol inhaler
as required. What is the next step in management?
Trial of a leukotriene receptor antagonist

Referral to a paediatrician

Inhaled long-acting beta2-agonist

Oral theophylline

Trial of oral steroids

Next question

Asthma in children: stepwise management


The British Thoracic Society differentiate between children younger and older than 5 years in their 2014 guidelines:
Children aged under 5 years
StepTherapy
1

As-required reliever therapy: short-acting beta2-agonist

Regular preventer therapy: inhaled corticosteroids, 200-400mcg/day*


Or, if inhaled corticosteroids cannot be used, a leukotriene receptor antagonist

Children aged 2-5 years: trial of a leukotriene receptor antagonist. If already taking leukotriene receptor antagonist reconsider inhaled
corticosteroids
Children aged under 2 years: refer to respiratory paediatrician

Refer to a respiratory paediatrician

Children aged over 5 years (similar to adult guidance)


StepTherapy
1

As-required reliever therapy: short-acting beta2-agonist

Regular preventer therapy: inhaled corticosteroids, 200-400mcg/day*


1. Add inhaled long-acting B2 agonist (LABA)
2. Assess control of asthma:

good response to LABA - continue LABA


benefit from LABA but control still inadequate: continue LABA and increase inhaled steroid dose to 400 mcg/day* (if not
already on this dose)
no response to LABA: stop LABA and increase inhaled steroid to 400 mcg/ day.* If control still inadequate, institute trial of
other therapies, leukotriene receptor antagonist or SR theophylline

Increase inhaled corticosteroids to high-dose, up to 800mcg/day*


Use daily steroid tablet at lowest dose providing control

Maintain inhaled corticosteroids at 800mcg/day


Refer to a paediatrician

*beclometasone dipropionate or equivalent

Question 23 of 231

Which one of the following is not a risk factor for sudden infant death syndrome?
Female sex

Hyperthermia

Social classes IV and V

Prematurity

Bottle feeding

Next question

Male rather than female sex is a risk factor for sudden infant death syndrome

Sudden infant death syndrome


Sudden infant death syndrome is the commonest cause of death in the first year of life. It is most common at 3 months of age
Risk factors

prematurity
parental smoking
hyperthermia (e.g. over-wrapping)
putting the baby to sleep prone
male sex
multiple births
bottle feeding
social classes IV and V
maternal drug use
incidence increases in winter

Following a cot death siblings should be screened for potential sepsis and inborn errors of metabolism

Question 24 of 231

A 4-year-old boy is reviewed in the Paediatric Admissions Unit. He has had a fever for the past week. On examination he has red, sore lips
and conjunctival injection. He also has swollen, red hands. Blood tests show:

Hb

13.1 g/dl

WBC

12.7 *109/l

Platelets

520 *109/l

CRP

96 mg/L

What is the likely diagnosis?


Rheumatic fever

Dermatomyositis

Lyme disease

Still's disease

Kawasaki disease

Next question

Kawasaki disease
Kawasaki disease is a type of vasculitis which is predominately seen in children. Whilst Kawasaki disease is uncommon it is important to
recognise as it may cause potentially serious complications, including coronary artery aneurysms
Features

high-grade fever which lasts for > 5 days. Fever is characteristically resistant to antipyretics
conjunctival injection
bright red, cracked lips
strawberry tongue
cervical lymphadenopathy
red palms of the hands and the soles of the feet which later peel

Kawasaki disease is a clinical diagnosis as there is no specific diagnostic test


Management

high-dose aspirin*
intravenous immunoglobulin
echocardiogram (rather than angiography) is used as the initial screening test for coronary artery aneurysms

Complications

coronary artery aneurysm

*Kawasaki disease is one of the few indications for the use of aspirin in children. Due to the risk of Reye's syndrome aspirin is normally
contraindicated in children.

Question 25 of 231

A 14-year-old girl requests to start the combined oral contraceptive pill. Which one of the following is not an integral part of the Fraser
guidelines?
Unless they receive contraceptive treatment their health is likely to suffer

They cannot be persuaded to inform their parents or allow the professional to contact them on their behalf

They are likely to begin, or continue having, sexual intercourse with or without contraceptive treatment

They do not have, or have had in the past, a mental illness or learning disability

They understand the professional's advice

Next question

Fraser guidelines
The Fraser guidelines are used to assess if patient who has not yet reached 16 years of age is competent to consent to treatment, for
example with respect to contraception
The following points should be fulfilled:

the young person understands the professional's advice


the young person cannot be persuaded to inform their parents or allow the professional to contact them on their behalf
the young person is likely to begin, or continue having, sexual intercourse with or without contraceptive treatment
unless the young person receives contraceptive treatment, their physical or mental health, or both, is likely to suffer
the young person's best interests require them to receive contraceptive advice or treatment with or without parental consent

Question 26 of 231

A 6-year-old boy is reviewed in clinic due to nocturnal enuresis. His mother has tried using a star-chart but unfortunately this has not
resulted in any significant improvement. Of the following options, what is the most appropriate initial management strategy?
Enuresis alarm

Trial of oral desmopressin

Trial of imipramine

Trial of intranasal desmopressin

Restrict fluids in the afternoon and evening

Next question

Restricting fluids is not recommended advice - Clinical Knowledge Summaries suggest: 'Do not restrict fluids. The child should have about
eight drinks a day, spaced out throughout the day, the last one about 1 hour before bed.'

Nocturnal enuresis
The majority of children achieve day and night time continence by 3 or 4 years of age. Enuresis may be defined as the 'involuntary
discharge of urine by day or night or both, in a child aged 5 years or older, in the absence of congenital or acquired defects of the nervous
system or urinary tract'
Nocturnal enuresis can be defined as either primary (the child has never achieved continence) or secondary (the child has been dry for at
least 6 months before)
NICE issued guidance in 2010. Management:

look for possible underlying causes/triggers (e.g. Constipation, diabetes mellitus, UTI if recent onset)
advise on fluid intake, diet and toileting behaviour
reward systems (e.g. Star charts). NICE recommend these 'should be given for agreed behaviour rather than dry nights' e.g.
Using the toilet to pass urine before sleep
NICE advise: 'Consider whether alarm or drug treatment is appropriate, depending on the age, maturity and abilities of the child
or young person, the frequency of bedwetting and the motivation and needs of the family'. Generally:
an enuresis alarm is first-line for children under the age of 7 years
desmopressin may be used first-line for children over the ago 7 years, particularly if short-term control is needed or an enuresis
alarm has been ineffective/is not acceptable to the family
please see the link for more details

Bedwetting (enuresis) - Summary

Bedwetting is involuntary wetting during sleep, without any inherent suggestion of


frequency or pathophysiology.
Bedwetting can be classified as:
o Primary bedwetting the child has never achieved sustained continence at night.
o Secondary bedwetting bedwetting occurs after the child has been dry at night for
more than 6 months.
Primary bedwetting without daytime symptoms is thought to be caused by sleep
arousal difficulties, polyuria, and/or bladder dysfunction.
o Daytime symptoms include urgency, frequency, daytime wetting, abdominal
straining or poor urinary stream, pain passing urine, or passing urine fewer than four
times a day.
Primary bedwetting with daytime symptoms is usually caused by disorders of the lower
urinary tract.
o It is most commonly caused by an overactive bladder, but may also be caused by
congenital malformations or neurological disorders.
o It is occasionally caused by chronic constipation, chronic urinary tract infection
(UTI), or chronic emotional problems.
Secondary bedwetting is bedwetting secondary to an underlying cause, such as
diabetes, UTI, constipation, or family problems (vulnerable child or family).
Risk factors associated with bedwetting include:
o A family history of bedwetting.
o Gender boys are more likely than girls to have bedwetting.
o Delay in attaining bladder control.
o Being obese approximately 30% of obese children have bedwetting.

Psychological or behavioural disorders such as attention deficit hyperactivity


disorder (ADHD), autism spectrum disorder, anxiety, depressive, and conduct
disorders.
It is necessary to determine the type of bedwetting by asking if:
o There are any daytime symptoms.
o The child has previously been dry at night without assistance for 6 months.
Managing primary bedwetting (without daytime symptoms) involves:
o General advice on bedwetting.
o Advice on lifting and waking.
o Addressing any issues concerning excessive or insufficient fluid intake, diet, and
toileting patterns.
o In children < 5 years of age, reassurance that many children younger than 5 years of
age wet the bed and that this usually resolves without treatment.
o In children > 5 years of age, if bedwetting is infrequent (less than twice a week),
reassurance that bedwetting may resolve without treatment and offering the option
of a wait-and-see approach. If long-term treatment is required, treatment with an
enuresis alarm (first-line treatment) should be offered in combination with positive
reward systems desmopressin may be considered if the alarm is unsuitable. If
rapid or short-term control of bedwetting is required, treatment with desmopressin
is an option.
Managing primary bedwetting (with daytime symptoms) involves:
o Referring the child to secondary care or an enuresis clinic for further investigations
and assessment.
o Considering referral for younger children (older than 2 years of age) who are
struggling to not wet themselves during the day as well as during the night, despite
awareness of toileting needs and showing appropriate toileting behaviour.
Managing secondary enuresis involves:
o Treatment of UTIs and constipation in primary care.
o Referral to a paediatrician or an enuresis clinic if children have other underlying
causes which have been identified but are generally not managed in primary care
(e.g. diabetes and learning difficulties
o

Question 27 of 231

A 2-month-old baby girl is admitted to hospital with suspected meningitis. Her parents describe her becoming pyrexial and drowsy over the
past 24 hours. On examination her temperature is 39.2C, heart rate is 160/min and respiratory rate is 50.min. Her anterior fontanelle is
bulging. No petechial rash is seen. In addition to cefotaxime, what antibiotic should be given intravenously?
Rifampicin

Flucloxacillin

Amoxicillin

Clarithromycin

Gentamicin

Next question

Meningitis in children < 3 months: give IV amoxicillin in addition to cefotaxime to cover for Listeria

Meningitis in children: investigation and management


Investigations
Contraindication to lumbar puncture (any signs of raised ICP)

focal neurological signs


papilloedema
significant bulging of the fontanelle
disseminated intravascular coagulation
signs of cerebral herniation

For patients with meningococcal septicaemia lumbar puncture is contraindicated - blood cultures and PCR for meningococcus should be
obtained
Management
1. Antibiotics

< 3 months: IV amoxicillin + IV cefotaxime


> 3 months: IV cefotaxime

2. Steroids

if > 1 month and Haemophilus influenzae then give dexamethasone

3. Fluids

treat any shock, e.g. with colloid

4. Cerebral monitoring

mechanical ventilation if respiratory impairment

5. Public health notification and antibiotic prophylaxis of contacts

rifampicin

Question 28 of 231

A 4-month-old girl is brought to surgery for her routine immunisations. She has had all the previous immunisations as per the routine
schedule and there is no past medical history. What should she be given at this visit?

Key

DTaP = Diphtheria, Tetanus, acellular Pertussis vaccine


IPV = Inactivated Polio Vaccine
Hib = Haemophilus influenzae B vaccine
PCV = Pneumococcal Conjugate Vaccine
Men C = Meningococcal C vaccine
MMR = Measles, Mumps, Rubella vaccine
DT = Diphtheria, Tetanus vaccine

DTaP/IPV/Hib + Men C

DTaP + Hib + PCV + Men C

DTaP/IPV/Hib + PCV

DTaP/IPV/Hib + PCV + Men C

Hib/Men C + MMR + PCV

Next question

The Men C booster has recently been dropped from the 4 month visit - please see the updated UK immunisation schedule.

Immunisation schedule
The current UK immunisation schedule is as follows. Please note that this table includes the changes announced in 2010 which merged the
12 and 13 month visits into one.
Age

Recommended immunisations

At birth

BCG / hepatitis B vaccine if risk factors (see below)

2 months

DTaP/IPV/Hib + PCV
Oral rotavirus vaccine

3 months

DTaP/IPV/Hib + Men C
Oral rotavirus vaccine

4 months

DTaP/IPV/Hib + PCV

12-13 months Hib/Men C + MMR + PCV


2-3 years

Flu vaccine (annual)

3-4 years

MMR + DTaP/IPV

12-13 years HPV vaccination for girls


13-18 years DT/IPV + MenC

At birth the BCG vaccine should be given if the baby is deemed at risk of tuberculosis (e.g. Tuberculosis in the family in the past 6 months).
Hepatitis B vaccine should be given at birth if the mother is HBsAg +ve.

Key

DTaP = Diphtheria, Tetanus, acellular Pertussis vaccine


IPV = Inactivated Polio Vaccine
Hib = Haemophilus influenzae B vaccine
PCV = Pneumococcal Conjugate Vaccine
Men C = Meningococcal C vaccine
MMR = Measles, Mumps, Rubella vaccine
DT = Diphtheria, Tetanus vaccine
HPV = Human Papilloma Vaccine

Question 29 of 231

Which one of the following statements regarding cerebral palsy is incorrect?


It is the most common cause of major motor impairment in children

Less than 5% of children will have epilepsy

It affects 2 in 1,000 live births

20% of children have hearing impairment

Postnatal factors account for 10% of cases

Next question

Around 30% of children with cerebral palsy have epilepsy

Cerebral palsy
Cerebral palsy may be defined as a disorder of movement and posture due to a non-progressive lesion of the motor pathways in the
developing brain. It affects 2 in 1,000 live births and is the most common cause of major motor impairment
Possible manifestations include:

abnormal tone early infancy


delayed motor milestones
abnormal gait
feeding difficulties

Children with cerebral palsy often have associated non-motor problems such as:

learning difficulties (60%)


epilepsy (30%)
squints (30%)
hearing impairment (20%)

Causes

antenatal (80%): e.g. cerebral malformation and congenital infection (rubella, toxoplasmosis, CMV)
intrapartum (10%): birth asphyxia/trauma
postnatal (10%): intraventricular haemorrhage, meningitis, head-trauma

Classification

spastic (70%): hemiplegia, diplegia or quadriplegia


dyskinetic
ataxic
mixed

Management

as with any child with a chronic condition a multidisciplinary approach is needed


treatments for spasticity include oral diazepam, oral and intrathecal baclofen, botulinum toxin type A, orthopaedic surgery and
selective dorsal rhizotomy
anticonvulsants, analgesia as required

Question 30 of 231

A mother brings her 8-month-old infant to the GP due to the development of a nappy rash. On examination an erythematous rash with
flexural sparing is seen. What is the most likely cause?
Irritant dermatitis

Psoriasis

Seborrhoeic dermatitis

Candida infection

Atopic eczema

Next question

Napkin rashes
Causes of a napkin ('nappy') rash include the following:

Irritant dermatitis

The most common cause, due to irritant effect of urinary ammonia and faeces
Creases are characteristically spared

Candida dermatitis

Typically an erythematous rash which involve the flexures and has characteristic satellite lesions

Seborrhoeic dermatitis Erythematous rash with flakes. May be coexistent scalp rash

Irritant dermatitis

The most common cause, due to irritant effect of urinary ammonia and faeces
Creases are characteristically spared

Psoriasis

A less common cause characterised by an erythematous scaly rash also present elsewhere on the skin

Atopic eczema

Other areas of the skin will also be affected

General management points

disposable nappies are preferable to towel nappies


expose napkin area to air when possible
apply barrier cream (e.g. Zinc and castor oil)
mild steroid cream (e.g. 1% hydrocortisone) in severe cases

Question 31 of 231

A 1-year-old girl is investigated for recurrent urinary tract infections. A micturating cystourethrogram is ordered:

Image used on license from Radiopaedia

What does this image demonstrate?


Vesicoureteric reflux

Bilateral renal scarring

Paediatric urolithiasis

Polycystic kidney disease

Bilateral hydronephrosis

Next question

This image demonstrates grade V vesicoureteric reflux - gross dilatation of the ureter, pelvis and calyces with ureteral tortuosity. A DMSA
scan is needed to identify renal scarring.

Vesicoureteric reflux
Vesicoureteric reflux (VUR) is the abnormal backflow of urine from the bladder into the ureter and kidney. It is relatively common
abnormality of the urinary tract in children and predisposes to urinary tract infection (UTI), being found in around 30% of children who
present with a UTI. As around 35% of children develop renal scarring it is important to investigate for VUR in children following a UTI
Pathophysiology of VUR

ureters are displaced laterally, entering the bladder in a more perpendicular fashion than at an angle
therefore shortened intramural course of ureter
vesicoureteric junction cannot therefore function adequately

The table below summarises the grading of VUR


Grade
I

Reflux into the ureter only, no dilatation

II

Reflux into the renal pelvis on micturition, no dilatation

III

Mild/moderate dilatation of the ureter, renal pelvis and calyces

IV

Dilation of the renal pelvis and calyces with moderate ureteral tortuosity

Gross dilatation of the ureter, pelvis and calyces with ureteral tortuosity

Investigation

VUR is normally diagnosed following a micturating cystourethrogram


a DMSA scan may also be performed to look for renal scarring

Question 32 of 231

A 4-year-old boy is brought to the surgery as his mother is concerned that he has had an intermittent fever for 48 hours. Which one of the
following is least relevant when examining the child?

Blood pressure

Respiratory rate

Capillary refill time

Temperature

Heart rate

Next question

Feverish illness in children


The 2007 NICE Feverish illness in children guidelines introduced a 'traffic light' system for risk stratification of children under the age of 5
years presenting with a fever. These guidelines were later modified in a 2013 update.
It should be noted that these guidelines only apply 'until a clinical diagnosis of the underlying condition has been made'. A link to the
guidelines is provided but some key points are listed below.
Assessment
The following should be recorded in all febrile children:

temperature
heart rate
respiratory rate
capillary refill time

Signs of dehydration (reduced skin turgor, cool extremities etc) should also be looked for
Measuring temperature should be done with an electronic thermometer in the axilla if the child is < 4 weeks or with an electronic/chemical
dot thermometer in the axilla or an infra-red tympanic thermometer.
Risk stratification
Please see the link for the complete table, below is a modified version
Green - low risk

Amber - intermediate risk

Red - high risk

Colour

Normal colour

Pallor reported by parent/carer

Pale/mottled/ashen/blue

Activity

Responds normally to
social cues
Content/smiles
Stays awake or awakens
quickly
Strong normal cry/not
crying

Not responding normally to social cues


No smile
Wakes only with prolonged stimulation
Decreased activity

No response to social cues


Appears ill to a healthcare professional
Does not wake or if roused does not
stay awake
Weak, high-pitched or continuous cry

Nasal flaring
Tachypnoea: respiratory rate
Respiratory

>50 breaths/minute, age 6-12


months;

Grunting
Tachypnoea: respiratory rate >60
breaths/minute
Moderate or severe chest indrawing

Green - low risk

Amber - intermediate risk

Red - high risk

>40 breaths/minute, age >12


months

Oxygen saturation <=95% in air


Crackles in the chest
Tachycardia:

Circulation and
hydration

Normal skin and eyes


Moist mucous membranes

>160 beats/minute, age <12


months
>150 beats/minute, age 12-24
months

Reduced skin turgor

>140 beats/minute, age 2-5 years

Capillary refill time >=3 seconds


Dry mucous membranes
Poor feeding in infants
Reduced urine output

Other

No amber or red signs

Age 3-6 months, temperature >=39C


Fever for >=5 days
Rigors
Swelling of a limb or joint
Non-weight bearing limb/not using an
extremity

Age <3 months, temperature >=38C


Non-blanching rash
Bulging fontanelle
Neck stiffness
Status epilepticus
Focal neurological signs
Focal seizures

Management
If green:

Child can be managed at home with appropriate care advice, including when to seek further help

If amber:

provide parents with a safety net or refer to a paediatric specialist for further assessment
a safety net includes verbal or written information on warning symptoms and how further healthcare can be accessed, a followup appointment, liaison with other healthcare professionals, e.g. out-of-hours providers, for further follow-up

If red:

refer child urgently to a paediatric specialist

Other key points include

oral antibiotics should not be prescribed to children with fever without apparent source
if a pneumonia is suspected but the child is not going to be referred to hospital then a chest x-ray does not need to be routinely
performed

Question 33 of 231

A 3-year-old child is diagnosed with whooping cough. What is the most appropriate antibiotic therapy?

No antibiotics are indicated

Oral amoxicillin

Oral levofloxacin

Oral erythromycin

Oral penicillin

Next question

Whooping cough (pertussis)


Overview

caused by the Gram negative bacterium Bordetella pertussis


incubation period = 10-14 days
infants are routinely immunised at 2, 3, 4 months and 3-5 years. Newborn infants are particularly vulnerable, which is why the
vaccincation campaign for pregnant women was introduced
neither infection nor immunisation results in lifelong protection - hence adolescents and adults may develop whooping cough
despite having had their routine immunisations
around 1,000 cases are reported each year in the UK

Features, 2-3 days of coryza precede onset of:

coughing bouts: usually worse at night and after feeding, may be ended by vomiting & associated central cyanosis
inspiratory whoop: not always present (caused by forced inspiration against a closed glottis)
persistent coughing may cause subconjunctival haemorrhages or even anoxia leading to syncope & seizures
symptoms may last 10-14 weeks* and tend to be more severe in infants
marked lymphocytosis

Diagnosis

per nasal swab culture for Bordetella pertussis - may take several days or weeks to come back
PCR and serology are now increasingly used as their availability becomes more widespread

Management

oral erythromycin to eradicate the organism and reduce spread


has not been shown to alter the course of the illness

Complications

subconjunctival haemorrhage
pneumonia
bronchiectasis

seizures

Vaccination of pregnant women


In 2012 there was an outbreak of whooping cough (pertussis) which resulted in the death of 14 newborn children. As a temporary measure
a vaccination programme was introduced in 2012 for pregnant women. This has successfully reduced the number of cases of whooping
cough (the vaccine is thought to be more than 90% effective in preventing newborns developing whooping cough). It was however decided
in 2014 to extend the whooping cough vaccination programme for pregnant women. This decision was taken as there was a 'great deal of
uncertainty' about the timing of future outbreaks.
Women who are between 28-38 weeks pregnant will be offered the vaccine.
*weeks, not days

Question 34 of 231

You are reviewing a 6-month-old child with suspected bronchiolitis. Which one of the following should prompt a referral to hospital?
Oxygen saturations of 96%

Lethargy

Feeding two-thirds of normal amount

Respiratory rate 54 / min

Crackles on auscultation

Next question

Bronchiolitis
Bronchiolitis is a condition characterised by acute bronchiolar inflammation. Respiratory syncytial virus (RSV) is the pathogen in 75-80% of
cases. SIGN released guidelines on bronchiolitis in 2006. Please see the link for more details.
Epidemiology

most common cause of a serious lower respiratory tract infection in < 1yr olds (90% are 1-9 months, with a peak incidence of 36 months). Maternal IgG provides protection to newborns against RSV
higher incidence in winter

Basics

respiratory syncytial virus (RSV) is the pathogen in 75-80% of cases


other causes: mycoplasma, adenoviruses
may be secondary bacterial infection
more serious if bronchopulmonary dysplasia (e.g. Premature), congenital heart disease or cystic fibrosis

Features

coryzal symptoms (including mild fever) precede:


dry cough
increasing breathlessness
wheezing, fine inspiratory crackles (not always present)
feeding difficulties associated with increasing dyspnoea are often the reason for hospital admission

SIGN suggested the following criteria for referral to hospital

poor feeding (< 50% normal)


lethargy
apnoea
respiratory rate > 70/min
nasal flaring or grunting
severe chest wall recession
cyanosis
oxygen saturation < 94%
uncertainty regarding diagnosis

Investigation

immunofluorescence of nasopharyngeal secretions may show RSV

Management is largely supportive

humidified oxygen is given via a head box


Question 35 of 231

A 6-year-old boy is noted to have pectus excavatum and pulmonary stenosis during a cardiorespiratory exam. What is the most
likely diagnosis?

Noonan syndrome

Pierre-Robin syndrome

Edward's syndrome

William's syndrome

Patau syndrome

Childhood syndromes

Next question


Below is a list of common features of selected childhood syndromes
Syndrome

Key features

Patau syndrome (trisomy 13)

Microcephalic, small eyes


Cleft lip/palate
Polydactyly
Scalp lesions

Micrognathia
Low-set ears
Edward's syndrome (trisomy 18)
Rocker bottom feet
Overlapping of fingers

Fragile X

Learning difficulties
Macrocephaly
Long face
Large ears
Macro-orchidism

Noonan syndrome

Webbed neck
Pectus excavatum
Short stature
Pulmonary stenosis

Pierre-Robin syndrome*

Micrognathia
Posterior displacement of the tongue (may result in upper airway obstruction)
Cleft palate

Prader-Willi syndrome

Hypotonia
Hypogonadism
Obesity

William's syndrome

Short stature
Learning difficulties
Friendly, extrovert personality
Transient neonatal hypercalcaemia
Supravalvular aortic stenosis

*this condition has many similarities with Treacher-Collins syndrome. One of the key differences is that Treacher-Collins syndrome
is autosomal dominant so there is usually a family history of similar problems
Question 36 of 231

When is the neonatal blood spot screening test typically performed in the United Kingdom
At birth

On first day of life

On fourth day of life

Between fifth and ninth day of life

Any time in first month of life

Next question

Neonatal blood spot screening


Neonatal blood spot screening (previously called the Guthrie test or 'heel-prick test') is performed at 5-9 days of life
The following conditions are currently screened for:

congenital hypothyroidism
cystic fibrosis
phenylketonuria
sickle cell disease
medium chain acyl-CoA dehydrogenase deficiency (MCADD)

Question 37 of 231

A mother brings her 2-week-old baby girl into the surgery for review. She has noted a bright red, well-circumscribed and lobulated lesion
developing on her right temple. This wasn't noted at birth but is now 5 mm in diameter. What is the most appropriate management?
Silver nitrate cautery

Refer to paediatrics (on an urgent basis)

Topical hydrocortisone + regular emollient

Refer to paediatrics (on a routine basis)

Reassure the mother that most lesions spontaneously regress

Next question

This baby has a strawberry naevus. Treatment is only usually required if the lesion is causing a mechanical problem or bleeding.

Strawberry naevus
Strawberry naevi (capillary haemangioma) are usually not present at birth but may develop rapidly in the first month of life. They appear as
erythematous, raised and multilobed tumours.
Typically they increase in size until around 6-9 months before regressing over the next few years (around 95% resolve before 10 years of
age).
Common sites include the face, scalp and back. Rarely they may be present in the upper respiratory tract leading to potential airway
obstruction
Capillary haemangiomas are present in around 10% of white infants. Female infants, premature infants and those of mothers who have
undergone chorionic villous sampling are more likely to be affected
Potential complications

mechanical e.g. Obstructing visual fields or airway


bleeding
ulceration

thrombocytopaenia

If treatment is required (e.g. Visual field obstruction) then systemic steroids are used
Cavernous haemangioma is a deep capillary haemangioma

Question 38 of 231

You are called to the treatment room of a GP surgery as a 12-month-old boy has developed a rash and breathing difficulties following a
routine vaccination. On examination he is developing swelling around the mouth and neck. What is the most appropriate initial action?
Phone 999 and reassure mother

IM adrenaline 150 mcg (0.15ml of 1 in 1,000)

IM adrenaline 300 mcg (0.3ml of 1 in 1,000)

IM adrenaline 50 mcg (0.05ml of 1 in 1,000)

Salbutamol nebuliser stat

Next question

Anaphylaxis
Anaphylaxis may be defined as a severe, life-threatening, generalised or systemic
hypersensitivity reaction.
Anaphylaxis is one of the few times when you would not have time to look up the dose of a medication. The Resuscitation Council
guidelines on anaphylaxis have recently been updated. Adrenaline is by far the most important drug in anaphylaxis and should be given as
soon as possible. The recommended doses for adrenaline, hydrocortisone and chlorphenamine are as follows:
Adrenaline

Hydrocortisone Chlorphenamine

< 6 months

150 mcg (0.15ml 1 in 1,000)25 mg

250 mcg/kg

6 months - 6 years

150 mcg (0.15ml 1 in 1,000)50 mg

2.5 mg

6-12 years

300 mcg (0.3ml 1 in 1,000) 100 mg

5 mg

Adult and child > 12 years 500 mcg (0.5ml 1 in 1,000) 200 mg

10 mg

Adrenaline can be repeated every 5 minutes if necessary. The best site for IM injection is the anterolateral aspect of the middle third of the
thigh.
Common identified causes of anaphylaxis

food (e.g. Nuts) - the most common cause in children


drugs
venom (e.g. Wasp sting)

Question 39 of 231

Which one of the following is an example of a primary accident prevention strategy?


Stair guards

Laminated safety glass

Cycling helmets

Smoke alarms

Wearing seat belts

Next question

Accidents in children
Around 15-20% of children attend Emergency Departments in the course of a year due to an accident. Accidents account for a third of all
childhood deaths and are the single most common cause of death in children aged between 1 - 15 years of age.
Key points

road traffic accidents are the most common cause of fatal accidents
boys and children from lower social classes are more likely to have an accident

Accident prevention
Accident prevention can be divided up into primary (preventing the accident from happening), secondary (prevent injury from the accident)
and tertiary (limit the impact of the injury) prevention strategies
The table below gives examples of accident prevention strategies
Primary prevention

Secondary preventionTertiary prevention

Stair guards
Wearing seat belts
Speed limits*
Cycling helmets
Teaching parents first aid
Teaching road safety Smoke alarms
Window safety catchesLaminated safety glass

*some strategies such as reducing driving speed may have a role in both primary and secondary accident prevention

Question 40 of 231

A 2-year-old boy is brought to the surgery by his mother with earache and pyrexia. On examination of the precordium a murmur is heard.
Which one of the following characteristics is not consistent with an innocent murmur?

Short buzzing murmur in the aortic area

Soft-blowing murmur in the pulmonary area

Varies with posture

Diastolic murmur

Continuous blowing noise heard just below the clavicles

Next question

Innocent murmurs
Innocent murmurs heard in children include
Ejection
murmurs

Due to turbulent blood flow at the outflow tract of the heart

Venous hums

Due to the turbulent blood flow in the great veins returning to the heart. Heard as a continuous blowing noise heard just
below the clavicles

Still's murmur

Low-pitched sound heard at the lower left sternal edge

Characteristics of an innocent ejection murmur include:

soft-blowing murmur in the pulmonary area or short buzzing murmur in the aortic area
may vary with posture
localised with no radiation
no diastolic component
no thrill
no added sounds (e.g. clicks)
asymptomatic child
no other abnormality

Question 41 of 231

Which one of the following statements regarding Perthes disease is incorrect?


Typically affects children between the ages of 4-8 years

Complications include premature fusion of the growth plates

Due to avascular necrosis of the femoral head

Twice as common in girls

10% of cases are bilateral

Next question

Perthes disease
Perthes disease is a degenerative condition affecting the hip joints of children, typically between the ages of 4-8 years. It is due to
avascular necrosis of the femoral head
Perthes disease is 5 times more common in boys. Around 10% of cases are bilateral
Features

hip pain: develops progressively over a few weeks


limp
stiffness and reduced range of hip movement
x-ray: early changes include widening of joint space, later changes include decreased femoral head size/flattening

Complications

osteoarthritis
premature fusion of the growth plates

Image used on license from Radiopaedia

Perthes disease - both femoral epiphyses show extensive destruction, the acetabula are deformed

Image used on license from Radiopaedia

Perthes disease - bilateral disease

Question 42 of 231

Which one of the following statements regarding child health surveillance in the UK is incorrect?
The health visitor distraction test is the first screening test done on infants hearing

The midwife should visit the mother for at least the first 2 weeks following birth

Antenatal screening for HIV is routinely offered in the UK

The GP routinely exams the child aged 6-8 weeks

Cystic fibrosis is one of the conditions the heel-prick test is used to screen for

Next question

The Newborn Hearing Screening Programme is gradually replacing distraction testing as the major screening test of infant hearing. In
everyday practice it is uncommon for midwives to visit up to 4 weeks

Child health surveillance


The following table gives a basic outline of child health surveillance in the UK

Antenatal

Ensure intrauterine growth


Check for maternal infections e.g. HIV
Ultrasound scan for fetal abnormalities
Blood tests for Neural Tube Defects

Newborn

Clinical examination of newborn


Newborn Hearing Screening Programme e.g. oto-acoustic emissions test
Give mother Personal Child Health Record

First month

Heel-prick test day 5-9 - hypothyroidism, PKU, metabolic diseases, cystic fibrosis, medium-chain acyl Co-A
dehydrogenase deficiency (MCADD)
Midwife visit up to 4 weeks*

Following
months

Health visitor input


GP examination at 6-8 weeks
Routine immunisations

Pre school

National orthoptist-led programme for pre-school vision screening to be introduced

Ongoing

Monitoring of growth, vision, hearing


Health professionals advice on immunisations, diet, accident prevention

*this doesn't seem to happen in practice with health visitors usually taking over at 2 weeks

Question 43 of 231

Which one of the following types of glomerulonephritis is most characteristically associated with streptococcal infection in children?
Focal segmental glomerulosclerosis

Diffuse proliferative glomerulonephritis

Membranous glomerulonephritis

Mesangiocapillary glomerulonephritis

Rapidly progressive glomerulonephritis

Next question

Glomerulonephritides
Knowing a few key facts is the best way to approach the difficult subject of glomerulonephritis:
Membranous glomerulonephritis

presentation: proteinuria / nephrotic syndrome / chronic kidney disease

cause: infections, rheumatoid drugs, malignancy


1/3 resolve, 1/3 respond to cytotoxics, 1/3 develop chronic kidney disease

IgA nephropathy - aka Berger's disease, mesangioproliferative GN

typically young adult with haematuria following an URTI

Diffuse proliferative glomerulonephritis

classical post-streptococcal glomerulonephritis in child


presents as nephritic syndrome / acute kidney injury
most common form of renal disease in SLE

Minimal change disease

typically a child with nephrotic syndrome (accounts for 80%)


causes: Hodgkin's, NSAIDs
good response to steroids

Focal segmental glomerulosclerosis

may be idiopathic or secondary to HIV, heroin


presentation: proteinuria / nephrotic syndrome / chronic kidney disease

Rapidly progressive glomerulonephritis - aka crescentic glomerulonephritis

rapid onset, often presenting as acute kidney injury


causes include Goodpasture's, ANCA positive vasculitis

Mesangiocapillary glomerulonephritis (membranoproliferative)

type 1: cryoglobulinaemia, hepatitis C


type 2: partial lipodystrophy

Question 44 of 231

Which one of the following statements regarding hand, foot and mouth disease is incorrect?
May be caused by enterovirus 71

Typically occurs in outbreaks

Palm and sole lesions tend to occur before oral ulcers

Children do not need to be excluded from school

Mild systemic upset is common

Next question

In hand, foot and mouth disease oral lesion usually occur before palm and sole lesions

Hand, foot and mouth disease


Hand, foot and mouth disease is a self-limiting condition affecting children. It is caused by the intestinal viruses of the Picornaviridae family
(most commonly coxsackie A16 and enterovirus 71). It is very contagious and typically occurs in outbreaks at nursery
Clinical features

mild systemic upset: sore throat, fever


oral ulcers
followed later by vesicles on the palms and soles of the feet

Image used on license from DermNet NZ

Image used on license from DermNet NZ

Management

general advice about hydration and analgesia


reassurance no link to disease in cattle
children do not need to be excluded from school*

*The HPA recommends that children who are unwell should be kept off school until they feel better. They also advise that you contact them
if you suspect that there may be a large outbreak.

Question 45 of 231

Which one of the following drugs is contra-indicated whilst breast feeding?


Heparin

Lithium

Penicillin

Warfarin

Carbamazepine

Next question

Breast feeding: contraindications


The major breastfeeding contraindications tested in exams relate to drugs (see below). Other contraindications of note include:

galactosaemia
viral infections - this is controversial with respect to HIV in the developing world. This is because there is such an increased
infant mortality and morbidity associated with bottle feeding that some doctors think the benefits outweigh the risk of HIV
transmission

Drug contraindications
The following drugs can be given to mothers who are breast feeding:

antibiotics: penicillins, cephalosporins, trimethoprim


endocrine: glucocorticoids (avoid high doses), levothyroxine*
epilepsy: sodium valproate, carbamazepine
asthma: salbutamol, theophyllines
psychiatric drugs: tricyclic antidepressants, antipsychotics**
hypertension: beta-blockers, hydralazine, methyldopa
anticoagulants: warfarin, heparin
digoxin

The following drugs should be avoided:

antibiotics: ciprofloxacin, tetracycline, chloramphenicol, sulphonamides


psychiatric drugs: lithium, benzodiazepines
aspirin
carbimazole
sulphonylureas
cytotoxic drugs
amiodarone

*the BNF advises that the amount is too small to affect neonatal hypothyroidism screening
**clozapine should be avoided

Question 46 of 231

A 5-year-old boy is diagnosed as having absence seizures. What is the chance he will be seizure free by the age of 16-years-old?
5-10%

20-25%

40-45%

65-70%

90-95%

Next question

Absence seizures - good prognosis: 90-95% become seizure free in adolescence

Absence seizures
Absence seizures (petit mal) are a form of generalised epilepsy that is mostly seen in children. The typical age of onset of 3-10 years old
and girls are affected twice as commonly as boys
Features

absences last a few seconds and are associated with a quick recovery
seizures may be provoked by hyperventilation or stress
the child is usually unaware of the seizure
they may occur many times a day
EEG: bilateral, symmetrical 3Hz spike and wave pattern

Management

sodium valproate and ethosuximide are first-line treatment


good prognosis - 90-95% become seizure free in adolescence

Question 47 of 231

Which one of the following statements regarding cleft lip and palate is incorrect?
Cleft lip is repaired earlier than cleft palate

Affect around 5 in every 1,000 babies

Cleft palate results from failure of the palatine processes and the nasal septum to fuse

Maternal antiepileptic use increases risk

Polygenic inheritance

Next question

Cleft lip and palate


Cleft lip and palate affect around 1 in every 1,000 babies.
Pathophysiology

polygenic inheritance
maternal antiepileptic use increases risk
cleft lip results from failure of the fronto-nasal and maxillary processes to fuse
cleft palate results from failure of the palatine processes and the nasal septum to fuse

Problems

feeding: orthodontic devices may be helpful


speech: with speech therapy 75% of children develop normal speech
increased risk of otitis media for cleft palate babies

Management

cleft lip is repaired earlier than cleft palate, with practices varying from repair in the first week of life to three months
cleft palates are typically repaired between 6-12 months of age

Question 48 of 231

Which of the following is a live attenuated vaccine?


Rabies

Mumps

Hepatitis B

Meningococcus

Diphtheria

Next question

Live attenuated vaccines

BCG
MMR
oral polio
yellow fever
oral typhoid

Vaccinations
It is important to be aware of vaccines which are of the live-attenuated type as these may pose a risk to immunocompromised patients. The
main types of vaccine are as follows:
Live attenuated

BCG
measles, mumps, rubella (MMR)
influenza (intranasal)
oral rotavirus
oral polio
yellow fever
oral typhoid*

Inactivated preparations

rabies
influenza (intramuscular)

Detoxified exotoxins

tetanus

Extracts of the organism/virus (sometimes termed fragment)**

diphtheria
pertussis ('acellular' vaccine)
hepatitis B
meningococcus, pneumococcus, haemophilus

Notes

influenza: different types are available, including whole inactivated virus, split virion (virus particles disrupted by detergent
treatment) and sub-unit (mainly haemagglutinin and neuraminidase)
cholera: contains inactivated Inaba and Ogawa strains of Vibrio cholerae together with recombinant B-subunit of the cholera
toxin
hepatitis B: contains HBsAg adsorbed onto aluminium hydroxide adjuvant and is prepared from yeast cells using recombinant
DNA technology

*whole cell typhoid vaccine is no longer used in the UK


**may also be produced using recombinant DNA technology

Question 49 of 231

A mother brings her 3-year-old child in to receive the DTP booster. Which one of the following would make it inappropriate to give the
vaccination today?
Being below the 2nd centile for weight

Family history of allergy to DTP

Recent onset of a seizure disorder currently being investigated

Planned general anaesthesia in 2 weeks time

Being born at 29 weeks gestation

Next question

Immunisation
The Department of Health published guidance in 2006 on the safe administration of vaccines in its publication 'Immunisation against
infectious disease'
General contraindications to immunisation

confirmed anaphylactic reaction to a previous dose of a vaccine containing the same antigens
confirmed anaphylactic reaction to another component contained in the relevant vaccine (e.g. egg protein)

Situations where vaccines should be delayed

febrile illness/intercurrent infection

Contraindications to live vaccines

pregnancy
immunosuppression

Specific vaccines

DTP: vaccination should be deferred in children with an evolving or unstable neurological condition

Not contraindications to immunisation

asthma or eczema
history of seizures (if associated with fever then advice should be given regarding antipyretics)
breastfed child
previous history of natural pertussis, measles, mumps or rubella infection
history of neonatal jaundice
family history of autism
neurological conditions such as Down's or cerebral palsy
low birth weight or prematurity
patients on replacement steroids e.g. (CAH)

Question 50 of 231

A 18-month-old boy is brought to surgery as his mother is concerned about his hearing. Which one of the following conditions is least
associated with hearing problems in children?

Cerebral palsy

Congenital rubella infection

Pendred syndrome

Tuberous sclerosis

Secretory otitis media

Next question

Tuberous sclerosis is not commonly associated with hearing problems in children

Hearing problems in children


The most common causes of hearing problems in children are listed below
Conductive

secretory otitis media


Down's syndrome*

Sensorineural

hereditary - Usher syndrome, Pendred syndrome, Jervell-Lange-Nielson syndrome, Wardenburg syndrome


congenital infection e.g. rubella
acquired - meningitis, head injury
cerebral palsy
perinatal insult

*may have elements of sensorineural loss as well

Question 51 of 231

Which one of the following is not a notifiable disease in the United Kingdom?
Tuberculosis

HIV

Measles

Whooping cough

Meningococcal septicaemia

Next question

HIV is not a notifiable disease

Notifiable diseases
Below is a list of notifiable diseases in the UK. The 'proper officer' of the Local Authority (usually a consultant in communicable disease)
needs to be notified. They in turn will notify the Health Protection Agency on a weekly basis.
Notable exceptions include:

HIV

In April 2010 the following diseases were removed from the list:

Dysentery
Ophthalmia neonatorum
Leptospirosis
Relapsing fever

Therefore, the current notifiable diseases are:

Acute encephalitis
Acute infectious hepatitis
Acute meningitis
Acute poliomyelitis
Anthrax
Botulism
Brucellosis
Cholera
Diphtheria
Enteric fever (typhoid or paratyphoid fever)
Food poisoning
Haemolytic uraemic syndrome (HUS)
Infectious bloody diarrhoea
Invasive group A streptococcal disease
Legionnaires Disease
Leprosy
Malaria
Measles
Meningococcal septicaemia
Mumps
Plague
Rabies
Rubella
SARS
Scarlet fever
Smallpox
Tetanus
Tuberculosis
Typhus

Viral haemorrhagic fever (VHF)


Whooping cough
Yellow fever
Question 1 of 180

A 3-year-old girl is taken to the GP due to a rash on her upper arm. On examination multiple raised lesions of about 2 mm in
diameter are seen. On close inspection a central dimple is present in the majority of lesions. What is the likely diagnosis?

Roseola infantum

Molluscum contagiosum

Kawasaki disease

Viral warts

Pityriasis rosea

Next question

Molluscum contagiosum

Molluscum contagiosum is caused by a pox DNA virus infection. It is typically seen in younger children and results in characteristic
small, pearly, umbilicated lesions
Molluscum contagiosum is highly infectious.
Lesions may be present for up to 12 months and usually resolve spontaneously. Whilst various treatments may be effective in
removing the lesions (e.g. surgery, cryotherapy, topical agents) no treatment is recommend in the initial phase due to the benign
nature of the condition

Image used on license from DermNet NZ and with the kind permission of Prof Raimo Suhonen

Image used on license from DermNet NZ

Question 2 of 180

At what age do the majority of children achieve day and night time urinary continence?
2-3 years old

3-4 years old

4-5 years old

5-6 years old

6-7 years old

Next question

Nocturnal enuresis
The majority of children achieve day and night time continence by 3 or 4 years of age. Enuresis may be defined as the 'involuntary
discharge of urine by day or night or both, in a child aged 5 years or older, in the absence of congenital or acquired defects of the nervous
system or urinary tract'
Nocturnal enuresis can be defined as either primary (the child has never achieved continence) or secondary (the child has been dry for at
least 6 months before)
NICE issued guidance in 2010. Management:

look for possible underlying causes/triggers (e.g. Constipation, diabetes mellitus, UTI if recent onset)
advise on fluid intake, diet and toileting behaviour
reward systems (e.g. Star charts). NICE recommend these 'should be given for agreed behaviour rather than dry nights' e.g.
Using the toilet to pass urine before sleep
NICE advise: 'Consider whether alarm or drug treatment is appropriate, depending on the age, maturity and abilities of the child
or young person, the frequency of bedwetting and the motivation and needs of the family'. Generally:
an enuresis alarm is first-line for children under the age of 7 years
desmopressin may be used first-line for children over the ago 7 years, particularly if short-term control is needed or an enuresis
alarm has been ineffective/is not acceptable to the family
please see the link for more details

Bedwetting (enuresis) - Summary

Bedwetting is involuntary wetting during sleep, without any inherent suggestion of


frequency or pathophysiology.
Bedwetting can be classified as:
o Primary bedwetting the child has never achieved sustained continence at night.
o Secondary bedwetting bedwetting occurs after the child has been dry at night for
more than 6 months.
Primary bedwetting without daytime symptoms is thought to be caused by sleep
arousal difficulties, polyuria, and/or bladder dysfunction.
o Daytime symptoms include urgency, frequency, daytime wetting, abdominal
straining or poor urinary stream, pain passing urine, or passing urine fewer than four
times a day.
Primary bedwetting with daytime symptoms is usually caused by disorders of the lower
urinary tract.
o It is most commonly caused by an overactive bladder, but may also be caused by
congenital malformations or neurological disorders.
o It is occasionally caused by chronic constipation, chronic urinary tract infection
(UTI), or chronic emotional problems.
Secondary bedwetting is bedwetting secondary to an underlying cause, such as
diabetes, UTI, constipation, or family problems (vulnerable child or family).
Risk factors associated with bedwetting include:
o A family history of bedwetting.
o Gender boys are more likely than girls to have bedwetting.
o Delay in attaining bladder control.
o Being obese approximately 30% of obese children have bedwetting.
o Psychological or behavioural disorders such as attention deficit hyperactivity
disorder (ADHD), autism spectrum disorder, anxiety, depressive, and conduct
disorders.
It is necessary to determine the type of bedwetting by asking if:
o There are any daytime symptoms.
o The child has previously been dry at night without assistance for 6 months.
Managing primary bedwetting (without daytime symptoms) involves:
o General advice on bedwetting.
o Advice on lifting and waking.
o Addressing any issues concerning excessive or insufficient fluid intake, diet, and
toileting patterns.
o In children < 5 years of age, reassurance that many children younger than 5 years of
age wet the bed and that this usually resolves without treatment.

In children > 5 years of age, if bedwetting is infrequent (less than twice a week),
reassurance that bedwetting may resolve without treatment and offering the option
of a wait-and-see approach. If long-term treatment is required, treatment with an
enuresis alarm (first-line treatment) should be offered in combination with positive
reward systems desmopressin may be considered if the alarm is unsuitable. If
rapid or short-term control of bedwetting is required, treatment with desmopressin
is an option.
Managing primary bedwetting (with daytime symptoms) involves:
o Referring the child to secondary care or an enuresis clinic for further investigations
and assessment.
o Considering referral for younger children (older than 2 years of age) who are
struggling to not wet themselves during the day as well as during the night, despite
awareness of toileting needs and showing appropriate toileting behaviour.
Managing secondary enuresis involves:
o Treatment of UTIs and constipation in primary care.
o Referral to a paediatrician or an enuresis clinic if children have other underlying
causes which have been identified but are generally not managed in primary care
(e.g. diabetes and learning difficulties).
o

Question 3 of 180

A 23-year-old female with Down's syndrome is reviewed in clinic. Which one of the following features is least associated with her condition?
Infertility

Hypothyroidism

Alzheimer's disease

Short stature

Ventricular septal defect

Next question

As this patient is female she is likely to be subfertile rather than infertile - please see the notes below

Down syndrome: features


Clinical features

face: upslanting palpebral fissures, epicanthic folds, Brushfield spots in iris, protruding tongue, small ears, round/flat face
flat occiput
single palmar crease, pronounced 'sandal gap' between big and first toe
hypotonia
congenital heart defects (40-50%, see below)
duodenal atresia
Hirschsprung's disease

Cardiac complications

multiple cardiac problems may be present


endocardial cushion defect (c. 40%, also known as atrioventricular septal canal defects)
ventricular septal defect (c. 30%)
secundum atrial septal defect (c. 10%)
tetralogy of Fallot (c. 5%)
isolated patent ductus arteriosus (c. 5%)

Later complications

subfertility: males are almost always infertile due to impaired spermatogenesis. Females are usually subfertile, and have an
increased incidence of problems with pregnancy and labour
learning difficulties
short stature
repeated respiratory infections (+hearing impairment from glue ear)
acute lymphoblastic leukaemia
hypothyroidism
Alzheimer's
atlantoaxial instability

Question 4 of 180

A 9-year-old girl is brought to surgery as her mother is concerned that she is too fat. This has now been a problem for over two years and
mum feels this is holding her back at school. What is the most appropriate method to ascertain how obese she is?
Body mass index

Body mass index percentile adjusted to age and gender

Weight plotted on percentile chart

Mother's perception

Waist circumference

Next question

Obesity in children
Defining obesity is more difficult in children than adults as body mass index (BMI) varies with age. BMI percentile charts are therefore
needed to make an accurate assessment. Recent NICE guidelines suggest to use 'UK 1990 BMI charts to give age- and gender-specific
information'
NICE recommend

consider tailored clinical intervention if BMI at 91st centile or above.


consider assessing for comorbidities if BMI at 98th centile or above

By far the most common cause of obesity in childhood is lifestyle factors. Other associations of obesity in children include:

Asian children: four times more likely to be obese than white children
female children
taller children: children with obesity are often above the 50th percentile in height

Cause of obesity in children

growth hormone deficiency


hypothyroidism
Down's syndrome
Cushing's syndrome
Prader-Willi syndrome

Consequences of obesity in children

orthopaedic problems: slipped upper femoral epiphyses, Blount's disease (a development abnormality of the tibia resulting in
bowing of the legs), musculoskeletal pains
psychological consequences: poor self-esteem, bullying
sleep apnoea
benign intracranial hypertension
long-term consequences: increased incidence of type 2 diabetes mellitus, hypertension and ischaemic heart disease

Question 5 of 180

Which one of the following statements regarding acne vulgaris is incorrect?


Follicular epidermal hyperproliferation results in obstruction of the pilosebaceous follicle

Acne vulgaris affects at least 80% of teenagers

Propionibacterium acnes is an anaerobic bacterium

Typical lesions include comedones and pustules

Beyond the age of 25 years acne vulgaris is more common in males

Next question

Acne is actually more common in females after the age of 25 years

Acne vulgaris
Acne vulgaris is a common skin disorder which usually occurs in adolescence. It typically affects the face, neck and upper trunk and is
characterised by the obstruction of the pilosebaceous follicle with keratin plugs which results in comedones, inflammation and pustules.
Epidemiology

affects around 80-90% of teenagers, 60% of whom seek medical advice


acne may also persist beyond adolescence, with 10-15% of females and 5% of males over 25 years old being affected

Pathophysiology is multifactorial

follicular epidermal hyperproliferation resulting in the formation of a keratin plug. This in turn causes obstruction of the
pilosebaceous follicle. Activity of sebaceous glands may be controlled by androgen, although levels are often normal in patients
with acne
colonisation by the anaerobic bacterium Propionibacterium acnes
inflammation

Acne vulgaris - Summary


Acne vulgaris is a chronic skin condition in which blockage or inflammation of the hair
follicles and accompanying sebaceous glands (known as pilosebaceous units) occurs. It
principally affects the face, the back, and the chest, and usually first occurs around the
age of puberty.
Acne is most prevalent among adolescents and young adults, affecting approximately
80% of people at some point between 1130 years of age.
Complications of acne include scarring, hyperpigmentation, and psychological problems.
In general, it is recommended that people with acne:
Do not wash more than twice a day.
Use a mild soap or cleanser and lukewarm water.
Do not use vigorous scrubbing when washing acne-affected skin, and the use of abrasive
soaps, cleansing granules, astringents, or exfoliating agents should be discouraged.
Should not attempt to 'clean' blackheads.
Ideally, should avoid excessive use of makeup and cosmetics.
Use a fragrance-free, water-based emollient if dry skin is a problem.
In mild acne, open and closed comedones (blackheads and whiteheads) predominate.
A single topical treatment should be prescribed topical retinoid (tretinoin,
isotretinoin, or adapalene) or benzoyl peroxide (especially if papules and pustules are
present) as first-line treatment.
Azelaic acid should be prescribed if both topical retinoids and benzoyl peroxide are
poorly tolerated.
Prescription of a standard combined oral contraceptive should be considered in women
who require contraception.
In moderate acne, inflammatory lesions (papules and pustules) predominate. The acne
may be widespread, there may be a risk of scarring, and there may be considerable
psychosocial morbidity.
A single topical drug should be considered for people with limited acne which is unlikely
to scar (benzoyl peroxide or a topical retinoid).
Combined treatment should be considered for all people with moderate acne. A topical
antibiotic combined with benzoyl peroxide or a topical retinoid is the preferred regimen,
as it is proven to be effective and may limit the development of bacterial resistance.
Where possible, a topical antibiotic course should be limited to a maximum of 12 weeks.

An oral antibiotic (tetracyclines or erythromycin) combined with a topical treatment


(but not a topical antibiotic) should be considered if there is moderate acne on the back
or shoulders, or if there is a significant risk of scarring or substantial pigment change.
A standard combined oral contraceptive (COC) should be considered in women who
require contraception.
In severe acne, there are nodules and cysts (nodulocystic acne), as well as a
preponderance of inflammatory papules and pustules. There is a high risk of scarring (or
scarring may already be evident), and there is likely to be considerable psychosocial
morbidity.
All people with severe acne should be referred for specialist assessment and treatment.
Prescription of an oral antibiotic in combination with a topical drug should be
considered whilst waiting for an appointment.
Prescription of a COC should be considered for women who require contraception.
Standard COCs are suitable for most women. Co-cyprindiol (Dianette) should be
considered only when topical treatment or systemic antibiotics has failed.
Have I got the right topic?
Age from 12 years onwards
This CKS topic covers the management of the most common type of acne acne
vulgaris.
This CKS topic does not cover chemically-induced acne, or acneiform lesions associated
with corticosteroids.
There is a separate CKS topic on Rosacea.
The target audience for this CKS topic is healthcare professionals working within the
NHS in the UK, and providing first contact or primary health care.
CKS gratefully acknowledges the contribution of the British Association of
Dermatologists in the development of this topic.
How up-to-date is this topic?
Changes
Update
Goals and outcome measures
Goals
Background information
Definition
Causes
Prevalence
Complications
Prognosis
Diagnosis and assessment
Diagnosis and assessment of acne vulgaris
Diagnosis
Differential diagnosis
Investigations in women

Assessment
Management
Scenario: Mild acne : covers the management of acne that consists mainly of noninflammatory comedones. It is usually limited in its extent.
Scenario: Moderate acne : covers the management of acne that consists of a mixture of
non-inflammatory comedones and inflammatory papules and pustules. It may extend to
the shoulders and back.
Scenario: Severe acne : covers the management of acne that is characterized by nodules
and cysts (nodulocystic acne), as well as a preponderance of inflammatory papules and
pustules. It may be extensive.
Scenario: Mild acne
Scenario: Moderate acne
Scenario: Severe acne
Prescribing information
Important aspects of prescribing information relevant to primary healthcare are covered
in this section specifically for the drugs recommended in this CKS topic. For further
information on contraindications, cautions, drug interactions, and adverse effects, see
the electronic Medicines Compendium (eMC) (http://medicines.org.uk/emc), or the
British National Formulary (BNF) (www.bnf.org).
Benzoyl peroxide
Topical retinoids
Topical antibiotics
Azelaic acid
Oral antibiotics
Combined oral contraceptives
Evidence
Supporting evidence
Search strategy
References
ABPI Medicines Compendium (2012) Summary of product characteristics for Dianette.
Electronic Medicines CompendiumDatapharm Communications Ltd.
www.medicines.org.uk [Free Full-text]
ABPI Medicines Compendium (2013a) Summary of product characteristics for Skinoren
cream. Electronic Medicines CompendiumDatapharm Communications Ltd.
www.medicines.org.uk [Free Full-text]
ABPI Medicines Compendium (2013b) Summary of product characteristics for Finacea
15% gel. Electronic Medicines CompendiumDatapharm Communications Ltd.
www.medicines.org.uk [Free Full-text]
ABPI Medicines Compendium (2013c) Summary of product characteristics for
Isotretinoin 5mg capsules. Electronic Medicines CompendiumDatapharm
Communications Ltd. www.medicines.org.uk [Free Full-text]

ABPI Medicines Compendium (2014a) Summary of product characteristics for Dalacin


cream 2%. Electronic Medicines CompendiumDatapharm Communications Ltd.
www.medicines.org.uk [Free Full-text]
ABPI Medicines Compendium (2014b) Summary of product characteristics for Epiduo
0.1%/2.5% gel. Electronic Medicines CompendiumDatapharm Communications Ltd.
www.medicines.org.uk [Free Full-text]
Acne Working Group (2008) Management of mild and moderate acne vulgaris. GP
Review January(), 1-11.
AHRQ (2001) Management of acne. . Evidence Report/Technology Assessment No. 17.
Agency for Health Care Research and Quality. www.ahrq.gov [Free Full-text]
Alirezai, M., George, S.A., Coutts, I. et al. (2007) Daily treatment with adapalene gel 0.1%
maintains initial improvement of acne vulgaris previously treated with oral lymecycline.
European Journal of Dermatology 17(1), 45-51. [Abstract] [Free Full-text]
Archer, C.B., Cohen, S.N., Baron, S.E. and British Association of Dermatologists and Royal
College of General Practitioners (2012) Guidance on the diagnosis and clinical
management of acne. Clinical and Experimental Dermatology 37(Suppl 1), 1-6. [Abstract]
[Free Full-text]
Arora, M.K., Yadav, A. and Saini, V. (2011) Role of hormones in acne vulgaris. Clinical
Biochemistry 44(13), 1035-1040. [Abstract]
Arowojolu, A.O., Gallo, M., Lopez, L.M. et al. (2007) Combined oral contraceptive pills
for treatment of acne (Cochrane Review). The Cochrane Library. Issue 1. John Wiley &
Sons, Ltd. www.thecochranelibrary.com [Free Full-text]
Arowojolu, A.O., Gallo, M.F., Lopez, L.M. and Grimes, D.A. (2012) Combined oral
contraceptive pills for treatment of acne (Cochrane review). The Cochrane Library. Issue
7. John Wiley & Sons, Ltd. www.thecochranelibrary.com [Free Full-text]
Baxter, K. and Preston, C.L. (Eds.) (2013) Stockley's drug interactions 2013: pocket
companion. London: Pharmaceutical Press.
Berson, D.S. and Marchant, M. (2003) Topical retinoids in primary care. Medscape.
www.medscape.com
Bhardwaj, S.S., Rohrer, T.E. and Arndt, K. (2005) Lasers and light therapy for acne
vulgaris. Seminars in Cutaneous Medicine & Surgery 24(2), 107-112. [Abstract]
Bikowski, J.B. (2005) Clinical experience results with clindamycin 1% benzoyl peroxide
5% gel (Duac) as monotherapy and in combination. Journal of Drugs in Dermatology
4(2), 164-171. [Abstract]
BNF 65 (2013) British National Formulary. 65th edn. London: British Medical Association
and Royal Pharmaceutical Society of Great Britain.
Bottomley, W.W. and Cunliffe, W.J. (1993) Oral trimethoprim as a third-line antibiotic in
the management of acne vulgaris. Dermatology 187(3), 193-196. [Abstract]
Bowman, S., Gold, M., Nasir, A. and Vamvakias, G. (2005) Comparison of
clindamycin/benzoyl peroxide, tretinoin plus clindamycin, and the combination of
clindamycin/benzoyl peroxide and tretinoin plus clindamycin in the treatment of acne

vulgaris: a randomized, blinded study. Journal of Drugs in Dermatology 4(5), 611-618.


[Abstract]
Brown, S.K. and Shalita, A.R. (1998) Acne vulgaris. Lancet 351(9119), 1871-1876.
Chia, C.Y., Lane, W., Chibnall, J. et al. (2005) Isotretinoin therapy and mood changes in
adolescents with moderate to severe acne: a cohort study. Archives of Dermatology
141(5), 557-560. [Abstract] [Free Full-text]
Coates, P., Vyakrnam, S., Eady, E.A. et al. (2002) Prevalence of antibiotic-resistant
propionibacteria on the skin of acne patients: 10-year surveillance data and snapshot
distribution study. British Journal of Dermatology 146(5), 840-848. [Abstract]
Cook, D., Krassas, G. and Huang, T. (2010) Acne - best practice. Australian family
physician 39(6), 656-660. [Abstract] [Free Full-text]
Cooper, A.J. (1998) Systematic review of Propionibacterium acnes resistance to systemic
antibiotics. Medical Journal of Australia 169(5), 259-261. [Abstract]
CSM (2002) Cyproterone acetate (Dianette): risk of venous thromboembolism (VTE).
Current Problems in Pharmacovigilance 28(Oct), 9-10. [Free Full-text]
Cunliffe, W.J. (1996) Minocycline for acne. Doctors should not change the way they
prescribe for acne. British Medical Journal 312(7038), 1101. [Free Full-text]
Cunliffe, W.J., Aldana, O.L. and Goulden, V. (1999) Oral trimethoprim: a relatively safe
and successful third-line treatment for acne vulgaris. British Journal of Dermatology
141(4), 757-758.
Cunliffe, W.J., Meynadier, J., Alirezai, M. et al. (2003) Is combined oral and topical
therapy better than oral therapy alone in patients with moderate to moderately severe
acne vulgaris? A comparison of the efficacy and safety of lymecycline plus adapalene gel
0.1%, versus lymecycline plus gel vehicle. Journal of the American Academy of
Dermatology 49(Suppl 3), S218-S226. [Abstract]
Dawson, A.L. (2013) Acne vulgaris. BMJ 346(), f2634.
do Nascimento, L.V., Guedes, A.C., Magalhaes, G.M. et al. (2003) Single-blind and
comparative clinical study of the efficacy and safety of benzoyl peroxide 4% gel (BID)
and adapalene 0.1% Gel (QD) in the treatment of acne vulgaris for 11 weeks. Journal of
Dermatological Treatment 14(3), 166-171. [Abstract]
Dos, S., Barbhuiya, J., Jana, S. and Dey, S. (2003) Comparative evaluation of clindamycin
phosphate 1% and clindamycin phosphate 1% with nicotinamide gel 4% in the
treatment of acne vulgaris. Indian Journal of Dermatology, Venereology and Leprology
69(1), 8-9. [Abstract] [Free Full-text]
Dreno, B. (2004) Topical antibacterial therapy for acne vulgaris. Drugs 64(21), 23892397. [Abstract]
Dreno, B., Bettoli, V., Ochsendorf, F. et al. (2004) European recommendations on the
use of oral antibiotics for acne. European Journal of Dermatology 14(6), 391-399.
[Abstract] [Free Full-text]
DTB (2002) Is Yasmin a "truly different" pill? Drug & Therapeutics Bulletin 40(8), 57-59.
[Abstract]

DTB (2006) Is minocycline overused in acne? Drug and Therapeutics Bulletin 44(8), 6064. [Abstract]
DTB (2009) Minocycline for acne - update. Drug and Therapeutics Bulletin 47(1), 7-8.
[Abstract]
Farmery, M.R., Jones, C.E., Eady, E.A. et al. (1994) In vitro activity of azelaic acid, benzoyl
peroxide and zinc acetate against antibiotic-resistant propionibacteria from acne
patients. Journal of Dermatological Treatment 5(2), 63-65.
FFPRHC (2007) First prescription of combined oral contraception. Faculty of Sexual &
Reproductive Healthcare. www.ffprhc.org.uk [Free Full-text]
Franks, S., Layton, A. and Glasier, A. (2008) Cyproterone acetate/ethinyl estradiol for
acne and hirsutism: time to revise prescribing policy. Human Reproduction 23(2), 231232. [Abstract] [Free Full-text]
FSRH (2009) UK medical eligibility criteria for contraceptive use. Faculty of Sexual and
Reproductive Healthcare. www.fsrh.org [Free Full-text]
FSRH (2011) Drug interactions with hormonal contraception. Faculty of Sexual and
Reproductive Healthcare. www.fsrh.org [Free Full-text]
Garner, S.E. (2003) Acne vulgaris. In: Williams, H., Bigby, M., Diepgen, T. et al. (Eds.)
Evidence-based dermatology. London: BMJ Publishing Group. Chapter 13. 87-114.
Garner, S.E., Eady, E.A., Popescu, C. et al. (2003) Minocycline for acne vulgaris: efficacy
and safety (Cochrane Review). The Cochrane Library. Issue 1. John Wiley & Sons, Ltd.
www.thecochranelibrary.com [Free Full-text]
Garner, S.E., Eady, A., Bennett, C. et al. (2012) Minocycline for acne vulgaris: efficacy
and safety (Cochrane review). The Cochrane Library. Issue 8. John Wiley & Sons, Ltd.
www.thecochranelibrary.com [Free Full-text]
Gold, L.S., Colon, L.E., Johnson, L.A. and Gottschalk, R.W. (2008) Is switching retinoids a
sound strategy for the treatment of acne vulgaris? Journal of Drugs in Dermatology 7(6
Suppl), s11-s17. [Abstract]
Gollnick, H., Cunliffe, W., Berson, D. et al. (2003) Management of acne: a report from a
global alliance to improve outcomes in acne. Journal of the American Academy of
Dermatology 49(Suppl 1), S1-S38.
Habbema, L., Koopmans, B., Menke, H.E. et al. (1989) A 4% erythromycin and zinc
combination (Zineryt) versus 2% erythromycin (Eryderm) in acne vulgaris: a randomized,
double-blind comparative study. British Journal of Dermatology 121(4), 497-502.
[Abstract]
Haider, A. and Shaw, J.C. (2004) Treatment of acne vulgaris. Journal of the American
Medical Association 292(6), 726-735. [Abstract] [Free Full-text]
Hamilton, F.L., Car, J., Lyons, C. et al. (2009) Laser and other light therapies for the
treatment of acne vulgaris: systematic review. British Journal of Dermatology 160(6),
1273-1285. [Abstract]
Healy, E. and Simpson, N. (1994) Acne vulgaris. British Medical Journal 308(6932), 831833. [Free Full-text]

Holland, K.T., Bojar, R.A., Cunliffe, W.J. et al. (1992) The effect of zinc and erythromycin
on the growth of erythromycin-resistant and erythromycin-sensitive isolates of
Propionibacterium acnes: an in-vitro study. British Journal of Dermatology 126(5), 505509. [Abstract]
James, W.D. (2005) Acne. New England Journal of Medicine 352(14), 1463-1472.
Jordan, R.E., Cummins, C.L., Burls, A.J.E. and Seukeran, D.C. (2000) Laser resurfacing for
facial acne scars (Cochrane Review). The Cochrane Library. Issue 3. John Wiley & Sons,
Ltd. www.thecochranelibrary.com [Free Full-text]
Ko, H.C., Song, M., Seo, S.H. et al. (2009) Prospective, open-label, comparative study of
clindamycin 1%/benzoyl peroxide 5% gel with adapalene 0.1% gel in Asian acne
patients: Efficacy and tolerability. Journal of The European Academy of Dermatology and
Venereology 23(3), 245-250. [Abstract]
Koltun, W., Lucky, A.W., Thiboutot, D. et al. (2008) Efficacy and safety of 3 mg
drospirenone/20 mcg ethinylestradiol oral contraceptive administered in 24/4 regimen
in the treatment of acne vulgaris: a randomized, double-blind, placebo-controlled trial.
Contraception 77(4), 249-256. [Abstract]
Korkut, C. and Piskin, S. (2005) Benzoyl peroxide, adapalene, and their combination in
the treatment of acne vulgaris. Journal of Dermatology 32(3), 169-173. [Abstract]
Kus, S., Yucelten, D. and Aytug, A. (2005) Comparison of efficacy of azithromycin vs.
doxycycline in the treatment of acne vulgaris. Clinical & Experimental Dermatology
30(3), 215-220. [Abstract]
Langner, A., Sheehan-Dare, R. and Layton, A. (2007) A randomized, single-blind
comparison of topical clindamycin + benzoyl peroxide (Duac) and erythromycin + zinc
acetate (Zineryt) in the treatment of mild to moderate facial acne vulgaris. Journal of
the European Academy of Dermatology and Venereology 21(3), 311-319. [Abstract]
Layton, A.M. (2000) Acne vulgaris and similar eruptions. Medicine 28(12), 46-50.
Layton, A.M. (2010) Disorders of the sebaceous glands. In: Burns, T., Breathnach, S., Cox,
N. and Griffiths, C. (Eds.) Rook's textbook of dermatology. 8th edn. Chichester: WileyBlackwell. 42.1-42.89.
Leyden, J.J. (1997) Therapy for acne vulgaris. New England Journal of Medicine 336(16),
1156-1162.
Magin, P., Pond, D., Smith, W. and Watson, A. (2005a) A systematic review of the
evidence for 'myths and misconceptions' in acne management: diet, face-washing and
sunlight. Family Practice 22(1), 1-9. [Abstract] [Free Full-text]
Magin, P., Pond, D. and Smith, W. (2005b) Isotretinoin, depression and suicide: a review
of the evidence. British Journal of General Practice 55(511), 134-138. [Abstract] [Free
Full-text]
Magin, P.J., Adams, J., Pond, C.D. and Smith, W. (2006) Topical and oral CAM in acne: a
review of the empirical evidence and a consideration of its context. Complementary
Therapies in Medicine 14(1), 62-76. [Abstract]
Maloney, J.M., Dietze, P., Watson, D. et al. (2008) Treatment of acne using a 3-milligram
drospirenone/20-microgram ethinyl estradiol oral contraceptive administered in a 24/4

regimen: a randomized controlled trial. Obstetrics and Gynecology 112(4), 773-781.


[Abstract]
Martin, K.W. and Ernst, E. (2003) Herbal medicines for treatment of bacterial infections:
a review of controlled clinical trials. Journal of Antimicrobial Chemotherapy 51(2), 241246. [Abstract] [Free Full-text]
Medicines Resource (1997) Management of acne vulgaris. Medicines Resource 37(Feb),
143-146.
MeReC (1999) The treatment of acne vulgaris: an update. MeReC Bulletin 10(8), 29-32.
[Free Full-text]
MeReC (2006) Contraception - current issues. MeReC Bulletin 17(2), 1-9. [Free Full-text]
MHRA (2011) Isotretinoin for severe acne. Medicines and Healthcare Regulatory
Agency. www.mhra.gov.uk [Free Full-text]
MHRA (2013) Cyproterone acetate with ethinylestradiol (co-cyprindiol): balance of
benefits and risks remains positiveupdated prescribing advice provided. Drug Safety
Update 6(11), A3. [Free Full-text]
Mills, O., Thornsberry, C., Cardin, C.W. et al. (2002) Bacterial resistance and therapeutic
outcome following three months of topical acne therapy with 2% erythromycin gel
versus its vehicle. Acta Dermato-Venereologica 82(4), 260-265. [Abstract]
NICE (2001) Referral advice - a guide to appropriate referral from general to specialist
services. National Institute for Health and Care Excellence. www.nice.org.uk [Free Fulltext]
Ochsendorf, F. (2006) Systemic antibiotic therapy of acne vulgaris. Journal der
Deutschen Dermatologischen Gesellschaft 4(10), 828-841. [Abstract]
Ozolins, M., Eady, E.A., Avery, A. et al. (2005) Randomised controlled multiple treatment
comparison to provide a cost-effectiveness rationale for the selection of antimicrobial
therapy in acne. Health Technology Assessment 9(1), . [Abstract] [Free Full-text]
Pariser, D., Colon, L.E., Johnson, L.A. and Gottschalk, R.W. (2008) Adapalene 0.1% gel
compared to tazarotene 01% cream in the treatment of acne vulgaris. Journal of Drugs
in Dermatology 7(6 Suppl), s18-s23. [Abstract]
Patel, M., Bowe, W.P., Heughebaert, C. and Shalita, A.R. (2010) The development of
antimicrobial resistance due to the antibiotic treatment of acne vulgaris: a review.
Journal of Drugs in Dermatology 9(6), 655-664. [Abstract]
Purdy, S. and de Berker, D. (2011) Acne vulgaris. Clinical EvidenceBMJ Publishing Group
Ltd. www.clinicalevidence.com [Free Full-text]
Purdy, S., Langston, J. and Tait, L. (2003) Presentation and management of acne in
primary care: a retrospective cohort study. British Journal of General Practice 53(492),
525-529. [Abstract] [Free Full-text]
Ravenscroft, J. (2005) Evidence based update on the management of acne. Archives of
Disease in Childhood Education and Practice Edition 90(4), ep98-ep101.
Ross, J.I., Snelling, A.M., Carnegie, E. et al. (2003) Antibiotic-resistant acne: lessons from
Europe. British Journal of Dermatology 148(3), 467-478. [Abstract]

Sardesai, V. and Kambli, V. (2003) Comparison of efficacy of topical clindamycin and


nicotinamide combination with plain clindamycin for the treatment of acne vulgaris and
acne resistant to topical antibiotics. Indian Journal of Dermatology, Venereology and
Leprology 69(2), 138-139. [Abstract] [Free Full-text]
Schachner, L., Pestana, A. and Kittles, C. (1990) A clinical trial comparing the safety and
efficacy of a topical erythromycin-zinc formulation with a topical clindamycin
formulation. Journal of the American Academy of Dermatology 22(3), 489-495.
[Abstract]
Scottish Medicines Consortium (2003) Drospirenone 3mg, ethinylestradiol 30mcg
(Yasmin) (No.23/03). Schering Health Care. NHS Scotland.
www.scottishmedicines.org.uk [Free Full-text]
Scottish Medicines Consortium (2011) Adapalene 0.1%/benozyl peroxide 2.5%
(Epiduo). NHS Scotland. www.scottishmedicines.org.uk [Free Full-text]
Seaman, H.E., de Vries, C.S. and Farmer, R.D.T. (2003) The risk of venous
thromboembolism in women prescribed cyproterone acetate in combination with
ethinyl estradiol: a nested cohort analysis and case-control study. Human Reproduction
18(3), 522-526. [Abstract] [Free Full-text]
Sharpe, G.R. (1995) Prescribing for acne vulgaris. Prescribers' Journal 35(2), 53-58.
Simonart, T. and Dramaix, M. (2005) Treatment of acne with topical antibiotics: lessons
from clinical studies. British Journal of Dermatology 153(2), 395-403. [Abstract]
Simonart, T., Dramaix, M. and De Maertelaer, V. (2008) Efficacy of tetracyclines in the
treatment of acne vulgaris: a review. British Journal of Dermatology 158(2), 208-216.
[Abstract]
Simpson, R.C., Grindlay, D.J. and Williams, H.C. (2011) What's new in acne? An analysis
of systematic reviews and clinically significant trials published in 2010-11. Clinical and
Experimental Dermatology 36(8), 840-843. [Abstract] [Free Full-text]
Smith, R.N., Mann, N.J., Braue, A. et al. (2007) The effect of a high-protein, low glycemicload diet versus a conventional, high glycemic-load diet on biochemical parameters
associated with acne vulgaris: a randomized, investigator-masked, controlled trial.
Journal of the American Academy of Dermatology 57(2), 247-256. [Abstract]
Stainforth, J., MacDonald-Hull, S., Papworth-Smith, J.W. et al. (1993) A single-blind
comparison of topical erythromycin/zinc lotion and oral minocycline in the treatment of
acne vulgaris. Journal of Dermatological Treatment 4(3), 119-122.
Strauss, J.S., Krowchuk, D.P., Leyden, J.J. et al. (2007) Guidelines of care for acne vulgaris
management. Journal of the American Academy of Dermatology 56(4), 651-653.
[Abstract] [Free Full-text]
Thiboutot, D. (2000) New treatments and therapeutic strategies for acne. Archives of
Family Medicine 9(2), 179-187. [Abstract]
Thiboutot, D., Zaenglein, A., Weiss, J. et al. (2008) An aqueous gel fixed combination of
clindamycin phosphate 1.2% and benzoyl peroxide 2.5% for the once-daily treatment of
moderate to severe acne vulgaris: assessment of efficacy and safety in 2813 patients.
Journal of the American Academy of Dermatology 59(5), 792-800. [Abstract]

Vasilakis-Scaramozza, C. and Jick, H. (2001) Risk of venous thromboembolism with


cyproterone or levonorgestrel contraceptives. Lancet 358(9291), 1427-1429. [Abstract]
Webster, G.F. (2002) Acne vulgaris. British Medical Journal 325(7362), 475-479. [Free
Full-text]
Wolf, J.E. (2002) Acne and rosacea: differential diagnosis and treatment in the primary
care setting. Medscape. www.medscape.com
Zaenglein, A.L. and Thiboutot, D.M. (2006) Expert committee recommendations for acne
management. Pediatrics 118(3), 1188-1199. [Abstract] [Free Full-text]

Question 6 of 180

A mother brings her 9-month-old child to surgery. She is concerned he is not developing normally and is 'falling behind' the children of her
friends. Whilst observing the child you notice he sits without support but makes little effort to move around the room. He has a crude pincer
grip, appears shy when you try to interact and says 'mama' but little else. What is the most accurate description of his development?
Normal development

Isolated delay in gross motor skills

Global developmental delay

Isolated delay in fine motor skills

Isolated delay in speech

Next question

The milestones described are normal for a 9-month-old

Development problems
Referral points

doesn't smile at 10 weeks


cannot sit unsupported at 12 months
cannot walk at 18 months

Fine motor skill problems

hand preference before 12 months is abnormal and may indicate cerebral palsy

Gross motor problems

most common causes of problems: variant of normal, cerebral palsy and neuromuscular disorders (e.g. Duchenne muscular
dystrophy)

Speech and language problems

always check hearing


other causes include environmental deprivation and general development delay

Question 7 of 180

A mother brings her 18-month-old daughter for review following a recent admission after a febrile convulsion. Which one of the following
statements regarding febrile convulsions is not correct?
They are rare in children older than 5 years of age

Carers should place the child in the recovery position and dial 999 if the seizure lasts > 5 minutes

They are seen in around 3% of children

The immunisation schedule should continue and can be safely done in the community

Giving antipyretics promptly can reduce the chance of further seizures

Next question

Antipyretics do not prevent febrile convulsions

There is no evidence that giving a pyrexial child antipyretics reduces the chance of a febrile convulsion

Febrile convulsions
Febrile convulsions are seizures provoked by fever in otherwise normal children. They typically occur between the ages of 6 months and 5
years and are seen in 3% of children
Clinical features

usually occur early in a viral infection as the temperature rises rapidly


seizures are usually brief, lasting less than 5 minutes
may be generalised tonic or tonic-clonic

Prognosis

risk of further febrile convulsion = 1/3 (higher if family history)


if recurrences, try teaching mother how to use rectal diazepam
if no focal signs + lasts less than 30 minutes* + single seizure then 1% risk of developing epilepsy
in the <1% who have all these features, risk of developing epilepsy is much higher (e.g. 50%)

*minutes not seconds, although some authorities use a cut-off of 15 minutes

Febrile seizure - Summary

A febrile seizure is a seizure associated with fever caused by infection or inflammation


outside the central nervous system in a young child who is otherwise neurologically
normal.
o Simple febrile seizures are isolated, generalized, tonic-clonic seizures lasting less
than 15 minutes, that do not recur within 24 hours or within the same febrile illness,
in the absence of previous neurologic problems.
o Complex febrile seizures have one or more of the following features: a partial (focal)
onset or focal features during the seizure; duration of more than 15 minutes;
incomplete recovery within 1 hour; recurrence within 24 hours, or within the same
febrile illness.
Characteristic features of a febrile seizure include:
o Age 6 months to 5 years.
o Duration of seizure usually no longer than 36 minutes.
o Seizure is usually generalized tonic-clonic (body stiffening; twitching of the face,
arms and legs; eye rolling; jerking of the arms and legs; staring; loss of
consciousness).
o Complete recovery of consciousness within 1 hour.
o Fever around the time of the seizure.
o History of previous febrile seizure.
Differential diagnoses that should be excluded include meningitis and encephalitis;
conditions that may be confused with seizures (e.g. rigors, syncope, and apnoea); and
other causes of afebrile seizures (e.g. epilepsy, hypoglycaemia, or brain injury).
A child with a suspected febrile seizure (not fitting on presentation) should be urgently
admitted to hospital if meningitis or encephalitis is suspected. Admission should also be
arranged:
o For children with a first febrile seizure.
o For children who have had a previous febrile seizure but who were not assessed by a
paediatrician following that seizure.
o Where there is diagnostic uncertainty about the cause of the seizure.
o If the seizure lasted for more than 15 minutes; or there were focal features during
the seizure; or the seizure recurred in the same febrile illness, or within 24 hours; or
there was incomplete recovery after one hour.
o For children less than 18 months of age.
o Where the child has no serious clinical findings but is currently taking antibiotics or
has recently been taking them.
o When the parents are anxious and feel that they cannot cope.
o When the child has a suspected serious cause for the fever.
For a child who is fitting for more than 5 minutes:
o Rectal diazepam (repeated once after 5 minutes if necessary) or buccal midazolam
should be given.
o If, 10 minutes after the first dose, the seizure has not stopped, the child has ongoing
twitching, or another seizure has begun before the child regains consciousness, the
child should be urgently admitted to hospital.
o If the child cannot be roused or is convulsing, blood glucose should be measured.
For children who can be managed at home:

Inform parents that seizures of short duration are not harmful to the child and that
about 1 in 3 children will have another febrile seizure in the future.
Advise parents about what to do if further seizures occur, including how to protect
them from injury during the seizure and when to call for medical help.
Advise parents about managing fever, but explain that reducing fever does not
prevent recurrence.
Drugs should not be prescribed to manage or prevent future seizures unless advised
to do so by a specialist.
Follow-up should be arranged, the timing of which will depend on the clinical
condition of the child.

o
o
o
o
o

Question 8 of 180

Which one of the following drugs is contra-indicated whilst breast feeding?


Salbutamol

Hydralazine

Lymecycline

Sodium valproate

Aminophylline

Next question

Breast feeding: contraindications


The major breastfeeding contraindications tested in exams relate to drugs (see below). Other contraindications of note include:

galactosaemia
viral infections - this is controversial with respect to HIV in the developing world. This is because there is such an increased
infant mortality and morbidity associated with bottle feeding that some doctors think the benefits outweigh the risk of HIV
transmission

Drug contraindications
The following drugs can be given to mothers who are breast feeding:

antibiotics: penicillins, cephalosporins, trimethoprim


endocrine: glucocorticoids (avoid high doses), levothyroxine*
epilepsy: sodium valproate, carbamazepine
asthma: salbutamol, theophyllines
psychiatric drugs: tricyclic antidepressants, antipsychotics**
hypertension: beta-blockers, hydralazine, methyldopa
anticoagulants: warfarin, heparin

digoxin

The following drugs should be avoided:

antibiotics: ciprofloxacin, tetracycline, chloramphenicol, sulphonamides


psychiatric drugs: lithium, benzodiazepines
aspirin
carbimazole
sulphonylureas
cytotoxic drugs
amiodarone

*the BNF advises that the amount is too small to affect neonatal hypothyroidism screening
**clozapine should be avoided

Question 9 of 180

The parents of a 3-year-old boy with cystic fibrosis ask for advice. They are considering having more children. What is the chance that their
next child will be a carrier of the cystic fibrosis gene?
50%

100%

1 in 25

25%

66.6%

Next question

As cystic fibrosis is an autosomal recessive condition there is a 50% chance that their next child will be a carrier of cystic fibrosis (i.e. be
heterozygous for the genetic defect) and a 25% chance that the child will actually have the disease (be homozygous).

Cystic fibrosis
Cystic fibrosis (CF) is an autosomal recessive disorder causing increased viscosity of secretions (e.g. lungs and pancreas). It is due to a
defect in the cystic fibrosis transmembrane conductance regulator gene (CFTR), which codes a cAMP-regulated chloride channel
In the UK 80% of CF cases are due to delta F508 on the long arm of chromosome 7. Cystic fibrosis affects 1 per 2500 births, and the
carrier rate is c. 1 in 25
Organisms which may colonise CF patients

Staphylococcus aureus
Pseudomonas aeruginosa
Burkholderia cepacia*
Aspergillus

*previously known as Pseudomonas cepacia

Question 10 of 180

Which one of the following best describes the emergency treatment of a child with severe croup?
Oxygen + nebulised saline

Oxygen + nebulised adrenaline

Oxygen + nebulised salbutamol

Oxygen + IM benzylpenicillin

Oxygen + nebulised steroids

Next question

Oral dexamethasone should also be given if the child is able to take it.

Croup
Croup is a form of upper respiratory tract infection seen in infants and toddlers. It is characterised by stridor which is caused by a
combination of laryngeal oedema and secretions. Parainfluenza viruses account for the majority of cases.
Epidemiology

peak incidence at 6 months - 3 years


more common in autumn

Features

stridor
barking cough (worse at night)
fever
coryzal symptoms

Clinical Knowledge Summaries (CKS) suggest using the following criteria to grade the severity*:
Mild

Moderate

Severe

Occasional barking cough


No audible stridor at rest
No or mild suprasternal and/or
intercostal recession
The child is happy and is prepared to
eat, drink, and play

Frequent barking cough


Frequent barking cough
Prominent inspiratory (and occasionally, expiratory)
Easily audible stridor at rest
stridor at rest
Suprasternal and sternal wall retraction at
Marked sternal wall retractions
rest
Significant distress and agitation, or lethargy or
No or little distress or agitation
restlessness (a sign of hypoxaemia)
The child can be placated and is interested
Tachycardia occurs with more severe obstructive
in its surroundings
symptoms and hypoxaemia

CKS suggest admitting any child with moderate or severe croup. Other features which should prompt admission include:

< 6 months of age


known upper airway abnormalities (e.g. Laryngomalacia, Down's syndrome)
uncertainty about diagnosis (important differentials include acute epiglottitis, bacterial tracheitis, peritonsillar abscess and
foreign body inhalation)

Management

CKS recommend giving a single dose of oral dexamethasone (0.15mg/kg) to all children regardless of severity
prednisolone is an alternative if dexamethasone is not available

Emergency treatment

high-flow oxygen
nebulised adrenaline

*these in turn are based partly on the Alberta Medical Association (2008) Guideline for the diagnosis and management of croup.

Croup - Summary
Croup (laryngotracheitis) is a common childhood disease that is usually caused by a virus. It is
characterized by the sudden onset of a seal-like barking cough usually accompanied by stridor
(predominantly inspiratory), hoarse voice, and respiratory distress due to upper airways
obstruction. Symptoms are usually worse at night. There may be a fever up to 40C.
There is often a preceding 14-day history of a non-specific cough, rhinorrhoea, and fever.
Croup most commonly affects children 6 months to 3 years of age, with a peak incidence during
the second year of life.
Mild:
Occasional barking cough and no audible stridor at rest.
No or mild suprasternal and/or intercostal recession.
The child is happy and is prepared to eat, drink, and play.
Moderate:
Frequent barking cough and easily audible stridor at rest.
Suprasternal and sternal wall retraction at rest.
No or little distress or agitation.

The child can be placated and is interested in its surroundings.


Severe:
Frequent barking cough with prominent inspiratory (and occasionally, expiratory) stridor at rest.
Marked sternal wall retractions.
Significant distress and agitation, or lethargy or restlessness (a sign of hypoxaemia).
Tachycardia occurs with more severe obstructive symptoms and hypoxaemia.
Impending respiratory failure may develop regardless of the severity of the symptoms:
Change in mental state, such as lethargy and listlessness or decreased level of consciousness.
Dusky appearance.
Tachycardia.
In children with impending respiratory failure, breathing may be laboured, a barking cough may
not be prominent, stridor at rest may be hard to hear, and sternal wall retractions may not be
marked. A child who appears to be deteriorating but whose stridor appears to be improving has
worsening airways obstruction and is at high risk of complete airway occlusion.
A child should be immediately admitted when presenting with:
Moderate or severe croup, or impending respiratory failure.
A serious disorder such as epiglottitis, bacterial tracheitis, peritonsillar abscess, retropharyngeal
abscess, foreign body or angioneurotic oedema.
Mild croup can usually be managed at home. However, reasons for admission include: a history
of severe obstruction, age less than 6 months, immunocompromised, an inadequate fluid
intake, an uncertain diagnosis, a poor response to initial treatment, parental anxiety, no
transport, or living far from the hospital.
All children with mild, moderate, or severe croup should receive a single dose of oral
dexamethasone (0.15 mg per kg body weight).
Oral prednisolone (12 mg per kg body weight) is an alternative if dexamethasone is not
available. A second dose should be considered if residual symptoms of stridor are still present
the following day.
Croup is self limiting and symptoms usually resolve within 48 hours. The use of paracetamol or
ibuprofen to control fever and pain should be advised, and arrangements made to review the
child within a few hours. Parents should be advised to seek urgent medical advice if there is any
deterioration.

Have I got the right topic?


Age from 3 months to 6 years
This CKS topic covers the management of croup in primary care.
This CKS topic does not cover the management of croup in secondary care, nor the use of
nebulized adrenaline.
There are separate CKS topics on Common cold, Feverish children - risk assessment, and Sore
throat - acute.
The target audience for this CKS topic is healthcare professionals working within the NHS in the
UK, and providing first contact or primary health care.
How up-to-date is this topic?
Changes
Update
Goals and outcome measures
Goals
Background information
Definition
Prevalence
Management
Scenario: Croup : covers the management of croup in primary care.
Scenario: Croup
Evidence
Supporting evidence
Search strategy
References
Alberta Medical Association (2008a) Guideline for the diagnosis and management of croup.
Alberta Medical Association. www.topalbertadoctors.org [Free Full-text]

Alshehr, M., Almegamsi, T. and Hammdi, A. (2005) Efficacy of a small dose of oral
dexamethasone in croup. Biomedical Research 16(1), 65-72.
Bjornson, C.L. and Johnson, D.W. (2008) Croup. Lancet 371(9609), 329-339. [Abstract]
Bjornson, C.L., Klassen, T.P., Williamson, J. et al. (2004) A randomized trial of a single dose of
oral dexamethasone for mild croup. New England Journal of Medicine 351(13), 1306-1313.
[Abstract] [Free Full-text]
BNF 64 (2012) British National Formulary. 64th edn. London: British Medical Association and
Royal Pharmaceutical Society of Great Britain.
Brown, J.C. (2002) The management of croup. British Medical Bulletin 61(1), 189-202. [Abstract]
[Free Full-text]
Brown, S. (2006b) GP guide to the assessment and management of croup. Prescriber 17(16), 2528. [Free Full-text]
Brown, J.C. and Klassen, T.P. (2000) Croup. In: Moyer, V.A., Elliott, E.J., Davis, R.L. et al. (Eds.)
Evidence based pediatrics and child health. London: BMJ Books. 215-227.
Cherry, J.D. (2005) State of the evidence for standard-of-care treatments for croup: are we
where we need to be? Pediatric Infectious Disease Journal 24(11 Suppl), S198-S201. [Abstract]
Chub-Uppakarn, S. and Sangsupawanich, P. (2007) A randomized comparison of dexamethasone
0.15 mg/kg versus 0.6 mg/kg for the treatment of moderate to severe croup. International
Journal of Pediatric Otorhinolaryngology 71(3), 473-477. [Abstract]
Fifoot, A.A. and Ting, J.Y. (2007) Comparison between single-dose oral prednisolone and oral
dexamethasone in the treatment of croup: a randomized, double-blinded clinical trial.
Emergency Medicine Australasia 19(1), 51-58. [Abstract]
Fitzgerald, D.A. (2006) The assessment and management of croup. Paediatric Respiratory
Reviews 7(1), 73-81. [Abstract]
Geelhoed, G.C. and Macdonald, W.B. (1995) Oral dexamethasone in the treatment of croup:
0.15 mg/kg versus 0.3 mg/kg. Pediatric Pulmonology 20(6), 362-368. [Abstract]
Geelhoed, G.C., Turner, J. and Macdonald, W.B.G. (1996) Efficacy of a small single dose of oral
dexamethasone for outpatient croup: a double blind placebo controlled trial. British Medical
Journal 313(7050), 140-142. [Abstract] [Free Full-text]
Greally, P., Cheng, K., Tanner, M.S. and Field, D.J. (1990) Children with croup presenting with
scalds. British Medical Journal 301(6743), 113. [Free Full-text]

Helms, P. and Henderson, J. (2003b) Infections causing upper airway obstruction. In: McIntosh,
N., Helms, P. and Smyth, R. (Eds.) Forfar & Arneil's textbook of pediatrics. 6th edn. Edinburgh:
Churchill Livingstone. 776-777.
Kairys, S.W., Olmstead, E.M. and O'Connor, G.T. (1989) Steroid treatment of laryngotracheitis: a
meta-analysis of the evidence from randomized trials. Pediatrics 83(5), 683-693. [Abstract]
MHRA (2011g) Press release: more exact paracetamol dosing for children to be introduced.
Medicines and Healthcare products Regulatory Agency. www.mhra.gov.uk [Free Full-text]
Moore, M. and Little, P. (2006) Humidified air inhalation for treating croup (Cochrane Review)
[Withdrawn]. The Cochrane Library. Issue 3. John Wiley & Sons, Ltd.
www.thecochranelibrary.com [Free Full-text]
Moore, M. and Little, P. (2007) Humidified air inhalation for treating croup: a systematic review
and meta-analysis. Family Practice 24(4), 295-301. [Abstract] [Free Full-text]
NICE (2007o) Feverish illness in children. Assessment and initial management in children
younger than 5 years (NICE guideline). . Clinical guideline 47. National Institute for Health and
Care Excellence. www.nice.org.uk
Russell, K.F., Liang, Y., O'Gorman, K. et al. (2011) Glucocorticoids for croup (Cochrane Review).
The Cochrane Library. Issue 1. John Wiley & Sons, Ltd. www.thecochranelibrary.com [Free Fulltext]
Scolnik, D., Coates, A.L., Stephens, D. et al. (2006) Controlled delivery of high vs low humidity vs
mist therapy for croup in emergency departments: a randomized controlled trial. Journal of the
American Medical Association 295(11), 1274-1280. [Abstract] [Free Full-text]
Sparrow, A. and Geelhoed, G. (2006) Prednisolone versus dexamethasone in croup: a
randomised equivalence trial. Archives of Disease in Childhood 91(7), 580-583. [Abstract] [Free
Full-text]
Sydney West Area Health Service (2004) Nurse practitioner clinical practice guidelines for the
management of croup. NSW Health. www.health.nsw.gov.au [Free Full-text]
Vyas, J. (2007) Croup. New Zealand Family Physician 34(4), 266-269. [Free Full-text]
Wagener, J.S., Landau, L.I., Olinsky, A. and Phelan, P.D. (1986) Management of children
hospitalized for laryngotracheobronchitis. Pediatric Pulmonology 2(3), 159-162. [Abstract]
Question 11 of 180

An 18-month-old girl is admitted as she has been vomiting and unwell for the past two days. Her parents think that she has some
abdominal pain as she is pointing to her abdomen. An abdominal x-ray and subsequent ultrasound are taken:

Image used on license from Radiopaedia

What is the diagnosis?


Duodenal atresia

Intussception

Pyloric stenosis

Incarcerated hernia

Meckel's diverticulum

Next question

The abdominal film demonstrates a large soft tissue opacity ('sausage shaped') in the left upper quadrant. Ultrasound confirms an
intussusception with a nice example of a target sign.

Intussusception
Intussusception describes the invagination of one portion of bowel into the lumen of the adjacent bowel, most commonly around the ileocaecal region.
Intussusception is usually affects infants between 6-18 months old. Boys are affected twice as often as girls
Features

paroxysmal abdominal colic pain


during paroxysm the infant will characteristically draw their knees up and turn pale
vomiting
blood stained stool - 'red-currant jelly'
sausage-shaped mass in the abdomen

Investigation

ultrasound is now the investigation of choice and may show a target-like mass

Management

the majority of children can be treated with reduction by air insufflation under radiological control, which is now widely used firstline compared to the traditional barium enema
if this fails, or the child has signs of peritonitis, surgery is performed

Question 12 of 180

What is the average age that the first signs of puberty appear in girls?
10 years

11.5 years

12.5 years

9 years

8 years

Next question

Puberty
Males

first sign is testicular growth at around 12 years of age (range = 10-15 years)
testicular volume > 4 ml indicates onset of puberty
maximum height spurt at 14

Females

first sign is breast development at around 11.5 years of age (range = 9-13 years)
height spurt reaches its maximum early in puberty (at 12) , before menarche
menarche at 13 (11-15)
there is an increase of only about 4% of height following menarche

Normal changes in puberty

gynaecomastia may develop in boys


asymmetrical breast growth may occur in girls
diffuse enlargement of the thyroid gland may be seen

Question 13 of 180

You are asked to organise a tutorial on child protection for medical students attached to the surgery. When discussing patterns of behaviour
which may point towards child abuse, which one of the following is least likely to be relevant?
Frequent attendances to see the GP

Late presentation following an injury

History of child abuse in the family

Torn frenulum

Poor weight gain

Next question

Frequent attendance to the A&E department, rather than GP, may point towards child abuse as parents presume they will see a different
doctor each time, making it less likely suspicions will be aroused

Child abuse: presentation


Children may disclose abuse themselves. Other factors which point towards child abuse include:

story inconsistent with injuries


repeated attendances at A&E departments
late presentation

child with a frightened, withdrawn appearance - 'frozen watchfulness'

Possible physical presentations of child abuse include:

bruising
fractures: particularly metaphyseal, posterior rib fractures or multiple fractures at different stages of healing
torn frenulum: e.g. from forcing a bottle into a child's mouth
burns or scalds
failure to thrive
sexually transmitted infections e.g. Chlamydia, Gonorrhoea, Trichomonas

Question 14 of 180

A 15-year-old girl presents with a palpable purpuric rash over her lower limbs accompanied by polyarthralgia following a recent sore throat.
What is the most likely diagnosis?
Rubella

Measles

Erythema multiforme

Idiopathic thrombocytopenic purpura

Henoch-Schonlein purpura

Next question

Henoch-Schonlein purpura
Henoch-Schonlein purpura (HSP) is an IgA mediated small vessel vasculitis. There is a degree of overlap with IgA nephropathy (Berger's
disease). HSP is usually seen in children following an infection.
Features

palpable purpuric rash (with localized oedema) over buttocks and extensor surfaces of arms and legs
abdominal pain
polyarthritis
features of IgA nephropathy may occur e.g. haematuria, renal failure

Image used on license from DermNet NZ

Treatment

analgesia for arthralgia


treatment of nephropathy is generally supportive. There is inconsistent evidence for the use of steroids and
immunosuppressants

Prognosis

usually excellent, HSP is a self-limiting condition, especially in children without renal involvement
around 1/3rd of patients have a relapse

Image used on license from DermNet NZ

Image used on license from DermNet NZ

Question 15 of 180

A 4-week-old baby girl is diagnosed as having developmental dysplasia of the left hip following an ultrasound examination. Clinical
examination of the hip was abnormal at birth. What treatment is she most likely to be given?
Double nappies

Pavlik harness

Spica cast in flexion and abduction

Re-scan at 3 months

Surgery

Next question

Developmental dysplasia of the hip


Developmental dysplasia of the hip (DDH) is gradually replacing the old term 'congenital dislocation of the hip' (CDH). It affects around 13% of newborns.
Risk factors

female sex: 6 times greater risk


breech presentation
positive family history
firstborn children
oligohydramnios
birth weight > 5 kg
congenital calcaneovalgus foot deformity

DDH is slightly more common in the left hip. Around 20% of cases are bilateral.
Clinical examination is made using the Barlow and Ortolani tests:

Barlow test: attempts to dislocate an articulated femoral head


Ortolani test: attempts to relocate a dislocated femoral head

Ultrasound is used to confirm the diagnosis if clinically suspected


Management

most unstable hips will spontaneously stabilise by 3-6 weeks of age


Pavlik harness (flexion-abduction orthosis) in children younger than 4-5 months
older children may require surgery

Question 16 of 180

You review a 3-year-old girl who is being treated for idiopathic constipation with Movicol Paediatric Plain. Her mother has increased the
dose but unfortunately there has been no response. She remains well and examination of the abdomen is normal. What is the most
appropriate next step?
Add senna

Ask the health visitor to insert a glycerin suppository

Stop Movicol Paediatric Plain and add lactulose + senna

Add lactulose

Prescribe a one-off dose of sodium picosulfate to be given whilst the health visitor is present

Next question

Constipation in children
The frequency at which children open their bowels varies widely, but generally decreases with age from a mean of 3 times per day for
infants under 6 months old to once a day after 3 years of age.
NICE produced guidelines in 2010 on the diagnosis and management of constipation in children. A diagnosis of constipation is suggested
by 2 or more of the following:
Child < 1 year

Child > 1 year

Fewer than 3 complete stools per week


Fewer than 3 complete stools per week (type 3 or 4 on Bristol Stool
(type 3 or 4)
Form Scale) (this does not apply to exclusively breastfed babies after 6 Overflow soiling (commonly very loose,
weeks
very smelly, stool passed without
of age)
sensation)
Hard large stool
'Rabbit droppings' (type 1)
'Rabbit droppings' (type 1)
Large, infrequent stools that can block the
toilet

Stool pattern

Distress on passing stool


Symptoms associated
Bleeding associated with hard stool
with defecation
Straining

Previous episode(s) of constipation


Previous or current anal fissure

History

Poor appetite that improves with passage


of large stool
Waxing and waning of abdominal pain with
passage of stool
Evidence of retentive posturing: typical
straight legged, tiptoed, back arching
posture
Straining
Anal pain
Previous episode(s) of constipation
Previous or current anal fissure
Painful bowel movements and bleeding
associated with hard stools

The vast majority of children have no identifiable cause - idiopathic constipation. Other causes of constipation in children include:

dehydration
low-fibre diet
medications: e.g. Opiates
anal fissure
over-enthusiastic potty training
hypothyroidism
Hirschsprung's disease
hypercalcaemia
learning disabilities

After making a diagnosis of constipation NICE then suggesting excluding secondary causes. If no red or amber flags are present then a
diagnosis of idiopathic constipation can be made:

Indicates idiopathic constipation

'Red flag' suggesting underlying


disorder

Indicates idiopathic constipation

'Red flag' suggesting underlying


disorder

Timing

Starts after a few weeks of life


Obvious precipitating factors coinciding with the start of symptoms: fissure, change
Reported from birth or first few
of diet, timing of potty/toilet training or acute events such as infections, moving
weeks of life
house, starting nursery/school, fears and phobias, major change in family, taking
medicines

Passage of
meconium

< 48 hours

Stool pattern
Growth

> 48 hours
'Ribbon' stools

Generally well, weight and height within normal limits, fit and active

Faltering growth is an amber flag

Neuro/locomotor No neurological problems in legs, normal locomotor development

Previously unknown or
undiagnosed weakness in legs,
locomotor delay

Abdomen

Distension

Diet

Changes in infant formula, weaning, insufficient fluid intake or poor diet


Amber flag: Disclosure or evidence
that raises concerns over possibility
of child maltreatment

Other

Prior to starting treatment the child needs to be assessed for faecal impaction. Factors which suggest faecal impaction include:

symptoms of severe constipation


overflow soiling
faecal mass palpable in abdomen (digital rectal examination should only be carried out by a specialist)

NICE guidelines on management


If faecal impaction is present

polyethylene glycol 3350 + electrolytes (Movicol Paediatric Plain) using an escalating dose regimen as the first-line treatment
add a stimulant laxative if Movicol Paediatric Plain does not lead to disimpaction after 2 weeks
substitute a stimulant laxative singly or in combination with an osmotic laxative such as lactulose if Movicol Paediatric Plain is
not tolerated
inform families that disimpaction treatment can initially increase symptoms of soiling and abdominal pain

Maintenance therapy

very similar to the above regime, with obvious adjustments to the starting dose, i.e.
first-line: Movicol Paediatric Plain
add a stimulant laxative if no response
substitute a stimulant laxative if Movicol Paediatric Plain is not tolerated. Add another laxative such as lactulose or docusate if
stools are hard
continue medication at maintenance dose for several weeks after regular bowel habit is established, then reduce dose gradually

General points

do not use dietary interventions alone as first-line treatment although ensure child is having adequate fluid and fibre intake
consider regular toileting and non-punitive behavioural interventions
for all children consider asking the Health Visitor or Paediatric Continence Advisor to help support the parents.

The NICE guidelines do not specifically discuss the management of very young child. The following recommendations are largely based on
the old Clinical Knowledge Summaries recommendations.
Infants not yet weaned (usually < 6 months)

bottle-fed infants: give extra water in between feeds. Try gentle abdominal massage and bicycling the infant's legs
breast-fed infants: constipation is unusual and organic causes should be considered

Infants who have or are being weaned

offer extra water, diluted fruit juice and fruits


if not effective consider adding lactulose

Constipation in children - Summary

Constipation is a decrease in the frequency of bowel movements characterized by the


passing of hardened stools which may be large and associated with straining and pain.
Soiling of the clothes may result from overflow from the overloaded bowel.
Normal stool frequency ranges from an average of four per day in the first week of life
to two per day at 1 year of age. Passing between three stools per day and three per
week is usually attained by 4 years of age.
Constipation is termed idiopathic (functional) if it cannot be explained by any
anatomical or physiological abnormality.
The exact cause of constipation is poorly understood. Contributing factors include pain,
fever, dehydration, dietary and fluid intake, toilet training problems, the effects of
medicines, psychological issues, and a family history of constipation.
Two or more of the following indicate that the child is constipated:
o Fewer than three complete stools per week (unless exclusively breastfed when
stools may be infrequent).
o Hard, large stool.
o 'Rabbit droppings' stool.
o Overflow soiling (commonly very loose, very smelly stools, which are passed without
sensation or awareness; may also be thick and sticky, or dry and flaky).
No specific investigations are required in primary care.
If constipation is diagnosed, red or amber flags and anatomical and physiological causes
should be excluded.
Faecal impaction should be identified because the child will require disimpaction prior
to prescribing maintenance treatment. Impaction should be suspected if there is:
o A history of severe symptoms of constipation.
o Overflow soiling.
o A faecal mass palpable on abdominal examination (or rectally, although digital rectal
examination is not routinely recommended in children).
If red flag features are present, referral for urgent specialist assessment should be
arranged.
Children should be treated in primary care only if a positive diagnosis of idiopathic
constipation has been made.
o Offer reassurance that underlying causes of constipation have been excluded.
o Advise that the condition is treatable, but laxatives may need to be taken for several
months in order to resolve the problem.

Written advice should be offered.


An adequate fluid intake and a balanced diet with sufficient fibre (unless exclusively
breastfed) should be recommended.
o Faecal impaction should be treated using the recommended disimpaction regimen.
o If impaction is not present or has been treated, the child should be promptly treated
with a laxative (even if the history of constipation is very short).
o Polyethylene glycol 3350 plus electrolytes (Movicol Paediatric Plain) is preferred
as first-line management. If this approach does not work, a stimulant laxative,
lactulose, or docusate should be considered.
Advice should be offered on behavioural interventions such as scheduled toileting, use
of a bowel diary, and reward systems.
Follow up should be arranged regularly to assess adherence and response to treatment.
o
o

Have I got the right topic?


Age from 0 months to 18 years
This CKS topic is based on a guideline published by the National Institute for
Health and Care Excellence (NICE) Constipation in children and young people:
diagnosis and management of idiopathic childhood constipation in primary and
secondary care [National Collaborating Centre for Women's and Children's Health,
2010].
This CKS topic covers the diagnosis, management, and indications for referral of
idiopathic constipation.
This CKS topic does not cover the diagnosis and treatment of underlying disorders,
or the management offered by specialist services after referral.
There is a separate CKS topic on Constipation.
The target audience for this CKS topic is healthcare professionals working within
the NHS in the UK, and providing first contact or primary health care.

How up-to-date is this topic?

Changes
Update

Goals and outcome measures

Goals
QIPP - Options for local implementation

Background information

Definition
Causes
Prevalence

Prognosis and complications

Diagnosis

Diagnosis and assessment - younger than 1 year


Diagnosis and assessment - older than 1 year

Management

Scenario: Management : covers the management of idiopathic constipation in children


and young people.

Scenario: Management

Prescribing information
Important aspects of prescribing information relevant to primary healthcare are
covered in this section specifically for the drugs recommended in this CKS topic.
For further information on contraindications, cautions, drug interactions, and
adverse effects, see the electronic Medicines Compendium (eMC)
(http://medicines.org.uk/emc), or the British National Formulary (BNF)
(www.bnf.org).

Types of laxatives
Choice of stimulant laxative
Palatability
Contraindications
Dose and titration
Adverse effects

Evidence

Supporting evidence
Search strategy

References
ABPI Medicines Compendium (2010a) Summary of product characteristics for
Dulcolax tablets, 5mg. Electronic Medicines CompendiumDatapharm
Communications Ltd. www.medicines.org.uk [Free Full-text]
ABPI Medicines Compendium (2010b) Summary of product characteristics for
Dulcolax liquid. Electronic Medicines CompendiumDatapharm Communications Ltd.
www.medicines.org.uk [Free Full-text]
ABPI Medicines Compendium (2010c) Summary of product characteristics for
Senokot syrup. Electronic Medicines CompendiumDatapharm Communications Ltd.
www.medicines.org.uk [Free Full-text]

BNF 59 (2010) British National Formulary. 59th edn. London: British Medical
Association and Royal Pharmaceutical Society of Great Britain.
Clayden, G.S., Keshtgar, A.S., Carcani-Rathwell, I. and Abhyankar, A. (2005) The
management of chronic constipation and related faecal incontinence in childhood.
Archives of Disease in Childhood - Education and Practice 90(3), 58-67. [Free Full-text]
DH (2004) At least five a week. Evidence on the impact of physical activity and its
relationship to health: a report from the Chief Medical Officer. Department of Health.
www.dh.gov.uk [Free Full-text]
Institute of Medicine of the National Academies (2005) Dietary reference intakes for
water, potassium, sodium, chloride, and sulfate. National Academy of Sciences.
www.nap.edu [Free Full-text]
Micromedex (2010) MICROMEDEX [CD-ROM]Thomson Healthcare.
National Collaborating Centre for Women's and Children's Health (2010)
Constipation in children and young people: diagnosis and management of idiopathic
childhood constipation in primary and secondary care (full NICE guideline). . Clinical
guideline 99. Royal College of Obstetricians and Gynaecologists. www.nice.org.uk
[Free Full-text]
NICE (2010) Constipation in children and young people: diagnosis and management of
idiopathic childhood constipation in primary and secondary care (NICE guideline). .
Clinical guideline 99. National Institute for Health and Care Excellence.
www.nice.org.uk [Free Full-text]
NICE (2013) Key therapeutic topics - medicines management options for local
implementation. National Institute for Health and Care Excellence. www.nice.org.uk
[Free Full-text]
NPC (2012) Key therapeutic topics - medicines management options for local
implementation. National Prescribing Centre. www.npc.nhs.uk [Free Full-text]
van den Berg, M.M., Benninga, M.A. and Di Lorenzo, C. (2006) Epidemiology of
childhood constipation: a systematic review. American Journal of Gastroenterology
101(10), 2401-2409. [Abstract]

Question 17 of 180

At what age would the average child start to say 'mama' and 'dada'?
3 months

4-5 months

6-7 months

9-10 months

13-14 months

Next question

Developmental milestones: speech and hearing


The table below summarises the major speech and hearing developmental milestones
Age

Milestone

3 months

Quietens to parents voice


Turns towards sound
Squeals

6 months

Double syllables 'adah', 'erleh'

9 months

Says 'mama' and 'dada'


Understands 'no'

12 months

Knows and responds to own name

12-15 months

Knows about 2-6 words (Refer at 18 months)


Understands simple commands - 'give it to mummy'

2 years

Combine two words


Points to parts of the body

2 years

Vocabulary of 200 words

3 years

Talks in short sentences (e.g. 3-5 words)


Asks 'what' and 'who' questions
Identifies colours
Counts to 10 (little appreciation of numbers though)

4 years

Asks 'why', 'when' and 'how' questions

Question 18 of 180

The chance of a 40-year-old mother giving birth to a child with Down's syndrome is approximately:
1 in 5

1 in 10

1 in 30

1 in 100

1 in 500

Next question

Down's syndrome risk - 1/1,000 at 30 years then divide by 3 for every 5 years

Down's syndrome: epidemiology and genetics


Risk of Down's syndrome with increasing maternal age

risk at 30 years = 1/1000


35 years = 1/350
40 years = 1/100
45 years = 1/30

One way of remembering this is by starting at 1/1,000 at 30 years and then dividing the denominator by 3 (i.e. 3 times more common) for
every extra 5 years of age
Cytogenetics
Mode

% of cases Risk of recurrence

Non-disjunction

94%

Robertsonian translocation
5%
(usually onto 14)
Mosaicism

1 in 100 if under mother < 35 years


10-15% if mother is translocation carrier
2.5% if father is translocation carrier

1%

The chance of a further child with Down's syndrome is approximately 1 in 100 if the mother is less than 35 years old. If the trisomy 21 is a
result of a translocation the risk is much higher

Question 19 of 180

A mother asks for information following a recent admission of her 2-year-old son with a febrile convulsion. What is the chance of her son
having a further febrile convulsion?
< 2%

5%

10%

20%

30%

Next question

Febrile convulsions - risk of further convulsions = 30%

Febrile convulsions
Febrile convulsions are seizures provoked by fever in otherwise normal children. They typically occur between the ages of 6 months and 5
years and are seen in 3% of children
Clinical features

usually occur early in a viral infection as the temperature rises rapidly


seizures are usually brief, lasting less than 5 minutes
may be generalised tonic or tonic-clonic

Prognosis

risk of further febrile convulsion = 1/3 (higher if family history)


if recurrences, try teaching mother how to use rectal diazepam
if no focal signs + lasts less than 30 minutes* + single seizure then 1% risk of developing epilepsy
in the <1% who have all these features, risk of developing epilepsy is much higher (e.g. 50%)

*minutes not seconds, although some authorities use a cut-off of 15 minutes

Febrile seizure - Summary


A febrile seizure is a seizure associated with fever caused by infection or inflammation outside
the central nervous system in a young child who is otherwise neurologically normal.
Simple febrile seizures are isolated, generalized, tonic-clonic seizures lasting less than 15
minutes, that do not recur within 24 hours or within the same febrile illness, in the absence of
previous neurologic problems.
Complex febrile seizures have one or more of the following features: a partial (focal) onset or
focal features during the seizure; duration of more than 15 minutes; incomplete recovery within
1 hour; recurrence within 24 hours, or within the same febrile illness.
Characteristic features of a febrile seizure include:
Age 6 months to 5 years.
Duration of seizure usually no longer than 36 minutes.
Seizure is usually generalized tonic-clonic (body stiffening; twitching of the face, arms and legs;
eye rolling; jerking of the arms and legs; staring; loss of consciousness).

Complete recovery of consciousness within 1 hour.


Fever around the time of the seizure.
History of previous febrile seizure.
Differential diagnoses that should be excluded include meningitis and encephalitis; conditions
that may be confused with seizures (e.g. rigors, syncope, and apnoea); and other causes of
afebrile seizures (e.g. epilepsy, hypoglycaemia, or brain injury).
A child with a suspected febrile seizure (not fitting on presentation) should be urgently admitted
to hospital if meningitis or encephalitis is suspected. Admission should also be arranged:
For children with a first febrile seizure.
For children who have had a previous febrile seizure but who were not assessed by a
paediatrician following that seizure.
Where there is diagnostic uncertainty about the cause of the seizure.
If the seizure lasted for more than 15 minutes; or there were focal features during the seizure;
or the seizure recurred in the same febrile illness, or within 24 hours; or there was incomplete
recovery after one hour.
For children less than 18 months of age.
Where the child has no serious clinical findings but is currently taking antibiotics or has recently
been taking them.
When the parents are anxious and feel that they cannot cope.
When the child has a suspected serious cause for the fever.
For a child who is fitting for more than 5 minutes:
Rectal diazepam (repeated once after 5 minutes if necessary) or buccal midazolam should be
given.
If, 10 minutes after the first dose, the seizure has not stopped, the child has ongoing twitching,
or another seizure has begun before the child regains consciousness, the child should be
urgently admitted to hospital.
If the child cannot be roused or is convulsing, blood glucose should be measured.
For children who can be managed at home:
Inform parents that seizures of short duration are not harmful to the child and that about
1 in 3 children will have another febrile seizure in the future.

Advise parents about what to do if further seizures occur, including how to protect them from
injury during the seizure and when to call for medical help.
Advise parents about managing fever, but explain that reducing fever does not prevent
recurrence.
Drugs should not be prescribed to manage or prevent future seizures unless advised to do so by
a specialist.
Follow-up should be arranged, the timing of which will depend on the clinical condition of the
child.
Have I got the right topic?
Age from 6 months to 6 years
This CKS topic covers the management of a child who presents with, or after, a febrile seizure.
This CKS topic does not cover the management of a child who has epilepsy or any other seizure
disorder.
There are separate CKS topics on Epilepsy, Feverish children - risk assessment, and Feverish
children - management.
The target audience for this CKS topic is healthcare professionals working within the NHS in the
UK, and providing first contact or primary health care.
How up-to-date is this topic?
Changes
Update
Goals and outcome measures
Goals
Background information
Definition
Cause
Prevalence
Risk factors for first seizure
Complications

Prognosis
Diagnosis
Diagnosis of febrile seizure
Diagnosis
Management
Scenario: Management after a seizure : covers the management of a child who has had a febrile
seizure, including assessment, advice, follow up, when to refer, and emergency management.
Scenario: Managing a seizure : covers the management of a child who is currently having a
febrile seizure, including safety advice, airway management, when to use anticonvulsant
medication, when to admit to hospital, and when to measure blood glucose levels.
Scenario: Management after a seizure
Scenario: Managing a seizure
Evidence
Supporting evidence
Search strategy
References
Armon, K., Stephenson, T., Hemingway, P. et al. (2003) An evidence and consensus based
guideline for the management of a child after a seizure. Emergency Medicine Journal 20(1), 1320. [Abstract] [Free Full-text]
Barlow, W.E., Davis, R.L., Glasser, J.W. et al. (2001) The risk of seizures after receipt of whole-cell
pertussis or measles, mumps, and rubella vaccine. New England Journal of Medicine 345(9),
656-661. [Abstract] [Free Full-text]
Baysun, S., Aydin, O.F., Atmaca, E. and Yavuz, G.Y.K. (2005) A comparison of buccal midazolam
and rectal diazepam for the acute treatment of seizures. Clinical Pediatrics 44(9), 771-776.
[Abstract]
Berg, A.T., Shinnar, S., Darefsky, A.S. et al. (1997) Predictors of recurrent febrile seizures. A
prospective cohort study. Archives of Pediatrics & Adolescent Medicine 151(4), 371-378.
[Abstract] [Free Full-text]
Bhattacharyya, M., Kalra, V. and Gulati, S. (2006) Intranasal midazolam vs rectal diazepam in
acute childhood seizures. Pediatric Neurology 34(5), 355-359. [Abstract]

Chungath, M. and Shorvon, S. (2008) The mortality and morbidity of febrile seizures. Nature
Clinical Practice Neurology 4(11), 610-621. [Abstract]
DTB (2005) Drugs for the doctors bag: 2 - children. Drug & Therapeutics Bulletin 43(11), 81-84.
[Abstract]
Epilepsy Action (2005) The role of primary care in epilepsy management 2. Epilepsy Action.
www.epilepsy.org.uk [Free Full-text]
Ferrie, C. (2008) Seizures, epilepsy and other paroxysmal disorders. In: McIntosh, N., Helms, P.J.,
Smyth, R.L. and Logan, S. (Eds.) Forfar and Arneil's textbook of pediatrics. 7th edn. Edinburgh:
Churchill Livingstone. 841-872.
Fisher, R.G. and Boyce, T.G. (Eds.) (2005) Roseola syndrome (exanthem subitum). In: Moffet's
pediatric infectious diseases: a problem-orientated approach. 4th edn. Philadelphia: Lippincott
Williams & Wilkins. 407-408.
Fukuyama, Y., Seki, T., Ohtsuka, C. et al. (1996) Practical guidelines for physicians in the
management of febrile seizures. Brain & Development 18(6), 479-484. [Abstract]
Ganesh, R. and Janakiraman, L. (2008) Serum zinc levels in children with simple febrile seizure.
Clinical Pediatrics 47(2), 164-166. [Abstract]
Graves, R.C., Oehler, K. and Tingle, L.E. (2012) Febrile seizures: risks, evaluation, and prognosis.
American Family Physician 85(2), 149-153. [Abstract] [Free Full-text]
Hartfield, D.S., Tan, J., Yager, J.Y. et al. (2009) The association between iron deficiency and
febrile seizures in childhood. Clinical Pediatrics 48(4), 420-426. [Abstract]
Jones, C. and Isaacs, D. (2008) Exanthemata. In: McIntosh, N., Helms, P.J., Smyth, R.L. and Logan,
S. (Eds.) Forfar and Arneil's textbook of pediatrics. 7th edn. Edinburgh: Churchill Livingstone.
1274-1279.
Jones, T. and Jacobsen, S.J. (2007) Childhood febrile seizures: overview and implications.
International Journal of Medical Sciences 4(2), 110-114. [Abstract] [Free Full-text]
Knudsen, F.U. (1996) Febrile seizures - treatment and outcome. Brain & Development 18(6),
438-449. [Abstract]
Koren, G. (2000) Intranasal midazolam for febrile seizures. A step forward in treating a common
and distressing conditions. British Medical Journal 321(7253), 64-65. [Free Full-text]
Lahat, E., Goldman, M., Barr, J. et al. (2000) Comparison of intranasal midazolam with
intravenous diazepam for treating febrile seizures in children: prospective randomised study.
British Medical Journal 321(7253), 83-86. [Abstract] [Free Full-text]

McIntyre, J., Robertson, S., Norris, E. et al. (2005) Safety and efficacy of buccal midazolam versus
rectal diazepam for emergency treatment of seizures in children: a randomised controlled trial.
Lancet 336(9481), 205-210. [Abstract]
Mewasingh, L.D. (2010) Febrile seizures. Clinical EvidenceBMJ Publishing Group Ltd.
www.clinicalevidence.com [Free Full-text]
MHRA (2011) Press release: more exact paracetamol dosing for children to be introduced.
Medicines and Healthcare products Regulatory Agency. www.mhra.gov.uk [Free Full-text]
National Collaborating Centre for Women's and Children's Health (2013) Feverish illness in
children: assessment and initial management in children younger than 5 years (full NICE
guideline). Clinical guidance 160National Centre for Health and Care Excellence.
www.nice.org.uk [Free Full-text]
Offringa, M. and Moyer, V.A. (2001) Evidence based paediatrics: evidence based management of
seizures associated with fever. British Medical Journal 323(7321), 1111-1114. [Free Full-text]
Offringa, M. and Newton, R. (2012) Prophylactic drug management for febrile seizures in
children (Cochrane Review). The Cochrane Library. Issue 4. John Wiley & Sons, Ltd.
www.thecochranelibrary.com [Free Full-text]
Rosenbloom, E., Finkelstein, Y., Adams-Webber, T. and Kozer, E. (2013) Do antipyretics prevent
the recurrence of febrile seizures in children? A systematic review of randomized controlled
trials and meta-analysis. European Journal of Paediatric Neurology 17(6), 585-588. [Abstract]
Royal College of Physicians and the British Paediatric Association (1991) Guidelines for the
management of convulsions with fever. British Medical Journal 303(6803), 634-636. [Free Fulltext]
Sadleir, L.G. and Scheffer, I.E. (2007) Febrile seizures. British Medical Journal 334(7588), 307311. [Free Full-text]
Siqueira, L.F. (2010) Febrile seizures: update on diagnosis and management. Revista Da
Associacao Medica Brasileira 56(4), 489-492. [Abstract] [Free Full-text]
Strengell, T., Uhari, M., Tarkka, R. et al. (2009) Antipyretic agents for preventing recurrences of
febrile seizures: randomized controlled trial. Archives of Pediatrics & Adolescent Medicine
163(9), 799-804. [Abstract] [Free Full-text]
van Stuijvenberg, M., Derksen-Lubsen, G., Steyerberg, E.W. et al. (1998) Randomized, controlled
trial of ibuprofen sysrup administered during febrile illnesses to prevent febrile seizure
recurrences. Pediatrics 102(5), E51. [Abstract]

Verity, C.M., Greenwood, R. and Golding, J. (1998) Long-term intellectual and behavioral
outcomes of children with febrile convulsions. New England Journal of Medicine 338(24), 17231728. [Abstract] [Free Full-text]
Vestergaard, M., Hviid, A., Madsen, K.M. et al. (2004) MMR vaccination and febrile seizures:
evaluation of susceptible subgroups and long-term prognosis. Journal of the American Medical
Association 292(3), 351-357. [Abstract] [Free Full-text]
Vestergaard, M., Pedersen, M.G., Ostergaard, J.R. et al. (2008) Death in children with febrile
seizures: a population-based cohort study. Lancet 372(9637), 457-463. [Abstract]
Warden, C.R., Zibulewsky, J., Mace, S. et al. (2003) Evaluation and management of febrile
seizures in the out-of-hospital and emergency department settings. Annals of Emergency
Medicine 41(2), 215-222. [Abstract]
Waruiru, C. and Appleton, R. (2004) Febrile seizures: an update. Archives of Disease in Childhood
89(8), 751-756. [Abstract] [Free Full-text]
Question 20 of 180

Which one of the following statements regarding Reye's syndrome is incorrect?


Associated with aspirin use

Mortality is 15-25%

Peak incidence is 2 years of age

Plasma exchange should be started as soon as possible

Causes fatty infiltration of the liver

Next question

Management of Reye's sydnrome is largely supportive.

Reye's syndrome
Reye's syndrome is a severe, progressive encephalopathy affecting children that is accompanied by fatty infiltration of the liver, kidneys
and pancreas. The aetiology of Reye's syndrome is not fully understood although there is a known association with aspirin use and a viral
cause has been postulated
The peak incidence is 2 years of age, features include:

may be history of preceding viral illness


encephalopathy: confusion, seizures, cerebral oedema, coma
fatty infiltration of the liver, kidneys and pancreas
hypoglycaemia

Management is supportive
Although the prognosis has improved over recent years there is still a mortality rate of 15-25%.

Question 21 of 180

An 11-year-old girl presents with a productive cough and fever. A chest x-ray is taken:

Image used on license from Radiopaedia

What is the main finding on the x-ray?


Bilateral pneumothoraces

Left lingual consolidation

Dilated cardiomyopathy with pulmonary oedema

Left humeral head fracture

Left middle lobe consolidation

Next question

The loss of the left heart border is a classic sign of left lingual consolidation. There is no left middle lobe!

Chest x-ray: white lung lesions


There are numerous causes of white shadowing in the lungs including:

consolidation
pleural effusion
collapse
pneumonectomy
specific lesions e.g. tumours
fluid e.g. pulmonary oedema

For more information check out the link to the superb Radiology Masterclass website.

Question 22 of 180

A 10-year-old girl is admitted with shortness-of-breath and fatigue. A chest x-ray is performed on admission:

Image used on license from Radiopaedia

Based on the x-ray findings, what is the most likely diagnosis?


Inhaled foreign object

Heart failure

Asthma

Cystic fibrosis

Pneumonia

Next question

The x-ray shows dilated cardiomyopathy and features of pulmonary oedema including fluid in the horizontal fissure.

Chest x-ray: pulmonary oedema


Features of pulmonary oedema on a chest x-ray may include:

interstitial oedema
bat's wing appearance
upper lobe diversion (increased blood flow to the superior parts of the lung)
Kerley B lines
pleural effusion
cardiomegaly may be seen if there is cardiogenic cause

Question 23 of 180

Which one of the following is least likely to cause snoring in children?


Hypertrophic nasal turbinates

Tonsillitis

Obesity

Kallman's syndrome

Down's syndrome

Next question

Kallman's syndrome is a cause of delayed puberty secondary to hypogonadotrophic hypogonadism. It is not associated with snoring

Snoring in children

Causes

obesity
nasal problems: polyps, deviated septum, hypertrophic nasal turbinates
recurrent tonsillitis
Down's syndrome
hypothyroidism

Question 24 of 180

For which one of the following indications is carbamazepine least likely to be a useful management option?
Trigeminal neuralgia

Absence seizures

Bipolar disorder

Temporal lobe epilepsy

Complex partial seizures

Next question

Carbamazepine is generally ineffective in absence seizures

Carbamazepine
Carbamazepine is chemically similar to the tricyclic antidepressant drugs. It is most commonly used in the treatment of epilepsy, particularly
partial seizures, where carbamazepine remains a first-line medication. Other uses include

neuropathic pain (e.g. trigeminal neuralgia, diabetic neuropathy)


bipolar disorder

Mechanism of action

sodium channel blocker, decreasing the sodium influx into neurons which in turn decreases excitability

Adverse effects

P450 enzyme inducer


dizziness and ataxia
drowsiness
headache
visual disturbances (especially diplopia)
Steven-Johnson syndrome
leucopenia and agranulocytosis
syndrome of inappropriate ADH secretion

Question 25 of 180

Which one of the following drugs is contra-indicated whilst breast feeding?


Aminophylline

Carbamazepine

Sodium valproate

Methyldopa

Amiodarone

Next question

Breast feeding: contraindications


The major breastfeeding contraindications tested in exams relate to drugs (see below). Other contraindications of note include:

galactosaemia
viral infections - this is controversial with respect to HIV in the developing world. This is because there is such an increased
infant mortality and morbidity associated with bottle feeding that some doctors think the benefits outweigh the risk of HIV
transmission

Drug contraindications
The following drugs can be given to mothers who are breast feeding:

antibiotics: penicillins, cephalosporins, trimethoprim


endocrine: glucocorticoids (avoid high doses), levothyroxine*
epilepsy: sodium valproate, carbamazepine
asthma: salbutamol, theophyllines
psychiatric drugs: tricyclic antidepressants, antipsychotics**
hypertension: beta-blockers, hydralazine, methyldopa
anticoagulants: warfarin, heparin
digoxin

The following drugs should be avoided:

antibiotics: ciprofloxacin, tetracycline, chloramphenicol, sulphonamides


psychiatric drugs: lithium, benzodiazepines
aspirin
carbimazole
sulphonylureas
cytotoxic drugs
amiodarone

*the BNF advises that the amount is too small to affect neonatal hypothyroidism screening
**clozapine should be avoided

Question 26 of 180

What is the first sign of puberty in girls?


Development of pubic hair

Menstrual bleeding

Development of axillary hair

Breast development

Height spurt

Next question

Puberty
Males

first sign is testicular growth at around 12 years of age (range = 10-15 years)
testicular volume > 4 ml indicates onset of puberty
maximum height spurt at 14

Females

first sign is breast development at around 11.5 years of age (range = 9-13 years)
height spurt reaches its maximum early in puberty (at 12) , before menarche
menarche at 13 (11-15)
there is an increase of only about 4% of height following menarche

Normal changes in puberty

gynaecomastia may develop in boys


asymmetrical breast growth may occur in girls
diffuse enlargement of the thyroid gland may be seen

Question 27 of 180

A 9-year-old boy who has recently arrived from India presents with fever. On examination a grey coating is seen surrounding the tonsils and
there is extensive cervical lymphadenopathy. What is the most likely diagnosis?
Dengue fever

Typhoid

Paratyphoid

Actinomycosis

Diphtheria

Next question

Diphtheria
Diphtheria is caused by the Gram positive bacterium Corynebacterium diphtheriae
Pathophysiology

releases an exotoxin encoded by a -prophage


exotoxin inhibits protein synthesis by catalyzing ADP-ribosylation of elongation factor EF-2

Diphtheria toxin commonly causes a 'diphtheric membrane' on tonsils caused by necrotic mucosal cells. Systemic distribution may produce
necrosis of myocardial, neural and renal tissue
Possible presentations

recent visitors to Eastern Europe/Russia/Asia


sore throat with a 'diphtheric membrane' - see above
bulky cervical lymphadenopathy
neuritis e.g. cranial nerves
heart block

Question 28 of 180

A 2-year-old boy presents with a harsh cough and pyrexia. His symptoms worsened overnight and on examination stridor is noted. Which
one of the following interventions may improve his symptoms?
Codeine linctus

Humidified oxygen

Nebulised salbutamol

Oral erythromycin

Oral dexamethasone

Next question

Croup
Croup is a form of upper respiratory tract infection seen in infants and toddlers. It is characterised by stridor which is caused by a
combination of laryngeal oedema and secretions. Parainfluenza viruses account for the majority of cases.
Epidemiology

peak incidence at 6 months - 3 years


more common in autumn

Features

stridor
barking cough (worse at night)
fever
coryzal symptoms

Clinical Knowledge Summaries (CKS) suggest using the following criteria to grade the severity*:
Mild

Moderate

Severe

Occasional barking cough


No audible stridor at rest
No or mild suprasternal and/or
intercostal recession
The child is happy and is prepared to
eat, drink, and play

Frequent barking cough


Frequent barking cough
Prominent inspiratory (and occasionally, expiratory)
Easily audible stridor at rest
stridor at rest
Suprasternal and sternal wall retraction at
Marked sternal wall retractions
rest
Significant distress and agitation, or lethargy or
No or little distress or agitation
restlessness (a sign of hypoxaemia)
The child can be placated and is interested
Tachycardia occurs with more severe obstructive
in its surroundings
symptoms and hypoxaemia

CKS suggest admitting any child with moderate or severe croup. Other features which should prompt admission include:

< 6 months of age


known upper airway abnormalities (e.g. Laryngomalacia, Down's syndrome)
uncertainty about diagnosis (important differentials include acute epiglottitis, bacterial tracheitis, peritonsillar abscess and
foreign body inhalation)

Management

CKS recommend giving a single dose of oral dexamethasone (0.15mg/kg) to all children regardless of severity
prednisolone is an alternative if dexamethasone is not available

Emergency treatment

high-flow oxygen
nebulised adrenaline

*these in turn are based partly on the Alberta Medical Association (2008) Guideline for the diagnosis and management of croup.

Question 29 of 180

Which one of the following statements concerning hypospadias is incorrect?


Is associated with a ventral urethral meatus

Circumcision should not be performed prior to surgery

Dorsal curvature of the penis is seen in more severe forms

Surgery should be performed before 2 years of age

Affects 3/1,000 male infants

Next question

Hypospadias
Hypospadias is a congenital abnormality of the penis which occurs in approximately 3/1,000 male infants
Hypospadias is characterised by

a ventral urethral meatus


a hooded prepuce
chordee (ventral curvature of the penis) in more severe forms

Corrective surgery is performed before 2 years of age. It is essential that the child is not circumcised prior to the surgery as the foreskin
may be used in the corrective procedure

Question 30 of 180

A 4-year-old boy is brought in to surgery by his mother. He has reportedly been drowsy and pyrexial for the past 2 hours. Whilst waiting to
be seen he becomes unresponsive. One of your GP colleagues performs an urgent assessment whilst the practice nurse calls 999. He is
making no respiratory effort so 5 rescue breaths are given. As you arrive your colleague cannot detect a brachial or carotid pulse. What is
the most appropriate course of action?
Start chest compressions/ventilations at a ratio of 5:1

Check for a femoral pulse

Start chest compressions/ventilations at a ratio of 15:2

Give a further 5 rescue breaths

Start chest compressions/ventilations at a ratio of 30:2

Next question

As two trained members of staff are present the ratio of chest compressions to ventilations should be 15:2

Paediatric basic life support


The 2010 Resuscitation Council guidelines made the following changes to paediatric basic life support

compression:ventilation ratio: lay rescuers should use a ratio of 30:2. If there are two or more rescuers with a duty to respond
then a ratio of 15:2 should be used
age definitions: an infant is a child under 1 year, a child is between 1 year and puberty

Key points of algorithm (please see link attached for more details)

unresponsive?
shout for help
open airway
look, listen, feel for breathing
give 5 rescue breaths
check for signs of circulation
15 chest compressions:2 rescue breaths (see above)

Question 31 of 180

A newborn female baby is diagnosed with cystic fibrosis following an episode of meconium ileus shortly after birth. Which one of the
following is least likely to occur as a consequence of her underlying diagnosis?
Delayed puberty

Nasal polyps

Diabetes mellitus

Rectal prolapse

Arthropathy

Next question

Arthropathy is not a common feature of cystic fibrosis

Cystic fibrosis: features


Presenting features

neonatal period (around 20%): meconium ileus, less commonly prolonged jaundice
recurrent chest infections (40%)
malabsorption (30%): steatorrhoea, failure to thrive
other features (10%): liver disease

Other features of cystic fibrosis

short stature
diabetes mellitus
delayed puberty
rectal prolapse (due to bulky stools)
nasal polyps
male infertility, female subfertility

Question 32 of 180

At what age would the average child acquire the ability to build a tower of three blocks?
18 months

2 years

2 years

3 years

4 years

Next question

Developmental milestones: fine motor and vision


The tables below summarises the major fine motor and vision developmental milestones
Age

Milestone

3 months

Reaches for object


Holds rattle briefly if given to hand
Visually alert, particularly human faces
Fixes and follows to 180 degrees

Holds in palmar grasp


6 months Pass objects from one hand to another
Visually insatiable, looking around in every direction

Age

Milestone

9 months

Points with finger


Early pincer

12 months

Good pincer grip


Bangs toys together

Bricks
Age

Milestone

15 monthsTower of 2
18 monthsTower of 3
2 years

Tower of 6

3 years

Tower of 9

Drawing
Age

Milestone

18 monthsCircular scribble
2 years

Copies vertical line

3 years

Copies circle

4 years

Copies cross

5 years

Copies square and triangle

Book
Age

Milestone

15 monthsLooks at book, pats page


18 monthsTurns pages, several at time
2 years

Turns pages, one at time

Notes

hand preference before 12 months is abnormal and may indicate cerebral palsy

Question 33 of 180

Which one of the following features is least associated with rickets?


Genu varum

Genu valgum

Reduced serum calcium

Reduced serum alkaline phosphatase

Harrison's sulcus

Next question

Rickets
Rickets is a term which describes inadequately mineralised bone in developing and growing bones. This results in soft and easily deformed
bones. It is usually due to vitamin D deficiency. In adults the equivalent condition is termed osteomalacia
Predisposing factors

dietary deficiency of calcium, for example in developing countries


prolonged breast feeding
unsupplemented cow's milk formula
lack of sunlight

Features

in toddlers - genu varum (bow legs), in older children - genu valgum (knock knees)
'rickety rosary' - swelling at the costochondral junction
kyphoscoliosis
craniotabes - soft skull bones in early life
Harrison's sulcus
reduced serum calcium - symptoms may results from hypocalcaemia
raised alkaline phosphatase

Management

oral vitamin D

Question 34 of 180

A 12-year-old girl presents with a two-day history of an itchy rash over her whole body associated with a low-grade pyrexia:

Image used on license from DermNet NZ

Which one of the following complications is least likely to occur?


Disseminated haemorrhagic chickenpox

Secondary bacterial infection

Encephalitis

Dilated cardiomyopathy

Pneumonia

Next question

Dilated cardiomyopathy is not an established complication of chickenpox, unlike the other four options.

Chickenpox
Chickenpox is caused by primary infection with varicella zoster virus. Shingles is reactivation of dormant virus in dorsal root ganglion
Chickenpox is highly infectious

spread via the respiratory route


can be caught from someone with shingles
infectivity = 4 days before rash, until 5 days after the rash first appeared*
incubation period = 10-21 days

Clinical features (tend to be more severe in older children/adults)

fever initially
itchy, rash starting on head/trunk before spreading. Initially macular then papular then vesicular
systemic upset is usually mild

Management is supportive

keep cool, trim nails


calamine lotion
school exclusion: current HPA advice is 5 days from start of skin eruption. They also state 'Traditionally children have been
excluded until all lesions are crusted. However, transmission has never been reported beyond the fifth day of the rash.'
immunocompromised patients and newborns with peripartum exposure should receive varicella zoster immunoglobulin (VZIG).
If chickenpox develops then IV aciclovir should be considered

A common complication is secondary bacterial infection of the lesions. Rare complications include

pneumonia
encephalitis (cerebellar involvement may be seen)
disseminated haemorrhagic chickenpox
arthritis, nephritis and pancreatitis may very rarely be seen

*it was traditionally taught that patients were infective until all lesions had scabbed over

Question 35 of 180

A mother brings her 3-year-old boy who is known to have Down's syndrome to surgery, as she is concerned about his vision. Which one of
the following eye problems is least associated with Down's syndrome?
Refractive errors

Cataracts

Retinal detachment

Strabismus

Recurrent blepharitis

Next question

Down's syndrome: vision and hearing problems


Individuals with Down's syndrome are more likely to suffer from vision and hearing problems, as detailed below:
Vision

refractive errors are more common

strabismus: seen in around 20-40%


cataracts: congenital and acquired are both more common
recurrent blepharitis
glaucoma

Hearing

otitis media and glue ear are very common resulting in hearing problems

Question 36 of 180

At what age would the average child acquire the ability to sit without support?
12 months

4-6 months

10-11 months

7-8 months

3 months

Next question

Developmental milestones: gross motor


The table below summarises the major gross motor developmental milestones
Age

Milestone

3 months

Little or no head lag on being pulled to sit


Lying on abdomen, good head control
Held sitting, lumbar curve

6 months

Lying on abdomen, arms extended


Lying on back, lifts and grasps feet
Pulls self to sitting
Held sitting, back straight
Rolls front to back

7-8 months

Sits without support (Refer at 12 months)

9 months

Pulls to standing
Crawls

12 months

Cruises
Walks with one hand held

13-15 months Walks unsupported (Refer at 18 months)

Age

Milestone

18 months

Squats to pick up a toy

2 years

Runs
Walks upstairs and downstairs holding on to rail

3 years

Rides a tricycle using pedals


Walks up stairs without holding on to rail

4 years

Hops on one leg

Notes

the majority of children crawl on all fours before walking but some children 'bottom-shuffle'. This is a normal variant and runs in
families

Question 37 of 180

Which one of the following statements regarding the MMR vaccine is incorrect?
Malaise, fever and rash may occur after around one week

Allergy to neomycin is a contraindication

The second dose is given at 3-4 years

MMR is a live attenuated vaccine

Children who received another live vaccine 2 weeks ago can safely have MMR

Next question

MMR vaccine
Children in the UK receive two doses of the Measles, Mumps and Rubella (MMR) vaccine before entry to primary school. This currently
occurs at 12-15 months and 3-4 years as part of the routine immunisation schedule
Contraindications to MMR

severe immunosuppression
allergy to neomycin
children who have received another live vaccine by injection within 4 weeks
pregnancy should be avoided for at least 1 month following vaccination
immunoglobulin therapy within the past 3 months (there may be no immune response to the measles vaccine if antibodies are
present)

Adverse effects

malaise, fever and rash may occur after the first dose of MMR. This typically occurs after 5-10 days and lasts around 2-3 days

Question 38 of 180

Which one of the following statements regarding retinoblastoma is true?


The most common presenting symptom is strabismus

70% of cases are heriditary

All cases require enucleation

The average age at diagnosis is 3 months

More than 90% of children survive to adulthood

Next question

Retinoblastoma
Retinoblastoma is a the most common ocular malignancy found in children. The average age of diagnosis is 18 months.
Pathophysiology

caused by a loss of function of the retinoblastoma tumour suppressor gene on chromosome 13


around 10% of cases are hereditary

Possible features

absence of red-reflex, repalced by a white pupil (leukocoria) - the most common presenting symptom
strabismus
visual problems

Management

enucleation is not the only option


depending on how advanced the tumour is other options include external beam radiation therapy, chemotherapy and
photocoagulation

Prognosis

excellent, with > 90% surviving into adulthood

Question 39 of 180

A 2-year-old child with a history of atopic eczema is brought to the local GP surgery. Her eczema is usually well controlled with emollients
but her parents are concerned as the facial eczema has got significantly worse overnight. She now has painful clustered blisters on both
cheeks, around her mouth on her neck. Her temperature is 37.9C. What is the most appropriate management?
Advise paracetamol + emollients and reassure

Admit to hospital

Add hydrocortisone 1%

Oral flucloxacillin

Topical fusidic acid

Next question

Eczema herpeticum is a serious condition that requires IV antivirals

Eczema herpeticum
Eczema herpeticum describes a severe primary infection of the skin by herpes simplex virus 1 or 2. It is more commonly seen in children
with atopic eczema. As it is potentially life threatening children should be admitted for IV aciclovir
Question 40 of 180

A 5-year-old girl is brought to surgery due to a high temperature. On examination she is noted to have an evolving purpuric rash. What is
the most appropriate course of action?
IM benzylpenicillin 150mg

IM benzylpenicillin 300mg

IM benzylpenicillin 600mg

IM benzylpenicillin 900mg

IM benzylpenicillin 1200mg

Next question

The RCGP have previously fed back that doctors are expected to be familiar with emergency drug doses, and have mentioned suspected
meningococcal septicaemia in particular

Paediatric drug doses: emergency


The current BNF should always be consulted prior to prescribing drugs you are unfamiliar with, the following is just a guide
IM benzylpenicillin for suspected meningococcal septicaemia in the community

Age

Dose

< 1 year

300 mg

1 - 10 years600 mg
> 10 years 1200 mg
Question 41 of 180

What is the most useful investigation to screen for the complications of Kawasaki disease?
Echocardiogram

Lumbar puncture

Coronary angiogram

MRI of posterior fossa

ECG

Next question

Kawasaki disease
Kawasaki disease is a type of vasculitis which is predominately seen in children. Whilst Kawasaki disease is uncommon it is important to
recognise as it may cause potentially serious complications, including coronary artery aneurysms
Features

high-grade fever which lasts for > 5 days. Fever is characteristically resistant to antipyretics
conjunctival injection
bright red, cracked lips
strawberry tongue
cervical lymphadenopathy
red palms of the hands and the soles of the feet which later peel

Kawasaki disease is a clinical diagnosis as there is no specific diagnostic test


Management

high-dose aspirin*
intravenous immunoglobulin
echocardiogram (rather than angiography) is used as the initial screening test for coronary artery aneurysms

Complications

coronary artery aneurysm

*Kawasaki disease is one of the few indications for the use of aspirin in children. Due to the risk of Reye's syndrome aspirin is normally
contraindicated in children.

Question 42 of 180

Precocious puberty in females may be defined as the development of secondary sexual characteristics before:
8 years of age

9 years of age

10 years of age

11 years of age

12 years of age

Next question

Precocious puberty
Definition

'development of secondary sexual characteristics before 8 years in females and 9 years in males'
more common in females

Some other terms

thelarche (the first stage of breast development)


adrenarche (the first stage of pubic hair development)

May be classified into:


1. Gonadotrophin dependent ('central', 'true')

due to premature activation of the hypothalamic-pituitary-gonadal axis


FSH & LH raised

2. Gonadotrophin independent ('pseudo', 'false')

due to excess sex hormones


FSH & LH low

Males - uncommon and usually has an organic cause

Testes

bilateral enlargement = gonadotrophin release from intracranial lesion


unilateral enlargement = gonadal tumour
small testes = adrenal cause (tumour or adrenal hyperplasia)

Females - usually idiopathic or familial and follows normal sequence of puberty


Organic causes

are rare, associated with rapid onset, neurological symptoms and signs and dissonance
e.g. McCune Albright syndrome

Question 43 of 180

Which one of the following vaccinations should be avoided in patients who are HIV positive?
Rabies

BCG

Hepatitis B

Pertussis

Diphtheria

Next question

Live attenuated vaccines

BCG
MMR
oral polio
yellow fever
oral typhoid

Vaccinations
It is important to be aware of vaccines which are of the live-attenuated type as these may pose a risk to immunocompromised patients. The
main types of vaccine are as follows:
Live attenuated

BCG
measles, mumps, rubella (MMR)
influenza (intranasal)

oral rotavirus
oral polio
yellow fever
oral typhoid*

Inactivated preparations

rabies
influenza (intramuscular)

Detoxified exotoxins

tetanus

Extracts of the organism/virus (sometimes termed fragment)**

diphtheria
pertussis ('acellular' vaccine)
hepatitis B
meningococcus, pneumococcus, haemophilus

Notes

influenza: different types are available, including whole inactivated virus, split virion (virus particles disrupted by detergent
treatment) and sub-unit (mainly haemagglutinin and neuraminidase)
cholera: contains inactivated Inaba and Ogawa strains of Vibrio cholerae together with recombinant B-subunit of the cholera
toxin
hepatitis B: contains HBsAg adsorbed onto aluminium hydroxide adjuvant and is prepared from yeast cells using recombinant
DNA technology

*whole cell typhoid vaccine is no longer used in the UK


**may also be produced using recombinant DNA technology

Question 44 of 180

A mother brings her 9-year-old daughter into surgery. She has been having recurrent headaches. Which one of the following features of
migraine is more common in children?
Prolonged migraines (e.g. 24-48 hours)

Strictly unilateral symptoms

Hemiplegia

Good response to metoclopramide

Gastrointestinal disturbance

Next question

Nausea, vomiting and abdominal pain are common in children with migraine.

Migraine: diagnostic criteria


The International Headache Society has produced the following diagnostic criteria for migraine without aura:
PointCriteria
A

At least 5 attacks fulfilling criteria B-D

Headache attacks lasting 4-72 hours* (untreated or unsuccessfully treated)


Headache has at least two of the following characteristics:

1. unilateral location*
2. pulsating quality (i.e., varying with the heartbeat)
3. moderate or severe pain intensity
4. aggravation by or causing avoidance of routine physical activity (e.g., walking or climbing stairs)

During headache at least one of the following:


D

1. nausea and/or vomiting*


2. photophobia and phonophobia

Not attributed to another disorder (history and examination do not suggest a secondary headache disorder or, if they do, it is ruled out
by appropriate investigations or headache attacks do not occur for the first time in close temporal relation to the other disorder)

*In children, attacks may be shorter-lasting, headache is more commonly bilateral, and gastrointestinal disturbance is more prominent.
Migraine with aura (seen in around 25% of migraine patients) tends to be easier to diagnose with a typical aura being progressive in nature
and may occur hours prior to the headache. Typical aura include a transient hemianopic disturbance or a spreading scintillating scotoma
('jagged crescent'). Sensory symptoms may also occur
If we compare these guidelines to the NICE criteria the following points are noted:

NICE suggests migraines may be unilateral or bilateral


NICE also give more detail about typical auras:

Auras may occur with or without headache and:

are fully reversible


develop over at least 5 minutes
last 5-60 minutes

The following aura symptoms are atypical and may prompt further investigation/referral;

motor weakness
double vision
visual symptoms affecting only one eye
poor balance

decreased level of consciousness.


Question 45 of 180

Which one of the following examination findings is abnormal?

Respiratory rate of 40 / min in a 2-year-old

Heart rate of 140 bpm in a 6-month-old

Respiratory rate of 24 / min in a 10-year-old

Heart rate of 120 bpm in a 18-month-old

Heart rate of 110 bpm in a 7-year-old

Paediatric vital signs

The table below indicate age-appropriate vital signs


Age Heart rate Respiratory rate
<1

110 - 160 30 - 40

1 - 2 100 - 150 25 - 35
2 - 5 90 - 140

25 - 30

5 - 12 80 - 120

20 - 25

> 12 60 - 100

15 - 20

Question 46 of 180

Which booster vaccines do young people usually receive between the ages of 13 - 18 years?
Key

DTaP = Diphtheria, Tetanus, acellular Pertussis vaccine


IPV = Inactivated Polio Vaccine
Hib = Haemophilus influenzae B vaccine
PCV = Pneumococcal Conjugate Vaccine
Men C = Meningococcal C vaccine
MMR = Measles, Mumps, Rubella vaccine
DT = Diphtheria, Tetanus vaccine

Next question

Hib + Men C

DT + Hib

DT + IPV + Men C

IPV + Hib

DT + Men C

Next question

Immunisation schedule
The current UK immunisation schedule is as follows. Please note that this table includes the changes announced in 2010 which merged the
12 and 13 month visits into one.
Age

Recommended immunisations

At birth

BCG / hepatitis B vaccine if risk factors (see below)

2 months

DTaP/IPV/Hib + PCV
Oral rotavirus vaccine

3 months

DTaP/IPV/Hib + Men C
Oral rotavirus vaccine

4 months

DTaP/IPV/Hib + PCV

12-13 months Hib/Men C + MMR + PCV


2-3 years

Flu vaccine (annual)

3-4 years

MMR + DTaP/IPV

12-13 years HPV vaccination for girls


13-18 years DT/IPV + MenC

At birth the BCG vaccine should be given if the baby is deemed at risk of tuberculosis (e.g. Tuberculosis in the family in the past 6 months).
Hepatitis B vaccine should be given at birth if the mother is HBsAg +ve.
Key

DTaP = Diphtheria, Tetanus, acellular Pertussis vaccine


IPV = Inactivated Polio Vaccine
Hib = Haemophilus influenzae B vaccine
PCV = Pneumococcal Conjugate Vaccine
Men C = Meningococcal C vaccine
MMR = Measles, Mumps, Rubella vaccine
DT = Diphtheria, Tetanus vaccine
HPV = Human Papilloma Vaccine

Question 47 of 180

Your next patient is a 2-year-old boy. His mother is very concerned as he has had a high temperature all day and is not taking fluids. On
review he is drowsy with a toxic appearance but no rash is seen. Which one of the following features is most supportive of a diagnosis of
meningococcal septicaemia?
Photophobia

Temperature of 39.4C

Neck stiffness

Bulging fontanelle

Cold peripheries

Next question

Whilst the typical non-blanching rash (petechial or purpuric) develops in over 80% of patients with meningococcal septicaemia it is
important to be aware of other symptoms/signs. A high temperature can be seen with any childhood illness and is non-specific. Neck
stiffness, photophobia and a bulging fontanelle suggest meninigits rather than septicaemia. Of course you would not be able to assess the
fontanelle in a patient of this age anyway. Please see the link for a comprehensive list of symptoms/signs.

Meningococcal septicaemia
Meningococcal septicaemia is a frightening condition for patients, parents and doctors. It is associated with a high morbidity and mortality
unless treated early - meningococcal disease is the leading infectious cause of death in early childhood. A high index of suspicion is
therefore needed. Much of the following is based on the 2010 NICE guidelines (please see link).
Presentation of meningococcal disease:

15% - meningitis
25% - septicaemia
60% - a combination of meningitis and septicaemia

NICE divide the features of meningococcal septicaemia into:

common non-specific symptoms/signs e.g. fever, vomiting, lethargy


less common non-specific symptoms/signs e.g. Chills, shivering
more specific symptoms/signs e.g. Non-blanching rash, altered mental state, capillary refill time more than 2 seconds, unusual
skin colour, shock, hypotension, leg pain, cold hands/feet

Management if suspected meningococcal septicaemia

give intramuscular or intravenous benzylpenicillin unless there is a history of anaphylaxis (do not give if this will delay hospital
transfer)
NICE recommend phoning 999

Question 48 of 180

What is the most common presenting feature of Wilms tumour?

Recurrent urinary tract infections

Abdominal mass

Fever

Loin pain

Haematuria

Next question

Wilms' tumour
Wilms' nephroblastoma is one of the most common childhood malignancies. It typically presents in children under 5 years of age, with a
median age of 3 years old.
Features

abdominal mass (most common presenting feature)


painless haematuria
flank pain
other features: anorexia, fever
unilateral in 95% of cases
metastases are found in 20% of patients (most commonly lung)

Associations

Beckwith-Wiedemann syndrome
as part of WAGR syndrome with Aniridia, Genitourinary malformations, mental Retardation
hemihypertrophy
around one-third of cases are associated with a mutation in the WT1 gene on chromosome 11

Management

nephrectomy
chemotherapy
radiotherapy if advanced disease
prognosis: good, 80% cure rate

Image used on license from PathoPic

Histological features include epithelial tubules, areas of necrosis, immature glomerular structures, stroma with spindle cells and small cell blastomatous tissues
resembling the metanephric blastema

Question 49 of 180

Which one of the following is least recognised as a side-effect of carbamazepine?


Headache

Hirsutism

Diplopia

Ataxia

Transient erythematous rash

Next question

Carbamazepine
Carbamazepine is chemically similar to the tricyclic antidepressant drugs. It is most commonly used in the treatment of epilepsy, particularly
partial seizures, where carbamazepine remains a first-line medication. Other uses include

neuropathic pain (e.g. trigeminal neuralgia, diabetic neuropathy)


bipolar disorder

Mechanism of action

sodium channel blocker, decreasing the sodium influx into neurons which in turn decreases excitability

Adverse effects

P450 enzyme inducer


dizziness and ataxia
drowsiness
headache
visual disturbances (especially diplopia)
Steven-Johnson syndrome
leucopenia and agranulocytosis
syndrome of inappropriate ADH secretion

Question 50 of 180

Which one of the following statements regarding the vaccine used to routinely immunise children against influenza is correct?
The most common side-effect is pyrexia

It should be given intramuscularly

It is a live vaccine

Two doses 4 weeks apart are required

Should be given at 12 months if the child was born before 35 weeks gestation

Next question

Influenza vaccination
Seasonal influenza still accounts for a significant morbidity and mortality in the UK each winter, with the influenza season typically starting
in the middle of November. This may vary year from year so it is recommended that vaccination occurs between September and early
November. There are three types of influenza virus; A, B and C. Types A and B account for the majority of clinical disease.
Prior to 2013 flu vaccination was only offered to the elderly and at risk groups.
Remember that the type of vaccine given routinely to children and the one given to the elderly and at risk groups is different (live vs.
inactivated) - this explains the different contraindications
Children
A new NHS influenza vaccination programme for children was announced in 2013. There are three key things to remember about the
children's vaccine:

it is given intranasally
the first dose is given at 2-3 years, then annually after that

it is a live vaccine (cf. injectable vaccine below)

Some other points

children who were traditionally offered the flu vaccine (e.g. asthmatics) will now be given intranasal vaccine unless this is
inappropriate, for example if they are immunosuppressed. In this situation the inactivated, injectable vaccine should be given
only children aged 2-9 years who have not received an influenza vaccine before need 2 doses
it is more effective than the injectable vaccine

Contraindications

immunocompromised
aged < 2 years
current febrile illness or blocked nose/rhinorrhoea
current wheeze (e.g. ongoing viral-induced wheeze/asthma) or history of severe asthma (BTS step 4)
egg allergy
pregnancy/breastfeeding
if the child is taking aspirin (e.g. for Kawasaki disease) due to a risk of Reye's syndrome

Side-effects

blocked-nose/rhinorrhoea
headache
anorexia

Adults and at-risk groups


Current vaccines are trivalent and consist of two subtypes of influenza A and one subtype of influenza B.
The Department of Health recommends annual influenza vaccination for people older than 65 years and those older than 6 months if they
have:

chronic respiratory disease (including asthmatics who use inhaled steroids)


chronic heart disease (heart failure, ischaemic heart disease, including hypertension if associated with cardiac complications)
chronic kidney disease
chronic liver disease: cirrhosis, biliary atresia, chronic hepatitis
chronic neurological disease: (e.g. Stroke/TIAs)
diabetes mellitus (including diet controlled)
immunosuppression due to disease or treatment (e.g. HIV)
asplenia or splenic dysfunction
pregnant women

Other at risk individuals include:

health and social care staff directly involved in patient care (e.g. NHS staff)
those living in long-stay residential care homes
carers of the elderly or disabled person whose welfare may be at risk if the carer becomes ill (at the GP's discretion)

The influenza vaccine

it is an inactivated vaccine, so cannot cause influenza. A minority of patients however develop fever and malaise which may last
1-2 days
should be stored between +2 and +8C and shielded from light
contraindications include hypersensitivity to egg protein.
in adults the vaccination is around 75% effective, although this figure decreases in the elderly
it takes around 10-14 days after immunisation before antibody levels are at protective levels

Question 51 of 180

Which of the following conditions is usually inherited in an autosomal dominant fashion?


Friedreich's ataxia

Hurler's syndrome

Phenylketonuria

Familial adenomatous polyposis

Haemochromatosis

Next question

Autosomal recessive conditions are 'metabolic' - exceptions: inherited ataxias


Autosomal dominant conditions are 'structural' - exceptions: Gilbert's, hyperlipidaemia type II

Autosomal dominant conditions

Autosomal recessive conditions are often thought to be 'metabolic' as opposed to autosomal dominant conditions being 'structural', notable
exceptions:

some 'metabolic' conditions such as Hunter's and G6PD are X-linked recessive whilst others such as hyperlipidaemia type II
and hypokalaemic periodic paralysis are autosomal dominant
some 'structural' conditions such as ataxia telangiectasia and Friedreich's ataxia are autosomal recessive

The following conditions are autosomal dominant:

Achondroplasia
Acute intermittent porphyria
Adult polycystic disease
Antithrombin III deficiency
Ehlers-Danlos syndrome
Familial adenomatous polyposis
Hereditary haemorrhagic telangiectasia
Hereditary spherocytosis
Hereditary non-polyposis colorectal carcinoma
Huntington's disease

Hyperlipidaemia type II
Hypokalaemic periodic paralysis
Malignant hyperthermia
Marfan's syndromes
Myotonic dystrophy
Neurofibromatosis
Noonan syndrome
Osteogenesis imperfecta
Peutz-Jeghers syndrome
Retinoblastoma
Romano-Ward syndrome
Tuberose sclerosis
Von Hippel-Lindau syndrome
Von Willebrand's disease*

*type 3 von Willebrand's disease (most severe form) is inherited as an autosomal recessive trait. Around 80% of patients have type 1
disease

Question 52 of 180

A mother brings her 14-month-old daughter into surgery as she is concerned about her hearing. For a child born in the United Kingdom, at
what age would their hearing first be formally assessed?
Newborn - otoacoustic emission

6-9 months - distraction test

18 months - pure-tone audiometry

School entry - pure-tone audiometry

No formal screening programme

Next question

Hearing testing in children


The table below summarises the hearing tests which may be performed on children
Age

Test

Comments

Newborn

All newborns should be tested as part of the Newborn Hearing Screening Programme. A computer
Otoacoustic emission
generated click is played through a small earpiece. The presence of a soft echo indicates a healthy
test
cochlea

Newborn &
infants

Auditory Brainstem
Response test

May be done if otoacoustic emission test is abnormal

Age

Test

Comments

6-9 months

Distraction test

Performed by health visitor, requires two trained staff

18 months 2.5 years

Recognition of familiar
Uses familiar objects e.g. teddy, cup. Ask child simple questions - e.g. 'where is the teddy?'
objects

> 2.5 years

Performance testing

> 2.5 years

Speech discrimination
Uses similar sounding objects e.g. Kendall Toy test, McCormick Toy Test
tests

> 3 years

Pure tone audiometry Done at school entry in most areas of the UK

As well as the above test there is a questionnaire for parents in the Personal Child Health Records - 'Can your baby hear you?'

Question 53 of 180

A 14-year-old male being investigated for iron-deficiency anaemia is found to have numerous polyps in his jejunum. On examination he is
also noted to have pigmented lesions on his palms and soles. What is the likely diagnosis?
Hereditary non-polyposis colorectal carcinoma

Gardner's syndrome

Familial adenomatous polyposis

Peutz-Jeghers syndrome

Hereditary haemorrhagic telangiectasia

Next question

Hereditary haemorrhagic telangiectasia is associated with mucocutaneous lesions and iron-deficiency anaemia but intestinal polyps are not
a feature

Peutz-Jeghers syndrome
Peutz-Jeghers syndrome is an autosomal dominant condition characterised by numerous hamartomatous polyps in the gastrointestinal
tract. It is also associated with pigmented freckles on the lips, face, palms and soles. Around 50% of patients will have died from a
gastrointestinal tract cancer by the age of 60 years.
Genetics

autosomal dominant
responsible gene encodes serine threonine kinase LKB1 or STK11

Features

hamartomatous polyps in GI tract (mainly small bowel)


pigmented lesions on lips, oral mucosa, face, palms and soles
intestinal obstruction e.g. intussusception

gastrointestinal bleeding

Management

conservative unless complications develop

Question 54 of 180

A 14-year-old boy is brought to surgery by his father who is concerned he is shorter than the other boys at school, despite having started
puberty. His father is 1.70 m tall and his mother is 1.50 m tall. Given his parents height, what is his adult height potential?
1.67 m

1.72 m

1.53 m

1.65 m

1.70 m

Next question

Short stature
Short stature may be caused by:

normal variant (often familial)


constitutional delay of growth and puberty
chronic illness e.g. cystic fibrosis, inflammatory bowel disease
endocrine: growth hormone deficiency , hypothyroidism, steroid excess
syndromes: Turner's, Down's, Prader-Willi
skeletal dysplasias e.g. achondroplasia

The adult height potential may be calculated for a male child by (father's height in cm + mother's height in cm) / 2 then add 7 cm
For a female child by (father's height in cm + mother's height in cm) / 2 then minus 7 cm
This can then be plotted on a height centile chart to find the mid-parental centile

Question 55 of 180

Which one of the following conditions does the neonatal blood spot screening test not screen for?

Phenylketonuria

Sickle cell disease

Cystic fibrosis

Galactosaemia

Hypothyroidism

Next question

Neonatal blood spot screening


Neonatal blood spot screening (previously called the Guthrie test or 'heel-prick test') is performed at 5-9 days of life
The following conditions are currently screened for:

congenital hypothyroidism
cystic fibrosis
phenylketonuria
sickle cell disease
medium chain acyl-CoA dehydrogenase deficiency (MCADD)

Question 56 of 180

Which one of the following neonatal complications is least commonly seen in diabetic pregnancies?
Stillbirth

Respiratory distress syndrome

Microsomia

Hypoglycaemia

Sacral agenesis

Next question

Diabetes predisposes to macrosomia rather than microsomia

Pregnancy: diabetes - complications


Maternal complications

polyhydramnios - 25%, possibly due to fetal polyuria


preterm labour - 15%, associated with polyhydramnios

Neonatal complications

macrosomia (although diabetes may also cause small for gestational age babies)
hypoglycaemia (secondary to beta cell hyperplasia)
respiratory distress syndrome: surfactant production is delayed
polycythaemia: therefore more neonatal jaundice
malformation rates increase 3-4 fold e.g. sacral agenesis, CNS and CVS malformations (hypertrophic cardiomyopathy)
stillbirth
hypomagnesaemia
hypocalcaemia
shoulder dystocia (may cause Erb's palsy)

Question 57 of 180

A 14-month-old girl is diagnosed as having roseola infantum. What is the most common complication of this disease?
Febrile convulsions

Transient synovitis

Otitis media with effusion

Pneumonia

Orchitis

Next question

Roseola infantum - febrile convulsions occur in around 10-15%

Roseola infantum
Roseola infantum (also known as exanthem subitum, occasionally sixth disease) is a common disease of infancy caused by the human
herpes virus 6 (HHV6). It has an incubation period of 5-15 days and typically affects children aged 6 months to 2 years.
Features

high fever: lasting a few days, followed by a


maculopapular rash
febrile convulsions occur in around 10-15%
diarrhoea and cough are also commonly seen

Other possible consequences of HHV6 infection

aseptic meningitis

hepatitis

Question 58 of 180

Which one of the following is least associated with constipation in children?


Dehydration

Anal fissure

Addison's disease

Hirschsprung's disease

Hypercalcaemia

Next question

Constipation in children
The frequency at which children open their bowels varies widely, but generally decreases with age from a mean of 3 times per day for
infants under 6 months old to once a day after 3 years of age.
NICE produced guidelines in 2010 on the diagnosis and management of constipation in children. A diagnosis of constipation is suggested
by 2 or more of the following:
Child < 1 year

Stool pattern

Fewer than 3 complete stools per week


Fewer than 3 complete stools per week (type 3 or 4 on Bristol Stool
(type 3 or 4)
Form Scale) (this does not apply to exclusively breastfed babies after 6 Overflow soiling (commonly very loose,
weeks
very smelly, stool passed without
of age)
sensation)
Hard large stool
'Rabbit droppings' (type 1)
'Rabbit droppings' (type 1)
Large, infrequent stools that can block the
toilet

Distress on passing stool


Symptoms associated
Bleeding associated with hard stool
with defecation
Straining

History

Child > 1 year

Previous episode(s) of constipation


Previous or current anal fissure

Poor appetite that improves with passage


of large stool
Waxing and waning of abdominal pain with
passage of stool
Evidence of retentive posturing: typical
straight legged, tiptoed, back arching
posture
Straining
Anal pain
Previous episode(s) of constipation
Previous or current anal fissure
Painful bowel movements and bleeding
associated with hard stools

The vast majority of children have no identifiable cause - idiopathic constipation. Other causes of constipation in children include:

dehydration
low-fibre diet
medications: e.g. Opiates
anal fissure
over-enthusiastic potty training
hypothyroidism
Hirschsprung's disease
hypercalcaemia
learning disabilities

After making a diagnosis of constipation NICE then suggesting excluding secondary causes. If no red or amber flags are present then a
diagnosis of idiopathic constipation can be made:

Indicates idiopathic constipation

'Red flag' suggesting underlying


disorder

Timing

Starts after a few weeks of life


Obvious precipitating factors coinciding with the start of symptoms: fissure, change
Reported from birth or first few
of diet, timing of potty/toilet training or acute events such as infections, moving
weeks of life
house, starting nursery/school, fears and phobias, major change in family, taking
medicines

Passage of
meconium

< 48 hours

Stool pattern
Growth

> 48 hours
'Ribbon' stools

Generally well, weight and height within normal limits, fit and active

Faltering growth is an amber flag

Neuro/locomotor No neurological problems in legs, normal locomotor development

Previously unknown or
undiagnosed weakness in legs,
locomotor delay

Abdomen

Distension

Diet

Changes in infant formula, weaning, insufficient fluid intake or poor diet


Amber flag: Disclosure or evidence
that raises concerns over possibility
of child maltreatment

Other

Prior to starting treatment the child needs to be assessed for faecal impaction. Factors which suggest faecal impaction include:

symptoms of severe constipation


overflow soiling
faecal mass palpable in abdomen (digital rectal examination should only be carried out by a specialist)

NICE guidelines on management


If faecal impaction is present

polyethylene glycol 3350 + electrolytes (Movicol Paediatric Plain) using an escalating dose regimen as the first-line treatment
add a stimulant laxative if Movicol Paediatric Plain does not lead to disimpaction after 2 weeks
substitute a stimulant laxative singly or in combination with an osmotic laxative such as lactulose if Movicol Paediatric Plain is
not tolerated
inform families that disimpaction treatment can initially increase symptoms of soiling and abdominal pain

Maintenance therapy

very similar to the above regime, with obvious adjustments to the starting dose, i.e.
first-line: Movicol Paediatric Plain
add a stimulant laxative if no response
substitute a stimulant laxative if Movicol Paediatric Plain is not tolerated. Add another laxative such as lactulose or docusate if
stools are hard
continue medication at maintenance dose for several weeks after regular bowel habit is established, then reduce dose gradually

General points

do not use dietary interventions alone as first-line treatment although ensure child is having adequate fluid and fibre intake
consider regular toileting and non-punitive behavioural interventions
for all children consider asking the Health Visitor or Paediatric Continence Advisor to help support the parents.

The NICE guidelines do not specifically discuss the management of very young child. The following recommendations are largely based on
the old Clinical Knowledge Summaries recommendations.
Infants not yet weaned (usually < 6 months)

bottle-fed infants: give extra water in between feeds. Try gentle abdominal massage and bicycling the infant's legs
breast-fed infants: constipation is unusual and organic causes should be considered

Infants who have or are being weaned

offer extra water, diluted fruit juice and fruits


if not effective consider adding lactulose

Question 59 of 180

You are doing the six week check on a baby girl. Which one of the following best describes the Barlow test for developmental dysplasia of
the hip?
Attempts to relocate a dislocated femoral head

Upward pressure on the femur with the hip flexed at 90 degrees

Observation of the relative height of the knees with the hips flexed at 90 degrees

Observation for buttock crease asymmetry with the hips flexed at 90 degrees

Attempts to dislocate an articulated femoral head

Next question

Developmental dysplasia of the hip

Developmental dysplasia of the hip (DDH) is gradually replacing the old term 'congenital dislocation of the hip' (CDH). It affects around 13% of newborns.
Risk factors

female sex: 6 times greater risk


breech presentation
positive family history
firstborn children
oligohydramnios
birth weight > 5 kg
congenital calcaneovalgus foot deformity

DDH is slightly more common in the left hip. Around 20% of cases are bilateral.
Clinical examination is made using the Barlow and Ortolani tests:

Barlow test: attempts to dislocate an articulated femoral head


Ortolani test: attempts to relocate a dislocated femoral head

Ultrasound is used to confirm the diagnosis if clinically suspected


Management

most unstable hips will spontaneously stabilise by 3-6 weeks of age


Pavlik harness (flexion-abduction orthosis) in children younger than 4-5 months
older children may require surgery

Question 60 of 180

A 30-year-old woman presents for the 6 week check of her son who was diagnosed with Down's syndrome shortly after birth. A letter from
genetics has confirmed non-disjunction as the cause of the trisomy. She asks you about future pregnancies. What is the chance that her
next child will also have Down's syndrome?
1 in 10

1 in 100

1 in 25

1 in 250

Same as background population for maternal age

Next question

Down's syndrome recurrence - normally 1 in 100

Down's syndrome: epidemiology and genetics


Risk of Down's syndrome with increasing maternal age

risk at 30 years = 1/1000


35 years = 1/350
40 years = 1/100
45 years = 1/30

One way of remembering this is by starting at 1/1,000 at 30 years and then dividing the denominator by 3 (i.e. 3 times more common) for
every extra 5 years of age
Cytogenetics
Mode

% of cases Risk of recurrence

Non-disjunction

94%

Robertsonian translocation
5%
(usually onto 14)
Mosaicism

1 in 100 if under mother < 35 years


10-15% if mother is translocation carrier
2.5% if father is translocation carrier

1%

The chance of a further child with Down's syndrome is approximately 1 in 100 if the mother is less than 35 years old. If the trisomy 21 is a
result of a translocation the risk is much higher

Question 61 of 180

The mother of a 6-week-old baby girl born at 32 weeks gestation asks for advice about immunisation. What should happen regarding the
first set of vaccines?
Give first set of vaccinations at 3 months (i.e. delay for 1 month)

Give DTaP/IPV/Hib at 2 months but not PCV

Give first set of vaccinations at 4 months (i.e. correct for gestational age)

Give first set of vaccinations as per normal timetable but within hospital environment

Give as per normal timetable

Next question

Immunisation schedule
The current UK immunisation schedule is as follows. Please note that this table includes the changes announced in 2010 which merged the
12 and 13 month visits into one.

Age

Recommended immunisations

At birth

BCG / hepatitis B vaccine if risk factors (see below)

2 months

DTaP/IPV/Hib + PCV
Oral rotavirus vaccine

3 months

DTaP/IPV/Hib + Men C
Oral rotavirus vaccine

4 months

DTaP/IPV/Hib + PCV

12-13 months Hib/Men C + MMR + PCV


2-3 years

Flu vaccine (annual)

3-4 years

MMR + DTaP/IPV

12-13 years HPV vaccination for girls


13-18 years DT/IPV + MenC

At birth the BCG vaccine should be given if the baby is deemed at risk of tuberculosis (e.g. Tuberculosis in the family in the past 6 months).
Hepatitis B vaccine should be given at birth if the mother is HBsAg +ve.
Key

DTaP = Diphtheria, Tetanus, acellular Pertussis vaccine


IPV = Inactivated Polio Vaccine
Hib = Haemophilus influenzae B vaccine
PCV = Pneumococcal Conjugate Vaccine
Men C = Meningococcal C vaccine
MMR = Measles, Mumps, Rubella vaccine
DT = Diphtheria, Tetanus vaccine
HPV = Human Papilloma Vaccine

Question 62 of 180

Which one of the following drugs is not contra-indicated whilst breast feeding?
Tetracycline

Ciprofloxacin

Chloramphenicol

Doxycycline

Ceftriaxone

Next question

Breast feeding: contraindications

The major breastfeeding contraindications tested in exams relate to drugs (see below). Other contraindications of note include:

galactosaemia
viral infections - this is controversial with respect to HIV in the developing world. This is because there is such an increased
infant mortality and morbidity associated with bottle feeding that some doctors think the benefits outweigh the risk of HIV
transmission

Drug contraindications
The following drugs can be given to mothers who are breast feeding:

antibiotics: penicillins, cephalosporins, trimethoprim


endocrine: glucocorticoids (avoid high doses), levothyroxine*
epilepsy: sodium valproate, carbamazepine
asthma: salbutamol, theophyllines
psychiatric drugs: tricyclic antidepressants, antipsychotics**
hypertension: beta-blockers, hydralazine, methyldopa
anticoagulants: warfarin, heparin
digoxin

The following drugs should be avoided:

antibiotics: ciprofloxacin, tetracycline, chloramphenicol, sulphonamides


psychiatric drugs: lithium, benzodiazepines
aspirin
carbimazole
sulphonylureas
cytotoxic drugs
amiodarone

*the BNF advises that the amount is too small to affect neonatal hypothyroidism screening
**clozapine should be avoided

Question 63 of 180

You are asked to by the Primary Care Trust to design a program to improve the health of infants in the local community. What is the most
common cause of death of infants greater than one month but less than one year old?
Accidents

Congenital disorders

Sudden infant death syndrome

Cancer

Infection

Next question

After the age of 1 year accidents are the most common cause of death in children

Sudden infant death syndrome


Sudden infant death syndrome is the commonest cause of death in the first year of life. It is most common at 3 months of age
Risk factors

prematurity
parental smoking
hyperthermia (e.g. over-wrapping)
putting the baby to sleep prone
male sex
multiple births
bottle feeding
social classes IV and V
maternal drug use
incidence increases in winter

Following a cot death siblings should be screened for potential sepsis and inborn errors of metabolism

Question 64 of 180

A 12-year-old female from Bulgaria presents to the surgery. She reports being unwell for the past 2 weeks. Initially she had a sore throat
but she is now experiencing joint pains intermittently in her knees, hips and ankles. On examination there are some pink, ring shaped
lesions on the trunk and occasional jerking movements of the face and hands. What is the most likely diagnosis?
Lyme disease

Infective endocarditis

Polyarticular juvenile idiopathic arthritis

Rheumatic fever

Still's disease

Next question

Rheumatic fever: criteria


Rheumatic fever develops following an immunological reaction to recent (2-6 weeks ago) Streptococcus pyogenes infection. Diagnosis is
based on evidence of recent streptococcal infection accompanied by:

2 major criteria

1 major with 2 minor criteria

Evidence of recent streptococcal infection

ASOT > 200iu/mL


history of scarlet fever
positive throat swab
increase in DNase B titre

Major criteria

erythema marginatum
Sydenham's chorea
polyarthritis
carditis (endo-, myo- or peri-)
subcutaneous nodules

Minor criteria

raised ESR or CRP


pyrexia
arthralgia (not if arthritis a major criteria)
prolonged PR interval

Image used on license from DermNet NZ

Erythema marginatum is seen in around 10% of children with rheumatic fever. It is rare in adults

Question 65 of 180

Which one of the following is least recognised as a complication of measles infection?

Infertility

Keratoconjunctivitis

Encephalitis

Subacute sclerosing panencephalitis

Pneumonia

Next question

Measles
Overview

RNA paramyxovirus
spread by droplets
infective from prodrome until 4 days after rash starts
incubation period = 10-14 days

Features

prodrome: irritable, conjunctivitis, fever


Koplik spots (before rash): white spots ('grain of salt') on buccal mucosa
rash: starts behind ears then to whole body, discrete maculopapular rash becoming blotchy & confluent

Image used on license from DermNet NZ

Koplik spots

Complications

encephalitis: typically occurs 1-2 weeks following the onset of the illness)
subacute sclerosing panencephalitis: very rare, may present 5-10 years following the illness
febrile convulsions
giant cell pneumonia
keratoconjunctivitis, corneal ulceration
diarrhoea
increased incidence of appendicitis
myocarditis

Image used on license from DermNet NZ

The rash typically starts behind the ears and then spreads to the whole body

Management of contacts

if a child not immunized against measles comes into contact with measles then MMR should be offered (vaccine-induced
measles antibody develops more rapidly than that following natural infection)
this should be given within 72 hours

Reference ranges
End and review

Question 66 of 180

You are reviewing a 9-month-old child with suspected bronchiolitis. Which one of the following features should make you consider other
possible diagnoses?

Fine inspiratory crackles

Rhinitis

Feeding difficulties

Temperature of 39.7C

Expiratory wheeze

Next question

A low-grade fever is typical in bronchiolitis. SIGN guidelines advise that the presence of high fever should make the clinician carefully
consider other causes before making the diagnosis.

Bronchiolitis
Bronchiolitis is a condition characterised by acute bronchiolar inflammation. Respiratory syncytial virus (RSV) is the pathogen in 75-80% of
cases. SIGN released guidelines on bronchiolitis in 2006. Please see the link for more details.
Epidemiology

most common cause of a serious lower respiratory tract infection in < 1yr olds (90% are 1-9 months, with a peak incidence of 36 months). Maternal IgG provides protection to newborns against RSV
higher incidence in winter

Basics

respiratory syncytial virus (RSV) is the pathogen in 75-80% of cases


other causes: mycoplasma, adenoviruses
may be secondary bacterial infection
more serious if bronchopulmonary dysplasia (e.g. Premature), congenital heart disease or cystic fibrosis

Features

coryzal symptoms (including mild fever) precede:


dry cough
increasing breathlessness
wheezing, fine inspiratory crackles (not always present)
feeding difficulties associated with increasing dyspnoea are often the reason for hospital admission

SIGN suggested the following criteria for referral to hospital

poor feeding (< 50% normal)


lethargy
apnoea
respiratory rate > 70/min
nasal flaring or grunting
severe chest wall recession
cyanosis
oxygen saturation < 94%

uncertainty regarding diagnosis

Investigation

immunofluorescence of nasopharyngeal secretions may show RSV

Management is largely supportive

humidified oxygen is given via a head box


Question 67 of 180

A newborn baby is noted to have low-set ears, rocker bottom feet and overlapping of her fingers. What is the most likely
diagnosis?

Patau syndrome

Edward's syndrome

William's syndrome

Fragile X

Pierre-Robin syndrome

Childhood syndromes

Below is a list of common features of selected childhood syndromes


Syndrome

Key features

Patau syndrome (trisomy 13)

Microcephalic, small eyes


Cleft lip/palate
Polydactyly
Scalp lesions

Edward's syndrome (trisomy 18)

Micrognathia
Low-set ears
Rocker bottom feet
Overlapping of fingers

Fragile X

Learning difficulties
Macrocephaly
Long face
Large ears
Macro-orchidism

Noonan syndrome

Webbed neck
Pectus excavatum
Short stature

Next question

Syndrome

Key features
Pulmonary stenosis

Pierre-Robin syndrome*

Micrognathia
Posterior displacement of the tongue (may result in upper airway obstruction)
Cleft palate

Prader-Willi syndrome

Hypotonia
Hypogonadism
Obesity

William's syndrome

Short stature
Learning difficulties
Friendly, extrovert personality
Transient neonatal hypercalcaemia
Supravalvular aortic stenosis

*this condition has many similarities with Treacher-Collins syndrome. One of the key differences is that Treacher-Collins syndrome
is autosomal dominant so there is usually a family history of similar problems

Question 68 of 180

An 8-year-old boy who is known to have asthma is reviewed. His current treatment is a salbutamol inhaler as required and beclometasone
inhaler 200mcg bd. Despite this he regular requires salbutamol for exacerbations and suffers with a night time cough. What is the most
appropriate next step in management?
Increase beclometasone to 400mcg bd

Referral to a paediatrician

Trial of a long-acting beta2-agonist

Trial of a leukotriene receptor antagonist

Trial of an oral theophylline

Next question

The British Thoracic Society guidelines advise adding a long-acting beta2-agonist before maximising the inhaled steroid dose.

Asthma in children: stepwise management


The British Thoracic Society differentiate between children younger and older than 5 years in their 2014 guidelines:
Children aged under 5 years
StepTherapy
1

As-required reliever therapy: short-acting beta2-agonist

Regular preventer therapy: inhaled corticosteroids, 200-400mcg/day*


Or, if inhaled corticosteroids cannot be used, a leukotriene receptor antagonist

StepTherapy

Children aged 2-5 years: trial of a leukotriene receptor antagonist. If already taking leukotriene receptor antagonist reconsider inhaled
corticosteroids
Children aged under 2 years: refer to respiratory paediatrician

Refer to a respiratory paediatrician

Children aged over 5 years (similar to adult guidance)


StepTherapy
1

As-required reliever therapy: short-acting beta2-agonist

Regular preventer therapy: inhaled corticosteroids, 200-400mcg/day*


1. Add inhaled long-acting B2 agonist (LABA)
2. Assess control of asthma:

good response to LABA - continue LABA


benefit from LABA but control still inadequate: continue LABA and increase inhaled steroid dose to 400 mcg/day* (if not
already on this dose)
no response to LABA: stop LABA and increase inhaled steroid to 400 mcg/ day.* If control still inadequate, institute trial of
other therapies, leukotriene receptor antagonist or SR theophylline

Increase inhaled corticosteroids to high-dose, up to 800mcg/day*


Use daily steroid tablet at lowest dose providing control

Maintain inhaled corticosteroids at 800mcg/day


Refer to a paediatrician

*beclometasone dipropionate or equivalent

Question 69 of 180

What is the prevalence of atopic eczema in children?


1-2%

2-5%

15-20%

11-12%

5-10%

Next question

Eczema in children
Eczema occurs in around 15-20% of children and is becoming more common. It typically presents before 6 months but clears in around
50% of children by 5 years of age and in 75% of children by 10 years of age
Features

in infants the face and trunk are often affected


in younger children eczema often occurs on the extensor surfaces
in older children a more typical distribution is seen, with flexor surfaces affected and the creases of the face and neck

Management

avoid irritants
simple emollients: large quantities should be prescribed (e.g. 250g / week), roughly in a ratio of with topical steroids of 10:1. If a
topical steroid is also being used the emollient should be applied first followed by waiting at least 30 minutes before applying the
topical steroid. Creams soak into the skin faster than ointments. Emollients can become contaminated with bacteria - fingers
should not be inserted into pots (many brands have pump dispensers)
topical steroids
in severe cases wet wraps and oral ciclosporin may be used

Question 70 of 180

Which one of the following congenital infections is most characteristically associated with sensorineural deafness?
Toxoplasma gondii

Parvovirus B19

Rubella

Treponema pallidum

Cytomegalovirus

Next question

Congenital rubella

sensorineural deafness
congenital cataracts

Congenital infections
The major congenital infections encountered in examinations are rubella, toxoplasmosis and cytomegalovirus
Cytomegalovirus is the most common congenital infection in the UK. Maternal infection is usually asymptomatic
Rubella

Toxoplasmosis

Cytomegalovirus

Rubella

Toxoplasmosis

Cytomegalovirus

Characteristic features

Sensorineural deafness
Cerebral calcification
Congenital cataracts
Growth retardation
Chorioretinitis
Congenital heart disease (e.g. patent ductus arteriosus)
Purpuric skin lesions
Hydrocephalus
Glaucoma

Other features

Growth retardation
Hepatosplenomegaly
Purpuric skin lesions
'Salt and pepper' chorioretinitis
Microphthalmia
Cerebral palsy

Sensorineural deafness
Encephalitis/seizures
Anaemia
Pneumonitis
HepatosplenomegalyHepatosplenomegaly
Cerebral palsy
Anaemia
Jaundice
Cerebral palsy

Question 71 of 180

Each one of the following is a feature of Tetralogy of Fallot, except:


Pulmonary stenosis

Overriding aorta

Ventricular septal defect

Atrial septal defect

Right ventricular hypertrophy

Next question

Tetralogy of Fallot
Tetralogy of Fallot (TOF) is the most common cause of cyanotic congenital heart disease*. It typically presents at around 1-2 months,
although may not be picked up until the baby is 6 months old
TOF is a result of anterior malalignment of the aorticopulmonary septum. The four characteristic features are:

ventricular septal defect (VSD)


right ventricular hypertrophy
right ventricular outflow tract obstruction, pulmonary stenosis
overriding aorta

The severity of the right ventricular outflow tract obstruction determines the degree of cyanosis and clinical severity
Other features

cyanosis
causes a right-to-left shunt
ejection systolic murmur due to pulmonary stenosis (the VSD doesn't usually cause a murmur)
a right-sided aortic arch is seen in 25% of patients
chest x-ray shows a 'boot-shaped' heart, ECG shows right ventricular hypertrophy

Management

surgical repair is often undertaken in two parts


cyanotic episodes may be helped by beta-blockers to reduce infundibular spasm

*however, at birth transposition of the great arteries is the more common lesion as patients with TOF generally present at around 1-2
months

Question 72 of 180

Which one of the following statements regarding childhood squints is correct?


Amblyopia is a sign of a paralytic squint

Divergent squints are more common than convergent squints

The corneal light reflection test is a suitable screening test

Management of the majority of childhood squints should be done in primary care

A child with a non-paralytic squint would have a degree of double vision

Next question

Amblyopia can develop in both paralytic and non-paralytic squints.

Squint
Squint (strabismus) is characterised by misalignment of the visual axes. Squints may be divided into concomitant (common) and paralytic
(rare)
Concomitant

Paralytic

Due to imbalance in extraocular muscles


Due to paralysis of extraocular muscles
Convergent is more common than divergent

Detection of a squint may be made by the corneal light reflection test - holding a light source 30cm from the child's face to see if the light
reflects symmetrically on the pupils
The cover test is used to identify the nature of the squint

ask the child to focus on a object


cover one eye
observe movement of uncovered eye
cover other eye and repeat test

Management

eye patches may help prevent amblyopia


referral to secondary care is appropriate

Question 73 of 180

Which one of the following is not associated with the development of obesity in children?
Male sex

Being tall for age

Asian ethnicity

Down's syndrome

Growth hormone deficiency

Next question

Obesity in children
Defining obesity is more difficult in children than adults as body mass index (BMI) varies with age. BMI percentile charts are therefore
needed to make an accurate assessment. Recent NICE guidelines suggest to use 'UK 1990 BMI charts to give age- and gender-specific
information'
NICE recommend

consider tailored clinical intervention if BMI at 91st centile or above.


consider assessing for comorbidities if BMI at 98th centile or above

By far the most common cause of obesity in childhood is lifestyle factors. Other associations of obesity in children include:

Asian children: four times more likely to be obese than white children
female children
taller children: children with obesity are often above the 50th percentile in height

Cause of obesity in children

growth hormone deficiency


hypothyroidism
Down's syndrome
Cushing's syndrome
Prader-Willi syndrome

Consequences of obesity in children

orthopaedic problems: slipped upper femoral epiphyses, Blount's disease (a development abnormality of the tibia resulting in
bowing of the legs), musculoskeletal pains
psychological consequences: poor self-esteem, bullying
sleep apnoea
benign intracranial hypertension
long-term consequences: increased incidence of type 2 diabetes mellitus, hypertension and ischaemic heart disease

Question 74 of 180

What is the most common cause of hypertension in children?


Renal vascular disease

Congenital adrenal hyperplasia

Renal parenchymal disease

Coarctation of the aorta

Phaeochromocytoma

Next question

Hypertension in children
Measuring blood pressure in children

correct cuff size is approximately 2/3 the length of the upper arm
the 4th Korotkoff sound is used to measure the diastolic blood pressure until adolescence, when the 5th Korotkoff sound can be
used
results should be compared with a graph of normal values for age

In younger children secondary hypertension is the most common cause, with renal parenchymal disease accounting for up to 80%
Causes of hypertension in children

renal parenchymal disease


renal vascular disease
coarctation of the aorta
phaeochromocytoma
congenital adrenal hyperplasia
essential or primary hypertension (becomes more common as children become older)
Question 75 of 180

You are asked to teach a 14-year-old boy how to use his metered dose salbutamol inhaler. After removing the cap and shaking,
what is the next step?

Take a deep breath in and a forced breath out

Breathe out gently

Breathe in and press down on canister after 1 second

Breathe in and simultaneously press down on canister

Press down on canister and hold breathe in for 10 seconds

Next question

Inhaler technique

The following inhaler technique guideline is for metered dose inhalers (source: Asthma.org.uk, a resource recommended to
patients by the British Thoracic Society)
1. Remove cap and shake

2. Breathe out gently


3. Put mouthpiece in mouth and as you begin to breathe in, which should be slow and deep, press canister down and continue to
inhale steadily and deeply
4. Hold breath for 10 seconds, or as long as is comfortable
5. For a second dose wait for approximately 30 seconds before repeating steps 1-4.
Only use the device for the number of doses on the label, then start a new inhaler.
Question 76 of 180

Which one of the following is a cause of cyanotic congenital heart disease?


Atrial septal defect

Ventricular septal defect

Transposition of the great arteries

Coarctation of the aorta

Patent ductus arteriosus

Next question

Congenital heart disease: types


Acyanotic - most common causes

ventricular septal defects (VSD) - most common, accounts for 30%


atrial septal defect (ASD)
patent ductus arteriosus (PDA)
coarctation of the aorta
aortic valve stenosis

VSDs are more common than ASDs. However, in adult patients ASDs are the more common new diagnosis as they generally presents
later
Cyanotic - most common causes

tetralogy of Fallot
transposition of the great arteries (TGA)
tricuspid atresia
pulmonary valve stenosis

Fallot's is more common than TGA. However, at birth TGA is the more common lesion as patients with Fallot's generally presenting at
around 1-2 months

Question 77 of 180

What is the investigation of choice to look for renal scarring in a child with vesicoureteric reflux?
Abdominal x-ray
Ultrasound
DMSA
CT
Micturating cystourethrogram
Next question

Vesicoureteric reflux

Vesicoureteric reflux (VUR) is the abnormal backflow of urine from the bladder into the ureter
and kidney. It is relatively common abnormality of the urinary tract in children and predisposes
to urinary tract infection (UTI), being found in around 30% of children who present with a UTI.
As around 35% of children develop renal scarring it is important to investigate for VUR in
children following a UTI
Pathophysiology of VUR

ureters are displaced laterally, entering the bladder in a more perpendicular fashion
than at an angle
therefore shortened intramural course of ureter
vesicoureteric junction cannot therefore function adequately

The table below summarises the grading of VUR


Grade
I

Reflux into the ureter only, no dilatation

Grade
II

Reflux into the renal pelvis on micturition, no dilatation

III

Mild/moderate dilatation of the ureter, renal pelvis and calyces

IV

Dilation of the renal pelvis and calyces with moderate ureteral tortuosity

Gross dilatation of the ureter, pelvis and calyces with ureteral tortuosity

Investigation

VUR is normally diagnosed following a micturating cystourethrogram


a DMSA scan may also be performed to look for renal scarring

Question 78 of 180

Which one of the following congenital infections is most characteristically associated with chorioretinitis?
Cytomegalovirus

Treponema pallidum

Rubella

Toxoplasma gondii

Parvovirus B19

Next question

Congenital toxoplasmosis

cerebral calcification
chorioretinitis

A form of 'salt and pepper' chorioretinitis is also seen in congenital rubella but this is not a common feature.
Chorioretinitis is found in around 75% of patients with congenital toxoplasmosis.

Congenital infections
The major congenital infections encountered in examinations are rubella, toxoplasmosis and cytomegalovirus
Cytomegalovirus is the most common congenital infection in the UK. Maternal infection is usually asymptomatic

Rubella

Toxoplasmosis

Cytomegalovirus

Characteristic features

Sensorineural deafness
Cerebral calcification
Congenital cataracts
Growth retardation
Chorioretinitis
Congenital heart disease (e.g. patent ductus arteriosus)
Purpuric skin lesions
Hydrocephalus
Glaucoma

Other features

Growth retardation
Hepatosplenomegaly
Purpuric skin lesions
'Salt and pepper' chorioretinitis
Microphthalmia
Cerebral palsy

Sensorineural deafness
Encephalitis/seizures
Anaemia
Pneumonitis
HepatosplenomegalyHepatosplenomegaly
Cerebral palsy
Anaemia
Jaundice
Cerebral palsy

Question 79 of 180

Which one of the following is not a risk factor for sudden infant death syndrome?
Bottle feeding

Male sex

Summer months

Maternal drug use

Being a twin

Next question

The incidence of sudden infant death syndrome increases in the winter months

Sudden infant death syndrome


Sudden infant death syndrome is the commonest cause of death in the first year of life. It is most common at 3 months of age
Risk factors

prematurity
parental smoking
hyperthermia (e.g. over-wrapping)
putting the baby to sleep prone
male sex
multiple births
bottle feeding
social classes IV and V
maternal drug use
incidence increases in winter

Following a cot death siblings should be screened for potential sepsis and inborn errors of metabolism

Question 80 of 180

Which one of the following statements regarding benign rolandic epilepsy is incorrect?
Seizures are usually partial in nature

EEG often shows centro-temporal spikes

Seizures characteristically occur at night

Carries a good prognosis

Typically occurs between the age of 1 and 3 years

Next question

Benign rolandic epilepsy


Benign rolandic epilepsy is a form of childhood epilepsy which typically occurs between the age of 4 and 12 years.
Features

seizures characteristically occur at night


seizures are typically partial (e.g. paraesthesia affecting face) but secondary generalisation may occur (i.e. parents may only
report tonic-clonic movements)
child is otherwise normal

EEG characteristically shows centro-temporal spikes


Prognosis is excellent, with seizures stopping by adolescence

Question 81 of 180

You review a 7-year-old girl who has cerebral palsy. She is having ongoing problems with spasticity in her legs which is causing pain and
contractures. On speaking to her mother you ascertain that she is having regular physiotherapy, using the appropriate orthoses and has
tried oral diazepam in the past. Which one of the following treatments may she be offered to try and improve her symptoms?
Baclofen

Clozapine

Clonidine

Dantrolene

Glyceryl trinitrate

Next question

Cerebral palsy
Cerebral palsy may be defined as a disorder of movement and posture due to a non-progressive lesion of the motor pathways in the
developing brain. It affects 2 in 1,000 live births and is the most common cause of major motor impairment
Possible manifestations include:

abnormal tone early infancy


delayed motor milestones
abnormal gait
feeding difficulties

Children with cerebral palsy often have associated non-motor problems such as:

learning difficulties (60%)


epilepsy (30%)
squints (30%)
hearing impairment (20%)

Causes

antenatal (80%): e.g. cerebral malformation and congenital infection (rubella, toxoplasmosis, CMV)
intrapartum (10%): birth asphyxia/trauma
postnatal (10%): intraventricular haemorrhage, meningitis, head-trauma

Classification

spastic (70%): hemiplegia, diplegia or quadriplegia


dyskinetic
ataxic
mixed

Management

as with any child with a chronic condition a multidisciplinary approach is needed


treatments for spasticity include oral diazepam, oral and intrathecal baclofen, botulinum toxin type A, orthopaedic surgery and
selective dorsal rhizotomy
anticonvulsants, analgesia as required

Question 82 of 180

Each of the following organisms commonly cause respiratory tract infections in patients with cystic fibrosis, except:

Aspergillus

Pseudomonas aeruginosa

Burkholderia cepacia

Staphylococcal aureus

Strongyloides stercoralis

Next question

Cystic fibrosis
Cystic fibrosis (CF) is an autosomal recessive disorder causing increased viscosity of secretions (e.g. lungs and pancreas). It is due to a
defect in the cystic fibrosis transmembrane conductance regulator gene (CFTR), which codes a cAMP-regulated chloride channel
In the UK 80% of CF cases are due to delta F508 on the long arm of chromosome 7. Cystic fibrosis affects 1 per 2500 births, and the
carrier rate is c. 1 in 25
Organisms which may colonise CF patients

Staphylococcus aureus
Pseudomonas aeruginosa
Burkholderia cepacia*
Aspergillus

*previously known as Pseudomonas cepacia

Question 83 of 180

A 10-year-old boy who is known to have asthma is reviewed. He currently uses salbutamol prn, beclometasone 100mcg bd and recently
had a trial of salmeterol 50mcg bd. Unfortunately he has noticed no benefit from adding salmeterol. What is the most appropriate next step
in management?
Stop salmeterol + trial of a leukotriene receptor antagonist

Increase salmeterol to 100mcg bd

Stop salmeterol + increase beclometasone to 200mcg bd

Continue salmeterol + increase beclometasone to 200mcg bd

Refer to a paediatrician

Next question

As the trial of salmeterol has failed the next step would be to increase the inhaled steroid to 400 mcg / day.

Asthma in children: stepwise management


The British Thoracic Society differentiate between children younger and older than 5 years in their 2014 guidelines:
Children aged under 5 years
StepTherapy
1

As-required reliever therapy: short-acting beta2-agonist

Regular preventer therapy: inhaled corticosteroids, 200-400mcg/day*


Or, if inhaled corticosteroids cannot be used, a leukotriene receptor antagonist

Children aged 2-5 years: trial of a leukotriene receptor antagonist. If already taking leukotriene receptor antagonist reconsider inhaled
corticosteroids
Children aged under 2 years: refer to respiratory paediatrician

Refer to a respiratory paediatrician

Children aged over 5 years (similar to adult guidance)


StepTherapy
1

As-required reliever therapy: short-acting beta2-agonist

Regular preventer therapy: inhaled corticosteroids, 200-400mcg/day*


1. Add inhaled long-acting B2 agonist (LABA)
2. Assess control of asthma:

good response to LABA - continue LABA


benefit from LABA but control still inadequate: continue LABA and increase inhaled steroid dose to 400 mcg/day* (if not
already on this dose)
no response to LABA: stop LABA and increase inhaled steroid to 400 mcg/ day.* If control still inadequate, institute trial of
other therapies, leukotriene receptor antagonist or SR theophylline

Increase inhaled corticosteroids to high-dose, up to 800mcg/day*


Use daily steroid tablet at lowest dose providing control

Maintain inhaled corticosteroids at 800mcg/day


Refer to a paediatrician

*beclometasone dipropionate or equivalent

Question 84 of 180

A 6-year-old boy is diagnosed as having nephrotic syndrome. A presumptive diagnosis of minimal change glomerulonephritis is made.
What is the most appropriate treatment?
Cyclophosphamide

Albumin infusion

Plasma exchange

Renal biopsy followed by prednisolone

Prednisolone

Next question

A renal biopsy is only indicated if response to steroids is poor

Minimal change disease


Minimal change disease nearly always presents as nephrotic syndrome, accounting for 75% of cases in children and 25% in adults.
The majority of cases are idiopathic, but in around 10-20% a cause is found:

drugs: NSAIDs, rifampicin


Hodgkin's lymphoma, thymoma
infectious mononucleosis

Pathophysiology

T-cell and cytokine mediated damage to the glomerular basement membrane polyanion loss
the resultant reduction of electrostatic charge increased glomerular permeability to serum albumin

Features

nephrotic syndrome
normotension - hypertension is rare
highly selective proteinuria*
renal biopsy: electron microscopy shows fusion of podocytes

Management

majority of cases (80%) are steroid responsive


cyclophosphamide is the next step for steroid resistant cases

Prognosis is overall good, although relapse is common. Roughly:

1/3 have just one episode


1/3 have infrequent relapses
1/3 have frequent relapses which stop before adulthood

*only intermediate-sized proteins such as albumin and transferrin leak through the glomerulus

Question 85 of 180

A baby is born by elective Caesarean section at 38 weeks performed due to pregnancy-induced hypertension. At one hour the female baby
is noted to be grunting with mild intercostal recession. Oxygen saturations are 95-96% on air. What is the most likely cause of her
respiratory distress?
Surfactant deficient lung disease

Congenital pneumonia

Transient tachypnoea of the newborn

Persistent pulmonary hypertension of the newborn

Pulmonary hypoplasia

Next question

Transient tachypnoea of the newborn


Transient tachypnoea of the newborn (TTN) is the commonest cause of respiratory distress in the newborn period. It is caused by delayed
resorption of fluid in the lungs
It is more common following Caesarean sections, possibly due to the lung fluid not being 'squeezed out' during the passage through the
birth canal
Chest x-ray may show hyperinflation of the lungs and fluid in the horizontal fissure
Supplementary oxygen may be required to maintain oxygen saturations. Transient tachypnoea of the newborn usually settles within 1-2
days
Question 86 of 180

Which one of the following is a contraindication to receiving the pneumococcal vaccine?


Splenectomy in past

Allergy to egg protein (no features of anaphylaxis)

Pregnancy

Current febrile illness

Eczema

Next question

Immunisation
The Department of Health published guidance in 2006 on the safe administration of vaccines in its publication 'Immunisation against
infectious disease'
General contraindications to immunisation

confirmed anaphylactic reaction to a previous dose of a vaccine containing the same antigens
confirmed anaphylactic reaction to another component contained in the relevant vaccine (e.g. egg protein)

Situations where vaccines should be delayed

febrile illness/intercurrent infection

Contraindications to live vaccines

pregnancy
immunosuppression

Specific vaccines

DTP: vaccination should be deferred in children with an evolving or unstable neurological condition

Not contraindications to immunisation

asthma or eczema
history of seizures (if associated with fever then advice should be given regarding antipyretics)
breastfed child
previous history of natural pertussis, measles, mumps or rubella infection
history of neonatal jaundice
family history of autism
neurological conditions such as Down's or cerebral palsy
low birth weight or prematurity
patients on replacement steroids e.g. (CAH)

Question 87 of 180

Which one of the following statements concerning accidents in children is incorrect?


Around 15-20% of children attend Emergency Departments in the course of a year due to an accident

Accidents account for two-thirds of all childhood deaths

Cycling helmets are an example of a secondary prevention strategy

Children from lower social classes are more likely to have an accident

Road traffic accidents are the most common cause of fatal accidents

Next question

Accidents account for approximately one-third of all childhood deaths

Accidents in children
Around 15-20% of children attend Emergency Departments in the course of a year due to an accident. Accidents account for a third of all
childhood deaths and are the single most common cause of death in children aged between 1 - 15 years of age.
Key points

road traffic accidents are the most common cause of fatal accidents
boys and children from lower social classes are more likely to have an accident

Accident prevention
Accident prevention can be divided up into primary (preventing the accident from happening), secondary (prevent injury from the accident)
and tertiary (limit the impact of the injury) prevention strategies
The table below gives examples of accident prevention strategies
Primary prevention

Secondary preventionTertiary prevention

Stair guards
Wearing seat belts
Speed limits*
Cycling helmets
Teaching parents first aid
Teaching road safety Smoke alarms
Window safety catchesLaminated safety glass

*some strategies such as reducing driving speed may have a role in both primary and secondary accident prevention

Question 88 of 180

Which one of the following is a cause of acyanotic congenital heart disease?


Tetralogy of Fallot

Pulmonary valve stenosis

Transposition of the great arteries

Coarctation of the aorta

Tricuspid atresia

Next question

Congenital heart disease: types


Acyanotic - most common causes

ventricular septal defects (VSD) - most common, accounts for 30%


atrial septal defect (ASD)
patent ductus arteriosus (PDA)
coarctation of the aorta
aortic valve stenosis

VSDs are more common than ASDs. However, in adult patients ASDs are the more common new diagnosis as they generally presents
later
Cyanotic - most common causes

tetralogy of Fallot
transposition of the great arteries (TGA)
tricuspid atresia
pulmonary valve stenosis

Fallot's is more common than TGA. However, at birth TGA is the more common lesion as patients with Fallot's generally presenting at
around 1-2 months
Question 89 of 180

Which one of the following vaccines uses an inactivated preparation of the organism or virus?
Influenza (intramuscular)

Yellow fever

Oral polio

Measles

Diphtheria

Next question

Vaccinations
It is important to be aware of vaccines which are of the live-attenuated type as these may pose a risk to immunocompromised patients. The
main types of vaccine are as follows:
Live attenuated

BCG

measles, mumps, rubella (MMR)


influenza (intranasal)
oral rotavirus
oral polio
yellow fever
oral typhoid*

Inactivated preparations

rabies
influenza (intramuscular)

Detoxified exotoxins

tetanus

Extracts of the organism/virus (sometimes termed fragment)**

diphtheria
pertussis ('acellular' vaccine)
hepatitis B
meningococcus, pneumococcus, haemophilus

Notes

influenza: different types are available, including whole inactivated virus, split virion (virus particles disrupted by detergent
treatment) and sub-unit (mainly haemagglutinin and neuraminidase)
cholera: contains inactivated Inaba and Ogawa strains of Vibrio cholerae together with recombinant B-subunit of the cholera
toxin
hepatitis B: contains HBsAg adsorbed onto aluminium hydroxide adjuvant and is prepared from yeast cells using recombinant
DNA technology

*whole cell typhoid vaccine is no longer used in the UK


**may also be produced using recombinant DNA technology

Question 90 of 180

What is the most appropriate way to confirm a diagnosis of pertussis?


Blood cultures

Sputum culture

Per nasal swab

Urine for serology

Throat swab

Next question

Whooping cough (pertussis)


Overview

caused by the Gram negative bacterium Bordetella pertussis


incubation period = 10-14 days
infants are routinely immunised at 2, 3, 4 months and 3-5 years. Newborn infants are particularly vulnerable, which is why the
vaccincation campaign for pregnant women was introduced
neither infection nor immunisation results in lifelong protection - hence adolescents and adults may develop whooping cough
despite having had their routine immunisations
around 1,000 cases are reported each year in the UK

Features, 2-3 days of coryza precede onset of:

coughing bouts: usually worse at night and after feeding, may be ended by vomiting & associated central cyanosis
inspiratory whoop: not always present (caused by forced inspiration against a closed glottis)
persistent coughing may cause subconjunctival haemorrhages or even anoxia leading to syncope & seizures
symptoms may last 10-14 weeks* and tend to be more severe in infants
marked lymphocytosis

Diagnosis

per nasal swab culture for Bordetella pertussis - may take several days or weeks to come back
PCR and serology are now increasingly used as their availability becomes more widespread

Management

oral erythromycin to eradicate the organism and reduce spread


has not been shown to alter the course of the illness

Complications

subconjunctival haemorrhage
pneumonia
bronchiectasis
seizures

Vaccination of pregnant women


In 2012 there was an outbreak of whooping cough (pertussis) which resulted in the death of 14 newborn children. As a temporary measure
a vaccination programme was introduced in 2012 for pregnant women. This has successfully reduced the number of cases of whooping
cough (the vaccine is thought to be more than 90% effective in preventing newborns developing whooping cough). It was however decided
in 2014 to extend the whooping cough vaccination programme for pregnant women. This decision was taken as there was a 'great deal of
uncertainty' about the timing of future outbreaks.

Women who are between 28-38 weeks pregnant will be offered the vaccine.
*weeks, not days

Question 91 of 180

A 6-month-old girl is brought to the surgery by her mother. She has a 24 hour history of poor feeding, taking around two-thirds her normal
intake. On examination she is alert and there are no focal signs of infection. She smiles initially then has a strong cry when you try to
examine her chest. She stops crying after being soothed by her mother.
The respiratory rate is 36 / min, capillary refill time is < 2 secs and the temperature is 39.0C. What is the most appropriate management?
Advise to give antipyretics if the child is distressed + discuss red flags + booked appointment for next day

Admit to hospital

Advise to alternate paracetamol and ibuprofen + discuss red flags + see if not improving

Amoxicillin for 7 days

IM benzylpenicillin

Next question

The temperature of 39.0C in a 6-month-old child is regarded as a 'amber' feature in the latest 2013 NICE guidelines. All the other features
however are green or low risk. It is therefore reasonable to manage this patient at home with the usual safety-netting advice.
NICE recommend that parents should only give antipyretics if the child is distressed.

Feverish illness in children


The 2007 NICE Feverish illness in children guidelines introduced a 'traffic light' system for risk stratification of children under the age of 5
years presenting with a fever. These guidelines were later modified in a 2013 update.
It should be noted that these guidelines only apply 'until a clinical diagnosis of the underlying condition has been made'. A link to the
guidelines is provided but some key points are listed below.
Assessment
The following should be recorded in all febrile children:

temperature
heart rate
respiratory rate
capillary refill time

Signs of dehydration (reduced skin turgor, cool extremities etc) should also be looked for
Measuring temperature should be done with an electronic thermometer in the axilla if the child is < 4 weeks or with an electronic/chemical
dot thermometer in the axilla or an infra-red tympanic thermometer.
Risk stratification
Please see the link for the complete table, below is a modified version

Green - low risk

Amber - intermediate risk

Red - high risk

Colour

Normal colour

Pallor reported by parent/carer

Pale/mottled/ashen/blue

Activity

Responds normally to
social cues
Content/smiles
Stays awake or awakens
quickly
Strong normal cry/not
crying

Not responding normally to social cues


No smile
Wakes only with prolonged stimulation
Decreased activity

No response to social cues


Appears ill to a healthcare professional
Does not wake or if roused does not
stay awake
Weak, high-pitched or continuous cry

Nasal flaring
Tachypnoea: respiratory rate

Respiratory

>50 breaths/minute, age 6-12


months;
>40 breaths/minute, age >12
months

Grunting
Tachypnoea: respiratory rate >60
breaths/minute
Moderate or severe chest indrawing

Oxygen saturation <=95% in air


Crackles in the chest
Tachycardia:

Circulation and
hydration

Normal skin and eyes


Moist mucous membranes

>160 beats/minute, age <12


months
>150 beats/minute, age 12-24
months

Reduced skin turgor

>140 beats/minute, age 2-5 years

Capillary refill time >=3 seconds


Dry mucous membranes
Poor feeding in infants
Reduced urine output

Other

No amber or red signs

Age 3-6 months, temperature >=39C


Fever for >=5 days
Rigors
Swelling of a limb or joint
Non-weight bearing limb/not using an
extremity

Age <3 months, temperature >=38C


Non-blanching rash
Bulging fontanelle
Neck stiffness
Status epilepticus
Focal neurological signs
Focal seizures

Management
If green:

Child can be managed at home with appropriate care advice, including when to seek further help

If amber:

provide parents with a safety net or refer to a paediatric specialist for further assessment
a safety net includes verbal or written information on warning symptoms and how further healthcare can be accessed, a followup appointment, liaison with other healthcare professionals, e.g. out-of-hours providers, for further follow-up

If red:

refer child urgently to a paediatric specialist

Other key points include

oral antibiotics should not be prescribed to children with fever without apparent source
if a pneumonia is suspected but the child is not going to be referred to hospital then a chest x-ray does not need to be routinely
performed

Recommendations
1.1 Thermometers and the detection of fever
1.2 Clinical assessment of children with fever
1.3 Management by remote assessment
1.4 Management by the non-paediatric practitioner
1.5 Management by the paediatric specialist
1.6 Antipyretic interventions
1.7 Advice for home care
1 Recommendations
The following guidance is based on the best available evidence. The full guideline gives details of
the methods and the evidence used to develop the guidance.
The wording used in the recommendations in this guideline denotes the certainty with which
the recommendation is made (the strength of the recommendation). See About this guideline
for details.
This guideline is intended for use by healthcare professionals for the assessment and initial
management in young children with feverish illness. The guideline should be followed until a
clinical diagnosis of the underlying condition has been made. Once a diagnosis has been made,
the child should be treated according to national or local guidance for that condition.
Parents or carers of a child with fever may approach a range of different healthcare
professionals as their first point of contact, for example, a GP, a pharmacist or an emergency
care practitioner. The training and experience of the healthcare professionals involved in the
child's care will vary and each should interpret the guidance according to the scope of their own
practice.
For the purposes of this guideline, fever was defined as 'an elevation of body temperature
above the normal daily variation'.
This guideline should be read in conjunction with:

Bacterial meningitis and meningococcal septicaemia (NICE clinical guideline 102).


Urinary tract infection in children (NICE clinical guideline 54).
Diarrhoea and vomiting in children under 5 (NICE clinical guideline 84).
1.1 Thermometers and the detection of fever
1.1.1 Oral and rectal temperature measurements
1.1.1.1 Do not routinely use the oral and rectal routes to measure the body temperature of
children aged 05 years. [2007]
1.1.2 Measurement of body temperature at other sites
1.1.2.1 In infants under the age of 4 weeks, measure body temperature with an electronic
thermometer in the axilla. [2007]
1.1.2.2 In children aged 4 weeks to 5 years, measure body temperature by one of the following
methods:
electronic thermometer in the axilla
chemical dot thermometer in the axilla
infra-red tympanic thermometer. [2007]
1.1.2.3 Healthcare professionals who routinely use disposable chemical dot thermometers
should consider using an alternative type of thermometer when multiple temperature
measurements are required. [2007]
1.1.2.4 Forehead chemical thermometers are unreliable and should not be used by healthcare
professionals. [2007]
1.1.3 Subjective detection of fever by parents and carers
1.1.3.1 Reported parental perception of a fever should be considered valid and taken seriously
by healthcare professionals. [2007]
1.2 Clinical assessment of children with fever
1.2.1 Life-threatening features of illness in children
1.2.1.1 First, healthcare professionals should identify any immediately life-threatening features,
including compromise of the airway, breathing or circulation, and decreased level of
consciousness. [2007]
1.2.2 Assessment of risk of serious illness

1.2.2.1 Assess children with feverish illness for the presence or absence of symptoms and signs
that can be used to predict the risk of serious illness using the traffic light system (see table 1).
[2013]
1.2.2.2 When assessing children with learning disabilities, take the individual child's learning
disability into account when interpreting the traffic light table. [new 2013]
1.2.2.3 Recognise that children with any of the following symptoms or signs are in a high-risk
group for serious illness:
pale/mottled/ashen/blue skin, lips or tongue
no response to social cues[3]
appearing ill to a healthcare professional
does not wake or if roused does not stay awake
weak, high-pitched or continuous cry
grunting
respiratory rate greater than 60 breaths per minute
moderate or severe chest indrawing
reduced skin turgor
bulging fontanelle. [new 2013]
1.2.2.4 Recognise that children with any of the following symptoms or signs are in at least an
intermediate-risk group for serious illness:
pallor of skin, lips or tongue reported by parent or carer
not responding normally to social cues[3]
no smile
wakes only with prolonged stimulation
decreased activity
nasal flaring
dry mucous membranes
poor feeding in infants

reduced urine output


rigors. [new 2013]
1.2.2.5 Recognise that children who have all of the following features, and none of the high- or
intermediate-risk features, are in a low-risk group for serious illness:
normal colour of skin, lips and tongue
responds normally to social cues[3]
content/smiles
stays awake or awakens quickly
strong normal cry or not crying
normal skin and eyes
moist mucous membranes. [new 2013]
1.2.2.6 Measure and record temperature, heart rate, respiratory rate and capillary refill time as
part of the routine assessment of a child with fever. [2007]
1.2.2.7 Recognise that a capillary refill time of 3 seconds or longer is an intermediate-risk group
marker for serious illness ('amber' sign). [2013]
1.2.2.8 Measure the blood pressure of children with fever if the heart rate or capillary refill time
is abnormal and the facilities to measure blood pressure are available. [2007]
1.2.2.9 In children older than 6 months do not use height of body temperature alone to identify
those with serious illness. [2013]
1.2.2.10 Recognise that children younger than 3 months with a temperature of 38C or higher
are in a high-risk group for serious illness. [2013]
1.2.2.11 Recognise that children aged 36 months with a temperature of 39C or higher are in at
least an intermediate-risk group for serious illness. [new 2013]
1.2.2.12 Do not use duration of fever to predict the likelihood of serious illness. However,
children with a fever lasting more than 5 days should be assessed for Kawasaki disease (see
recommendation 1.2.3.10). [new 2013]
1.2.2.13 Recognise that children with tachycardia are in at least an intermediate-risk group for
serious illness. Use the Advanced Paediatric Life Support (APLS)[4] criteria below to define
tachycardia: [new 2013]

Age

Heart rate (bpm)

<12 months

>160

1224 months >150


25 years

>140

1.2.2.14 Assess children with fever for signs of dehydration. Look for:
prolonged capillary refill time
abnormal skin turgor
abnormal respiratory pattern
weak pulse
cool extremities. [2007]
1.2.3 Symptoms and signs of specific illnesses
1.2.3.1 Look for a source of fever and check for the presence of symptoms and signs that are
associated with specific diseases (see table 2). [2007]
1.2.3.2 Consider meningococcal disease in any child with fever and a non-blanching rash,
particularly if any of the following features are present[5]:
an ill-looking child
lesions larger than 2 mm in diameter (purpura)
a capillary refill time of 3 seconds or longer
neck stiffness. [2007]
1.2.3.3 Consider bacterial meningitis in a child with fever and any of the following features[5]:
neck stiffness
bulging fontanelle
decreased level of consciousness
convulsive status epilepticus. [2007, amended 2013]
1.2.3.4 Be aware that classic signs of meningitis (neck stiffness, bulging fontanelle, high-pitched
cry) are often absent in infants with bacterial meningitis[5]. [2007]

1.2.3.5 Consider herpes simplex encephalitis in children with fever and any of the following
features:
focal neurological signs
focal seizures
decreased level of consciousness. [2007]
1.2.3.6 Consider pneumonia in children with fever and any of the following signs:
tachypnoea (respiratory rate greater than 60 breaths per minute, age 05 months; greater than
50 breaths per minute, age 612 months; greater than 40 breaths per minute, age older than
12 months)
crackles in the chest
nasal flaring
chest indrawing
cyanosis
oxygen saturation of 95% or less when breathing air. [2007]
1.2.3.7 Consider urinary tract infection in any child younger than 3 months with fever[6]. [2007]
1.2.3.8 Consider urinary tract infection in a child aged 3 months or older with fever and 1 or
more of the following[6]:
vomiting
poor feeding
lethargy
irritability
abdominal pain or tenderness
urinary frequency or dysuria. [new 2013]
1.2.3.9 Consider septic arthritis/osteomyelitis in children with fever and any of the following
signs:
swelling of a limb or joint
not using an extremity

non-weight bearing. [2007]


1.2.3.10 Consider Kawasaki disease in children with fever that has lasted longer than 5 days and
who have 4 of the following 5 features:
bilateral conjunctival injection
change in mucous membranes in the upper respiratory tract (for example, injected pharynx, dry
cracked lips or strawberry tongue)
change in the extremities (for example, oedema, erythema or desquamation)
polymorphous rash
cervical lymphadenopathy
Be aware that, in rare cases, incomplete/atypical Kawasaki disease may be diagnosed with fewer
features. [2007]
1.2.4 Imported infections
1.2.4.1 When assessing a child with feverish illness, enquire about recent travel abroad and
consider the possibility of imported infections according to the region visited. [2007]
Table 1 Traffic light system for identifying risk of serious illness
[new 2013]
Children with fever and any of the symptoms or signs in the red column should be recognised as
being at high risk. Similarly, children with fever and any of the symptoms or signs in the amber
column and none in the red column should be recognised as being at intermediate risk. Children
with symptoms and signs in the green column and none in the amber or red columns are at low
risk. The management of children with fever should be directed by the level of risk.
This traffic light table should be used in conjunction with the recommendations in this guideline
on investigations and initial management in children with fever.
A colour version of this table is available.

Colour
(of skin, lips or
tongue)

Green low risk

Amber intermediate risk Red high risk

Normal colour

Pallor reported by
parent/carer

Pale/mottled/ashen/blue

Activity

Responds normally to Not responding normally


social cues
to social cues
Content/smiles

No smile

Appears ill to a healthcare


professional

Stays awake or
awakens quickly

Wakes only with


prolonged stimulation

Does not wake or if roused


does not stay awake

Strong normal cry/not Decreased activity


crying
Respiratory

No response to social cues

Weak, high-pitched or
continuous cry

Nasal flaring

Grunting

Tachypnoea: respiratory
rate

Tachypnoea: respiratory rate


>60 breaths/minute

>50 breaths/minute, age


612 months;

Moderate or severe chest


indrawing

>40 breaths/minute, age


>12 months
Oxygen saturation 95% in
air
Crackles in the chest
Circulation
Normal skin and eyes Tachycardia:
and hydration
Moist mucous
>160 beats/minute, age
membranes
<12 months
>150 beats/minute,
age 1224 months
>140 beats/minute, age 2
5 years
Capillary refill time
3 seconds
Dry mucous membranes
Poor feeding in infants
Reduced urine output

Reduced skin turgor

Other

None of the amber or Age 36 months,


red symptoms or
temperature 39C
signs
Fever for 5 days
Rigors

Age <3 months, temperature


38C
Non-blanching rash
Bulging fontanelle

Swelling of a limb or joint Neck stiffness


Non-weight bearing
limb/not using an
extremity

Status epilepticus
Focal neurological signs
Focal seizures

Table 2 Summary table for symptoms and signs suggestive of specific diseases
[2013]
Diagnosis to be
considered

Symptoms and signs in conjunction with fever

Meningococcal
disease

Non-blanching rash, particularly with 1 or more of the following:


an ill-looking child
lesions larger than 2 mm in diameter (purpura)
capillary refill time of 3 seconds
neck stiffness

Bacterial meningitis Neck stiffness


Bulging fontanelle
Decreased level of consciousness
Convulsive status epilepticus
Herpes simplex
encephalitis

Focal neurological signs


Focal seizures
Decreased level of consciousness

Pneumonia

Tachypnoea (respiratory rate >60 breaths/minute, age 05 months; >50


breaths/minute, age 612 months; >40 breaths/minute, age >12 months)

Crackles in the chest


Nasal flaring
Chest indrawing
Cyanosis
Oxygen saturation 95%
Urinary tract
infection

Vomiting
Poor feeding
Lethargy
Irritability
Abdominal pain or tenderness
Urinary frequency or dysuria

Septic arthritis

Swelling of a limb or joint


Not using an extremity
Non-weight bearing

Kawasaki disease

Fever for more than 5 days and at least 4 of the following:


bilateral conjunctival injection
change in mucous membranes
change in the extremities
polymorphous rash
cervical lymphadenopathy

1.3 Management by remote assessment


Remote assessment refers to situations in which a child is assessed by a healthcare professional
who is unable to examine the child because the child is geographically remote from the assessor
(for example, telephone calls to NHS Direct[7]). Therefore, assessment is largely an
interpretation of symptoms rather than physical signs. The guidance in this section may also
apply to healthcare professionals whose scope of practice does not include the physical
examination of a young child (for example, community pharmacists).

1.3.1 Management according to risk of serious illness


1.3.1.1 Healthcare professionals performing a remote assessment of a child with fever should
seek to identify symptoms and signs of serious illness and specific diseases as described in
section 1.2 and summarised in tables 1 and 2. [2007]
1.3.1.2 Children whose symptoms or combination of symptoms suggest an immediately lifethreatening illness (see recommendation 1.2.1.1) should be referred immediately for emergency
medical care by the most appropriate means of transport (usually 999 ambulance). [2007]
1.3.1.3 Children with any 'red' features but who are not considered to have an immediately lifethreatening illness should be urgently assessed by a healthcare professional in a face-to-face
setting within 2 hours. [2007]
1.3.1.4 Children with 'amber' but no 'red' features should be assessed by a healthcare
professional in a face-to-face setting. The urgency of this assessment should be determined by
the clinical judgement of the healthcare professional carrying out the remote assessment.
[2007]
1.3.1.5 Children with 'green' features and none of the 'amber' or 'red' features can be cared for
at home with appropriate advice for parents and carers, including advice on when to seek
further attention from the healthcare services (see section 1.7). [2007, amended 2013]
1.4 Management by the non-paediatric practitioner
In this guideline, a non-paediatric practitioner is defined as a healthcare professional who has
not had specific training or who does not have expertise in the assessment and treatment of
children and their illnesses. This term includes healthcare professionals working in primary care,
but it may also apply to many healthcare professionals in general emergency departments.
1.4.1 Clinical assessment
1.4.1.1 Management by a non-paediatric practitioner should start with a clinical assessment as
described in section 1.2. Healthcare practitioners should attempt to identify symptoms and signs
of serious illness and specific diseases as summarised in tables 1 and 2. [2007]
1.4.2 Management according to risk of serious illness
1.4.2.1 Children whose symptoms or combination of symptoms and signs suggest an
immediately life-threatening illness (see recommendation 1.2.1.1) should be referred
immediately for emergency medical care by the most appropriate means of transport (usually
999 ambulance). [2007]
1.4.2.2 Children with any 'red' features but who are not considered to have an immediately lifethreatening illness should be referred urgently to the care of a paediatric specialist. [2007]

1.4.2.3 If any 'amber' features are present and no diagnosis has been reached, provide parents
or carers with a 'safety net' or refer to specialist paediatric care for further assessment. The
safety net should be 1 or more of the following:
providing the parent or carer with verbal and/or written information on warning symptoms and
how further healthcare can be accessed (see section 1.7.2)
arranging further follow-up at a specified time and place
liaising with other healthcare professionals, including out-of-hours providers, to ensure direct
access for the child if further assessment is required. [2007]
1.4.2.4 Children with 'green' features and none of the 'amber' or 'red' features can be cared for
at home with appropriate advice for parents and carers, including advice on when to seek
further attention from the healthcare services (see section 1.7). [2007, amended 2013]
1.4.3 Tests by the non-paediatric practitioner
1.4.3.1 Children with symptoms and signs suggesting pneumonia who are not admitted to
hospital should not routinely have a chest X-ray. [2007]
1.4.3.2 Test urine in children with fever as recommended in Urinary tract infection in children
(NICE clinical guideline 54). [2007]
1.4.4 Use of antibiotics by the non-paediatric practitioner
1.4.4.1 Do not prescribe oral antibiotics to children with fever without apparent source. [2007]
1.4.4.2 Give parenteral antibiotics to children with suspected meningococcal disease at the
earliest opportunity (either benzylpenicillin or a third-generation cephalosporin)[5]. [2007]
1.5 Management by the paediatric specialist
In this guideline, the term paediatric specialist refers to a healthcare professional who has had
specific training or has recognised expertise in the assessment and treatment of children and
their illnesses. Examples include paediatricians, or healthcare professionals working in children's
emergency departments.
1.5.1 Children younger than 5 years
1.5.1.1 Management by the paediatric specialist should start with a clinical assessment as
described in section 1.2. The healthcare professional should attempt to identify symptoms and
signs of serious illness and specific diseases as summarised in tables 1 and 2. [2007]
1.5.2 Children younger than 3 months

1.5.2.1 Infants younger than 3 months with fever should be observed and have the following
vital signs measured and recorded:
temperature
heart rate
respiratory rate. [2007]
1.5.2.2 Perform the following investigations in infants younger than 3 months with fever:
full blood count
blood culture
C-reactive protein
urine testing for urinary tract infection[6]
chest X-ray only if respiratory signs are present
stool culture, if diarrhoea is present. [2013]
1.5.2.3 Perform lumbar puncture in the following children with fever (unless contraindicated):
infants younger than 1 month
all infants aged 13 months who appear unwell
infants aged 13 months with a white blood cell count (WBC) less than 5 109/litre or greater
than 15 109/litre. [2007, amended 2013]
1.5.2.4 When indicated, perform a lumbar puncture without delay and, whenever possible,
before the administration of antibiotics. [2007]
1.5.2.5 Give parenteral antibiotics to:
infants younger than 1 month with fever
all infants aged 13 months with fever who appear unwell
infants aged 13 months with WBC less than 5 109/litre or greater than 15 109/litre. [2007,
amended 2013]
1.5.2.6 When parenteral antibiotics are indicated for infants younger than 3 months of age, a
third-generation cephalosporin (for example cefotaxime or ceftriaxone) should be given plus an
antibiotic active against listeria (for example, ampicillin or amoxicillin). [2007]
1.5.3 Children aged 3 months or older

1.5.3.1 Perform the following investigations in children with fever without apparent source who
present to paediatric specialists with 1 or more 'red' features:
full blood count
blood culture
C-reactive protein
urine testing for urinary tract infection[6]. [2013]
1.5.3.2 The following investigations should also be considered in children with 'red' features, as
guided by the clinical assessment:
lumbar puncture in children of all ages (if not contraindicated)
chest X-ray irrespective of body temperature and WBC
serum electrolytes and blood gas. [2007]
1.5.3.3 Children with fever without apparent source presenting to paediatric specialists who
have 1 or more 'amber' features, should have the following investigations performed unless
deemed unnecessary by an experienced paediatrician.
urine should be collected and tested for urinary tract infection[6]
blood tests: full blood count, C-reactive protein and blood cultures
lumbar puncture should be considered for children younger than 1 year
chest X-ray in a child with a fever greater than 39C and WBC greater than 20 109/litre. [2007]
1.5.3.4 Children who have been referred to a paediatric specialist with fever without apparent
source and who have no features of serious illness (that is, the 'green' group), should have urine
tested for urinary tract infection[6] and be assessed for symptoms and signs of pneumonia (see
table 2). [2007]
1.5.3.5 Do not routinely perform blood tests and chest X-rays in children with fever who have no
features of serious illness (that is, the 'green' group). [2007]
1.5.4 Viral co-infection
1.5.4.1 Febrile children with proven respiratory syncytial virus or influenza infection should be
assessed for features of serious illness. Consideration should be given to urine testing for urinary
tract infection[6]. [2007]
1.5.5 Observation in hospital

1.5.5.1 In children aged 3 months or older with fever without apparent source, a period of
observation in hospital (with or without investigations) should be considered as part of the
assessment to help differentiate non-serious from serious illness. [2007]
1.5.5.2 When a child has been given antipyretics, do not rely on a decrease or lack of decrease in
temperature at 12 hours to differentiate between serious and non-serious illness.
Nevertheless, in order to detect possible clinical deterioration, all children in hospital with
'amber' or 'red' features should still be reassessed after 12 hours. [new 2013]
1.5.6 Immediate treatment by the paediatric specialist (for children of all ages)
1.5.6.1 Children with fever and shock presenting to specialist paediatric care or an emergency
department should be:
given an immediate intravenous fluid bolus of 20 ml/kg; the initial fluid should normally be 0.9%
sodium chloride
actively monitored and given further fluid boluses as necessary. [2007]
1.5.6.2 Give immediate parenteral antibiotics to children with fever presenting to specialist
paediatric care or an emergency department if they are:
shocked
unrousable
showing signs of meningococcal disease. [2007]
1.5.6.3 Immediate parenteral antibiotics should be considered for children with fever and
reduced levels of consciousness. In these cases symptoms and signs of meningitis and herpes
simplex encephalitis should be sought (see table 2 and Bacterial meningitis and meningococcal
septicaemia [NICE clinical guideline 102]). [2007]
1.5.6.4 When parenteral antibiotics are indicated, a third-generation cephalosporin (for
example, cefotaxime or ceftriaxone) should be given, until culture results are available. For
children younger than 3 months, an antibiotic active against listeria (for example, ampicillin or
amoxicillin) should also be given. [2007]
1.5.6.5 Give intravenous aciclovir to children with fever and symptoms and signs suggestive of
herpes simplex encephalitis (see recommendation 1.2.3.5). [2007]
1.5.6.6 Oxygen should be given to children with fever who have signs of shock or oxygen
saturation (SpO2) of less than 92% when breathing air. Treatment with oxygen should also be
considered for children with an SpO2 of greater than 92%, as clinically indicated. [2007]
1.5.7 Causes and incidence of serious bacterial infection

1.5.7.1 In a child presenting to hospital with a fever and suspected serious bacterial infection,
requiring immediate treatment, antibiotics should be directed against Neisseria meningitidis,
Streptococcus pneumoniae, Escherichia coli, Staphylococcus aureus and Haemophilus influenzae
type b. A third-generation cephalosporin (for example, cefotaxime or ceftriaxone) is
appropriate, until culture results are available. For infants younger than 3 months, an antibiotic
active against listeria (for example, ampicillin or amoxicillin) should be added. [2007]
1.5.7.2 Refer to local treatment guidelines when rates of bacterial antibiotic resistance are
significant. [2007]
1.5.8 Admission to and discharge from hospital
1.5.8.1 In addition to the child's clinical condition, consider the following factors when deciding
whether to admit a child with fever to hospital:
social and family circumstances
other illnesses that affect the child or other family members
parental anxiety and instinct (based on their knowledge of their child)
contacts with other people who have serious infectious diseases
recent travel abroad to tropical/subtropical areas, or areas with a high risk of endemic infectious
disease
when the parent or carer's concern for their child's current illness has caused them to seek
healthcare advice repeatedly
where the family has experienced a previous serious illness or death due to feverish illness
which has increased their anxiety levels
when a feverish illness has no obvious cause, but the child remains ill longer than expected for a
self-limiting illness. [2007]
1.5.8.2 If it is decided that a child does not need to be admitted to hospital, but no diagnosis has
been reached, provide a safety net for parents and carers if any 'red' or 'amber' features are
present. The safety net should be 1 or more of the following:
providing the parent or carer with verbal and/or written information on warning symptoms and
how further healthcare can be accessed (see section 1.7.2)
arranging further follow-up at a specified time and place
liaising with other healthcare professionals, including out-of-hours providers, to ensure direct
access for the child if further assessment is required. [2007]

1.5.8.3 Children with 'green' features and none of the 'amber' or 'red' features can be cared for
at home with appropriate advice for parents and carers, including advice on when to seek
further attention from the healthcare services (see section 1.7). [2007, amended 2013]
1.5.9 Referral to paediatric intensive care
1.5.9.1 Children with fever who are shocked, unrousable or showing signs of meningococcal
disease should be urgently reviewed by an experienced paediatrician and consideration given to
referral to paediatric intensive care. [2007]
1.5.9.2 Give parenteral antibiotics to children with suspected meningococcal disease at the
earliest opportunity (either benzylpenicillin or a third-generation cephalosporin). [2007]
1.5.9.3 Children admitted to hospital with meningococcal disease should be under paediatric
care, supervised by a consultant and have their need for inotropes assessed. [2007]
1.6 Antipyretic interventions
1.6.1 Effects of body temperature reduction
1.6.1.1 Antipyretic agents do not prevent febrile convulsions and should not be used specifically
for this purpose. [2007]
1.6.2 Physical interventions to reduce body temperature
1.6.2.1 Tepid sponging is not recommended for the treatment of fever. [2007]
1.6.2.2 Children with fever should not be underdressed or over-wrapped. [2007]
1.6.3 Drug interventions to reduce body temperature
1.6.3.1 Consider using either paracetamol or ibuprofen in children with fever who appear
distressed. [new 2013]
1.6.3.2 Do not use antipyretic agents with the sole aim of reducing body temperature in children
with fever. [new 2013]
1.6.3.3 When using paracetamol or ibuprofen in children with fever:
continue only as long as the child appears distressed
consider changing to the other agent if the child's distress is not alleviated
do not give both agents simultaneously
only consider alternating these agents if the distress persists or recurs before the next dose is
due. [new 2013]

1.7 Advice for home care


1.7.1 Care at home
1.7.1.1 Advise parents or carers to manage their child's temperature as described in section 1.6.
[2007]
1.7.1.2 Advise parents or carers looking after a feverish child at home:
to offer the child regular fluids (where a baby or child is breastfed the most appropriate fluid is
breast milk)
how to detect signs of dehydration by looking for the following features:
sunken fontanelle
dry mouth
sunken eyes
absence of tears
poor overall appearance
to encourage their child to drink more fluids and consider seeking further advice if they detect
signs of dehydration
how to identify a non-blanching rash
to check their child during the night
to keep their child away from nursery or school while the child's fever persists but to notify the
school or nursery of the illness. [2007]
1.7.2 When to seek further help
1.7.2.1 Following contact with a healthcare professional, parents and carers who are looking
after their feverish child at home should seek further advice if:
the child has a fit
the child develops a non-blanching rash
the parent or carer feels that the child is less well than when they previously sought advice
the parent or carer is more worried than when they previously sought advice
the fever lasts longer than 5 days

the parent or carer is distressed, or concerned that they are unable to look after their child.
[2007]

Question 92 of 180

What is the correct dose of intramuscular adrenaline to give a 10-year-old boy who is having an anaphylactic reaction?
IM adrenaline 300 mcg (0.3ml of 1 in 1,000)

IM adrenaline 100 mcg (0.1ml of 1 in 1,000)

IM adrenaline 200 mcg (0.2ml of 1 in 1,000)

IM adrenaline 150 mcg (0.15ml of 1 in 1,000)

IM adrenaline 500 mcg (0.5ml of 1 in 1,000)

Next question

Anaphylaxis
Anaphylaxis may be defined as a severe, life-threatening, generalised or systemic
hypersensitivity reaction.
Anaphylaxis is one of the few times when you would not have time to look up the dose of a medication. The Resuscitation Council
guidelines on anaphylaxis have recently been updated. Adrenaline is by far the most important drug in anaphylaxis and should be given as
soon as possible. The recommended doses for adrenaline, hydrocortisone and chlorphenamine are as follows:
Adrenaline

Hydrocortisone Chlorphenamine

< 6 months

150 mcg (0.15ml 1 in 1,000)25 mg

250 mcg/kg

6 months - 6 years

150 mcg (0.15ml 1 in 1,000)50 mg

2.5 mg

6-12 years

300 mcg (0.3ml 1 in 1,000) 100 mg

5 mg

Adult and child > 12 years 500 mcg (0.5ml 1 in 1,000) 200 mg

10 mg

Adrenaline can be repeated every 5 minutes if necessary. The best site for IM injection is the anterolateral aspect of the middle third of the
thigh.
Common identified causes of anaphylaxis

food (e.g. Nuts) - the most common cause in children


drugs
venom (e.g. Wasp sting)

Question 93 of 180

A 9-year-old is referred to the Children's Ward with pyrexia and cough:

Image used on license from Radiopaedia

What does the x-ray show?


Right middle lobe consolidation

Perihilar lymphadenopathy

No abnormality

Left lingual consolidation

Right upper lobe consolidation

Next question

The loss of the right heart border is a common finding in right middle lobe consolidation

Chest x-ray: white lung lesions

There are numerous causes of white shadowing in the lungs including:

consolidation
pleural effusion
collapse
pneumonectomy
specific lesions e.g. tumours
fluid e.g. pulmonary oedema

For more information check out the link to the superb Radiology Masterclass website.

Question 94 of 180

Which one of the following statements regarding cow's milk protein intolerance/allergy in infants is true?
An adrenaline pen should be given to all parents

It is more common in breastfed infants

Green-coloured stools are common

Around 0.5-1% of infants are affected

The majority of cases resolve before the age of 5 years

Next question

Cow's milk protein intolerance/allergy


Cow's milk protein intolerance/allergy (CMPI/CMPA) occurs in around 3-6% of all children and typically presents in the first 3 months of life
in formula fed infants, although rarely it is seen in exclusively breastfed infants.
Both immediate (IgE mediated) and delayed (non-IgE mediated) reactions are seen. The term CMPA is usually used for immediate
reactions and CMPI for mild-moderate delayed reactions.
Features

regurgitation and vomiting


diarrhoea
urticaria, atopic eczema
'colic' symptoms: irritability, crying
wheeze, chronic cough
rarely angioedema and anaphylaxis may occur

Diagnosis is often clinical (e.g. improvement with cow's milk protein elimination). Investigations include:

skin prick/patch testing


total IgE and specific IgE (RAST) for cow's milk protein

Management
If the symptoms are severe (e.g. failure to thrive) refer to a paediatrician.
Management if formula-fed

extensive hydrolysed formula (eHF) milk is the first-line replacement formula for infants with mild-moderate symptoms
amino acid-based formula (AAF) in infants with severe CMPA or if no response to eHF
around 10% of infants are also intolerant to soya milk

Management if breast-fed

continue breastfeeding
eliminate cow's milk protein from maternal diet
use eHF milk when breastfeeding stops, until 12 months of age and at least for 6 months

CMPI usually resolves by 1-2 years of age. A challenge is often performed in the hospital setting as anaphylaxis can occur.

Question 95 of 180

Which one of the following is not part of the Apgar score for assessing the newborn?
Colour

Respiratory effort

Tone

Heart rate

Capillary refill time

Next question

Apgar score
The Apgar score is used to assess the health of a newborn baby
Score Pulse Respiratory effort Colour

Muscle tone

Reflex irritability

> 100 Strong, crying

Pink

Active movementSneezes, coughs

< 100 Weak, irregular

Blue limbs Limb flexion

Grimace

Score Pulse Respiratory effort Colour


0

Absent Nil

Muscle tone

Blue all over Flaccid

Reflex irritability
Nil

Question 96 of 180

What is the first sign of puberty in boys?


Development of axillary hair

Height spurt

Development of pubic hair

Increase in penis length

Increase in testicular volume

Next question

Puberty
Males

first sign is testicular growth at around 12 years of age (range = 10-15 years)
testicular volume > 4 ml indicates onset of puberty
maximum height spurt at 14

Females

first sign is breast development at around 11.5 years of age (range = 9-13 years)
height spurt reaches its maximum early in puberty (at 12) , before menarche
menarche at 13 (11-15)
there is an increase of only about 4% of height following menarche

Normal changes in puberty

gynaecomastia may develop in boys


asymmetrical breast growth may occur in girls
diffuse enlargement of the thyroid gland may be seen

Question 97 of 180

A 11-week-old boy is reviewed in surgery. As part of a general assessment his head circumference is noted to be between the 0.4th and
2nd centile. Which one of the following would not explain this finding?
Congenital infection

Fragile X syndrome

Normal variant

Craniosynostosis

Hypoxic ischaemic encephalopathy

Next question

Whilst not a classic cause of macrocephaly, children with Fragile X syndrome tend to have a head larger than normal

Microcephaly
Microcephaly may be defined as an occipital-frontal circumference < 2nd centile
Causes include

normal variation e.g. small child with small head


familial e.g. parents with small head
congenital infection
perinatal brain injury e.g. hypoxic ischaemic encephalopathy
fetal alcohol syndrome
syndromes: Patau
craniosynostosis

Question 98 of 180

A mother brings her son in to surgery as she suspects he has a squint. She thinks his right eye is 'turned inwards'. You perform a cover test
to gather further information. Which one of the following findings would be consistent with a right esotropia?
On covering the left eye the right eye moves medially to take up fixation

The cover test could not be used to identify this type of defect

On covering the left eye the right eye moves laterally to take up fixation

On covering the right eye the left eye moves laterally to take up fixation

On covering the right eye the left eye moves medially to take up fixation

Next question

Squints may be classified as to where the eye deviates toward

the nose: esotropia


temporally: exotropia
superiorly: hypertropia
inferiorly: hypotropia

On covering the left eye in this example the right eye moves laterally from the nasal (esotropic) position to take up fixation.

Squint
Squint (strabismus) is characterised by misalignment of the visual axes. Squints may be divided into concomitant (common) and paralytic
(rare)
Concomitant

Paralytic

Due to imbalance in extraocular muscles


Due to paralysis of extraocular muscles
Convergent is more common than divergent

Detection of a squint may be made by the corneal light reflection test - holding a light source 30cm from the child's face to see if the light
reflects symmetrically on the pupils
The cover test is used to identify the nature of the squint

ask the child to focus on a object


cover one eye
observe movement of uncovered eye
cover other eye and repeat test

Management

eye patches may help prevent amblyopia


referral to secondary care is appropriate

Question 99 of 180

Which one of the following statements regarding bronchiolitis is true?


A trial of bronchodilators should be given

Oral corticosteroids have been shown to reduce the duration of the illness

Peak incidence is 3-6 months of age

Most common in autumn

Infantile eczema is a risk factor

Next question

Bronchiolitis
Bronchiolitis is a condition characterised by acute bronchiolar inflammation. Respiratory syncytial virus (RSV) is the pathogen in 75-80% of
cases. SIGN released guidelines on bronchiolitis in 2006. Please see the link for more details.
Epidemiology

most common cause of a serious lower respiratory tract infection in < 1yr olds (90% are 1-9 months, with a peak incidence of 36 months). Maternal IgG provides protection to newborns against RSV
higher incidence in winter

Basics

respiratory syncytial virus (RSV) is the pathogen in 75-80% of cases


other causes: mycoplasma, adenoviruses
may be secondary bacterial infection
more serious if bronchopulmonary dysplasia (e.g. Premature), congenital heart disease or cystic fibrosis

Features

coryzal symptoms (including mild fever) precede:


dry cough
increasing breathlessness
wheezing, fine inspiratory crackles (not always present)
feeding difficulties associated with increasing dyspnoea are often the reason for hospital admission

SIGN suggested the following criteria for referral to hospital

poor feeding (< 50% normal)


lethargy
apnoea
respiratory rate > 70/min
nasal flaring or grunting
severe chest wall recession
cyanosis
oxygen saturation < 94%
uncertainty regarding diagnosis

Investigation

immunofluorescence of nasopharyngeal secretions may show RSV

Management is largely supportive

humidified oxygen is given via a head box


Question 100 of 180

A 4-year-old boy is noted to have macrocephaly and learning difficulties. What is the most likely diagnosis?

Patau syndrome

Pierre-Robin syndrome

Edward's syndrome

Fragile X

William's syndrome

Next question

Childhood syndromes

Below is a list of common features of selected childhood syndromes


Syndrome

Key features

Patau syndrome (trisomy 13)

Microcephalic, small eyes


Cleft lip/palate
Polydactyly
Scalp lesions

Edward's syndrome (trisomy 18)

Micrognathia
Low-set ears
Rocker bottom feet
Overlapping of fingers

Fragile X

Learning difficulties
Macrocephaly
Long face
Large ears
Macro-orchidism

Noonan syndrome

Webbed neck
Pectus excavatum
Short stature
Pulmonary stenosis

Pierre-Robin syndrome*

Micrognathia
Posterior displacement of the tongue (may result in upper airway obstruction)
Cleft palate

Prader-Willi syndrome

Hypotonia
Hypogonadism
Obesity

William's syndrome

Short stature
Learning difficulties
Friendly, extrovert personality
Transient neonatal hypercalcaemia
Supravalvular aortic stenosis

*this condition has many similarities with Treacher-Collins syndrome. One of the key differences is that Treacher-Collins syndrome
is autosomal dominant so there is usually a family history of similar problems

Question 1 of 80

Please look at a close-up of the scalp of a 7-year-old-girl

Image used on license from DermNet NZ

Which one of the following is not a recommended management option?


Isopropyl myristate and cyclomethicone

Wet combing

Dimeticone

Malathion

Benzyl benzoate

Next question

Head lice
Head lice (also known as pediculosis capitis or 'nits') is a common condition in children caused by the parasitic insect Pediculus capitis,
which lives on and among the hair of the scalp of humans
Diagnosis

fine-toothed combing of wet or dry hair

Management

treatment is only if living lice are found


a choice of treatments should be offered - malathion, wet combing, dimeticone, isopropyl myristate and cyclomethicone

School exclusion is not advised for children with head lice

Head lice - Summary

Head lice infestation (also known as pediculosis capitis) is the condition caused by the
parasitic insect Pediculus capitis, which lives on, and among, the hair of the scalp and
neck of humans.
o Live lice can be found anywhere on the scalp; the eggs are most commonly found
behind the ears and at the back of the neck.
o The severity of infestation varies from a few lice to thousands of lice, but a typical
infestation might have about 30 lice per head.
If left untreated, infestation with head lice may persist for long periods.
To confirm a person has head lice:
o Detection combing (systematic combing of wet or dry hair with a detection comb)
confirms the presence of lice.
o A diagnosis of active head lice infestation should only be made if a live head louse is
found.
o An itching scalp is not sufficient to diagnose active infestation.
o The presence of louse eggs alone, whether hatched (nits) or unhatched, are not
proof of active infestation.
o A person should only be treated if a live head louse is found.
All affected household members need simultaneous treatment.
Depending on the preference of the individual or parent and on the treatment history,
head lice can be treated with:
o Dimeticone 4% lotion (Hedrin).
o Dimeticone 92% spray (NYDA).
o Wet combing using the Bug Buster comb and method.
o Isopropyl myristate and cyclomethicone solution (Full Marks Solution ).
o Coconut, anise, and ylang ylang spray (Lyclear SprayAway).
o Malathion 0.5% aqueous liquid.
All treatments need more than one treatment session and no treatment can guarantee
success. Treatment has the best chance of success if it is performed correctly and if all
affected household members are treated on the same day.
People should be advised to check whether treatment was successful by detection
combing on day 2 or day 3 after completing a course of treatment, and again after an
interval of 7 days (day 9 or day 10 after completing a course of treatment).
If treatment is unsuccessful:
o The same treatment should be repeated, or a different treatment tried.
o All affected household contacts need to be treated simultaneously again.
Children who are being treated for head lice can still attend school.

Question 2 of 80

A mother brings her 2-year-old son to surgery. For the past two weeks he has been complaining of an itchy bottom. He is otherwise well
and clinical examination including that of the perianal area is unremarkable. What is the most appropriate management?
Hygiene measures + single dose dimeticone for child

Hygiene measures + single dose mebendazole for child

Hygiene measures + single dose mebendazole for all the family

Hygiene measures + single dose mebendazole repeated after 2 weeks for all the family

Contact child protection officer

Next question

Threadworms are extremely common and in the absence of any other concern an itchy bottom would not warrant referral to the child
protection officer

Threadworms
Infestation with threadworms (Enterobius vermicularis, sometimes called pinworms) is extremely common amongst children in the UK.
Infestation occurs after swallowing eggs that are present in the environment.
Threadworm infestation is asymptomatic in around 90% of cases, possible features include:

perianal itching, particularly at night


girls may have vulval symptoms

Diagnosis may be made by the applying Sellotape to the perianal area and sending it to the laboratory for microscopy to see the eggs.
However, most patients are treated empirically and this approach is supported in the CKS guidelines.
Management

CKS recommend a combination of anthelmintic with hygiene measures for all members of the household
mebendazole is used first-line for children > 6 months old. A single dose is given unless infestation persists

Threadworm - Summary

Threadworm or pinworm (Enterobius vermicularis) is a parasitic worm which infests the


intestines of humans.
Threadworm infestation occurs after swallowing eggs, usually by eating contaminated
food or from contaminated hands.
Threadworm is the most common parasitic worm infestation in the UK and is more
likely to affect school or pre-school children than adults, because of their inattention to
good personal hygiene and close contact with other children. Threadworms often affect
family groups or institutions, especially if conditions are crowded. It is not necessary to
exclude children with threadworms from school.

The most common symptom is perianal itching, which is worse during the night.
Infestation may be symptomless and only detected when threadworms are seen on the
perianal skin or in the stools.There may be signs of scratching in the perianal area and
sometimes localized secondary bacterial infection. It is unusual to see worms in the
perianal area when the person is examined during the day.
If the diagnosis is uncertain, the adhesive tape test for eggs may be useful. Transparent
tape is applied to the perianal area first thing in the morning and then examined under a
microscope to detect threadworm eggs. Stool examination is not generally
recommended.
Other causes of perineal itch include dermatitis, candidal infection, and haemorrhoids.
Threadworms are rarely confused with other types of worm infestation because of
their different appearance.
Treatment is recommended if threadworms have been seen or eggs detected
treatment. All household members should be treated at the same time.
o For adults and children aged over 6 months, an anthelmintic (mebendazole)
combined with hygiene measures is recommended.
o For children aged 6 months and under, hygiene measures alone are recommended.
o For people who do not wish to take an anthelmintic, physical removal of the eggs,
combined with hygiene measures is recommended.
o For pregnant or breastfeeding women, physical removal of eggs combined with
hygiene methods is the preferred treatment. Treatment with mebendazole is
contraindicated in the first trimester of pregnancy; however, it can be considered in
the second or third trimester, or during breastfeeding if drug treatment is
considered necessary (off-label use).
If infestation persists after hygiene measures have been continued for the
recommended duration, a further course of drug treatment is recommended. It is
important that household members are treated and adhere to hygiene measures.

Question 3 of 80

What is the most likely outcome following the diagnosis of minimal change nephropathy in a 10-year-old male?
Chronic kidney disease requiring renal replacement therapy within 30 years

Full recovery and no further episodes

Full recovery but with later recurrent episode

Chronic kidney disease not requiring renal replacement therapy

Chronic kidney disease requiring renal replacement therapy within 10 years

Next question

As 1/3 of patients have infrequent relapses and 1/3 of patients have frequent relapses a majority (2/3) will have later recurrent episodes. It
is important however to stress to patients that generally speaking the longer term prognosis in minimal change glomerulonephritis is good.

Minimal change disease


Minimal change disease nearly always presents as nephrotic syndrome, accounting for 75% of cases in children and 25% in adults.
The majority of cases are idiopathic, but in around 10-20% a cause is found:

drugs: NSAIDs, rifampicin


Hodgkin's lymphoma, thymoma
infectious mononucleosis

Pathophysiology

T-cell and cytokine mediated damage to the glomerular basement membrane polyanion loss
the resultant reduction of electrostatic charge increased glomerular permeability to serum albumin

Features

nephrotic syndrome
normotension - hypertension is rare
highly selective proteinuria*
renal biopsy: electron microscopy shows fusion of podocytes

Management

majority of cases (80%) are steroid responsive


cyclophosphamide is the next step for steroid resistant cases

Prognosis is overall good, although relapse is common. Roughly:

1/3 have just one episode


1/3 have infrequent relapses
1/3 have frequent relapses which stop before adulthood

*only intermediate-sized proteins such as albumin and transferrin leak through the glomerulus

Question 4 of 80

When assessing an 18-month-old child with diarrhoea and vomiting which one of the following signs is most likely to indicate shock?
Tachycardia

Sunken eyes

Reduced skin turgor

Prolonged capillary refill time

Mother reports child is irritable

Next question

Diarrhoea and vomiting in children


Diarrhoea and vomiting is very common in younger children. The most common cause of gastroenteritis in children in the UK is rotavirus.
Much of the following is based around the 2009 NICE guidelines (please see the link for more details).
Clinical features
NICE suggest that typically:

diarrhoea usually lasts for 5-7 days and stops within 2 weeks
vomiting usually lasts for 1-2 days and stops within 3 days

When assessing hydration status NICE advocate using normal, dehydrated or shocked categories rather than the traditional normal, mild,
moderate or severe categories.
Clinical dehydration

Clinical shock

Appears to be unwell or deteriorating


Decreased level of consciousness
Decreased urine output
Skin colour unchanged
Cold extremities
Warm extremities
Altered responsiveness (for example, irritable, lethargic)Pale or mottled skin
Sunken eyes
Dry mucous membranes
Tachycardia
Tachypnoea
Normal peripheral pulses
Normal capillary refill time
Reduced skin turgor
Normal blood pressure

Tachycardia
Tachypnoea
Weak peripheral pulses
Prolonged capillary refill time
Hypotension

The following children are at an increased risk of dehydration:

children younger than 1 year, especially those younger than 6 months


infants who were of low birth weight
children who have passed six or more diarrhoeal stools in the past 24 hours
children who have vomited three times or more in the past 24 hours
children who have not been offered or have not been able to tolerate supplementary fluids before presentation
infants who have stopped breastfeeding during the illness
children with signs of malnutrition

Features suggestive of hypernatraemic dehydration:

jittery movements
increased muscle tone
hyperreflexia
convulsions
drowsiness or coma

Diagnosis
NICE suggest doing a stool culture in the following situations:

you suspect septicaemia or


there is blood and/or mucus in the stool or
the child is immunocompromised

You should consider doing a stool culture if:

the child has recently been abroad or


the diarrhoea has not improved by day 7 or
you are uncertain about the diagnosis of gastroenteritis

Management
If clinical shock is suspected children should be admitted for intravenous rehydration.
For children with no evidence of dehydration

continue breastfeeding and other milk feeds


encourage fluid intake
discourage fruit juices and carbonated drinks

If dehydration is suspected:

give 50 ml/kg low osmolarity oral rehydration solution (ORS) solution over 4 hours, plus ORS solution for maintenance, often
and in small amounts
continue breastfeeding
consider supplementing with usual fluids (including milk feeds or water, but not fruit juices or carbonated drinks)

Question 5 of 80

An 8-year-old boy is reviewed in the Enuresis clinic. He is still wetting the bed at night despite using an enuresis alarm for the past three
months. There are no problems with micturition during the daytime and he passes one soft stool everyday. Which one of the following
treatments is most likely to be offered?
Lactulose

Desmopressin

Cognitive behavioural therapy

Imipramine

Duloxetine

Next question

Nocturnal enuresis
The majority of children achieve day and night time continence by 3 or 4 years of age. Enuresis may be defined as the 'involuntary
discharge of urine by day or night or both, in a child aged 5 years or older, in the absence of congenital or acquired defects of the nervous
system or urinary tract'
Nocturnal enuresis can be defined as either primary (the child has never achieved continence) or secondary (the child has been dry for at
least 6 months before)
NICE issued guidance in 2010. Management:

look for possible underlying causes/triggers (e.g. Constipation, diabetes mellitus, UTI if recent onset)
advise on fluid intake, diet and toileting behaviour
reward systems (e.g. Star charts). NICE recommend these 'should be given for agreed behaviour rather than dry nights' e.g.
Using the toilet to pass urine before sleep
NICE advise: 'Consider whether alarm or drug treatment is appropriate, depending on the age, maturity and abilities of the child
or young person, the frequency of bedwetting and the motivation and needs of the family'. Generally:
an enuresis alarm is first-line for children under the age of 7 years
desmopressin may be used first-line for children over the ago 7 years, particularly if short-term control is needed or an enuresis
alarm has been ineffective/is not acceptable to the family
please see the link for more details

Bedwetting (enuresis) - Summary


Bedwetting is involuntary wetting during sleep, without any inherent suggestion of
frequency or pathophysiology.
Bedwetting can be classified as:
Primary bedwetting the child has never achieved sustained continence at night.
Secondary bedwetting bedwetting occurs after the child has been dry at night for
more than 6 months.
Primary bedwetting without daytime symptoms is thought to be caused by sleep arousal
difficulties, polyuria, and/or bladder dysfunction.
Daytime symptoms include urgency, frequency, daytime wetting, abdominal straining or
poor urinary stream, pain passing urine, or passing urine fewer than four times a day.
Primary bedwetting with daytime symptoms is usually caused by disorders of the lower
urinary tract.
It is most commonly caused by an overactive bladder, but may also be caused by
congenital malformations or neurological disorders.
It is occasionally caused by chronic constipation, chronic urinary tract infection (UTI), or
chronic emotional problems.
Secondary bedwetting is bedwetting secondary to an underlying cause, such as
diabetes, UTI, constipation, or family problems (vulnerable child or family).
Risk factors associated with bedwetting include:
A family history of bedwetting.
Gender boys are more likely than girls to have bedwetting.
Delay in attaining bladder control.
Being obese approximately 30% of obese children have bedwetting.

Psychological or behavioural disorders such as attention deficit hyperactivity disorder


(ADHD), autism spectrum disorder, anxiety, depressive, and conduct disorders.
It is necessary to determine the type of bedwetting by asking if:
There are any daytime symptoms.
The child has previously been dry at night without assistance for 6 months.
Managing primary bedwetting (without daytime symptoms) involves:
General advice on bedwetting.
Advice on lifting and waking.
Addressing any issues concerning excessive or insufficient fluid intake, diet, and toileting
patterns.
In children < 5 years of age, reassurance that many children younger than 5 years of age
wet the bed and that this usually resolves without treatment.
In children > 5 years of age, if bedwetting is infrequent (less than twice a week),
reassurance that bedwetting may resolve without treatment and offering the option of
a wait-and-see approach. If long-term treatment is required, treatment with an enuresis
alarm (first-line treatment) should be offered in combination with positive reward
systems desmopressin may be considered if the alarm is unsuitable. If rapid or shortterm control of bedwetting is required, treatment with desmopressin is an option.
Managing primary bedwetting (with daytime symptoms) involves:
Referring the child to secondary care or an enuresis clinic for further investigations and
assessment.
Considering referral for younger children (older than 2 years of age) who are struggling
to not wet themselves during the day as well as during the night, despite awareness of
toileting needs and showing appropriate toileting behaviour.
Managing secondary enuresis involves:
Treatment of UTIs and constipation in primary care.
Referral to a paediatrician or an enuresis clinic if children have other underlying causes
which have been identified but are generally not managed in primary care (e.g. diabetes
and learning difficulties).
Have I got the right topic?
Age from 24 months to 19 years
This CKS topic is based on guidance published by the National Institute for Health and
Care Excellence, Nocturnal enuresis the management of bedwetting in children and
young people [National Clinical Guideline Centre, 2010].
This CKS topic covers the management of children with nocturnal enuresis (bedwetting).
This CKS topic covers the management of children who have never been consistently dry
at night (primary enuresis), as well as children who have been previously dry for at least
6 months before developing enuresis again (secondary enuresis).
This CKS topic does not cover the management of adult bedwetting. This CKS topic does
not cover the management of toilet training, or children with constipation, urinary tract
infections, or diabetes.

There are separate CKS topics on Constipation in children, Diabetes - type 1, Diabetes type 2, and Urinary tract infection - children.
The target audience for this CKS topic is healthcare professionals working within the
NHS in the UK, and providing first contact or primary health care.
How up-to-date is this topic?
Changes
Update
Goals and outcome measures
Goals
Background information
Definition
Causes
Prevalence
Risk factors
Prognosis
Impact
Assessment
Assessment of bedwetting (enuresis)
Assessment
When to consider child maltreatment
Investigations
Management
Scenario: Primary bedwetting (without daytime symptoms) : covers the management of
children with primary bedwetting without daytime symptoms in primary care.
Scenario: Primary bedwetting (with daytime symptoms) : covers the management of
children with daytime symptoms in primary care.
Scenario: Secondary bedwetting : covers the primary care management of children who
have previously been dry at night for 6 months and who start to wet the bed again.
Scenario: Primary bedwetting (without daytime symptoms)
Scenario: Primary bedwetting (with daytime symptoms)
Scenario: Secondary bedwetting
Prescribing information
Important aspects of prescribing information relevant to primary healthcare are covered
in this section specifically for the drugs recommended in this CKS topic. For further
information on contraindications, cautions, drug interactions, and adverse effects, see
the electronic Medicines Compendium (eMC) (http://medicines.org.uk/emc), or the
British National Formulary (BNF) (www.bnf.org).
Desmopressin prescribing issues
Evidence
Supporting evidence
Search strategy
References

ABPI Medicines Compendium (2012a) Summary of product characteristics for


Desmotabs 0.2mg. Electronic Medicines CompendiumDatapharm Communications Ltd.
www.medicines.org.uk [Free Full-text]
ABPI Medicines Compendium (2012b) Summary of product characteristics for
DesmoMelt 120mcg and 240mcg oral lyophilisate. Electronic Medicines
CompendiumDatapharm Communications Ltd. www.medicines.org.uk [Free Full-text]
American Psychiatric Association (Ed.) (2013) Diagnostic and statistical manual of mental
disorders: DSM-5. 5th edn. Washington, DC: American Psychiatric Association.
Baxter, K. and Preston, C.L. (Eds.) (2013) Stockley's drug interactions: a source book of
interactions, their mechanisms, clinical importance and management. 10th edn.
London: Pharmaceutical Press.
BNF 68 (2014) British National Formulary. 68th edn. London: British Medical Association
and Royal Pharmaceutical Society of Great Britain.
Butler, R.J. and Heron, J. (2008) The prevalence of infrequent bedwetting and nocturnal
enuresis in childhood. Scandinavian Journal of Urology and Nephrology 42(3), 257-264.
[Abstract]
Caldwell, P., Deshpande, A. and Von Gontard, A. (2013a) Management of nocturnal
enuresis (clinical review). BMJ. 347: f6259. BMJ. www.bmj.com
Caldwell, P., Nankivell, G. and Sureshkumar, P. (2013b) Simple behavioural interventions
for nocturnal enuresis in children (Cochrane Review). The Cochrane Library. Issue 7.
John Wiley & Sons, Ltd. www.thecochranelibrary.com [Free Full-text]
European Association of Urology and European Society for Paediatric Urology (2013)
Guidelines on paediatric urology. European Association of Urology. www.uroweb.org
[Free Full-text]
Evans, J., Malmsten, B., Maddocks, A. et al. (2011) Randomized comparison of long-term
desmopressin and alarm treatment for bedwetting. Journal of Pediatric Urology 7(1), 2129. [Abstract]
IOM (2004) Dietary reference intakes for water, potassium, sodium, chloride, and
sulfate. Washington, DC.: Institute of Medicine [Free Full-text].
Joinson, C., Heron, J., Emond, A. and Butler, R. (2007) Psychological problems in children
with bedwetting and combined (day and night) wetting: A UK population-based study.
Journal of Pediatric Psychology 32(5), 605-616. [Abstract] [Free Full-text]
Kiddoo, D. (2012) Nocturnal enuresis. Canadian Medical Association Journal 184(8), 908911. [Free Full-text]
Kwak, K., Lee, Y., Park, K. and Baek, M. (2010) Efficacy of desmopressin and enuresis
alarm as first and second line treatment for primary monosymptomatic nocturnal
enuresis: prospective randomized crossover study. Journal of Urology 184(6), 25212526. [Abstract]
Merck (2012) Urinary incontinence in children (enuresis). Merck Manual.
www.merck.com [Free Full-text]

MHRA (2007) Desmopressin nasal spray: removal of the primary nocturnal enuresis
(bedwetting) indication. Medicines and Healthcare products Regulatory Agency.
www.mhra.gov.uk [Free Full-text]
Montaldo, P., Tafuro, L., Rea, M. et al. (2012) Desmopressin and oxybutynin in
monosymptomatic nocturnal enuresis: a randomized, double-blind, placebo-controlled
trial and an assessment of predictive factors. BJU International 110(8(Pt B)), E381-E386.
[Abstract]
National Clinical Guideline Centre (2010) Nocturnal enuresis: the management of
bedwetting in children and young people. National Institute for Health and Care
Excellence. www.nice.org.uk [Free Full-text]
Nevus, T. (2009) Diagnosis and management of nocturnal enuresis. Current Opinion in
Pediatrics 21(2), 199-202. [Abstract]
NICE (2009) When to suspect child maltreatment (NICE guideline). . Clinical guideline 89.
National Institute for Health and Care Excellence. www.nice.org.uk [Free Full-text]
NICE (2012) Nocturnal enuresis: evidence update July 2012. A summary of selected new
evidence relevant to NICE clinical guideline 111 'The management of bedwetting in
children and young people' (2010). Evidence update 21National Institute for Health and
Care Excellence. www.evidence.nhs.uk [Free Full-text]
Perrin, N., Sayer, L. and While, A. (2013) The efficacy of alarm therapy versus
desmopressin therapy in the treatment of primary mono-symptomatic nocturnal
enuresis: a systematic review. Primary Health Care Research and Development (epub
ahead of print), . [Abstract]
Sapi, M.C., Vasconcelos, J.S., Silva, F.G. et al. (2009) Assessment of domestic violence
against children and adolescents with enuresis. Jornal de Pediatria 85(5), 433-437.
[Abstract]
van Dommelen, P., Kamphuis, M., van Leerdam, F. et al. (2009) The short- and long-term
effects of simple behavioral interventions for nocturnal enuresis in young children: a
randomized controlled trial. Journal of Pediatrics 154(5), 662-666. [Abstract]

Question 6 of 80

A 14-year-old attends surgery. She was diagnosed with having migraines three years ago and requests advice about options for treating an
acute attack. Which one of the following medications is it least suitable to recommend?
Aspirin

Paracetamol + prochlorperazine

Paracetamol + codeine

Ibuprofen

Paracetamol

Next question

Avoid aspirin in children < 16 years as risk of Reye's syndrome

Aspirin should be avoided in children due to the risk of Reye's syndrome.


Codeine would also be a poor choice as it has limited benefit in migraine.

Migraine: management
It should be noted that as a general rule 5-HT receptor agonists are used in the acute treatment of migraine whilst 5-HT receptor
antagonists are used in prophylaxis. NICE produced guidelines in 2012 on the management of headache, including migraines.
Acute treatment

first-line: offer combination therapy with an oral triptan and an NSAID, or an oral triptan and paracetamol
for young people aged 12-17 years consider a nasal triptan in preference to an oral triptan
if the above measures are not effective or not tolerated offer a non-oral preparation of metoclopramide* or prochlorperazine and
consider adding a non-oral NSAID or triptan

Prophylaxis

prophylaxis should be given if patients are experiencing 2 or more attacks per month. Modern treatment is effective in about
60% of patients.
NICE advise either topiramate or propranolol 'according to the person's preference, comorbidities and risk of adverse events'.
Propranolol should be used in preference to topiramate in women of child bearing age as it may be teratogenic and it can
reduce the effectiveness of hormonal contraceptives
if these measures fail NICE recommend 'a course of up to 10 sessions of acupuncture over 5-8 weeks' or gabapentin
NICE recommend: 'Advise people with migraine that riboflavin (400 mg once a day) may be effective in reducing migraine
frequency and intensity for some people'
for women with predictable menstrual migraine treatment NICE recommend either frovatriptan (2.5 mg twice a day) or
zolmitriptan (2.5 mg twice or three times a day) as a type of 'mini-prophylaxis'
pizotifen is no longer recommend. Adverse effects such as weight gain & drowsiness are common

*caution should be exercised with young patients as acute dystonic reactions may develop

Migraine - Summary
Migraine is a primary episodic headache disorder. It is characterized by episodic severe
headaches with associated symptoms such as photophobia, phonophobia, and nausea and
vomiting. The most common subtypes of migraine are migraine without aura and migraine with
aura.
Migraine is a complex condition, and the exact pathophysiological cause is not fully understood.
It has a significant genetic component, with about half of people with migraine having a firstdegree relative with the condition.
The prevalence of migraine differs between the sexes, being about three times more common in
women (18%, mean onset of age 18 years) than men (6%, mean onset of age 14 years).

In adults, migraine without aura is diagnosed when at least five attacks fulfil the following
criteria:
Headache lasts 472 hours.
At least two of the following characteristics are present: unilateral location, moderate or severe
pain intensity, pulsating quality, aggravation by routine physical activity.
At least one of the following is present: nausea or vomiting, photophobia or phonophobia.
In adults, migraine with aura (reported by about one third of people with migraine) is diagnosed
when:
There are two or more attacks with one or more symptoms of aura including visual or sensory
symptoms, or dysphasic speech disturbance.
Each individual symptom of aura lasts less than 60 minutes.
Other conditions may present with signs and symptoms similar to migraine. These may be an
alternative form of primary headache disorder (such a cluster headache or tension-type
headache), or a secondary cause, which may be serious and life-threatening.
A headache diary may be useful to identify potential triggers (such as stress, specific foods,
dehydration, missed meals), which can be managed as appropriate.
First-line treatment of acute migraine consists of combination therapy with an oral triptan and
analgesia (paracetamol or a nonsteroidal anti-inflammatory drug). An anti-emetic
(prochlorperazine, domperidone, or metoclopramide) may be added even in the absence of
nausea or vomiting. Monotherapy, if preferred, consists of an oral triptan, NSAID, aspirin, or
paracetamol.
When two or more triptans have been trialled unsuccessfully, or treatment is successful but
attacks are frequent, preventive treatment is often used. First-line preventive treatment
consists of topiramate or propranolol. If both of these have been trialled unsuccessfully or are
inappropriate, gabapentin or acupuncture can be tried.
Hospitalisation or urgent referral is only necessary if a serious cause of headache is suspected,
or if the person is in severe, uncontrolled status migrainosus (migraine lasting for more than 72
hours).
Have I got the right topic?
Age from 12 years onwards
This CKS topic is based on the guidance Headaches - Diagnosis and management of headaches in
young people and adults, published by the National Institute for Health and Clinical Excellence
(NICE) [NICE, 2012], and Guidelines for all healthcare professionals in the diagnosis and

management of migraine, tension-type, cluster and medication-overuse headache, published by


the British Association for the Study of Headache [BASH, 2010].
This CKS topic covers the primary care management of migraine in adults and children over the
age of 12 years.
This CKS topic does not cover the management of other primary headaches (for example tension
and medication-overuse headaches), secondary headaches (for example meningitis),
complications of migraine (for example status migrainosus), or precursors of migraine (for
example cyclical vomiting syndrome and abdominal migraine).
There are separate CKS topics on Headache - assessment, Headache - cluster, Headache medication overuse, Headache - tension-type, Giant cell arteritis, and Trigeminal neuralgia;
there is also a referral guideline, published by the National Institute for Health and Care
Excellence, for Brain tumour - suspected.
The target audience for this CKS topic is healthcare professionals working within the NHS in the
UK, and providing first contact or primary health care.
How up-to-date is this topic?
Changes
Update
Goals and outcome measures
Goals
QIPP Options for local implementation
Background information
Definition
Causes
Prevalence
Prognosis
Complications
Diagnosis
Diagnosis of migraine
Diagnosis

Management
Scenario: Adults : covers the management of people aged 18 years and older with migraine.
Scenario: Young people aged 1217 years : covers the management of young people aged
between 12 and 17 years of age with migraine.
Scenario: Pregnant or breastfeeding women : covers the management of pregnant or
breastfeeding women with migraine.
Scenario: Adults
Scenario: Young people aged 1217 years
Scenario: Pregnant or breastfeeding women
Prescribing information
Important aspects of prescribing information relevant to primary healthcare are covered in this
section specifically for the drugs recommended in this CKS topic. For further information on
contraindications, cautions, drug interactions, and adverse effects, see the electronic Medicines
Compendium (eMC) (http://medicines.org.uk/emc), or the British National Formulary (BNF)
(www.bnf.org).
Standard analgesics, NSAIDs, and anti-emetic drugs
Triptans (5-hydoxytryptamine agonists)
Drugs for the prevention of migraine
Evidence
Supporting evidence
Search strategy
References
ABPI Medicines Compendium (2011a) Summary of product characteristics for Boots Dispersible
Aspirin Tablets BP 75mg. Electronic Medicines CompendiumDatapharm Communications Ltd.
www.medicines.org.uk
ABPI Medicines Compendium (2011b) Summary of product characteristics for half inderal LA
80mg. Electronic Medicines CompendiumDatapharm Communications Ltd.
www.medicines.org.uk [Free Full-text]

ABPI Medicines Compendium (2012) Summary of product characteristics for propranolol tablets
BP 40mg. Electronic Medicines CompendiumDatapharm Communications Ltd.
www.medicines.org.uk [Free Full-text]
ABPI Medicines Compendium (2013a) Summary of product characteristics for Ponstan Forte
tablets 500mg. Electronic Medicines CompendiumDatapharm Communications Ltd.
www.medicines.org.uk [Free Full-text]
ABPI Medicines Compendium (2013b) Summary of product characteristics for Zomig tablets
2.5mg. Electronic Medicines CompendiumDatapharm Communications Ltd.
www.medicines.org.uk [Free Full-text]
ABPI Medicines Compendium (2013c) Summary of product characteristics for Imigran 10mg and
20mg nasal spray. Electronic Medicines CompendiumDatapharm Communications Ltd.
www.medicines.org.uk [Free Full-text]
ABPI Medicines Compendium (2013d) Summary of product characteristics for Neurontin 100mg
Hard Capsules. Electronic Medicines CompendiumDatapharm Communications Ltd.
www.medicines.org.uk [Free Full-text]
ABPI Medicines Compendium (2013e) Summary of product characteristics for Imigran tablets
50mg Imigran tablets 100mg. Electronic Medicines CompendiumDatapharm Communications
Ltd. www.medicines.org.uk [Free Full-text]
ABPI Medicines Compendium (2013f) Summary of product characteristics for Topamax 25 mg,
50mg, 100mg, 200mg tablets and sprinkle capsules 15, 25 or 50 mg. Electronic Medicines
CompendiumDatapharm Communications Ltd. www.medicines.org.uk [Free Full-text]
Ashcroft, D.M. and Millson, D. (2004) Naratriptan for the treatment of acute migraine: metaanalysis of randomised controlled trials. Pharmacoepidemiology and Drug Safety 13(2), 73-82.
[Abstract]
Aube, M. (1999) Migraine in pregnancy. Neurology 53(4 Suppl 1), S26-S28. [Abstract]
Azzopardi, T.D. and Brooks, N.A. (2008) Oral metoclopramide as an adjunct to analgesics for the
outpatient treatment of acute migraine. Annals of Pharmacotherapy 42(3), 397-402. [Abstract]
Bandolier (2002) Migraine special issue. Bandolier. www.medicine.ox.ac.uk/bandolier [Free Fulltext]
BASH (2010) Guidelines for all healthcare professionals in the diagnosis and management of
migraine, tension-type headache, cluster headache, medication-overuse headache. . 3rd edn.
British Association for the Study of Headache. www.bash.org.uk [Free Full-text]
Bates, D., Ashford, E., Dawson, R. et al. (1994) Subcutaneous sumatriptan during the migraine
aura. Sumatriptan Aura Study Group. Neurology 44(9), 1587-1592.

Baxter, K. and Preston, C.L. (Eds.) (2013) Stockley's drug interactions 2013: pocket companion.
London: Pharmaceutical Press.
BMA, NHS Employers and DH (2007) 2006/7 UK general practice workload survey. British
Medical Association. www.ic.nhs.uk [Free Full-text]
BNF 65 (2013) British National Formulary. 65th edn. London: British Medical Association and
Royal Pharmaceutical Society of Great Britain.
Brandes, J.L. (2006) The influence of estrogen on migraine: a systematic review. Journal of the
American Medical Association 295(15), 1824-1830. [Abstract] [Free Full-text]
Campbell, J.K., Penzien, D.B. and Wall, E.M. (2000) Evidence-based guidelines for migraine
headache: behavioral and physical treatments. American Academy of Neurology. www.aan.com
[Free Full-text]
Chen, L.C. and Ashcroft, D.M. (2007) Meta-analysis examining the efficacy and safety of
almotriptan in the acute treatment of migraine. Headache 47(8), 1169-1177. [Abstract]
Chen, L.C. and Ashcroft, D.M. (2008) Meta-analysis of the efficacy and safety of zolmitriptan in
the acute treatment of migraine. Headache 48(2), 236-247. [Abstract]
Colman, I., Brown, M.D., Innes, G.D. et al. (2004) Parenteral metoclopramide for acute migraine:
meta-analysis of randomised controlled trials. British Medical Journal 329(7479), 1369-1373.
[Abstract] [Free Full-text]
Colman, I., Friedman, B.W., Brown, M.D. et al. (2008) Parenteral dexamethasone for acute
severe migraine headache: meta-analysis of randomised controlled trials for preventing
recurrence. British Medical Journal 336(7657), 1359-1361. [Abstract] [Free Full-text]
CSM (2005) Updated advice on the safety of selective COX-2 inhibitors. Committee on Safety of
Medicines. www.mhra.gov.uk [Free Full-text]
Cutrer, F.M., Goadsby, P.J., Ferrari, M.D. et al. (2004) Priorities for triptan treatment attributes
and the implications for selecting an oral triptan for acute migraine: a study of US primary care
physicians (the TRIPSTAR project). Clinical Therapeutics 26(9), 1533-1545. [Abstract]
Damen, L., Bruijn, J.K., Verhagen, A.P. et al. (2005) Symptomatic treatment of migraine in
children: a systematic review of medication trials. Pediatrics 116(2), e295-e302. [Abstract] [Free
Full-text]
Derry, C.J., Derry, S. and Moore, R.A. (2012a) Sumatriptan (intranasal route of administration)
for acute migraine attacks in adults (Cochrane review). The Cochrane Library. Issue 2. John Wiley
& Sons, Ltd. www.thecochranelibrary.com [Free Full-text]

Derry, C.J., Derry, S. and Moore, R.A. (2012b) Sumatriptan (subcutaneous route of
administration) for acute migraine attacks in adults (Cochrane review). The Cochrane Library.
Issue 2. John Wiley & Sons, Ltd. www.thecochranelibrary.com [Free Full-text]
Derry, C.J., Derry, S. and Moore, R.A. (2012c) Sumatriptan (oral route of administration) for
acute migraine attacks in adults (Cochrane review). The Cochrane Library. Issue 2. John Wiley &
Sons, Ltd. www.thecochranelibrary.com [Free Full-text]
Derry, C.J., Derry, S. and Moore, R.A. (2012d) Sumatriptan (rectal route of administration) for
acute migraine attacks in adults (Cochrane review). The Cochrane Library. Issue 2. John Wiley &
Sons, Ltd. www.thecochranelibrary.com [Free Full-text]
Di Monda, V., Nicolodi, M., Aloisio, A. et al. (2003) Efficacy of a fixed combination of
indomethacin, prochlorperazine, and caffeine versus sumatriptan in acute treatment of multiple
migraine attacks: a multicenter, randomized, crossover trial. Headache 43(8), 835-844.
[Abstract]
Di Trapani, G., Mei, D., Marra, C. et al. (2000) Gabapentin in the prophylaxis of migraine: a
double-blind randomized placebo-controlled study. La Clinica Terapeutica 151(3), 145-148.
[Abstract]
Dowson, A.J., Lipscombe, S., Sender, J. et al. (2002) New guidelines for the management of
migraine in primary care. Current Medical Research and Opinion 18(7), 414-439. [Abstract]
DTB (2004) Managing migraine in children. Drug & Therapeutics Bulletin 42(4), 25-28. [Abstract]
DTB (2010) Acupuncture for tension-type headaches and migraine. Drugs and Therapeutics
Bulletin 48(6), 62-65. [Abstract]
Edelman, A., Gallo, M.F., Jensen, J.T. et al. (2005) Continuous or extended cycle vs. cyclic use of
combined hormonal contraceptives for contraception (Cochrane Review). The Cochrane Library.
Issue 3. John Wiley & Sons, Ltd. www.thecochranelibrary.com [Free Full-text]
EMA (2013) European Medicines Agency recommends changes to the use of metoclopramide.
European Medicines Agency. www.ema.europa.eu [Free Full-text]
Endres, H.G., Diener, H.C. and Molsberger, A. (2007) Role of acupuncture in the treatment of
migraine. Expert Review of Neurotherapeutics 7(9), 1121-1134. [Abstract]
Etminan, M., Takkouche, B., Isorna, F.C. and Samii, A. (2005) Risk of ischaemic stroke in people
with migraine: systematic review and meta-analysis of observational studies. British Medical
Journal 330(7482), 63. [Abstract] [Free Full-text]
European Medicines Agency (2014) CMDh confirms recommendations on restricting use of
domperidone-containing medicines. European Medicines Agency. www.ema.europa.eu [Free
Full-text]

Fenstermacher, N., Levin, M. and Ward, T. (2011) Pharmacological prevention of migraine. BMJ
342(), d583. [Erratum appears in BMJ (2011) 346, f507].
Ferrari, M.D., Roon, K.I., Lipton, R.B. and Goadsby, P.J. (2001) Oral triptans (serotonin 5HT(1B/1D) agonists) in acute migraine treatment: a meta-analysis of 53 trials. Lancet 358(9294),
1668-1675. [Abstract]
Ferrari, M.D., Goadsby, P.J., Roon, K.I. and Lipton, R.B. (2002) Triptans (serotonin, 5-HT1B/1D
agonists) in migraine: detailed results and methods of a meta-analysis of 53 trials. Cephalalgia
22(8), 633-658. [Abstract]
FSRH (2011) Combined hormonal contraception. Faculty of Sexual & Reproductive Health.
www.fsrh.org [Free Full-text]
Gawel, M.J., Worthington, I. and Maggisano, A. (2001) A systematic review of the use of triptans
in acute migraine. Canadian Journal of Neurological Sciences 28(1), 30-41. [Abstract]
Gerth, W.C., Carides, G.W., Dasbach, E.J. et al. (2001) The multinational impact of migraine
symptoms on healthcare utilisation and work loss. Pharmacoeconomics 19(2), 197-206.
[Abstract]
GHC (2011) Migraine and tension headache. Diagnosis and treatment guideline. Group Health
Cooperative. www.ghc.org [Free Full-text]
Giffin, N.J., Ruggiero, L., Lipton, R.B. et al. (2003) Premonitory symptoms in migraine: an
electronic diary study. Neurology 60(6), 935-940. [Abstract]
GMC (2013) Good practice in prescribing and managing medicines and devices. General Medical
Council. www.gmc-uk.org [Free Full-text]
Goadsby, P.J. (2005) Migraine pathophysiology. Headache 45(Suppl 1), S14-S24. [Abstract]
Goadsby, P.J., Lipton, R.B. and Ferrari, M.D. (2002) Migraine - current understanding and
treatment. New England Journal of Medicine 346(4), 257-270.
Headache Classification Committee of the International Headache Society (IHS) (2013) The
International Classification of Headache Disorders, 3rd edition (beta version). Cephalalgia 33(9),
629-808. [Free Full-text]
ICSI (2013) Health care guideline: diagnosis and treatment of headache. . 11th edn. Institute for
Clinical Systems Improvement. www.icsi.org [Free Full-text]
Jackson, J.L., Kuriyama, A. and Hayashino, Y. (2012) Botulinum toxin A for prophylactic
treatment of migraine and tension headaches in adults: a meta-analysis. JAMA 307(16), 17361745. [Abstract] [Free Full-text]
Kernick, D. (2008) Headache in children. GP Magazine 20 June(), 29-30.

Kurth, T., Kase, C.S., Schurks, M. et al. (2010) Migraine and risk of haemorrhagic stoke in women:
prospective cohort study. BMJ 341(), c3659. [Abstract] [Free Full-text]
LactMed (2007) Sumatriptan. National Library of Medicine. http://toxnet.nlm.nih.gov
Lampl, C., Voelker, M. and Diener, H.C. (2007) Efficacy and safety of 1, 000 mg effervescent
aspirin: individual patient data meta-analysis of three trials in migraine headache and migraine
accompanying symptoms. Journal of Neurology 254(6), 705-712. [Abstract]
Li, Y., Liang, F., Yang, X. et al. (2009) Acupuncture for treating acute attacks of migraine: a
randomized controlled trial. Headache 49(6), 805-816. [Abstract]
Linde, K. and Rossnagel, K. (2004) Propranolol for migraine prophylaxis (Cochrane Review). The
Cochrane Library. Issue 2. John Wiley & Sons, Ltd. www.thecochranelibrary.com [Free Full-text]
Linde, K., Allais, G., Brinkhaus, B. et al. (2009) Acupuncture for migraine prophylaxis (Cochrane
review). The Cochrane Library. Issue 1. John Wiley & Sons, Ltd. www.thecochranelibrary.com
[Free Full-text]
Lipton, R.B., Stewart, W.F., Ryan, R.E., Jr. et al. (1998) Efficacy and safety of acetaminophen,
aspirin, and caffeine in alleviating migraine headache pain: three double-blind, randomized,
placebo-controlled trials. Archives of Neurology 55(2), 210-217. [Abstract] [Free Full-text]
Lipton, R.B., Baggish, J.S., Stewart, W.F. et al. (2000) Efficacy and safety of acetaminophen in the
treatment of migraine: results of a randomized, double-blind, placebo-controlled, populationbased study. Archives of Internal Medicine 160(22), 3486-3492. [Abstract] [Free Full-text]
Lipton, R.B., Bigal, M.E., Diamond, M. et al. (2007) Migraine prevalence, disease burden, and the
need for preventive therapy (Cochrane review). Neurology 68(5), 343-349. [Abstract]
Lyngberg, A.C., Rasmussen, B.K., Jorgensen, T. and Jensen, R. (2005) Prognosis of migraine and
tension-type headache: a population-based follow-up study. Neurology 65(4), 580-585.
[Abstract]
MacGregor, E.A. (2012) Headache in pregnancy. Neurologic Clinics 30(3), 835-866. [Abstract]
MacGregor, E.A., Wilkinson, M. and Bancroft, K. (1993) Domperidone plus paracetamol in the
treatment of migraine. Cephalalgia 13(2), 124-127. [Abstract]
McCormack, P.L. and Keating, G.M. (2006) Eletriptan: a review of its use in the acute treatment
of migraine. Drugs 66(8), 1129-1149. [Abstract]
McNeil Products Ltd and Winthrop Pharmaceuticals UK Ltd (2011) Direct healthcare professional
communication on domperidone and cardiac safety. Medicines and Healthcare products
Regulatory Agency. www.mhra.gov.uk [Free Full-text]
MeReC (1997) Acute treatment of migraine: new products. MeReC Bulletin 8(10), 37-40.

MHRA (2011) Press release: more exact paracetamol dosing for children to be introduced.
Medicines and Healthcare products Regulatory Agency. www.mhra.gov.uk [Free Full-text]
MHRA (2013) Diclofenac: new contraindications and warnings after a Europe-wide review of
cardiovascular safety. Drug Safety Update 6(11), A2. [Free Full-text]
Modi, S. and Lowder, D.M. (2006) Medications for migraine prophylaxis. American Family
Physician 73(1), 72-78. [Abstract] [Free Full-text]
National Clinical Guideline Centre (2012) Headaches. Diagnosis and management of headaches
in young people and adults (full NICE guideline). . Clinical guideline 150. National Institute for
Health and Care Excellence. www.nice.org.uk [Free Full-text]
NHF (2008) Improving patient care in menstrual migraine. National Headache Foundation.
www.headaches.org [Free Full-text]
NICE (2008) Osteoarthritis. The care and management of osteoarthritis in adults (NICE guideline)
[Replaced by CG177]. . Clinical guideline 59. National Institute for Health and Care Excellence.
www.nice.org.uk [Free Full-text]
NICE (2009a) Rheumatoid arthritis: the management of rheumatoid arthritis (NICE guideline). .
Clinical guideline 79. National Institute for Health and Care Excellence. www.nice.org.uk [Free
Full-text]
NICE (2009b) Low back pain: early management of persistent non-specific low back pain (NICE
guideline). . Clinical guideline 88. National Institute for Health and Care Excellence.
www.nice.org.uk [Free Full-text]
NICE (2009c) Depression in adults with a chronic physical health problem. Treatment and
management. (NICE guideline). . Clinical guideline 91. National Institute for Health and Care
Excellence. www.nice.org.uk [Free Full-text]
NICE (2012) Headaches. Diagnosis and management of headaches in young people and adults
(NICE guideline). . Clinical guideline 150. National Institute for Health and Care Excellence.
www.nice.org.uk [Free Full-text]
NICE (2013) Key therapeutic topics - medicines management options for local implementation.
National Institute for Health and Care Excellence. www.nice.org.uk [Free Full-text]
NPC (2011) Key therapeutic topics 2010/11 - Medicines management options for local
implementation. National Prescribing Centre. www.npc.nhs.uk [Free Full-text]
NPIS (2010a) Use of sumatriptan in pregnancy. Health Protection AgencyUK teratology
information service. www.toxbase.org

NPIS (2010b) Treatment of migraine in pregnancy. ToxbaseUK Teratology Service.


www.toxbase.org
NPIS (2010c) Use of diclofenac in pregnancy. UK teratology information serviceHPA.
www.toxbase.org
NPIS (2011) Use of naratriptan in pregnancy. HPAUk teratology information service.
www.toxbase.org
NTIS (2008) Use of aspirin in pregnancy. National Teratology Information Service.
www.toxbase.org
Oldman, A.D., Smith, L.A., McQuay, H.J. and Moore, R.A. (2002) Pharmacological treatments for
acute migraine: quantitative systematic review. Pain 97(3), 247-257. [Abstract]
Olesen, J., Diener, H.C., Schoenen, J. and Hettiarachchi, J. (2004) No effect of eletriptan
administration during the aura phase of migraine. European Journal of Neurology 11(10), 671677. [Abstract]
Peikert, A., Wilimzig, C. and Kohne-Volland, R. (1996) Prophylaxis of migraine with oral
magnesium: results from a prospective, mulit-centre, placebo-controlled and double-blind
randomized study. Cephalalgia 16(4), 257-263. [Abstract]
Pfaffenrath, V. and Scherzer, S. (1995) Analgesics and NSAIDs in the treatment of the acute
migraine attack. Cephalalgia 15(Suppl 15), 14-20. [Abstract]
Poolsup, N., Leelasangaluk, V., Jittangtrong, J. et al. (2005) Efficacy and tolerability of
frovatriptan in acute migraine treatment: systematic review of randomized controlled trials.
Journal of Clinical Pharmacy and Therapeutics 30(6), 521-532. [Abstract]
Pringsheim, T., Davenport, W.J. and Becker, W.J. (2010) Prophylaxis of migraine headache. CMAJ
182(7), E269-E276. [Free Full-text]
RPSGB (2006) Practice guidance: OTC sumatriptan. Royal Pharmaceutical Society of Great
Britain. www.rpharms.com
Sandrini, G., Cerbo, R., Del Bene, E. et al. (2007) Efficacy of dosing and re-dosing of two oral fixed
combinations of indomethacin, prochlorperazine and caffeine compared with oral sumatriptan
in the acute treatment of multiple migraine attacks: a double-blind, double-dummy,
randomised, parallel group, multicentre study. International Journal of Clinical Practice 61(8),
1256-1269. [Abstract] [Free Full-text]
Schaefer, C., Peters, P. and Miller, R.K. (Eds.) (2007) Drugs during pregnancy and lactation:
treatment options and risk assessment. 2nd edn. Oxford: Academic Press.

Schoenen, J., Jacquy, J. and Lenaerts, M. (1998) Effectiveness of high-dose riboflavin in migraine
prophylaxis. A randomized controlled trial. Neurology 50(2), 466-470. [Abstract]
Scholpp, J., Schellenberg, R., Moeckesch, B. and Banik, N. (2004) Early treatment of a migraine
attack while pain is still mild increases the efficacy of sumatriptan. Cephalalgia 24(11), 925-933.
[Abstract]
Shamliyan, T.A., Choi, J.Y., Ramakrishnan, R. et al. (2013) Preventive pharmacologic treatments
for episodic migraine in adults. Journal of General Internal Medicine 28(9), 1225-1237.
[Abstract]
Sharma, S., Prasad, A., Nehru, R. et al. (2002) Efficacy and tolerability of prochlorperazine buccal
tablets in treatment of acute migraine. Headache 42(9), 896-902. [Abstract]
SIGN (2008) Diagnosis and management of headache in adults. Scottish Intercollegiate
Guidelines Network. www.sign.ac.uk [Free Full-text]
Silberstein, S.D., Lipton, R.B. and Goadsby, P.J. (2002) Headache in clinical practice. 2nd edn.
London: Martin Dunitz.
Silberstein, S.D., Elkind, A.H., Schreiber, C. and Keywood, C. (2004) A randomized trial of
frovatriptan for the intermittent prevention of menstrual migraine. Neurology 63(2), 261-269.
[Abstract]
Silver, S., Gano, D. and Gerretsen, P. (2008) Acute treatment of paediatric migraine: a metaanalysis of efficacy. Journal of Paediatrics and Child Health 44(1-2), 3-9. [Abstract]
Smetana, G.W. (2000) The diagnostic value of historical features in primary headache
syndromes: a comprehensive review. Archives of Internal Medicine 160(18), 2729-2737.
[Abstract] [Free Full-text]
Snow, V., Weiss, K., Wall, E.M. et al. (2002) Pharmacologic management of acute attacks of
migraine and prevention of migraine headache. Annals of Internal Medicine 137(10), 840-849.
[Free Full-text]
Spector, J.T., Kahn, S.R., Jones, M.R. et al. (2010) Migraine headache and ischemic stroke risk: an
updated meta-analysis. American Journal of Medicine 123(7), 612-624. [Abstract] [Free Full-text]
Stewart, W.F., Staffa, J., Lipton, R.B. and Ottman, R. (1997) Familial risk of migraine: a
population-based study. Annals of Neurology 41(2), 166-172. [Abstract]
Sun, Y. and Gan, T.J. (2008) Acupuncture for the management of chronic headache: a systematic
review. Anesthesia and Analgesia 107(6), 2038-2047. [Abstract] [Free Full-text]

Suthisisang, C., Poolsup, N., Kittikulsuth, W. et al. (2007) Efficacy of low-dose ibuprofen in acute
migraine treatment: systematic review and meta-analysis. Annals of Pharmacotherapy 41(11),
1782-1791. [Abstract]
Tfelt-Hansen, P. (2006) A review of evidence-based medicine and meta-analytic reviews in
migraine. Cephalalgia 26(11), 1265-1274. [Abstract]
Tfelt-Hansen, P., De Vries, P. and Saxena, P.R. (2000) Triptans in migraine: a comparative review
of pharmacology, pharmacokinetics and efficacy. Drugs 60(6), 1259-1287. [Abstract]
UKMi (2012) What is the preferred triptan for the treatment of migraine in breastfeeding
mothers? UK Medicines Information. www.evidence.nhs.uk
UKTIS (2012) Use of ibuprofen in pregnancy. TOXBASEUK Teratology Information Service.
www.toxbase.org
Vickers, A.J., Rees, R.W., Zollman, C.E. et al. (2004) Acupuncture of chronic headache disorders
in primary care: randomised controlled trial and economic analysis. Health Technology
Assessment 8(48), . [Abstract] [Free Full-text]
Wenzel, R.G., Sarvis, C.A. and Krause, M.L. (2003) Over-the-counter drugs for acute migraine
attacks: literature review and recommendations. Pharmacotherapy 23(4), 494-505. [Abstract]

Question 7 of 80

Which one of the following is the causative agent of roseola infantum?


Human herpes virus 2

Parvovirus B19

Human herpes virus 6

Human herpes virus 8

Cytomegalovirus

Next question

Roseola infantum
Roseola infantum (also known as exanthem subitum, occasionally sixth disease) is a common disease of infancy caused by the human
herpes virus 6 (HHV6). It has an incubation period of 5-15 days and typically affects children aged 6 months to 2 years.

Features

high fever: lasting a few days, followed by a


maculopapular rash
febrile convulsions occur in around 10-15%
diarrhoea and cough are also commonly seen

Other possible consequences of HHV6 infection

aseptic meningitis
hepatitis

Question 8 of 80

Which one of the following is not a risk factor for developmental dysplasia of the hip?
Positive family history

Breech presentation

Female sex

Afro-Caribbean origin

Firstborn child

Next question

Developmental dysplasia of the hip


Developmental dysplasia of the hip (DDH) is gradually replacing the old term 'congenital dislocation of the hip' (CDH). It affects around 13% of newborns.
Risk factors

female sex: 6 times greater risk


breech presentation
positive family history
firstborn children
oligohydramnios
birth weight > 5 kg
congenital calcaneovalgus foot deformity

DDH is slightly more common in the left hip. Around 20% of cases are bilateral.
Clinical examination is made using the Barlow and Ortolani tests:

Barlow test: attempts to dislocate an articulated femoral head


Ortolani test: attempts to relocate a dislocated femoral head

Ultrasound is used to confirm the diagnosis if clinically suspected


Management

most unstable hips will spontaneously stabilise by 3-6 weeks of age


Pavlik harness (flexion-abduction orthosis) in children younger than 4-5 months
older children may require surgery

Question 9 of 80

What percentage of male term infants are found to have an undescended testicle on newborn examination?
0.1-0.3%

0.5-1%

2-4%

6-8%

10-14%

Next question

Undescended testis occurs in around 2-3% of term male infants

Undescended testis
Undescended testis occurs in around 2-4% of term male infants., but is much more common if the baby is preterm. Around 25% of cases
are bilateral
Complications of undescended testis

infertility
torsion
testicular cancer
psychological

Management

orchidopexy: referral should be considered from around 6 months of age. Surgical practices vary although the majority of
procedures are performed at around 1 year of age

Question 10 of 80

You are reviewing a 11-month-old baby with a viral upper respiratory tract infection. She is clinically well but at the end of the consultation
her mother asks you about her development. You notice that she points and babbles 'mama' and 'dada' but has no other words. She is shy
and cries when you try to examine her. There is an early pincer grip and she can roll from front to back but she cannot yet sit without
support. How would you describe her development?
Normal development

Global developmental delay

Isolated delay in gross motor skills

Delay in speech + social skills, possibly early autism

Isolated delay in fine motor skills

Next question

Most babies can sit without support at 7-8 months so this probably represents a delay in gross motor skills. If still present at 12 months she
should be considered for referral to a paediatrician. The other development features are normal for her age.

Developmental milestones: gross motor


The table below summarises the major gross motor developmental milestones
Age

Milestone

3 months

Little or no head lag on being pulled to sit


Lying on abdomen, good head control
Held sitting, lumbar curve

6 months

Lying on abdomen, arms extended


Lying on back, lifts and grasps feet
Pulls self to sitting
Held sitting, back straight
Rolls front to back

7-8 months

Sits without support (Refer at 12 months)

9 months

Pulls to standing
Crawls

12 months

Cruises
Walks with one hand held

13-15 months Walks unsupported (Refer at 18 months)


18 months

Squats to pick up a toy

2 years

Runs
Walks upstairs and downstairs holding on to rail

3 years

Rides a tricycle using pedals


Walks up stairs without holding on to rail

4 years

Hops on one leg

Notes

the majority of children crawl on all fours before walking but some children 'bottom-shuffle'. This is a normal variant and runs in
families

Question 10 of 80

You are reviewing a 11-month-old baby with a viral upper respiratory tract infection. She is clinically well but at the end of the consultation
her mother asks you about her development. You notice that she points and babbles 'mama' and 'dada' but has no other words. She is shy
and cries when you try to examine her. There is an early pincer grip and she can roll from front to back but she cannot yet sit without
support. How would you describe her development?
Normal development

Global developmental delay

Isolated delay in gross motor skills

Delay in speech + social skills, possibly early autism

Isolated delay in fine motor skills

Next question

Most babies can sit without support at 7-8 months so this probably represents a delay in gross motor skills. If still present at 12 months she
should be considered for referral to a paediatrician. The other development features are normal for her age.

Developmental milestones: gross motor


The table below summarises the major gross motor developmental milestones
Age

Milestone

3 months

Little or no head lag on being pulled to sit


Lying on abdomen, good head control
Held sitting, lumbar curve

6 months

Lying on abdomen, arms extended


Lying on back, lifts and grasps feet
Pulls self to sitting
Held sitting, back straight
Rolls front to back

7-8 months

Sits without support (Refer at 12 months)

9 months

Pulls to standing
Crawls

12 months

Cruises
Walks with one hand held

13-15 months Walks unsupported (Refer at 18 months)


18 months

Squats to pick up a toy

2 years

Runs
Walks upstairs and downstairs holding on to rail

Age

Milestone

3 years

Rides a tricycle using pedals


Walks up stairs without holding on to rail

4 years

Hops on one leg

Notes

the majority of children crawl on all fours before walking but some children 'bottom-shuffle'. This is a normal variant and runs in
families
Question 11 of 80

A newborn infant is noted to have posterior displacement of the tongue and a clef palate. What is the most likely diagnosis?

Pierre-Robin syndrome

Patau syndrome

Edward's syndrome

Noonan syndrome

William's syndrome

Childhood syndromes

Below is a list of common features of selected childhood syndromes


Syndrome

Key features

Patau syndrome (trisomy 13)

Microcephalic, small eyes


Cleft lip/palate
Polydactyly
Scalp lesions

Micrognathia
Low-set ears
Edward's syndrome (trisomy 18)
Rocker bottom feet
Overlapping of fingers

Fragile X

Learning difficulties
Macrocephaly
Long face
Large ears
Macro-orchidism

Noonan syndrome

Webbed neck
Pectus excavatum
Short stature
Pulmonary stenosis

Next question

Syndrome

Key features

Pierre-Robin syndrome*

Micrognathia
Posterior displacement of the tongue (may result in upper airway obstruction)
Cleft palate

Prader-Willi syndrome

Hypotonia
Hypogonadism
Obesity

William's syndrome

Short stature
Learning difficulties
Friendly, extrovert personality
Transient neonatal hypercalcaemia
Supravalvular aortic stenosis

*this condition has many similarities with Treacher-Collins syndrome. One of the key differences is that Treacher-Collins syndrome
is autosomal dominant so there is usually a family history of similar problems
Question 12 of 80

An oral rotavirus vaccine has recently been introduced into the NHS immunisation schedule. When should it be given?
2 months + 4 months

2 months + 3 months

3 months + 12-13 months

3 + 4 months

4 months + 12-13 months

Next question

The oral rotavirus vaccine is given at 2 months + 3 months

Rotavirus vaccine
Rotavirus is a major public health problem, accounting for significant morbidity and hospital admissions in the developed world and
childhood mortality in the developing world.
A vaccine was introduced into the NHS immunisation programme in 2013. The key points to remember as as follows:

it is an oral, live attenuated vaccine


2 doses are required, the first at 2 months, the second at 3 months
the first dose should not be given after 14 weeks + 6 days and the second dose cannot be given after 23 weeks + 6 days due to
a theoretical risk of intussusception

Other points

the vaccine is around 85-90% effective and is predicted to decrease hospitalisation by 70%

offers long-term protection against rotavirus

Question 13 of 80

A 9-year-old girl is brought to surgery due to persistent leg pains. Which one of the following would not be consistent with a diagnosis of
'growing pains'?
Present upon waking in the morning

Worse after a day of vigorous activity

Mainly affecting the shins and ankles

Bilateral symptoms

Intermittent symptoms

Next question

Growing pains
A common presentation in General Practice is a child complaining of pain in the legs with no obvious cause. Such presentations, in the
absence of any worrying features, are often attributed to 'growing pains'. This is a misnomer as the pains are often not related to growth the current term used in rheumatology is 'benign idiopathic nocturnal limb pains of childhood'
Growing pains are equally common in boys and girls and occur in the age range of 3-12 years.
Features of growing pains

never present at the start of the day after the child has woken
no limp
no limitation of physical activity
systemically well
normal physical examination
motor milestones normal
symptoms are often intermittent and worse after a day of vigorous activity

Question 13 of 80

A 9-year-old girl is brought to surgery due to persistent leg pains. Which one of the following would not be consistent with a diagnosis of
'growing pains'?
Present upon waking in the morning

Worse after a day of vigorous activity

Mainly affecting the shins and ankles

Bilateral symptoms

Intermittent symptoms

Next question

Growing pains
A common presentation in General Practice is a child complaining of pain in the legs with no obvious cause. Such presentations, in the
absence of any worrying features, are often attributed to 'growing pains'. This is a misnomer as the pains are often not related to growth the current term used in rheumatology is 'benign idiopathic nocturnal limb pains of childhood'
Growing pains are equally common in boys and girls and occur in the age range of 3-12 years.
Features of growing pains

never present at the start of the day after the child has woken
no limp
no limitation of physical activity
systemically well
normal physical examination
motor milestones normal
symptoms are often intermittent and worse after a day of vigorous activity
Question 14 of 80

A 9-year-old boy is brought to surgery as his asthma has been getting worse over the past 2 days. His mother is concerned that
his breathing is getting worse and not responding to inhaled salbutamol as normal. Given the overall clinical context, which one of
the following is most consistent with a life-threatening asthma attack in this particular patient?

Quiet breath sounds on auscultation

SpO2 of 94%

Heart rate of 120 bpm

Respiratory rate of 30 / minute

Peak flow 40% of predicted

Next question

Quiet breath sounds in a child with an asthma exacerbation is a worrying feature. Children with asthma normally have an obvious
bilateral wheeze - the absence of this may suggest a life-threatening asthma attack.

Asthma in children: assessment of acute attacks

The 2014 BTS/SIGN guidelines suggest the following criteria are used to assess the severity of asthma in general practice:
Children between 2 and 5 years of age

Moderate attack

Severe attack

Life-threatening attack

SpO2 <92%
SpO2 < 92%
Silent chest
Too breathless to talk or feed
SpO2 > 92%
Poor respiratory effort
Heart rate > 140/min
No clinical features of severe asthma
Agitation
Respiratory rate > 40/min
Altered consciousness
Use of accessory neck muscles
Cyanosis

Children greater than 5 years of age


Attempt to measure PEF in all children aged > 5 years.
Moderate attack

Severe attack

Life-threatening attack

SpO2 < 92%


SpO2 < 92%
SpO2 > 92%
PEF 33-50% best or predicted
PEF < 33% best or predicted
PEF > 50% best or predicted Can't complete sentences in one breath or too breathless to talk or feed Silent chest
No clinical features of
Heart rate > 125/min
Poor respiratory effort
severe asthma
Respiratory rate > 30/min
Altered consciousness
Use of accessory neck muscles
Cyanosis
Question 15 of 80

How many doses of tetanus vaccine should a child receive as part of the routine UK immunisation schedule?
None

3 with an optional 4th dose

Next question

Tetanus: vaccination
The tetanus vaccine is a cell-free purified toxin that is normally given as part of a combined vaccine.
Tetanus vaccine is currently given in the UK as part of the routine immunisation schedule at:

2 months
3 months
4 months
3-5 years
13-18 years

This therefore provides 5 doses of tetanus-containing vaccine. Five doses is now considered to provide adequate long-term protection
against tetanus.

Intramuscular human tetanus immunoglobulin should be given to patients with high-risk wounds (e.g. Compound fractures, delayed surgical
intervention, significant degree of devitalised tissue) irrespective of whether 5 doses of tetanus vaccine have previously been given
If vaccination history is incomplete or unknown then a dose of tetanus vaccine should be given combined with intramuscular human tetanus
immunoglobulin for high-risk wounds

Question 16 of 80

Which one of the following conditions is not associated with obesity in children?
Down's syndrome

Hypothyroidism

Cushing's syndrome

Prader-Willi syndrome

Growth hormone excess

Next question

Obesity in children
Defining obesity is more difficult in children than adults as body mass index (BMI) varies with age. BMI percentile charts are therefore
needed to make an accurate assessment. Recent NICE guidelines suggest to use 'UK 1990 BMI charts to give age- and gender-specific
information'
NICE recommend

consider tailored clinical intervention if BMI at 91st centile or above.


consider assessing for comorbidities if BMI at 98th centile or above

By far the most common cause of obesity in childhood is lifestyle factors. Other associations of obesity in children include:

Asian children: four times more likely to be obese than white children
female children
taller children: children with obesity are often above the 50th percentile in height

Cause of obesity in children

growth hormone deficiency


hypothyroidism
Down's syndrome

Cushing's syndrome
Prader-Willi syndrome

Consequences of obesity in children

orthopaedic problems: slipped upper femoral epiphyses, Blount's disease (a development abnormality of the tibia resulting in
bowing of the legs), musculoskeletal pains
psychological consequences: poor self-esteem, bullying
sleep apnoea
benign intracranial hypertension
long-term consequences: increased incidence of type 2 diabetes mellitus, hypertension and ischaemic heart disease

Question 17 of 80

What is the average age that puberty starts in boys?


10 years

11 years

12 years

13 years

14 years

Next question

Puberty
Males

first sign is testicular growth at around 12 years of age (range = 10-15 years)
testicular volume > 4 ml indicates onset of puberty
maximum height spurt at 14

Females

first sign is breast development at around 11.5 years of age (range = 9-13 years)
height spurt reaches its maximum early in puberty (at 12) , before menarche
menarche at 13 (11-15)
there is an increase of only about 4% of height following menarche

Normal changes in puberty

gynaecomastia may develop in boys

asymmetrical breast growth may occur in girls


diffuse enlargement of the thyroid gland may be seen

Question 18 of 80

A 3-year-old boy from a Turkish family is referred to the local paediatric unit due to recurrent lethargy and pallor. His parents report no other
symptoms such as fever, pain or poor feeding. He had been treated with a course of ciprofloxacin for otitis externa two weeks ago.
Admission bloods show:

Hb

5.2 g/dl

WBC

10.7 *109/l

Platelets

346 *109/l

Reticulocytes

5%

What is the most likely underlying diagnosis?


Sickle cell disease

Beta-thalassaemia major

Acute lymphoblastic leukaemia

Pyruvate kinase deficiency

Glucose-6-phoshate dehydrogenase deficiency

Next question

Ciprofloxacin is a common cause of haemolysis in patients with glucose-6-phoshate dehydrogenase deficiency

G6PD deficiency
Glucose-6-phosphate dehydrogenase (G6PD) deficiency is the commonest red blood cell enzyme defect. It is more common in people from
the Mediterranean and Africa and is inherited in a X-linked recessive fashion. Many drugs can precipitate a crisis as well as infections and
broad (fava) beans
Pathophysiology

G6PD glutathione increased red cell susceptibility to oxidative stress

Features

neonatal jaundice is often seen


intravascular haemolysis
gallstones are common

splenomegaly may be present


Heinz bodies on blood films

Diagnosis is made by using a G6PD enzyme assay


Some drugs causing haemolysis

anti-malarials: primaquine
ciprofloxacin
sulph- group drugs: sulphonamides, sulphasalazine, sulfonylureas

Some drugs thought to be safe

penicillins
cephalosporins
macrolides
tetracyclines
trimethoprim

Comparing G6PD deficiency to hereditary spherocytosis


G6PD deficiency

Hereditary spherocytosis

Gender

Male (X-linked recessive)

Male + female (autosomal dominant)

Ethnicity

African + Mediterranean descent

Northern European descent

Neonatal jaundice
Neonatal jaundice
Chronic symptoms although haemolytic crises may be precipitated by infection
Typical history Infection/drugs precipitate haemolysis
Gallstones
Gallstones
Splenomegaly is common
Blood film

Heinz bodies

Diagnostic testMeasure enzyme activity of G6PD

Spherocytes (round, lack of central pallor)


Osmotic fragility test

Question 19 of 80

Which one of the following statements regarding absence seizures is incorrect?


Typical age of onset of 3-10 years old

Sodium valproate and ethosuximide are first-line treatments

Seizures may be provoked by a child holding their breath

There is a good prognosis

The EEG characteristically shows a bilateral, symmetrical 3Hz spike and wave pattern

Next question

Seizures are characteristically provoked by hyperventilation

Absence seizures
Absence seizures (petit mal) are a form of generalised epilepsy that is mostly seen in children. The typical age of onset of 3-10 years old
and girls are affected twice as commonly as boys
Features

absences last a few seconds and are associated with a quick recovery
seizures may be provoked by hyperventilation or stress
the child is usually unaware of the seizure
they may occur many times a day
EEG: bilateral, symmetrical 3Hz spike and wave pattern

Management

sodium valproate and ethosuximide are first-line treatment


good prognosis - 90-95% become seizure free in adolescence

Question 20 of 80

Which one of the following is least associated with fetal varicella syndrome?
Limb hypoplasia

Microphthalmia

Sensorineural deafness

Skin scarring

Microcephaly

Next question

Sensorineural deafness is more characteristic of congenital rubella infection

Chickenpox exposure in pregnancy


Chickenpox is caused by primary infection with varicella zoster virus. Shingles is reactivation of dormant virus in dorsal root ganglion. In
pregnancy there is a risk to both the mother and also the fetus, a syndrome now termed fetal varicella syndrome
Fetal varicella syndrome (FVS)

risk of FVS following maternal varicella exposure is around 1% if occurs before 20 weeks gestation
studies have shown a very small number of cases occurring between 20-28 weeks gestation and none following 28 weeks
features of FVS include skin scarring, eye defects (microphthalmia), limb hypoplasia, microcephaly and learning disabilities

Management of chickenpox exposure

if there is any doubt about the mother previously having chickenpox maternal blood should be checked for varicella antibodies
if the pregnant women is not immune to varicella she should be given varicella zoster immunoglobulin (VZIG) as soon as
possible. RCOG and Greenbook guidelines suggest VZIG is effective up to 10 days post exposure
consensus guidelines suggest oral aciclovir should be given if pregnant women with chickenpox present within 24 hours of
onset of the rash
Question 21 of 80

Which one of the following is not part of the core Child Health Promotion Program as outlined in the Children's National Service
Framework?

GP examination at 6-8 weeks

Newborn clinical examination

Heel-prick test at 5-9 days

8-9 month surveillance review

Newborn hearing check

Next question

The routine surveillance reviews at 8 months, 2 years and 3-4 years have now being stopped. However, if a child is deemed 'at
risk' more frequent reviews are advisable

Child health surveillance

The following table gives a basic outline of child health surveillance in the UK

Antenatal

Ensure intrauterine growth


Check for maternal infections e.g. HIV
Ultrasound scan for fetal abnormalities
Blood tests for Neural Tube Defects

Newborn

Clinical examination of newborn


Newborn Hearing Screening Programme e.g. oto-acoustic emissions test
Give mother Personal Child Health Record

First month

Heel-prick test day 5-9 - hypothyroidism, PKU, metabolic diseases, cystic fibrosis, medium-chain acyl Co-A
dehydrogenase deficiency (MCADD)
Midwife visit up to 4 weeks*

Following
months

Health visitor input


GP examination at 6-8 weeks
Routine immunisations

Pre school

National orthoptist-led programme for pre-school vision screening to be introduced

Ongoing

Monitoring of growth, vision, hearing


Health professionals advice on immunisations, diet, accident prevention

*this doesn't seem to happen in practice with health visitors usually taking over at 2 weeks

Question 22 of 80

Which one of the following is not a risk factor for the development of glue ear?
Day care attendance

Parental smoking

Female sex

Bottle feeding

Family history

Next question

Glue ear
Glue ear describes otitis media with an effusion (other terms include serous otitis media). It is common with the majority of children having
at least one episode during childhood
Risk factors

male sex
siblings with glue ear
higher incidence in Winter and Spring
bottle feeding
day care attendance
parental smoking

Features

peaks at 2 years of age


hearing loss is usually the presenting feature (glue ear is the commonest cause of conductive hearing loss and elective surgery
in childhood)
secondary problems such as speech and language delay, behavioural or balance problems may also be seen

Treatment options include:

grommet insertion - to allow air to pass through into the middle ear and hence do the job normally done by the Eustachian tube.
The majority stop functioning after about 10 months
adenoidectomy

Question 23 of 80

Which one of the following statements regarding chickenpox in children is incorrect?

Systemic upset is usually mild

Incubation period is 10-21 days

Children are infectious once rash begins until all lesions have scabbed over

Can be caught from someone with shingles

Spread by the respiratory route

Next question

Chickenpox
Chickenpox is caused by primary infection with varicella zoster virus. Shingles is reactivation of dormant virus in dorsal root ganglion
Chickenpox is highly infectious

spread via the respiratory route


can be caught from someone with shingles
infectivity = 4 days before rash, until 5 days after the rash first appeared*
incubation period = 10-21 days

Clinical features (tend to be more severe in older children/adults)

fever initially
itchy, rash starting on head/trunk before spreading. Initially macular then papular then vesicular
systemic upset is usually mild

Management is supportive

keep cool, trim nails


calamine lotion
school exclusion: current HPA advice is 5 days from start of skin eruption. They also state 'Traditionally children have been
excluded until all lesions are crusted. However, transmission has never been reported beyond the fifth day of the rash.'
immunocompromised patients and newborns with peripartum exposure should receive varicella zoster immunoglobulin (VZIG).
If chickenpox develops then IV aciclovir should be considered

A common complication is secondary bacterial infection of the lesions. Rare complications include

pneumonia
encephalitis (cerebellar involvement may be seen)
disseminated haemorrhagic chickenpox
arthritis, nephritis and pancreatitis may very rarely be seen

*it was traditionally taught that patients were infective until all lesions had scabbed over

Question 24 of 80

A 9-month-old girl is generally unwell with a low-grade pyrexia, oral ulcers and the following appearance of her feet:

Image used on license from DermNet NZ

Which one of the following is most likely to be responsible for this presentation?
Measles

Human herpesvirus 8 (HHV-8)

Parvovirus B19

Vasculitis

Coxsackie A16

Next question

Hand, foot and mouth disease


Hand, foot and mouth disease is a self-limiting condition affecting children. It is caused by the intestinal viruses of the Picornaviridae family
(most commonly coxsackie A16 and enterovirus 71). It is very contagious and typically occurs in outbreaks at nursery
Clinical features

mild systemic upset: sore throat, fever


oral ulcers
followed later by vesicles on the palms and soles of the feet

Image used on license from DermNet NZ

Image used on license from DermNet NZ

Management

general advice about hydration and analgesia

reassurance no link to disease in cattle


children do not need to be excluded from school*

*The HPA recommends that children who are unwell should be kept off school until they feel better. They also advise that you contact them
if you suspect that there may be a large outbreak.

Question 25 of 80

Which of the following is a live attenuated vaccine?


Pneumococcus

Meningococcus

Rabies

Oral polio

Diphtheria

Next question

Live attenuated vaccines

BCG
MMR
oral polio
yellow fever
oral typhoid

Vaccinations
It is important to be aware of vaccines which are of the live-attenuated type as these may pose a risk to immunocompromised patients. The
main types of vaccine are as follows:
Live attenuated

BCG
measles, mumps, rubella (MMR)
influenza (intranasal)
oral rotavirus
oral polio
yellow fever
oral typhoid*

Inactivated preparations

rabies
influenza (intramuscular)

Detoxified exotoxins

tetanus

Extracts of the organism/virus (sometimes termed fragment)**

diphtheria
pertussis ('acellular' vaccine)
hepatitis B
meningococcus, pneumococcus, haemophilus

Notes

influenza: different types are available, including whole inactivated virus, split virion (virus particles disrupted by detergent
treatment) and sub-unit (mainly haemagglutinin and neuraminidase)
cholera: contains inactivated Inaba and Ogawa strains of Vibrio cholerae together with recombinant B-subunit of the cholera
toxin
hepatitis B: contains HBsAg adsorbed onto aluminium hydroxide adjuvant and is prepared from yeast cells using recombinant
DNA technology

*whole cell typhoid vaccine is no longer used in the UK


**may also be produced using recombinant DNA technology
Question 26 of 80

Which one of the following is the most likely result if a fetus is homozygous for alpha-thalassaemia?
Anencephalic fetus

Normal pregnancy

Prematurity

Hydrops fetalis

Macrosomia

Next question

Alpha-thalassaemia
Alpha-thalassaemia is due to a deficiency of alpha chains in haemoglobin
Overview

2 separate alpha-globulin genes are located on each chromosome 16

Clinical severity depends on the number of alpha chains present


If 1 or 2 alpha chains are absent then the blood picture would be hypochromic and microcytic, but the Hb level would be typically normal
Loss of 3 alpha chains results in a hypochromic microcytic anaemia with splenomegaly. This is known as Hb H disease
If all 4 alpha chains absent (i.e. homozygote) then death in utero (hydrops fetalis, Bart's hydrops

Question 27 of 80

Which one of the following is associated with tall stature?


Noonan syndrome

Ehler-Danlos syndrome

Maternal diabetes mellitus

Homocystinuria

Kallman's syndrome

Next question

Tall stature
Causes of tall stature

constitutional
excessive growth hormone (pituitary gigantism)
hyperthyroidism
Marfan syndrome
homocystinuria
Klinefelter's syndrome

Congenital adrenal hyperplasia may be associated with a premature growth spurt but early fusion of the epiphyses results in short stature

Question 28 of 80

The parents of a 9-year-old child come for advice regarding malaria prophylaxis. They are of Indian origin and are due to go back to their
own country for a wedding. Which one of the following is contraindicated?

Mefloquine

Doxycycline

Atovaquone + proguanil (Malarone)

Proguanil + chloroquine

Topical diethyltoluamide (DEET)

Next question

Malaria: prophylaxis
There are around 1,500-2,000 cases each year of malaria in patients returning from endemic countries. The majority of these cases
(around 75%) are caused by the potentially fatal Plasmodium falciparum protozoa. The majority of patients who develop malaria did not
take prophylaxis. It should also be remembered that UK citizens who originate from malaria endemic areas quickly lose their innate
immunity.
Up-to-date charts with recommended regimes for malarial zones should be consulted prior to prescribing
Drug

Side-effects + notes

Atovaquone + proguanil (Malarone) GI upset

Time to begin before travelTime to end after travel


1 - 2 days

7 days

1 week

4 weeks

1 - 2 days

4 weeks

Headache
Chloroquine

Contraindicated in epilepsy
Taken weekly

Doxycycline

Photosensitivity
Oesophagitis

Mefloquine (Lariam)

Dizziness
Neuropsychiatric disturbance
2 - 3 weeks
Contraindicated in epilepsy
Taken weekly

Proguanil (Paludrine)
Proguanil + chloroquine

See above

4 weeks

1 week

4 weeks

1 week

4 weeks

Pregnant women should be advised to avoid travelling to regions where malaria is endemic. Diagnosis can also be difficult as parasites
may not be detectable in the blood film due to placental sequestration. However, if travel cannot be avoided:

chloroquine can be taken


proguanil: folate supplementation (5mg od) should be given
Malarone (atovaquone + proguanil): the BNF advises to avoid these drugs unless essential. If taken then folate supplementation
should be given
mefloquine: caution advised
doxycycline is contraindicated

It is again advisable to avoid travel to malaria endemic regions with children if avoidable. However, if travel is essential then children should
take malarial prophylaxis as they are more at risk of serious complications.

diethyltoluamide (DEET) 20-50% can be used in children over 2 months of age


doxycycline is only licensed in the UK for children over the age of 12 years

Question 29 of 80

A newborn female baby is noted to have a clicky left hip during the routine newborn examination. What is the most appropriate
investigation?
X-ray

Urine dipstick

Ultrasound

Serum bone profile

MRI

Next question

Developmental dysplasia of the hip


Developmental dysplasia of the hip (DDH) is gradually replacing the old term 'congenital dislocation of the hip' (CDH). It affects around 13% of newborns.
Risk factors

female sex: 6 times greater risk


breech presentation
positive family history
firstborn children
oligohydramnios
birth weight > 5 kg
congenital calcaneovalgus foot deformity

DDH is slightly more common in the left hip. Around 20% of cases are bilateral.
Clinical examination is made using the Barlow and Ortolani tests:

Barlow test: attempts to dislocate an articulated femoral head


Ortolani test: attempts to relocate a dislocated femoral head

Ultrasound is used to confirm the diagnosis if clinically suspected


Management

most unstable hips will spontaneously stabilise by 3-6 weeks of age


Pavlik harness (flexion-abduction orthosis) in children younger than 4-5 months

older children may require surgery

Question 30 of 80

The mother of a 4-year-old boy comes to surgery as she is concerned he is still wetting the bed at night. This is in contrast to his older
brother who was dry at night by the age of 3 years. She is wondering if there is any treatment you can offer. What is the most appropriate
management?
Trial of oral desmopressin

Enuresis alarm

Discourage fluids at night

Trial of intranasal desmopressin

Reassurance and advice on fluid intake, diet and toileting behaviour

Next question

Nocturnal bedwetting is still very common at 4 years and the mother should be reassured

Nocturnal enuresis
The majority of children achieve day and night time continence by 3 or 4 years of age. Enuresis may be defined as the 'involuntary
discharge of urine by day or night or both, in a child aged 5 years or older, in the absence of congenital or acquired defects of the nervous
system or urinary tract'
Nocturnal enuresis can be defined as either primary (the child has never achieved continence) or secondary (the child has been dry for at
least 6 months before)
NICE issued guidance in 2010. Management:

look for possible underlying causes/triggers (e.g. Constipation, diabetes mellitus, UTI if recent onset)
advise on fluid intake, diet and toileting behaviour
reward systems (e.g. Star charts). NICE recommend these 'should be given for agreed behaviour rather than dry nights' e.g.
Using the toilet to pass urine before sleep
NICE advise: 'Consider whether alarm or drug treatment is appropriate, depending on the age, maturity and abilities of the child
or young person, the frequency of bedwetting and the motivation and needs of the family'. Generally:
an enuresis alarm is first-line for children under the age of 7 years
desmopressin may be used first-line for children over the ago 7 years, particularly if short-term control is needed or an enuresis
alarm has been ineffective/is not acceptable to the family
please see the link for more details

Question 31 of 80

A 12-month-old boy is brought to surgery as mum is concerned about his eyesight. On examination the red reflex is present bilaterally and
no squint is seen. Which one of the following is least associated with visual problems in childhood?

Cerebral palsy

Albinism

Hydrocephalus

Prematurity

Prader-Willi syndrome

Next question

Vision problems in children


Causes of visual problems in children

congenital: infection, cataracts


prematurity - retinopathy of prematurity
cerebral palsy
optic atrophy e.g. hydrocephalus, optic nerve hypoplasia
albinism

Question 32 of 80

A 5-year-old boy is brought to the surgery with chickenpox. His mother wants advice regarding school exclusion. What is the most
appropriate advice to give?
Should be excluded until 2 days after all lesions have scabbed over

Should be excluded until 5 days after skin lesions first appeared

Should be excluded until skin lesions have disappeared

School exclusion is not indicated

Should be excluded until 5 days after all lesions have scabbed over

Next question

Chickenpox school exclusion - 5 days after skin lesions first appeared

Chickenpox
Chickenpox is caused by primary infection with varicella zoster virus. Shingles is reactivation of dormant virus in dorsal root ganglion

Chickenpox is highly infectious

spread via the respiratory route


can be caught from someone with shingles
infectivity = 4 days before rash, until 5 days after the rash first appeared*
incubation period = 10-21 days

Clinical features (tend to be more severe in older children/adults)

fever initially
itchy, rash starting on head/trunk before spreading. Initially macular then papular then vesicular
systemic upset is usually mild

Management is supportive

keep cool, trim nails


calamine lotion
school exclusion: current HPA advice is 5 days from start of skin eruption. They also state 'Traditionally children have been
excluded until all lesions are crusted. However, transmission has never been reported beyond the fifth day of the rash.'
immunocompromised patients and newborns with peripartum exposure should receive varicella zoster immunoglobulin (VZIG).
If chickenpox develops then IV aciclovir should be considered

A common complication is secondary bacterial infection of the lesions. Rare complications include

pneumonia
encephalitis (cerebellar involvement may be seen)
disseminated haemorrhagic chickenpox
arthritis, nephritis and pancreatitis may very rarely be seen

*it was traditionally taught that patients were infective until all lesions had scabbed over

Question 33 of 80

A 2-day-old baby girl is noted to become cyanotic whilst feeding and crying. A diagnosis of congenital heart disease is suspected. What is
the most likely cause?
Transposition of the great arteries

Coarctation of the aorta

Patent ductus arteriosus

Tetralogy of Fallot

Ventricular septal defect

Next question

Congenital heart disease

cyanotic: TGA most common at birth, Fallot's most common overall


acyanotic: VSD most common cause

The key point to this question is that whilst tetralogy of Fallot is more common than transposition of the great arteries (TGA), Fallot's doesn't
usually present until 1-2 months following the identification of a murmur. In the neonate, TGA is the most common presenting cause of
cyanotic congenital heart disease
The other 3 options are causes of acyanotic congenital heart disease

Congenital heart disease: types


Acyanotic - most common causes

ventricular septal defects (VSD) - most common, accounts for 30%


atrial septal defect (ASD)
patent ductus arteriosus (PDA)
coarctation of the aorta
aortic valve stenosis

VSDs are more common than ASDs. However, in adult patients ASDs are the more common new diagnosis as they generally presents
later
Cyanotic - most common causes

tetralogy of Fallot
transposition of the great arteries (TGA)
tricuspid atresia
pulmonary valve stenosis

Fallot's is more common than TGA. However, at birth TGA is the more common lesion as patients with Fallot's generally presenting at
around 1-2 months

Question 34 of 80

A 3-year-old boy is investigated for lethargy. Examination is unremarkable with a blood pressure of 90/46 mmHg (normal for his age). Blood
tests reveal:

Na+

140 mmol/l

K+

2.6 mmol/l

Bicarbonate

33 mmol/l

Urea

4.2 mmol/l

Creatinine

91 mol/l

Which one of the following conditions is most likely to be responsible?


Cushing's syndrome

Conn's syndrome

11-beta hydroxylase deficiency

Bartter's syndrome

Liddle's syndrome

Next question

Bartter's syndrome is associated with normotension

Bartter's syndrome is an inherited cause (usually autosomal recessive) of severe hypokalaemia due to defective chloride absorption at the
Na+ K+ 2Cl- cotransporter in the ascending loop of Henle

Hypokalaemia and hypertension


For exams it is useful to be able to classify the causes of hypokalaemia in to those associated with hypertension, and those which are not
Hypokalaemia with hypertension

Cushing's syndrome
Conn's syndrome (primary hyperaldosteronism)
Liddle's syndrome
11-beta hydroxylase deficiency*

Carbenoxolone, an anti-ulcer drug, and liquorice excess can potentially cause hypokalaemia associated with hypertension
Hypokalaemia without hypertension

diuretics
GI loss (e.g. Diarrhoea, vomiting)
renal tubular acidosis (type 1 and 2**)
Bartter's syndrome
Gitelman syndrome

*21-hydroxylase deficiency, which accounts for 90% of congenital adrenal hyperplasia cases, is not associated with hypertension
**type 4 renal tubular acidosis is associated with hyperkalaemia

Question 35 of 80

You are contacted by the health visitor who is concerned about the head growth of a 4-month-old boy. His head growth has crossed from
the 50th to the 91st centile in the past 2 months. Which one of the following is least likely to be the cause?

Incorrect measurement of the head circumference

Neurofibromatosis

Subdural effusion

Craniosynostosis e.g. Crouzon's syndrome

Hydrocephalus

Next question

Crouzon's syndrome is associated with a small head

Macrocephaly
Causes of macrocephaly in children include:

normal variant
chronic hydrocephalus
chronic subdural effusion
neurofibromatosis
gigantism (e.g. Soto's syndrome)
metabolic storage diseases
bone problems e.g. thalassaemia

Question 36 of 80

Each one of the following features is seen in phenylketonuria, except:


Learning difficulties

Seizures

Eczema

Recurrent infections

'Musty' urine

Next question

Phenylketonuria
Phenylketonuria (PKU) is an autosomal recessive condition caused by a disorder of phenylalanine metabolism. This is usually due to defect
in phenylalanine hydroxylase, an enzyme which converts phenylalanine to tyrosine. In a small number of cases the underlying defect is a

deficiency of the tetrahydrobiopterin-deficient cofactor, e.g. secondary to defective dihydrobiopterin reductase. High levels of phenylalanine
lead to problems such as learning difficulties and seizures. The gene for phenylalanine hydroxylase is located on chromosome 12. The
incidence of PKU is around 1 in 10,000 live births.

Features

usually presents by 6 months e.g. with developmental delay


child classically has fair hair and blue eyes
learning difficulties
seizures, typically infantile spasms
eczema
'musty' odour to urine and sweat*

Diagnosis

Guthrie test: the 'heel-prick' test done at 5-9 days of life - also looks for other biochemical disorders such as hypothyroidism
hyperphenylalaninaemia
phenylpyruvic acid in urine

Management

poor evidence base to suggest strict diet prevents learning disabilities


dietary restrictions are however important during pregnancy as genetically normal fetuses may be affected by high maternal
phenylalanine levels

*secondary to phenylacetate, a phenylketone

Question 37 of 80

At what age would the average child acquire the ability to take off their socks?
3 months

6 months

9 months

12 months

18 months

Next question

Developmental milestones: social behaviour and play


The table below summarises the major social behaviour and play milestones
Age

Milestone

6 weeks Smiles (Refer at 10 weeks)


3 months

Laughs
Enjoys friendly handling

6 monthsNot shy
9 months

Shy
Takes everything to mouth

Feeding
Age

Milestone

May put hand on bottle when being fed

6 months

Drinks from cup + uses spoon, develops over 3 month period 12 -15 months
Competent with spoon, doesn't spill with cup

2 years

Uses spoon and fork

3 years

Uses knife and fork

5 years

Dressing
Age

Milestone

Helps getting dressed/undressed

12-15 months

Takes off shoes, hat but unable to replace

18 months

Puts on hat and shoes

2 years

Can dress and undress independently except for laces and buttons 4 years

Play

Age

Milestone

Plays 'peek-a-boo'

9 months

Waves 'bye-bye'
Plays 'pat-a-cake'

12 months

Plays contentedly alone

18 months

Plays near others, not with them 2 years


Plays with other children

4 years

Question 38 of 80

A 1-year-old girl is brought to surgery due to a high fever. She has a past history of febrile convulsions. Whilst examining her she starts to
fit. You carry her to the treatment room and apply oxygen. After 5 minutes she continues to fit. What is the most appropriate next step?
Call 999 for an ambulance

Give 1 g rectal paracetamol

Give 2.5 mg rectal diazepam + rectal paracetamol 1g

Give 5 mg rectal diazepam

Give 10 mg rectal diazepam

Next question

Seizures: acute management


Most seizures are self-limiting and stop spontaneously but prolonged seizures may be potentially life-threatening.
Basics

check the airway and apply oxygen if appropriate


place the patient in the recovery position
if the seizure is prolonged give benzodiazepines

BNF recommend dose for rectal diazepam, repeated once after 10-15 minutes if necessary
Neonate

1.25 - 2.5 mg

Child 1 month - 2 years 5 mg


Child 2 years - 12 years 5 - 10 mg
Child 12 years - 18 years10 mg
Adult

10 - 20 mg (max. 30 mg)

Neonate

1.25 - 2.5 mg

Elderly

10 mg (max. 15 mg)

Question 39 of 80

Which one of the following is the most important factor in determining fetal growth?
Age of mother

Placental factors

Fetal environment e.g. maternal nutrition

Hormonal factors

Genetic factors

Next question

Growth
Factors which affect fetal growth are as follows:

environmental: this is the most important factor affecting fetal growth e.g. maternal nutrition and uterine capacity
placental
hormonal
genetic: predominately maternal

Following birth genetic factors are the most important determinant of final adult height
Monitoring of growth

infants aged 0-1 years should have at least 5 recordings of weight


children aged 1-2 years should have at least 3 recordings of weight
children older than 2 years should have annual recording of weight
children below 2nd centile for height should be reviewed by their GP
children below 0.4th centile for height should be reviewed by a paediatrician

Question 40 of 80

Which one of the following statements regarding the UK-WHO growth charts is true?
They only include data from UK children

They have five centile lines marked on them

They should not be used for Asian children

They are based on data from breast fed infants only

They can be used for children up to the age of 16 years

Next question

Growth charts
The UK has recently switched to the new growth charts based on the WHO growth standard for children under the age of 5 years. The new
UK-WHO charts have a separate preterm section and a 0-1 year section.
Key points

based on data from breast fed infants and all ethnic groups
the data matches UK children well for height and length but after 6 months UK children and slightly more heavy and more likel y
to be above the 98% centile
preterm infants born at 32-36 weeks have a separate chart until 2 weeks post-term

Please see the comprehensive review by Wright CM et al. BMJ 2010; 340:c1140 for more information

Question 41 of 80

A mother brings her 13-year-old son in who has developed acne. On both his face and back he has a number of inflammatory lesions
consistent with moderate acne. Which one of the following is the most appropriate guidance when prescribing oral tetracyclines?
Patients must be 11 years or older

Patients must be 13 years or older

Patients must be 12 years or older

Patients must have completed puberty

Patients must be 14 years or older

Next question

Acne vulgaris: management


Acne vulgaris is a common skin disorder which usually occurs in adolescence. It typically affects the face, neck and upper trunk and is
characterised by the obstruction of the pilosebaceous follicles with keratin plugs which results in comedones, inflammation and pustules.
Acne may be classified into mild, moderate or severe:

mild: open and closed comedones with or without sparse inflammatory lesions
moderate acne: widespread non-inflammatory lesions and numerous papules and pustules
severe acne: extensive inflammatory lesions, which may include nodules, pitting, and scarring

A simple step-up management scheme often used in the treatment of acne is as follows:

single topical therapy (topical retinoids, benzyl peroxide)


topical combination therapy (topical antibiotic, benzoyl peroxide, topical retinoid)
oral antibiotics: e.g. Oxytetracycline, doxycycline. Improvement may not be seen for 3-4 months. Minocycline is now considered
less appropriate due to the possibility of irreversible pigmentation. Gram negative folliculitis may occur as a complication of
long-term antibiotic use - high-dose oral trimethoprim is effective if this occurs
oral isotretinoin: only under specialist supervision

There is no role for dietary modification in patients with acne

Question 42 of 80

Fragile X is associated with each one of the following, except:


Small, firm testes

Learning difficulties

Joint laxity

Long thin face

Large low set ears

Next question

Fragile X
Fragile X is a trinucleotide repeat disorder
Features in males

learning difficulties
large low set ears, long thin face, high arched palate
macroorchidism
hypotonia
autism is more common
mitral valve prolapse

Features in females (who have one fragile chromosome and one normal X chromosome) range from normal to mild
Diagnosis

can be made antenatally by chorionic villus sampling or amniocentesis


analysis of the number of CGG repeats using restriction endonuclease digestion and Southern blot analysis

Question 43 of 80

At what age would the average child acquire the ability to crawl?
6 months

9 months

12 months

18 months

2 years

Next question

Developmental milestones: gross motor


The table below summarises the major gross motor developmental milestones
Age

Milestone

3 months

Little or no head lag on being pulled to sit


Lying on abdomen, good head control
Held sitting, lumbar curve

6 months

Lying on abdomen, arms extended


Lying on back, lifts and grasps feet
Pulls self to sitting
Held sitting, back straight
Rolls front to back

7-8 months

Sits without support (Refer at 12 months)

9 months

Pulls to standing
Crawls

12 months

Cruises
Walks with one hand held

13-15 months Walks unsupported (Refer at 18 months)


18 months

Squats to pick up a toy

2 years

Runs
Walks upstairs and downstairs holding on to rail

3 years

Rides a tricycle using pedals


Walks up stairs without holding on to rail

4 years

Hops on one leg

Notes

the majority of children crawl on all fours before walking but some children 'bottom-shuffle'. This is a normal variant and runs in
families

Question 43 of 80

At what age would the average child acquire the ability to crawl?
6 months

9 months

12 months

18 months

2 years

Next question

Developmental milestones: gross motor


The table below summarises the major gross motor developmental milestones
Age

Milestone

3 months

Little or no head lag on being pulled to sit


Lying on abdomen, good head control
Held sitting, lumbar curve

6 months

Lying on abdomen, arms extended


Lying on back, lifts and grasps feet
Pulls self to sitting
Held sitting, back straight
Rolls front to back

7-8 months

Sits without support (Refer at 12 months)

9 months

Pulls to standing
Crawls

12 months

Cruises
Walks with one hand held

13-15 months Walks unsupported (Refer at 18 months)


18 months

Squats to pick up a toy

2 years

Runs
Walks upstairs and downstairs holding on to rail

3 years

Rides a tricycle using pedals

Age

Milestone
Walks up stairs without holding on to rail

4 years

Hops on one leg

Notes

the majority of children crawl on all fours before walking but some children 'bottom-shuffle'. This is a normal variant and runs in
families

Question 44 of 80

A 4-year-old boy is admitted after developing a haemarthrosis in his right knee whilst playing in the garden. The following blood results are
obtained:

Platelets

220 * 109/l

PT

12 secs

APTT

78 secs

Factor VIIIc
activity

Normal

What is the most likely diagnosis?


Antithrombin III deficiency

Von Willebrand's disease

Antiphospholipid syndrome

Haemophilia A

Haemophilia B

Next question

A grossly elevated APTT may be caused by heparin therapy, haemophilia or antiphospholipid syndrome. A normal factor VIIIc activity
points to a diagnosis of haemophilia B (lack of factor IX). Antiphospholipid syndrome is a prothrombotic condition

Haemophilia

Haemophilia is a X-linked recessive disorder of coagulation. Up to 30% of patients have no family history of the condition. Haemophilia A is
due to a deficiency of factor VIII whilst in haemophilia B (Christmas disease) there is a lack of factor IX
Features

haemoarthroses, haematomas
prolonged bleeding after surgery or trauma

Blood tests

prolonged APTT
bleeding time, thrombin time, prothrombin time normal

Up to 10-15% of patients with haemophilia A develop antibodies to factor VIII treatment

Question 45 of 80

A 4-year-old girl is brought into the Emergency Department after falling down some stairs at home. You have been asked by the Consultant
to make an initial assessment of her. On examination her eyes are open spontaneously but she is crying and is inconsolable at times. She
is repeatedly moaning 'mummy' but does not appear agitated. She is not moving spontaneously but does withdraw her hand when you
touch it saying 'ow'. What is her paediatric Glasgow Coma Scale score?
10

11

12

13

14

Next question

Her GCS score is 12, M5 V3 E4.

Glasgow Coma Scale: children

Modality

Options

6. Infant moves spontaneously or purposefully


5. Infant withdraws from touch
4. Infant withdraws from pain
Motor response
3. Abnormal flexion to pain for an infant (decorticate response)
2. Extension to pain (decerebrate response)
1. No motor response
Verbal response

5. Smiles, oriented to sounds, follows objects, interacts.


4. Cries but consolable, inappropriate interactions.
3. Inconsistently inconsolable, moaning.

Modality

Options
2. Inconsolable, agitated.
1. No verbal response.

Eye opening

4. Eyes opening spontaneously


3. Eye opening to speech
2. Eye opening to pain
1. No eye opening or response

Glasgow coma scale (GCS) scores are generally expressed in the following format 'GCS = 13, M5 V4 E4 at 21:30'.
Question 46 of 80

Which one of the following drugs is contra-indicated whilst breast feeding?


Warfarin

Cephalosporins

Aspirin

Tricyclic antidepressants

Chlorpromazine

Next question

Aspirin is best avoided in children due to the risk of Reye's syndrome

Breast feeding: contraindications


The major breastfeeding contraindications tested in exams relate to drugs (see below). Other contraindications of note include:

galactosaemia
viral infections - this is controversial with respect to HIV in the developing world. This is because there is such an increased
infant mortality and morbidity associated with bottle feeding that some doctors think the benefits outweigh the risk of HIV
transmission

Drug contraindications
The following drugs can be given to mothers who are breast feeding:

antibiotics: penicillins, cephalosporins, trimethoprim


endocrine: glucocorticoids (avoid high doses), levothyroxine*
epilepsy: sodium valproate, carbamazepine
asthma: salbutamol, theophyllines
psychiatric drugs: tricyclic antidepressants, antipsychotics**
hypertension: beta-blockers, hydralazine, methyldopa
anticoagulants: warfarin, heparin
digoxin

The following drugs should be avoided:

antibiotics: ciprofloxacin, tetracycline, chloramphenicol, sulphonamides


psychiatric drugs: lithium, benzodiazepines
aspirin
carbimazole
sulphonylureas
cytotoxic drugs
amiodarone

*the BNF advises that the amount is too small to affect neonatal hypothyroidism screening
**clozapine should be avoided

Question 47 of 80

A mother requests a home visit for her 12-year-old son who is too poorly to come to surgery, On arrival the boy is noted to be pyrexial, have
cool peripheries and purpura on his legs. What action should be taken?
IM benzylpenicillin 150mg

IM benzylpenicillin 300mg

IM benzylpenicillin 600mg

IM benzylpenicillin 900mg

IM benzylpenicillin 1200mg

Next question

The RCGP have previously fed back that doctors are expected to be familiar with emergency drug doses, and have mentioned suspected
meningococcal septicaemia in particular

Paediatric drug doses: emergency


The current BNF should always be consulted prior to prescribing drugs you are unfamiliar with, the following is just a guide
IM benzylpenicillin for suspected meningococcal septicaemia in the community
Age

Dose

< 1 year

300 mg

1 - 10 years600 mg
> 10 years 1200 mg
Question 48 of 80

Which one of the following statements regarding infantile spasms is incorrect?


EEG shows hypsarrhythmia in the majority of children

Carries a good prognosis

More common in male children

Typically presents in the first 4 to 8 months

Causes characteristic 'salaam' attacks

Next question

Infantile spasms
Infantile spasms, or West syndrome, is a type of childhood epilepsy which typically presents in the first 4 to 8 months of life and is more
common in male infants. They are often associated with a serious underlying condition and carry a poor prognosis
Features

characteristic 'salaam' attacks: flexion of the head, trunk and arms followed by extension of the arms
this lasts only 1-2 seconds but may be repeated up to 50 times
progressive mental handicap

Investigation

the EEG shows hypsarrhythmia in two-thirds of infants


CT demonstrates diffuse or localised brain disease in 70% (e.g. tuberous sclerosis)

Management

poor prognosis
vigabatrin is now considered first-line therapy
ACTH is also used

Question 49 of 80

Each one of the following is a cause of hypertension in children, except:


Bartter's syndrome

Renal vascular disease

Congenital adrenal hyperplasia

Coarctation of the aorta

Renal parenchymal disease

Next question

Bartter's syndrome is an inherited cause (usually autosomal recessive) of severe hypokalaemia due to defective chloride absorption at the
Na+ K+ 2Cl- cotransporter in the ascending loop of Henle. It should be noted that it is associated with normotension (unlike other endocrine
causes of hypokalaemia such as Conn's, Cushing's and Liddle's syndrome which are associated with hypertension)
Features

usually presents in childhood, e.g. Failure to thrive


polyuria, polydipsia
hypokalaemia
normotension
weakness

Hypertension in children
Measuring blood pressure in children

correct cuff size is approximately 2/3 the length of the upper arm
the 4th Korotkoff sound is used to measure the diastolic blood pressure until adolescence, when the 5th Korotkoff sound can be
used
results should be compared with a graph of normal values for age

In younger children secondary hypertension is the most common cause, with renal parenchymal disease accounting for up to 80%
Causes of hypertension in children

renal parenchymal disease


renal vascular disease
coarctation of the aorta
phaeochromocytoma
congenital adrenal hyperplasia
essential or primary hypertension (becomes more common as children become older

Question 50 of 80

The chance of a 30-year-old mother giving birth to a child with Down's syndrome is approximately:
1 in 125

1 in 350

1 in 550

1 in 1,000

1 in 2,000

Next question

Down's syndrome risk - 1/1,000 at 30 years then divide by 3 for every 5 years

Down's syndrome: epidemiology and genetics


Risk of Down's syndrome with increasing maternal age

risk at 30 years = 1/1000


35 years = 1/350
40 years = 1/100
45 years = 1/30

One way of remembering this is by starting at 1/1,000 at 30 years and then dividing the denominator by 3 (i.e. 3 times more common) for
every extra 5 years of age
Cytogenetics
Mode

% of cases Risk of recurrence

Non-disjunction

94%

Robertsonian translocation
5%
(usually onto 14)
Mosaicism

1 in 100 if under mother < 35 years


10-15% if mother is translocation carrier
2.5% if father is translocation carrier

1%

The chance of a further child with Down's syndrome is approximately 1 in 100 if the mother is less than 35 years old. If the trisomy 21 is a
result of a translocation the risk is much higher

Question 51 of 80

A 13-year-old girl develops purpura on her lower limbs and buttocks associated with microscopic haematuria. A diagnosis of HenochSchonlein purpura is made. Her urea and electrolytes show mild renal impairment that is still present 4 weeks later, although she does not
require any specific therapy. What is the most likely renal outcome?
Hypertension within 20 years

Persistent proteinuria

End stage renal failure

Full renal recovery

Frequent relapses

Next question

Henoch-Schonlein purpura
Henoch-Schonlein purpura (HSP) is an IgA mediated small vessel vasculitis. There is a degree of overlap with IgA nephropathy (Berger's
disease). HSP is usually seen in children following an infection.
Features

palpable purpuric rash (with localized oedema) over buttocks and extensor surfaces of arms and legs
abdominal pain
polyarthritis
features of IgA nephropathy may occur e.g. haematuria, renal failure

Image used on license from DermNet NZ

Treatment

analgesia for arthralgia


treatment of nephropathy is generally supportive. There is inconsistent evidence for the use of steroids and
immunosuppressants

Prognosis

usually excellent, HSP is a self-limiting condition, especially in children without renal involvement
around 1/3rd of patients have a relapse

Image used on license from DermNet NZ

Image used on license from DermNet NZ

Question 52 of 80

Which of the following conditions is inherited in a X-linked recessive fashion?


Testicular feminisation syndrome

Myotonic dystrophy

von Willebrand's disease

Ehlers-Danlos syndrome

Huntington's disease

Next question

X-linked conditions: Duchenne/Becker, haemophilia, G6PD

X-linked recessive conditions


The following conditions are inherited in a X-linked recessive fashion:
Androgen insensitivity syndrome
Becker muscular dystrophy
Colour blindness
Duchenne muscular dystrophy
Fabry's disease
G6PD deficiency
Haemophilia A,B
Hunter's disease
Lesch-Nyhan syndrome
Nephrogenic diabetes insipidus
Ocular albinism
Retinitis pigmentosa
Wiskott-Aldrich syndrome
The following diseases have varying patterns of inheritance, with the majority being in an X-linked recessive fashion:
Chronic granulomatous disease (in > 70%)
Question 53 of 80

When does the Moro reflex typically disappear?


2 months

4 months

6 months

8 months

12 months

Next question

Moro reflex disappears at 4 months

Primitive reflexes

Reflex Description

Moro

Head extension causes abduction followed by adduction of the arms


Present from birth to around 3-4 months of age

Reflex Description

Grasp

Flexion of fingers when object placed in palm


Present from birth to around 4-5 months of age

Rooting

Assists in breastfeeding
Present from birth to around 4 months of age

Stepping

Also known as walking reflex


Present from birth to around 2 months of age

Question 53 of 80

When does the Moro reflex typically disappear?


2 months

4 months

6 months

8 months

12 months

Next question

Moro reflex disappears at 4 months

Primitive reflexes

Reflex Description

Moro

Head extension causes abduction followed by adduction of the arms


Present from birth to around 3-4 months of age

Grasp

Flexion of fingers when object placed in palm


Present from birth to around 4-5 months of age

Rooting

Assists in breastfeeding
Present from birth to around 4 months of age

Stepping

Also known as walking reflex


Present from birth to around 2 months of age

Question 54 of 80

A 12-month-old child is brought into surgery for her next routine immunisations. She has received all the recommended immunisations to
date. What should be given at this stage?

MMR + PCV

MMR + Hib

Hib/Men C

MMR + Men C

Hib/Men C + MMR + PCV

Next question

Immunisation schedule

12-13 months: Hib/Men C + MMR + PCV

In 2010 the NHS immunisation guidelines changed and it is now recommended that infants aged 12-13 months are given Hib/Men C +
MMR + PCV at the same time, rather than split between two visits at 12 and 13 months.

Immunisation schedule
The current UK immunisation schedule is as follows. Please note that this table includes the changes announced in 2010 which merged the
12 and 13 month visits into one.
Age

Recommended immunisations

At birth

BCG / hepatitis B vaccine if risk factors (see below)

2 months

DTaP/IPV/Hib + PCV
Oral rotavirus vaccine

3 months

DTaP/IPV/Hib + Men C
Oral rotavirus vaccine

4 months

DTaP/IPV/Hib + PCV

12-13 months Hib/Men C + MMR + PCV


2-3 years

Flu vaccine (annual)

3-4 years

MMR + DTaP/IPV

12-13 years HPV vaccination for girls


13-18 years DT/IPV + MenC

At birth the BCG vaccine should be given if the baby is deemed at risk of tuberculosis (e.g. Tuberculosis in the family in the past 6 months).
Hepatitis B vaccine should be given at birth if the mother is HBsAg +ve.
Key

DTaP = Diphtheria, Tetanus, acellular Pertussis vaccine


IPV = Inactivated Polio Vaccine
Hib = Haemophilus influenzae B vaccine
PCV = Pneumococcal Conjugate Vaccine
Men C = Meningococcal C vaccine

MMR = Measles, Mumps, Rubella vaccine


DT = Diphtheria, Tetanus vaccine
HPV = Human Papilloma Vaccine

Question 55 of 80

Which one of the following is a risk factor for the development surfactant deficient lung disease in the newborn?
Maternal diabetes mellitus

Maternal pregnancy-induced hypertension

Vaginal delivery

Maternal asthma

Female sex

Next question

Surfactant deficient lung disease


Surfactant deficient lung disease (SDLD., also known as respiratory distress syndrome and previously as hyaline membrane disease) is a
condition seen in premature infants. It is caused by insufficient surfactant production and structural immaturity of the lungs
The risk of SDLD decreases with gestation

50% of infants born at 26-28 weeks


25% of infants born at 30-31 weeks

Other risk factors for SDLD include

male sex
diabetic mothers
Caesarean section
second born of premature twins

Clinical features are those common to respiratory distress in the newborn, i.e. tachypnoea, intercostal recession, expiratory grunting and
cyanosis
Chest x-ray characteristically shows 'ground-glass' appearance with an indistinct heart border
Management

prevention during pregnancy: maternal corticosteroids to induce fetal lung maturation


oxygen
assisted ventilation
exogenous surfactant given via endotracheal tube

Question 56 of 80

Which one of the following features is least associated with roseola infantum?
Tonsillopharyngitis

Cough

Febrile convulsions

High fever

Diarrhoea

Next question

Whilst Nagayama spots (erythematous papules on the soft palate and uvula) may be seen, tonsillopharyngitis is not a common feature

Roseola infantum
Roseola infantum (also known as exanthem subitum, occasionally sixth disease) is a common disease of infancy caused by the human
herpes virus 6 (HHV6). It has an incubation period of 5-15 days and typically affects children aged 6 months to 2 years.
Features

high fever: lasting a few days, followed by a


maculopapular rash
febrile convulsions occur in around 10-15%
diarrhoea and cough are also commonly seen

Other possible consequences of HHV6 infection

aseptic meningitis
hepatitis

Question 56 of 80

Which one of the following features is least associated with roseola infantum?
Tonsillopharyngitis

Cough

Febrile convulsions

High fever

Diarrhoea

Next question

Whilst Nagayama spots (erythematous papules on the soft palate and uvula) may be seen, tonsillopharyngitis is not a common feature

Roseola infantum
Roseola infantum (also known as exanthem subitum, occasionally sixth disease) is a common disease of infancy caused by the human
herpes virus 6 (HHV6). It has an incubation period of 5-15 days and typically affects children aged 6 months to 2 years.
Features

high fever: lasting a few days, followed by a


maculopapular rash
febrile convulsions occur in around 10-15%
diarrhoea and cough are also commonly seen

Other possible consequences of HHV6 infection

aseptic meningitis
hepatitis

Question 57 of 80

Precocious puberty in males may be defined as the development of secondary sexual characteristics before:
8 years of age

9 years of age

10 years of age

11 years of age

12 years of age

Next question

Precocious puberty
Definition

'development of secondary sexual characteristics before 8 years in females and 9 years in males'
more common in females

Some other terms

thelarche (the first stage of breast development)


adrenarche (the first stage of pubic hair development)

May be classified into:


1. Gonadotrophin dependent ('central', 'true')

due to premature activation of the hypothalamic-pituitary-gonadal axis


FSH & LH raised

2. Gonadotrophin independent ('pseudo', 'false')

due to excess sex hormones


FSH & LH low

Males - uncommon and usually has an organic cause


Testes

bilateral enlargement = gonadotrophin release from intracranial lesion


unilateral enlargement = gonadal tumour
small testes = adrenal cause (tumour or adrenal hyperplasia)

Females - usually idiopathic or familial and follows normal sequence of puberty


Organic causes

are rare, associated with rapid onset, neurological symptoms and signs and dissonance
e.g. McCune Albright syndrome

Question 58 of 80

A 4-year-old girl with sickle cell anaemia presents with abdominal pain. On examination she is noted to have splenomegaly and is clinically
anaemic. What is the most likely diagnosis?
Liver cirrhosis

Parvovirus infection

Sequestration crisis

Salmonella infection

Thrombotic crisis

Next question

Sickle-cell crises
Sickle cell anaemia is characterised by periods of good health with intervening crises
Four main types of crises are recognised:

thrombotic, 'painful crises'


sequestration
aplastic
haemolytic

Thrombotic crises

also known as painful crises or vaso-occlusive crises


precipitated by infection, dehydration, deoxygenation
infarcts occur in various organs including the bones (e.g. avascular necrosis of hip, hand-foot syndrome in children, lungs,
spleen and brain

Sequestration crises

sickling within organs such as the spleen or lungs causes pooling of blood with worsening of the anaemia
acute chest syndrome: dyspnoea, chest pain, pulmonary infiltrates, low pO2 - the most common cause of death after childhood

Aplastic crises

caused by infection with parvovirus


sudden fall in haemoglobin

Haemolytic crises

rare
fall in haemoglobin due an increased rate of haemolysis

Sickle cell disease - Summary


Sickle cell disease encompasses a group of inherited conditions which have in common the
inheritance of sickle haemoglobin. Sickle haemoglobin has an abnormal beta-globin chain that
causes it to polymerize when deoxygenated, which distorts the erythrocyte into a sickle shape.
People with sickle cell disease have inherited the gene for sickle haemoglobin (Hb S) from one
parent and a gene for an abnormal haemoglobin variant from the other parent. If the second
abnormal gene is also for Hb S, the person is homozygous for the sickle haemoglobin (Hb SS) and
the person has sickle cell disease.

Sickle cell trait (sickle cell carrier) occurs when a person inherits a gene for normal haemoglobin
(Hb A) from one parent and a gene for sickle cell haemoglobin from their other parent. Their
genotype is Hb AS. People who are sickle cell carriers rarely have clinical symptoms.
Sickle cell disease affects 1 in every 2400 live births in England and is now the most common
genetic condition at birth.
Sickle cell disease should be suspected if the person is in a high-risk ethnic group and:
Is a child aged 918 months with painful dactylitis (painful swelling of the bones of the hands
and feet). There may be chronic shortening of a digit due to epiphyseal damage.
Has a sudden severe infection.
Presents with features of an acute crisis, or presents with a history of features consistent with
an acute crisis.
Presents with features of a chronic complication of sickle cell disease.
Sickle cell disease is always diagnosed after both an initial and confirmatory test are positive.
A sickle cell crisis should be suspected if there is a sudden onset of pain, infection, or anaemia,
or other symptoms, such as a stroke or priapism. There is often a history of a previous crisis. The
person is often able to state whether their pain is typical of sickle cell disease.
The most common type of crisis is an acute painful crisis which usually starts with vague pain,
often in the back of limb bones, which gets gradually worse.
Acute chest syndrome is common in early childhood, when it may present like pneumonia.
Onset may be abrupt, and the person may deteriorate over a few hours. Typical features of
acute chest syndrome include: new onset of chest pain or pleuritic pain, cough, shortness of
breath and/or respiratory distress, tachypnea, fever, hypoxia, crepitations in the lung bases that
become generalized, and rib tenderness.
All people with clinical features of a sickle cell crisis should be admitted to hospital unless they
are:
A well adult with only mild or moderate pain who has a temperature of 38C or less.
A well child with mild or moderate pain who does not have an increased temperature.
All people with sickle cell disease will be followed up regularly in secondary care who will advise
on:
Immunizations.
Lifelong antibiotic prophylaxis.

Daily folic acid.


Have I got the right topic?
All ages
This CKS topic is based on Sickle cell disease in childhood: standards and guidelines for clinical
care [NHS Antenatal and Newborn Screening Programmes, 2006] and Standards for the clinical
care of adults with sickle cell disease in the UK [Sickle Cell Society, 2008].
This CKS topic covers the management of sickle cell disease and sickle cell trait in primary care.
This CKS topic does not cover the detailed management of sickle cell disease in secondary care.
There are separate CKS topics on Anaemia - B12 and folate deficiency and Anaemia - iron
deficiency.
The target audience for this CKS topic is healthcare professionals working within the NHS in the
UK, and providing first contact or primary healthcare.
How up-to-date is this topic?
Changes
Update
Goals and outcome measures
Goals
Background information
Definition
Inheritance
Sickle cell trait
Prevalence
Pregnancy with sickle cell disease
Prognosis
Complications
Diagnosis
Diagnosis of sickle cell disease

When to suspect
Confirmation
Assessment of acute crisis
Management
Scenario: Screening : provides information on the national screening programme to detect sickle
cell disease and other disorders, such as sickle cell trait and thalassemia.
Scenario: Management - sickle cell crisis : covers the management of a sickle cell crisis that has
been identified in primary care.
Scenario: Management - chronic complications : covers the management of chronic
complications of sickle cell disease in primary care.
Scenario: Prevention of complications : covers the prevention of complications of sickle cell
disease, including the use of immunizations, antibiotics, and folic acid in primary care.
Scenario: Management - sickle cell trait : covers the management of sickle cell trait in primary
care.
Scenario: Contraception : covers recommendations for the use of contraception in people with
sickle cell disease.
Scenario: Screening
Scenario: Management - sickle cell crisis
Scenario: Management - chronic complications
Scenario: Prevention of complications
Scenario: Management - sickle cell trait
Scenario: Contraception
Evidence
Supporting evidence
Search strategy
References
Alhashimi, D., Fedorowicz, Z., Alhashimi, F. and Dastgiri, S. (2010) Blood transfusions for treating
acute chest syndrome in people with sickle cell disease (Cochrane Review). The Cochrane
Library. Issue 1. John Wiley & Sons, Ltd. www.thecochranelibrary.com [Free Full-text]

BNF 59 (2010) British National Formulary. 59th edn. London: British Medical Association and
Royal Pharmaceutical Society of Great Britain.
BNF for Children (2010) British National Formulary for Children. London: British Medical
Association and the Royal Pharmaceutical Society of Great Britain.
British Committee for Standards in Haematology (1996) Guidelines for the prevention and
treatment of infection in patients with an absent or dysfunctional spleen. Working Party of the
British Committee for Standards in Haematology Clinical Haematology Task Force. BMJ
312(7028), 430-434. [Abstract] [Free Full-text]
British Committee for Standards in Haematology (2003) Guidelines for the management of acute
painful crisis in sickle cell disease. British Journal of Haematology 120(5), 744-752. [Free Fulltext]
Chinegwundoh, F.I. and Anie, K.A. (2004) Treatments for priapism in boys and men with sickle
cell disease (Cochrane Review). The Cochrane Library. Issue 4. John Wiley & Sons, Ltd.
www.thecochranelibrary.com [Free Full-text]
Claster, S. and Vichinsky, E.P. (2003) Managing sickle cell disease. British Medical Journal
327(7424), 1151-1155. [Free Full-text]
Davies, J.M., Barnes, R. and Milligan, D. (2002) Update of guidelines for the prevention and
treatment of infection in patients with an absent or dysfunctional spleen. Clinical Medicine 2(5),
440-443. [Abstract] [Free Full-text]
Davis, C.J., Mostofi, F.K. and Sesterhenn, I.A. (1995) Renal medullary carcinoma. The seventh
sickle cell nephropathy. American Journal of Surgical Pathology 19(1), 1-11. [Abstract]
de Montalembert, M. (2008) Management of sickle cell disease. British Medical Journal 337(Sep
8), a1397.
Dimashkieh, H., Choe, J. and Mutema, G. (2003) Renal medullary carcinoma: a report of 2 cases
and review of the literature. Archives of Pathology & Laboratory Medicine 127(3), e135-e138.
[Abstract] [Free Full-text]
Dunlop, R. and Bennett, K.C.L.B. (2006) Pain management for sickle cell disease in children and
adults (Cochrane Review). The Cochrane Library. Issue 2. John Wiley & Sons, Ltd.
www.thecochranelibrary.com [Free Full-text]
Falletta, J.M., Woods, G.M., Verter, J.I. et al. (1995) Discontinuing penicillin prophylaxis in
children with sickle cell anemia. Prophylactic Penicillin Study II. Journal of Pediatrics 127(5), 685690. [Abstract]
FSRH (2009) UK medical eligibility criteria for contraceptive use. Faculty of Sexual and
Reproductive Healthcare. www.fsrh.org [Free Full-text]

Gaston, M.H., Verter, J.I., Woods, G. et al. (1986) Prophylaxis with oral penicillin in children with
sickle cell anemia. A randomized trial. The New England Journal of Medicine 314(25), 15931599. [Abstract]
Gill, F.M., Brown, A., Gallagher, D. et al. (1989) Newborn experience in the Cooperative Study of
Sickle Cell Disease. Pediatrics 83(5), 827-829.
Hirst, C. and Owusu-Ofori, S. (2002) Prophylactic antibiotics for preventing pneumococcal
infection in children with sickle cell disease (Cochrane Review). The Cochrane Library. Issue 3.
John Wiley & Sons, Ltd. www.thecochranelibrary.com [Free Full-text]
Hirst, C. and Wang, W.C. (2002) Blood transfusion for preventing stroke in people with sickle cell
disease (Cochrane Review). The Cochrane Library. Issue 1. John Wiley & Sons, Ltd.
www.thecochranelibrary.com [Free Full-text]
Joint Royal Colleges Ambulance Liaison Committee (2009) UK Ambulance Service clinical
practice guidelines: sickle cell crisis - update. University of Warwick. www2.warwick.ac.uk [Free
Full-text]
Jones, A.P., Davies, S.C. and Olujohungbe, A. (2001) Hydroxyurea for sickle cell disease
(Cochrane Review). The Cochrane Library. Issue 2. John Wiley & Sons, Ltd.
www.thecochranelibrary.com [Free Full-text]
Lees, C., Davies, S.C. and Dezateux, C. (2000) Neonatal screening for sickle cell disease (Cochrane
Review). The Cochrane Library. Issue 1. John Wiley & Sons, Ltd. www.thecochranelibrary.com
[Free Full-text]
Legardy, J.K. and Curtis, K.M. (2006) Progestogen-only contraceptive use among women with
sickle cell anemia: a systematic review. Contraception 73(2), 195-204. [Abstract]
Lucas, S.B., Mason, D.G., Mason, M. and Weyman, D. (2008) A sickle crisis? A report of the
national confidential enquiry into patient outcome and death (2008). National Confidential
Enquiry into Patient Outcome and Death. www.ncepod.org.uk [Free Full-text]
Mart-Carvajal, A.J., Conterno, L.O. and Knight-Madden, J.M. (2007) Antibiotics for treating acute
chest syndrome in people with sickle cell disease (Cochrane Review). The Cochrane Library.
Issue 2. John Wiley & Sons, Ltd. www.thecochranelibrary.com [Free Full-text]
Mehta, S.R., Afenyi-Annan, A., Byrns, P.J. and Lottenberg, R. (2006) Opportunities to improve
outcomes in sickle cell disease. American Family Physician 74(2), 303-310. [Abstract] [Free Fulltext]
NHS Antenatal and Newborn Screening Programmes (2006) Sickle cell disease in childhood:
standards and guidelines for clinical care. . .London: NHS Sickle Cell and Thalassaemia Screening

Programme in partnership with the Sickle Cell Society. http://sct.screening.nhs.uk [Free Fulltext]
NHS Sickle Cell and Thalassaemia Screening Programme (2006) Laboratory data report:
development towards a quality report. NHS Sickle Cell and Thalassaemia Screening Programme.
http://sct.screening.nhs.uk
O'Brien, P. (2001) New WHO medical eligibility criteria for contraceptives: adaptation for use in
a local service in UK. Journal of Family Planning and Reproductive Health Care 27(3), 149-152.
Okomo, U. and Meremikwu, M.M. (2007) Fluid replacement therapy for acute episodes of pain
in people with sickle cell disease (Cochrane Review). The Cochrane Library. Issue 2. John Wiley &
Sons, Ltd. www.thecochranelibrary.com [Free Full-text]
Oringanje, C., Nemecek, E. and Oniyangi, O. (2009) Hematopoietic stem cell transplantation for
children with sickle cell disease (Cochrane Review). The Cochrane Library. Issue 1. John Wiley &
Sons, Ltd. www.thecochranelibrary.com [Free Full-text]
Platt, O.S., Brambilla, D.J., Rosse, W.F. et al. (1994) Mortality in sickle cell disease - life
expectancy and risk factors for early death. New England Journal of Medicine 330(23), 16391644. [Abstract] [Free Full-text]
RCOG (2004) Venous thromboembolism and hormonal contraception. . Guideline 40. Royal
College of Obstetricians and Gynaecologists. www.rcog.org.uk [Free Full-text]
Ryan, K., Bain, B.J., Worthington, D. et al. (2010) Significant haemoglobinopathies: guidelines for
screening and diagnosis. British Journal of Haematology 149(1), 35-49. [Free Full-text]
Sears, D.A. (1978) The morbidity of sickle cell trait: a review of the literature. American Journal
of Medicine 64(6), 1021-1036.
Sickle Cell Society (2005) Sickle cell: a guide for GPs, nurses and other health professionals. .
.London: Sickle Cell Society. www.sicklecellsociety.org
Sickle Cell Society (2008) Standards for the clinical care of adults with sickle cell disease in the
UK. . .London: Sickle Cell Society. http://sct.screening.nhs.uk [Free Full-text]
Swartz, M.A., Karth, J., Schneider DT et al. (2002) Renal medullary carcinoma: clinical,
pathologic, immunohistochemical, and genetic analysis with pathogenetic implications. Urology
60(6), 1083-1089. [Abstract]
Teach, S.J., Lillis, K.A. and Grossi, M. (1998) Compliance with penicillin prophylaxis in patients
with sickle cell disease. Archives of Pediatrics Adolescent Medicine 152(3), 274-278. [Abstract]
[Free Full-text]

Vichinsky, E. (2001) Transfusion therapy in sickle cell disease. Information Center for Sickle Cell
and Thalassemic Disorders. http://sickle.bwh.harvard.edu [Free Full-text]
Watanabe, I.C., Billis, A., Guimaraes, M.S. et al. (2007) Renal medullary carcinoma: report of
seven cases from Brazil. Modern Pathology 20(9), 914-920. [Abstract] [Free Full-text]
Weatherall, D.J. (2010) Disorders of the synthesis or function of haemoglobin. In: Warrell, D.A.,
Cox, T.M., Firth, J.D. and Benz, E.J.Jr (Eds.) Oxford textbook of medicine. 5th edn. Oxford: Oxford
University Press. 4420-4444.
Wethers, D.L. (2000) Sickle cell disease in childhood: part II. Diagnosis and treatment of major
complications and recent advances in treatment. American Family Physician 62(6), 1309-1314.
[Abstract] [Free Full-text]
Wierenga, K.J., Hambleton, I.R. and Lewis, N.A. (2001) Survival estimates for patients with
homozygous sickle cell disease in Jamaica: a clinic based population study. Lancet 357(9257),
680-683. [Abstract]
Question 59 of 80

Each one of the following statements regarding glue ear is correct, except:
Incidence peaks at 2 years of age

Also referred to as serous otitis media

Around 1 in 7 children will have an episode before the age of 8 years

Male sex is a risk factor

Hearing loss is usually the presenting feature

Next question

The majority of children will have at least one episode during childhood

Glue ear
Glue ear describes otitis media with an effusion (other terms include serous otitis media). It is common with the majority of children having
at least one episode during childhood
Risk factors

male sex
siblings with glue ear
higher incidence in Winter and Spring
bottle feeding
day care attendance

parental smoking

Features

peaks at 2 years of age


hearing loss is usually the presenting feature (glue ear is the commonest cause of conductive hearing loss and elective surgery
in childhood)
secondary problems such as speech and language delay, behavioural or balance problems may also be seen

Treatment options include:

grommet insertion - to allow air to pass through into the middle ear and hence do the job normally done by the Eustachian tube.
The majority stop functioning after about 10 months
adenoidectomy

Question 60 of 80

Which one of the following is most strongly associated with coeliac disease?
HLA-DQ3

HLA-B4

HLA-DR4

HLA-DR3

HLA-DQ2

Next question

Coeliac disease in children


Coeliac disease is caused by sensitivity to the protein gluten. Repeated exposure leads to villous atrophy which in turn causes
malabsorption. Children normally present before the age of 3 years, following the introduction of cereals into the diet
Genetics

incidence between 1:1,000 and 1:2,000


it is strongly associated with HLA-DQ2 (95% of patients) and HLA-B8 (80%)

Features may coincide with the introduction of cereals (i.e. gluten)

failure to thrive
diarrhoea
abdominal distension
older children may present with anaemia

many cases are not diagnosed to adulthood

Diagnosis

jejunal biopsy showing subtotal villous atrophy


anti-endomysial and anti-gliadin antibodies are useful screening tests

Question 61 of 80

Which one of the following statements regarding developmental dysplasia of the hip is true?
Affects around 10% of newborns

Birth weight less than 3 kg is a risk factor

20% of cases are bilateral

The Ortolani test attempts to dislocate an articulated femoral head

Polyhydramnios is a risk factor

Next question

Developmental dysplasia of the hip


Developmental dysplasia of the hip (DDH) is gradually replacing the old term 'congenital dislocation of the hip' (CDH). It affects around 13% of newborns.
Risk factors

female sex: 6 times greater risk


breech presentation
positive family history
firstborn children
oligohydramnios
birth weight > 5 kg
congenital calcaneovalgus foot deformity

DDH is slightly more common in the left hip. Around 20% of cases are bilateral.
Clinical examination is made using the Barlow and Ortolani tests:

Barlow test: attempts to dislocate an articulated femoral head


Ortolani test: attempts to relocate a dislocated femoral head

Ultrasound is used to confirm the diagnosis if clinically suspected

Management

most unstable hips will spontaneously stabilise by 3-6 weeks of age


Pavlik harness (flexion-abduction orthosis) in children younger than 4-5 months
older children may require surgery

Question 62 of 80

How many deciduous teeth do humans have?


20

24

28

32

36

Next question

Dentition
Basics

20 deciduous (milk) teeth


32 permanent teeth (including 4 wisdom teeth)

The age at which children develop their first teeth varies from birth (rare) to around 1 year of age, but it tends to occur at around 6 months.
The first teeth to appear are usually the lower incisors
Children usually develop all their deciduous teeth by two-and-a-half years of age.

Question 63 of 80

A mother brings her child as she is concerned he is clumsy compared to other similar aged children. At what age would the average child
acquire a good pincer grip?
5-6 months

7-8 months

12 months

18 month

2 years

Next question

Developmental milestones: fine motor and vision


The tables below summarises the major fine motor and vision developmental milestones
Age

Milestone

3 months

Reaches for object


Holds rattle briefly if given to hand
Visually alert, particularly human faces
Fixes and follows to 180 degrees

Holds in palmar grasp


6 months Pass objects from one hand to another
Visually insatiable, looking around in every direction

9 months

Points with finger


Early pincer

12 months

Good pincer grip


Bangs toys together

Bricks
Age

Milestone

15 monthsTower of 2
18 monthsTower of 3
2 years

Tower of 6

3 years

Tower of 9

Drawing
Age

Milestone

18 monthsCircular scribble
2 years

Copies vertical line

3 years

Copies circle

4 years

Copies cross

5 years

Copies square and triangle

Book

Age

Milestone

15 monthsLooks at book, pats page


18 monthsTurns pages, several at time
2 years

Turns pages, one at time

Notes

hand preference before 12 months is abnormal and may indicate cerebral palsy

Question 64 of 80

Which one of the following is least associated with Reye's syndrome?


Hypoglycaemia

Preceding aspirin use

Purpuric skin lesions

Seizures

Preceding viral infection

Next question

Reye's syndrome

viral/aspirin use may trigger


features include encephalopathy, fatty infiltration and hypoglycaemia

Reye's syndrome
Reye's syndrome is a severe, progressive encephalopathy affecting children that is accompanied by fatty infiltration of the liver, kidneys
and pancreas. The aetiology of Reye's syndrome is not fully understood although there is a known association with aspirin use and a viral
cause has been postulated
The peak incidence is 2 years of age, features include:

may be history of preceding viral illness


encephalopathy: confusion, seizures, cerebral oedema, coma
fatty infiltration of the liver, kidneys and pancreas
hypoglycaemia

Management is supportive
Although the prognosis has improved over recent years there is still a mortality rate of 15-25%.
Question 65 of 80

What is the risk of a newborn infant at 27 weeks gestation having surfactant deficient lung disease?
25%

50%

75%

90%

95%

Next question

Surfactant deficient lung disease


Surfactant deficient lung disease (SDLD., also known as respiratory distress syndrome and previously as hyaline membrane disease) is a
condition seen in premature infants. It is caused by insufficient surfactant production and structural immaturity of the lungs
The risk of SDLD decreases with gestation

50% of infants born at 26-28 weeks


25% of infants born at 30-31 weeks

Other risk factors for SDLD include

male sex
diabetic mothers
Caesarean section
second born of premature twins

Clinical features are those common to respiratory distress in the newborn, i.e. tachypnoea, intercostal recession, expiratory grunting and
cyanosis
Chest x-ray characteristically shows 'ground-glass' appearance with an indistinct heart border
Management

prevention during pregnancy: maternal corticosteroids to induce fetal lung maturation


oxygen
assisted ventilation
exogenous surfactant given via endotracheal tube

Question 66 of 80

You review a 4-year-old boy in clinic. He has been diagnosed with asthma after having multiple wheezy episodes over the past 3 years.
Around 4 months ago he was admitted with shortness-of-breath and wheeze and was diagnosed as having a viral exacerbation of asthma
by the paediatric team. Prior to his discharge he was given a Clenil (beclometasone dipropionate) inhaler 100mcg bd in addition to
salbutamol 100mcg prn via a spacer.
His mother reports that he has a persistent night-time cough and is regularly having to use his salbutamol inhaler. Clinical examination of
his chest today is normal.
What is the most appropriate next step in management?
Add a long-acting beta agonist

Add a short-acting muscarinic antagonist

Add a leukotriene receptor antagonist

Add a long-acting muscarinic antagonist

Switch Clenil to Pulmicort (budesonide)

Next question

Step 3 of asthma management in children 2-5 years: add a leukotriene receptor antagonist

Asthma in children: stepwise management


The British Thoracic Society differentiate between children younger and older than 5 years in their 2014 guidelines:
Children aged under 5 years
StepTherapy
1

As-required reliever therapy: short-acting beta2-agonist

Regular preventer therapy: inhaled corticosteroids, 200-400mcg/day*


Or, if inhaled corticosteroids cannot be used, a leukotriene receptor antagonist

Children aged 2-5 years: trial of a leukotriene receptor antagonist. If already taking leukotriene receptor antagonist reconsider inhaled
corticosteroids
Children aged under 2 years: refer to respiratory paediatrician

Refer to a respiratory paediatrician

Children aged over 5 years (similar to adult guidance)


StepTherapy
1

As-required reliever therapy: short-acting beta2-agonist

Regular preventer therapy: inhaled corticosteroids, 200-400mcg/day*


1. Add inhaled long-acting B2 agonist (LABA)

3
2. Assess control of asthma:

StepTherapy

good response to LABA - continue LABA


benefit from LABA but control still inadequate: continue LABA and increase inhaled steroid dose to 400 mcg/day* (if not
already on this dose)
no response to LABA: stop LABA and increase inhaled steroid to 400 mcg/ day.* If control still inadequate, institute trial of
other therapies, leukotriene receptor antagonist or SR theophylline

Increase inhaled corticosteroids to high-dose, up to 800mcg/day*


Use daily steroid tablet at lowest dose providing control

Maintain inhaled corticosteroids at 800mcg/day


Refer to a paediatrician

*beclometasone dipropionate or equivalent

Question 67 of 80

A 2 month-old boy presents to the department with a desquamating rash affecting the palms of the hand and soles of the feet. On
examination he has a low grade fever of 37.8C and there is generalised lymphadenopathy, hepatosplenomegaly and a muco-purulent
discharge from the nose.
What is the most likely diagnosis?
Cytomegalovirus

Enterovirus

Parvovirus B19

Rubella virus

Congenital syphilis

Next question

The above signs are classically seen in congenital syphilis.

Syphilis
Syphilis is a sexually transmitted infection caused by the spirochaete Treponema pallidum. Infection is characterised by primary, secondary
and tertiary stages. The incubation period is between 9-90 days
Primary features

chancre - painless ulcer at the site of sexual contact


local non-tender lymphadenopathy
often not seen in women (the lesion may be on the cervix)

Secondary features - occurs 6-10 weeks after primary infection

systemic symptoms: fevers, lymphadenopathy


rash on trunk, palms and soles
buccal 'snail track' ulcers (30%)
condylomata lata

Image used on license from DermNet NZ

Classical palm lesions of secondary syphilis

Image used on license from DermNet NZ

More generalised rash of secondary syphilis

Tertiary features

gummas
aortic aneurysms
general paralysis of the insane
tabes dorsalis

Features of congenital syphilis

blunted upper incisor teeth


keratitis
saber shins
saddle nose
deafness

Question 68 of 80

A 17-year-old male with a history of cystic fibrosis presents to clinic for annual review. What is the most appropriate advice regarding his
diet?
High calorie and low fat with pancreatic enzyme supplementation for every meal

High calorie and low fat with pancreatic enzyme supplementation for evening meal

Normal calorie and low fat with pancreatic enzyme supplementation for every meal

High calorie and high fat with pancreatic enzyme supplementation for evening meal

High calorie and high fat with pancreatic enzyme supplementation for every meal

Next question

Please see the link for more details.

Cystic fibrosis: management


Management of cystic fibrosis involves a multidisciplinary approach
Key points

regular (at least twice daily) chest physiotherapy and postural drainage. Parents are usually taught to do this. Deep breathing
exercises are also useful
high calorie diet, including high fat intake*
vitamin supplementation
pancreatic enzyme supplements taken with meals
heart and lung transplant

*this is now the standard recommendation - previously high calorie, low-fat diets have been recommended to reduce the amount of
steatorrhoea
Question 69 of 80

A 4-year-old girl is brought to surgery due to dysuria. On examination her temperature is 37.2C, abdominal examination is unremarkable
and urine dipstick is positive for leukocytes and nitrites. She has no past medical history of note. What is the most appropriate
management, other than urine microscopy?
Oral antibiotics for 10 days + follow-up if not settled + static radioisotope scan after 4 months

Oral antibiotics for 3 days + follow-up if not settled

Oral antibiotics for 7 days + follow-up if not settled

Oral antibiotics for 10 days + follow-up if not settled + micturating cystourethrography

Oral antibiotics for 10 days + follow-up if not settled + ultrasound within 6 weeks

Next question

The 2007 NICE guidelines are controversial - many paediatricians are still taking the approach of imaging more children than strictly
suggested by the guidelines.

Urinary tract infection in children: features, diagnosis and management


Urinary tract infections (UTI) are more common in boys until 3 months of age (due to more congenital abnormalities) after which the
incidence is substantially higher in girls. At least 8% of girls and 2% of boys will have a UTI in childhood
Presentation in childhood depends on age:

infants: poor feeding, vomiting, irritability


younger children: abdominal pain, fever, dysuria
older children: dysuria, frequency, haematuria
features which may suggest an upper UTI include: temperature > 38C, loin pain/tenderness

NICE guidelines for checking urine sample in a child

if there are any symptoms or signs suggestive or a UTI


with unexplained fever of 38C or higher (test urine after 24 hours at the latest)
with an alternative site of infection but who remain unwell (consider urine test after 24 hours at the latest)

Urine collection method

clean catch is preferable


if not possible then urine collection pads should be used
cotton wool balls, gauze and sanitary towels are not suitable
invasive methods such as suprapubic aspiration should only be used if non-invasive methods are not possible

Management

infants less than 3 months old should be referred immediately to a paediatrician


children aged more than 3 months old with an upper UTI should be considered for admission to hospital. If not admitted oral
antibiotics such as cephalosporin or co-amoxiclav should be given for 7-10 days

children aged more than 3 months old with a lower UTI should be treated with oral antibiotics for 3 days according to local
guidelines, usually trimethoprim, nitrofurantoin, cephalosporin or amoxicillin. Parents should be asked to bring the children back
if they remain unwell after 24-48 hours
antibiotic prophylaxis is not given after the first UTI but should be considered with recurrent UTIs

Urinary tract infection - children - Summary


Urinary tract infection (UTI) is illness caused by micro-organisms in the urinary tract.
Most UTIs are caused by a single species of bacteria from the gastrointestinal tract.
Common organisms causing UTI in children include Escherichia coli (about 75% or more
of cases), Klebsiella species, and Staphylococcus saprophyticus.
Around 1 in 10 girls and 1 in 30 boys will have had a UTI by the time they turn 16 years
of age.
Most UTIs are not associated with any risk factor.
The prognosis after childhood UTI is generally excellent and a course of antibiotics cures
UTI in most infants and children. However, UTI may recur and rarely, long-term
complications (such as renal scarring and hypertension) may occur.
In a child less than 3 months old, UTI should be suspected if there is a combination of:
Fever (without obvious cause), vomiting, irritability, lethargy.
Poor feeding, failure to thrive.
Failure to respond adequately to appropriate treatment of another presumed cause of
the illness.
Abdominal pain, jaundice, haematuria, offensive urine.
An urgent admission should be made in all infants less than 3 months of age with
suspected UTI. Urine testing is not necessary in primary care.
In a child over 3 months of age, UTI should be suspected if there is a combination of:
Frequency, dysuria.
Fever without an obvious cause (if the temperature is greater than 38C urine should be
tested within 24 hours).
Fever with a presumed cause (other than UTI) but poor response to treatment (if there
is doubt about the cause of fever urine should be tested within 24 hours).
Abdominal pain, loin tenderness.
Voluntary withholding or incontinence of urine or faeces.
Vomiting, poor feeding.
Malaise, lethargy, irritability.
Haematuria, offensive urine, cloudy urine.
Failure to thrive.
Management of suspected UTI in a child over 3 months of age involves:
Assessing the risk of serious illness and arranging a referral (or admission) with the
appropriate urgency. This should include taking a full history; measurement of blood
pressure; abdominal palpation; inspection of external genitalia, anus and urethral
meatus; assessment of lower back and limbs, and examination of urine.
Considering the need for antibiotics. A urine specimen should be sent for culture and
sensitivities prior to starting antibiotics.

Arranging a review (e.g. after 48 hours) to ensure response to treatment and to reassess
the antibiotic choice, if applicable.
Offering appropriate advice to parents or carers (such as to return for assessment if the
child is still unwell after 2448 hours of treatment or if they suspect a repeat UTI).
Infants/children aged over 3 months with suspected UTI are at:
High risk of serious illness if they are systemically unwell, dehydrated, or vomiting and
cannot tolerate oral fluids and medication or have a history or clinical features
suggesting urinary tract obstruction.
Low risk of serious illness if temperature is less than 38C with no history of fever and
there is no loin pain/tenderness.
Intermediate risk of serious illness if they do not satisfy the criteria for being at high or
low risk.
Have I got the right topic?
Age from 0 months to 16 years
This CKS topic is based on the National Institute for Health and Care Excellence guideline
Urinary tract infection in children: diagnosis, treatment and long-term management
[National Collaborating Centre for Women's and Children's Health, 2007].
This CKS topic covers the management in primary care of urinary tract infection (UTI) in
infants and children.
This CKS topic does not cover the management of recurrent UTI in sexually active girls,
or UTI in infants or children with:
Indwelling urinary catheters.
Neurogenic bladders.
Using intermittent catheterization.
Significant uropathy.
Underlying renal disease.
Immunosuppression.
This CKS topic does not cover in any detail the investigations used to detect renal tract
abnormalities in children with UTI as these will usually be arranged and performed in
secondary care. However referral to secondary care for consideration of further
investigation is discussed.
There are separate CKS topics on Urinary tract infection (lower) - women, and Urinary
tract infection (lower) - men.
The target audience for this CKS topic is healthcare professionals working within the
NHS in the UK, and providing first contact or primary health care.
How up-to-date is this topic?
Changes
Update
Goals and outcome measures
Goals
QIPP - options for local implementation
Background information

Definition
Causal organisms
Prevalence
Risk factors
Prognosis
Complications
Diagnosis
Diagnosis (< 3 months of age)
Diagnosis (3 months to 3 years of age)
Diagnosis (> 3 years of age)
Management
Scenario: UTI less than 3 months of age : covers the management of infants less than
three months of age, with a urinary tract infection.
Scenario: UTI 3 months to 3 years of age : covers the management of children between
three months and three years of age, with a urinary tract infection.
Scenario: UTI over 3 years of age : covers the management of children over three years
of age, with a urinary tract infection.
Scenario: UTI less than 3 months of age
Scenario: UTI 3 months to 3 years of age
Scenario: UTI over 3 years of age
Prescribing information
Important aspects of prescribing information relevant to primary healthcare are covered
in this section specifically for the drugs recommended in this CKS topic. For further
information on contraindications, cautions, drug interactions, and adverse effects, see
the electronic Medicines Compendium (eMC) (http://medicines.org.uk/emc), or the
British National Formulary (BNF) (www.bnf.org).
Trimethoprim
Nitrofurantoin
Cefalexin
Cefixime
Amoxicillin and co-amoxiclav
Paracetamol
Evidence
Supporting evidence
Search strategy
References
ABPI Medicines Compendium (2010) Summary of product characteristics for Augmentin
125/31 SF Suspension. Electronic Medicines CompendiumDatapharm Communications
Ltd. www.medicines.org.uk [Free Full-text]
ABPI Medicines Compendium (2012a) Summary of product characteristics for Cefalexin
250mg Capsules. Electronic Medicines CompendiumDatapharm Communications Ltd.
www.medicines.org.uk [Free Full-text]

ABPI Medicines Compendium (2012b) Summary of product characteristics for


Macrodantin Capsules 50mg B.P Electronic Medicines CompendiumDatapharm
Communications Ltd. www.medicines.org.uk [Free Full-text]
ABPI Medicines Compendium (2012c) Summary of product characteristics for Macrobid
Capsules 100mg B.P Electronic Medicines CompendiumDatapharm Communications Ltd.
www.medicines.org.uk [Free Full-text]
ABPI Medicines Compendium (2013a) Summary of product characteristics for
Augmentin 625mg Tablets. Electronic Medicines CompendiumDatapharm
Communications Ltd. www.medicines.org.uk [Free Full-text]
ABPI Medicines Compendium (2013b) Summary of product characteristics for Suprax
Powder for Paediatric Oral Suspension. Electronic Medicines CompendiumDatapharm
Communications Ltd. www.medicines.org.uk [Free Full-text]
BNF 64 (2012) British National Formulary. 64th edn. London: British Medical Association
and Royal Pharmaceutical Society of Great Britain.
BNF for Children (2012) British National Formulary for Children 2012/13. London: British
Medical Association and the Royal Pharmaceutical Society of Great Britain.
Brumfitt, W., Hamilton-Miller, J. and Bailey, R.R. (Eds.) (1998) Urinary tract infections.
London: Chapman & Hall Medical.
CSM (1997) Revised indications for co-amoxiclav (Augmentin). Current Problems in
Pharmacovigilance 23(May), 8. [Free Full-text]
DTB (1996) Penicillin allergy. Drug & Therapeutics Bulletin 34(11), 87-88.
Fitzgerald, A., Mori, R., Lakhanpaul, M. and Tullus, K. (2012) Antibiotics for treating
lower urinary tract infection in children (Cochrane Review). The Cochrane Library. Issue
8. John Wiley & Sons, Ltd. www.thecochranelibrary.com [Free Full-text]
Grabe, M., Bjerklund-Johansen, T.E., Botto, H. et al. (2013) Guidelines on urological
infections. European Association of Urology. www.uroweb.org [Free Full-text]
Hodson, E.M., Willis, N.S. and Craig, J.C. (2007) Antibiotics for acute pyelonephritis in
children (Cochrane Review). The Cochrane Library. Issue 4. John Wiley & Sons, Ltd.
www.thecochranelibrary.com [Free Full-text]
HPA (2011) Diagnosis of UTI: quick reference guide for primary care for consultation and
local adaptation. Health Protection Agency. www.hpa.org.uk [Free Full-text]
HPA and Association of Medical Microbiologists (2008) Management of infection
guidance for primary care for consultation and local adaptation. Health Protection
Agency. www.hpa.org.uk [Free Full-text]
HPA and British Infection Association (2013) Management of infection guidance for
primary care for consultation and local adaptation. Health Protection Agency.
www.hpa.org.uk [Free Full-text]
Lambert, H.J. and Coulthard, M.G. (2002) The child with urinary tract infection. In:
Webb, N. and Postlethwaite, R. (Eds.) Clinical paediatric nephrology. 3rd edn. Oxford:
Oxford University Press.
Larcombe, J. (2010) Urinary tract infection in children. Clinical EvidenceBMJ Publishing
Group Ltd. www.clinicalevidence.com [Free Full-text]

MHRA (2011) Press release: more exact paracetamol dosing for children to be
introduced. Medicines and Healthcare products Regulatory Agency. www.mhra.gov.uk
[Free Full-text]
MHRA (2014) Nitrofurantoin now contraindicated in most patients with an estimated
glomerular filtration rate (eGFR) of less than 45 ml/min. Drug Safety Update 8(2), A3.
[Free Full-text]
Miron, D., Daas, A., Sakran, W. et al. (2007) Is omitting post urinary-tract-infection renal
ultrasound safe after normal antenatal ultrasound? An observational study. Archives of
Disease in Childhood 92(6), 502-504. [Abstract] [Free Full-text]
National Collaborating Centre for Women's and Children's Health (2007) Urinary tract
infection in children: diagnosis, treatment and long-term management (full NICE
guideline). . Clinical guideline 54. National Institute for Health and Care Excellence.
www.nice.org.uk [Free Full-text]
NICE (2009) When to suspect child maltreatment (NICE guideline). . Clinical guideline 89.
National Institute for Health and Care Excellence. www.nice.org.uk [Free Full-text]
NICE (2013) Key therapeutic topics - medicines management options for local
implementation. National Institute for Health and Care Excellence. www.nice.org.uk
[Free Full-text]
NPC (2011) Key therapeutic topics 2010/11 - Medicines management options for local
implementation. National Prescribing Centre. www.npc.nhs.uk [Free Full-text]
NPC (2012) Key therapeutic topics - medicines management options for local
implementation. National Prescribing Centre. www.npc.nhs.uk [Free Full-text]
Shaikh, N., Ewing, A.L., Bhatnagar, S. and Hoberman, A. (2010) Risk of renal scarring in
children with a first urinary tract infection: a systematic review. Pediatrics 126(6), 10841091. [Abstract] [Free Full-text]
Stamm, W.E. (1998) Urinary tract infections and pyelonephritis. In: Fauci, A.S.,
Braunwald, E. and Isselbacher, K.J. (Eds.) Harrison's principles of internal medicine. New
York: McGraw-Hill.
Tomson, C. and Armitage, A. (2010) Urinary tract infection. In: Warrell, D.A., Cox, T.M.
and Firth, J.D. (Eds.) Oxford textbook of medicine. 5th edn. Oxford: Oxford University
Press. 4103-4122.
Tullus, K. (2007) Personal communication. Consultant Paediatric Nephrologist, Great
Ormond Street Hospital: London.
Williams, G. and Craig, J.C. (2011) Long-term antibiotics for preventing recurrent urinary
tract infection in children (Cochrane Review). The Cochrane Library. Issue 3. John Wiley
& Sons, Ltd. www.thecochranelibrary.com [Free Full-text]
Leave feedback
Topics
Specialities
Educational slides

Indicators

Follow Us
About
Accessibility
Freedom of information
Contact Us
Glossary
Terms and conditions
Subscribe
Copyright 2014 National Institute for Health and Care Excellence. All rights reserved.

Question 70 of 80

What is the investigation of choice to diagnose vesicoureteric reflux?


CT

Abdominal x-ray

DMSA

Micturating cystourethrogram

Ultrasound

Next question

Vesicoureteric reflux
Vesicoureteric reflux (VUR) is the abnormal backflow of urine from the bladder into the ureter and kidney. It is relatively common
abnormality of the urinary tract in children and predisposes to urinary tract infection (UTI), being found in around 30% of children who
present with a UTI. As around 35% of children develop renal scarring it is important to investigate for VUR in children following a UTI
Pathophysiology of VUR

ureters are displaced laterally, entering the bladder in a more perpendicular fashion than at an angle
therefore shortened intramural course of ureter
vesicoureteric junction cannot therefore function adequately

The table below summarises the grading of VUR


Grade

Grade
I

Reflux into the ureter only, no dilatation

II

Reflux into the renal pelvis on micturition, no dilatation

III

Mild/moderate dilatation of the ureter, renal pelvis and calyces

IV

Dilation of the renal pelvis and calyces with moderate ureteral tortuosity

Gross dilatation of the ureter, pelvis and calyces with ureteral tortuosity

Investigation

VUR is normally diagnosed following a micturating cystourethrogram


a DMSA scan may also be performed to look for renal scarring

Question 71 of 80

Which one of the following is not a risk factor for child abuse?
Disability

Low birth weight

Small family size

Chronic illness

Prematurity

Next question

Child abuse: risk factors


Risk factors for child abuse - child factors

prematurity
low birth weight
disability
chronic illness

Risk factors for child abuse - parental factors

personal history of child abuse


teenage parents
single parent
substance abuse

psychiatric disorder
isolation
large family size

Question 72 of 80

Each one of the following is associated with tall stature, except:


Hyperthyroidism

Marfan syndrome

Kallman's syndrome

Klinefelter's syndrome

Homocystinuria

Next question

Tall stature
Causes of tall stature

constitutional
excessive growth hormone (pituitary gigantism)
hyperthyroidism
Marfan syndrome
homocystinuria
Klinefelter's syndrome

Congenital adrenal hyperplasia may be associated with a premature growth spurt but early fusion of the epiphyses results in short stature

Question 73 of 80

Which one of the following statements regarding iron deficiency anaemia in children is incorrect?
The prevalence is around 2%

The most common cause is dietary

Cow's milk is a poor source of iron

Management can be done in primary care and involves dietary advice and iron supplementation

Asian, Afro-Caribbean and Chinese children are more commonly affected

Next question

Iron deficiency anaemia in children


Iron deficiency anaemia is the most common nutritional disorder of childhood, affecting around 10% of children in the UK. The prevalence is
higher in Asian, Afro-Caribbean and Chinese children
Causes

socioeconomic - iron supplemented milk formulas may be more expensive


unmodified cow's milk - a poor source of iron due to it being in a form that is not absorbed well, therefore should be introduced
after 1 year of age*
ethnic origin - e.g. Asian mothers may introduce solids later

Prevention

supplementary iron in milk


dietary education
free formulas for at risk infants

*whilst breast milk is relatively low in iron it is present in a form that is easily absorbed

Question 74 of 80

A 13-month-old baby girl has her first MMR vaccination. If side-effects do occur, which on of the following best describes the likely
symptoms?
Diarrhoea: occurs after 5-10 days and lasts around 2-3 days

Diarrhoea: occurs after 2-3 days and lasts around 1-2 days

Malaise, fever and rash: occurs after 5-10 days and lasts around 2-3 days

Arthralgia: occurs after 5-10 days and lasts around 2-3 days

Malaise, fever and rash: occurs after 2-3 days and lasts around 1-2 days

Next question

MMR vaccine
Children in the UK receive two doses of the Measles, Mumps and Rubella (MMR) vaccine before entry to primary school. This currently
occurs at 12-15 months and 3-4 years as part of the routine immunisation schedule
Contraindications to MMR

severe immunosuppression
allergy to neomycin
children who have received another live vaccine by injection within 4 weeks
pregnancy should be avoided for at least 1 month following vaccination
immunoglobulin therapy within the past 3 months (there may be no immune response to the measles vaccine if antibodies are
present)

Adverse effects

malaise, fever and rash may occur after the first dose of MMR. This typically occurs after 5-10 days and lasts around 2-3 days

Question 75 of 80

What is the most common cause of childhood hypothyroidism in the United Kingdom?
Iodine deficiency

Autoimmune thyroiditis

Follicular thyroid cancer

Post total-body irradiation

Grave's disease

Next question

Hypothyroidism in children
The most common cause of hypothyroidism in children (juvenile hypothyroidism) is autoimmune thyroiditis.
Other causes include

post total-body irradiation (e.g. in a child previous treated for acute lymphoblastic leukaemia)
iodine deficiency (the most common cause in the developing world)

Question 76 of 80

Which one of the following is not a recognised benefit of breast feeding?

Helps involution of the uterus

Reduced incidence of gastro-intestinal infections

Improved infant weight gain

Reduced incidence of type 1 diabetes mellitus

Protection against breast cancer

Next question

Improved weight gain is not a recognised benefit of breast feeding

Breast feeding

Advantages

Disadvantages

Mother

bonding
involution of uterus
protection against breast and ovarian cancer
cheap, no need to sterilise bottle

Transmission of drugs

contraceptive effect (unreliable)


Transmission of infection (e.g. HIV)

Immunological

Nutrient inadequacies (prolonged breast


feeding may lead to vitamin D deficiency)

IgA (protects mucosal surfaces), lysozyme (bacteriolytic enzyme) and lactoferrin Vitamin K deficiency
(ensures rapid absorption of iron so not available to bacteria)
Breast milk jaundice
reduced incidence of ear, chest and gastro-intestinal infections
reduced incidence of eczema and asthma
reduced incidence of type 1 diabetes mellitus

Reduced incidence of sudden infant death syndrome


Baby is in control of how much milk it takes
Question 77 of 80

At what age would the average child start to play alongside, but not interacting with, other children?
3 months

6 months

12 months

2 years

4 years

Next question

Developmental milestones: social behaviour and play


The table below summarises the major social behaviour and play milestones
Age

Milestone

6 weeks Smiles (Refer at 10 weeks)


3 months

Laughs
Enjoys friendly handling

6 monthsNot shy
9 months

Shy
Takes everything to mouth

Feeding
Age

Milestone

May put hand on bottle when being fed

6 months

Drinks from cup + uses spoon, develops over 3 month period 12 -15 months
Competent with spoon, doesn't spill with cup

2 years

Uses spoon and fork

3 years

Uses knife and fork

5 years

Dressing
Age

Milestone

Helps getting dressed/undressed

12-15 months

Takes off shoes, hat but unable to replace

18 months

Puts on hat and shoes

2 years

Can dress and undress independently except for laces and buttons 4 years

Play
Age

Milestone

Plays 'peek-a-boo'

9 months

Waves 'bye-bye'
Plays 'pat-a-cake'

12 months

Age

Milestone

Plays contentedly alone

18 months

Plays near others, not with them 2 years


Plays with other children

4 years

Question 78 of 80

A newborn male baby is found to have an undescended right testicle during the routine newborn exam. It is neither palpable in the scrotum
or inguinal canal. What is the most appropriate management?
Outpatient referral to urology to be seen within 6 weeks

Reassurance, advise GP follow-up

Immediate referral to urology

Arrange ultrasound abdomen and scrotum

Start testosterone therapy

Next question

Undescended testicle - wait 6 months prior to referral

If the testicle has not descended by around 6 months then referral should be considered for orchidopexy

Undescended testis
Undescended testis occurs in around 2-4% of term male infants., but is much more common if the baby is preterm. Around 25% of cases
are bilateral
Complications of undescended testis

infertility
torsion
testicular cancer
psychological

Management

orchidopexy: referral should be considered from around 6 months of age. Surgical practices vary although the majority of
procedures are performed at around 1 year of age

Question 79 of 80

How many permanent teeth (including wisdom teeth) do humans have?

24

28

32

36

40

Next question

Dentition
Basics

20 deciduous (milk) teeth


32 permanent teeth (including 4 wisdom teeth)

The age at which children develop their first teeth varies from birth (rare) to around 1 year of age, but it tends to occur at around 6 months.
The first teeth to appear are usually the lower incisors
Children usually develop all their deciduous teeth by two-and-a-half years of age.

Question 80 of 80

Which one of the following conditions is most strongly associated with supravalvular aortic stenosis?
Pierre-Robin syndrome

Edward's syndrome

William's syndrome

Patau syndrome

Noonan syndrome

Next question

Childhood syndromes
Below is a list of common features of selected childhood syndromes

Syndrome

Key features

Patau syndrome (trisomy 13)

Microcephalic, small eyes


Cleft lip/palate
Polydactyly
Scalp lesions

Edward's syndrome (trisomy 18)

Micrognathia
Low-set ears
Rocker bottom feet
Overlapping of fingers

Fragile X

Learning difficulties
Macrocephaly
Long face
Large ears
Macro-orchidism

Noonan syndrome

Webbed neck
Pectus excavatum
Short stature
Pulmonary stenosis

Pierre-Robin syndrome*

Micrognathia
Posterior displacement of the tongue (may result in upper airway obstruction)
Cleft palate

Prader-Willi syndrome

Hypotonia
Hypogonadism
Obesity

William's syndrome

Short stature
Learning difficulties
Friendly, extrovert personality
Transient neonatal hypercalcaemia
Supravalvular aortic stenosis

*this condition has many similarities with Treacher-Collins syndrome. One of the key differences is that Treacher-Collins syndrome is
autosomal dominant so there is usually a family history of similar problems

You might also like

pFad - Phonifier reborn

Pfad - The Proxy pFad of © 2024 Garber Painting. All rights reserved.

Note: This service is not intended for secure transactions such as banking, social media, email, or purchasing. Use at your own risk. We assume no liability whatsoever for broken pages.


Alternative Proxies:

Alternative Proxy

pFad Proxy

pFad v3 Proxy

pFad v4 Proxy